DOKK Library

Basic Algebra With Applications

Authors Ivan G. Zaigralin

License CC-BY-SA-4.0 GPL-3.0-or-later

Plaintext
BASIC ALGEBRA
     With Applications

          Ivan G. Zaigralin
                               © 2017–2020 Ivan G. Zaigralin

                           Cover design © 2018 Derrick Santiago

                   This typeset version of the text is licensed under the

                     Attribution-ShareAlike 4.0 International license,

        or, if a user so desires, any later version published by Creative Commons.

       A human-readable summary of (and not a substitute for) the license follows.



                                       YOU   ARE FREE TO :


                                             SHARE:

              COPY AND REDISTRIBUTE THE MATERIAL IN ANY MEDIUM OR FORMAT


                                             ADAPT:

 REMIX , TRANSFORM , AND BUILD UPON THE MATERIAL FOR ANY PURPOSE , EVEN COMMERCIALLY.


                   THIS   LICENSE IS ACCEPTABLE FOR   FREE CULTURAL WORKS.

THE   LICENSOR CANNOT REVOKE THESE FREEDOMS AS LONG AS YOU FOLLOW THE LICENSE TERMS .



            The source code for the text bears a different free copyleft license.




                                         Edition: 5th

                                     Publisher: Samizdat
                                       Contents

Contents                                          5

Chapter 1. Concepts                               15
  1. Expressions, Relations, and Substitution     15
  1.1. Expressions versus Relations               15
  1.2. Algebraic Substitution                     17
  1.3. Common Types of Expressions                18
  Homework 1.1                                    21
  Homework 1.1 Answers                            22
  2. Real Number Axioms                           23
  2.1. Axioms                                     23
  2.2. Applying Axioms                            27
  2.3. Inverses, Subtraction, Division            28
  2.4. Conclusion                                 30
  Homework 1.2                                    32
  Homework 1.2 Answers                            33
  3. The Number Line and Sets of Numbers          34
  3.1. Roster                                     34
  3.2. Integers                                   34
  3.3. Set Builder                                36
  3.4. Rationals                                  37
  3.5. Reals                                      38
  3.6. Absolute Value                             40
  3.7. Inequality Relations                       42
  Homework 1.3                                    44
  Homework 1.3 Answers                            46
  4. Properties of Real Numbers                   47
  4.1. Basic Properties                           47
  4.2. Positive Integer Exponent                  50
  Homework 1.4                                    52
  Homework 1.4 Answers                            53
  5. Order of Operations                          54
  5.1. Evaluating Sums and Products               54
  5.2. Like Terms                                 57
  Homework 1.5                                    59
  Homework 1.5 Answers                            61
  6. Prime Numbers                                62
                                            5
                                                    CHAPTER 0. CONTENTS
  6.1. Integer Division and Prime Numbers                            62
  6.2. Fractions of Integers in Lowest Terms                         64
  Homework 1.6                                                       67
  Homework 1.6 Answers                                               69
  7. Fractions and Rationals                                         70
  7.1. Fraction Notation                                             70
  7.2. Changing the Denominator                                      74
  Homework 1.7                                                       76
  Homework 1.7 Answers                                               78
  8. Fraction Addition                                               79
  8.1. Adding Fractions with a Common Denominator                    79
  8.2. Least Common Multiple                                         80
  8.3. Adding Fractions of Integers                                  81
  8.4. Mixed Numbers                                                 82
  Homework 1.8                                                       84
  Homework 1.8 Answers                                               86
  9. Translation                                                     87
  9.1. Expressions                                                   87
  9.2. Equations                                                     87
  9.3. Applications                                                  90
  Homework 1.9                                                       91
  Homework 1.9 Answers                                               92
  Practice Test 1                                                    93
  Practice Test 1 Answers                                            94

Chapter 2. Linear Equations                                          95
  1. Properties of Equations                                         95
  1.1. Equations and Solutions                                       95
  1.2. Addition Property for Equations                               97
  1.3. Multiplication Property for Equations                         98
  1.4. Applications                                                  99
  Homework 2.1                                                      101
  Homework 2.1 Answers                                              103
  2. Solving Linear Equations                                       104
  2.1. Linear and Non-linear Equations                              104
  2.2. Isolating The Variable                                       105
  2.3. Applications                                                 110
  Homework 2.2                                                      114
  Homework 2.2 Answers                                              116
  3. Linear Formulas                                                117
  3.1. Isolating a Variable                                         117
  Homework 2.3                                                      121
  Homework 2.3 Answers                                              122
  4. Percent                                                        123
  4.1. Percent Units and Notation                                   123
                                               6
CHAPTER 0. CONTENTS
  4.2. Percent Increase and Decrease            125
  Homework 2.4                                  128
  Homework 2.4 Answers                          129
  5. Applications                               130
  5.1. Distance and Work                        130
  5.2. Applications of w = r t                  132
  Homework 2.5                                  135
  Homework 2.5 Answers                          137
  6. Linear Inequalities                        138
  6.1. Solution Sets                            138
  6.2. Interval Notation and Graphs             139
  6.3. Addition and Multiplication Properties   141
  Homework 2.6                                  144
  Homework 2.6 Answers                          145
  Practice Test 2                               146
  Practice Test 2 Answers                       147

Chapter 3. Graphing                             149
  1. Reading and Constructing Graphs            149
  1.1. Cartesian Plane                          149
  Homework 3.1                                  155
  Homework 3.1 Answers                          158
  2. Graphing Linear Equations                  159
  2.1. Solution Sets                            159
  2.2. Graphing Solution Sets                   160
  2.3. Graphing Lines                           162
  Homework 3.2                                  165
  Homework 3.2 Answers                          166
  3. Intercepts                                 168
  3.1. x and y-intercepts                       168
  3.2. Finding Intercepts                       168
  Homework 3.3                                  171
  Homework 3.3 Answers                          172
  4. Slope                                      174
  4.1. Slope of a Line                          174
  4.2. Using Slope and Intercept to Plot        175
  Homework 3.4                                  177
  Homework 3.4 Answers                          180
  5. Slope-Intercept Form                       182
  5.1. Slope-Intercept Form                     182
  5.2. Parallel Lines                           186
  5.3. Perpendicular Lines                      188
  Homework 3.5                                  191
  Homework 3.5 Answers                          194
  6. Point-Slope Form                           195
                                            7
                                                  CHAPTER 0. CONTENTS
  6.1. A Point and a Slope Determine a Line                       195
  6.2. Two Points Determine a Line                                196
  6.3. Using Point and Slope to Plot                              198
  Homework 3.6                                                    199
  Homework 3.6 Answers                                            200
  7. Function Notation                                            202
  7.1. Notation                                                   202
  7.2. Applications of Linear Functions                           204
  Homework 3.7                                                    208
  Homework 3.7 Answers                                            209
  Practice Test 3                                                 210
  Practice Test 3 Answers                                         211

Chapter 4. Linear Systems                                         213
  1. Graphing Systems of Equations                                213
  1.1. Solution Sets                                              213
  1.2. Solving Systems by Graphing                                215
  Homework 4.1                                                    218
  Homework 4.1 Answers                                            219
  2. Substitution                                                 221
  2.1. Solving Systems using Substitution                         221
  Homework 4.2                                                    224
  Homework 4.2 Answers                                            225
  3. Elimination                                                  226
  3.1. Equivalent Systems                                         226
  3.2. Eliminating Variable with Addition                         226
  Homework 4.3                                                    230
  Homework 4.3 Answers                                            231
  4. Applications of Systems                                      232
  4.1. Constant Rate Problems                                     232
  4.2. Mixing Problems                                            233
  Homework 4.4                                                    236
  Homework 4.4 Answers                                            237
  5. Multivariate Linear Inequalities                             238
  5.1. Solution Sets                                              238
  5.2. Solving Inequalities                                       238
  Homework 4.5                                                    244
  Homework 4.5 Answers                                            245
  Practice Test 4                                                 247
  Practice Test 4 Answers                                         248

Chapter 5. Polynomial Expressions                                 249
  1. Integer Exponent                                             249
  1.1. Definition                                                 249
  1.2. Products and Quotients of Exponents                        250
  1.3. Exponents of Sums                                          254
                                              8
CHAPTER 0. CONTENTS
  Homework 5.1                                  255
  Homework 5.1 Answers                          256
  2. Properties of Exponent                     257
  2.1. Exponent of Exponent                     257
  2.2. Distributivity                           258
  Homework 5.2                                  263
  Homework 5.2 Answers                          264
  3. Introduction to Polynomials                265
  3.1. Definitions                              265
  3.2. Combining Similar Monomial Terms         267
  3.3. Evaluating Polynomials in Applications   269
  Homework 5.3                                  271
  Homework 5.3 Answers                          273
  4. Sums of Polynomials                        274
  Homework 5.4                                  276
  Homework 5.4 Answers                          278
  5. Products of Polynomials                    279
  5.1. Monomial Times Monomial                  279
  5.2. Monomial Times Polynomial                279
  5.3. Polynomial Times Polynomial              280
  Homework 5.5                                  282
  Homework 5.5 Answers                          283
  6. Special Products                           284
  6.1. Difference of Squares                    284
  6.2. Square of a Binomial                     286
  Homework 5.6                                  288
  Homework 5.6 Answers                          289
  7. Quotients of Polynomials                   290
  7.1. Monomial Divisor                         290
  7.2. Polynomial Divisor                       292
  Homework 5.7                                  298
  Homework 5.7 Answers                          299
  8. Negative Exponent                          300
  8.1. Negative Integer Exponent                300
  8.2. Scientific Notation                      304
  Homework 5.8                                  307
  Homework 5.8 Answers                          309
  Practice Test 5                               310
  Practice Test 5 Answers                       311

Chapter 6. Factoring Polynomials                313
  1. Greatest Common Factor                     313
  1.1. Factoring Polynomials                    313
  1.2. Greatest Common Factor                   314
  1.3. Monomial GCF                             316
                                           9
                                             CHAPTER 0. CONTENTS
  Homework 6.1                                               319
  Homework 6.1 Answers                                       320
  2. Grouping                                                321
  2.1. Factoring by Grouping                                 321
  Homework 6.2                                               323
  Homework 6.2 Answers                                       324
  3. Factoring x 2 + bx + c                                  325
  3.1. Guessing the Coefficients                             325
  3.2. Variations of the Pattern                             327
  3.3. Basic Factoring Strategy                              328
  Homework 6.3                                               330
  Homework 6.3 Answers                                       331
  4. Factoring ax 2 + bx + c                                 332
  4.1. Guessing the Coefficients                             332
  4.2. Factoring Trinomials                                  333
  Homework 6.4                                               336
  Homework 6.4 Answers                                       337
  5. Factoring Special Products                              338
  5.1. Difference of Squares                                 338
  5.2. Square of a Binomial                                  340
  Homework 6.5                                               341
  Homework 6.5 Answers                                       342
  6. General Factoring Strategy                              343
  6.1. Irreducible Polynomials                               343
  6.2. Factoring Strategy                                    343
  Homework 6.6                                               346
  Homework 6.6 Answers                                       347
  7. Solving Equations by Factoring                          348
  7.1. Zero Product Property                                 348
  7.2. Equation Solving Strategy                             349
  Homework 6.7                                               352
  Homework 6.7 Answers                                       353
  8. Factoring Applications                                  354
  8.1. Applications to Areas                                 354
  Homework 6.8                                               358
  Homework 6.8 Answers                                       359
  Practice Test 6                                            360
  Practice Test 6 Answers                                    361

Chapter 7. Rational Expressions                              363
  1. Simplifying Rational Expressions                        363
  1.1. Definition                                            363
  1.2. Variable Restrictions                                 364
  1.3. Canceling Common Polynomial Factors                   366
  Homework 7.1                                               368
                                        10
CHAPTER 0. CONTENTS
  Homework 7.1 Answers                           370
  2. Products of Rational Expressions            371
  2.1. Multiplying Rational Expressions          371
  2.2. Dividing Rational Expressions             372
  Homework 7.2                                   375
  Homework 7.2 Answers                           376
  3. LCD for Rational Expressions                377
  3.1. LCM for Polynomials                       377
  3.2. LCD for Rational Expressions              378
  Homework 7.3                                   381
  Homework 7.3 Answers                           382
  4. Sums of Rational Expressions                383
  4.1. Sums with a Common Denominator            383
  4.2. General Sums                              384
  Homework 7.4                                   388
  Homework 7.4 Answers                           390
  5. Complex Fractions                           391
  5.1. Definition                                391
  5.2. Simplifying Complex Fractions             391
  Homework 7.5                                   396
  Homework 7.5 Answers                           398
  6. Equations with Rational Expressions         399
  6.1. Extraneous Solutions                      399
  Homework 7.6                                   404
  Homework 7.6 Answers                           406
  Practice Test 7                                407
  Practice Test 7 Answers                        408

Chapter 8. Radicals                              409
  1. Introduction to Radicals                    409
  1.1. Square Root and Radicals                  409
  1.2. Principal nth Root                        411
  Homework 8.1                                   414
  Homework 8.1 Answers                           415
  2. Products with Square Roots                  416
  2.1. Distributivity Over Multiplication        416
  Homework 8.2                                   420
  Homework 8.2 Answers                           421
  3. Quotients with Square Roots                 422
  3.1. Distributivity Over Division              422
  Homework 8.3                                   424
  Homework 8.3 Answers                           426
  4. Sums with Square Roots                      427
  4.1. Radical Like Terms                        427
  4.2. Rationalizing Sums                        429
                                            11
                                                              CHAPTER 0. CONTENTS
  Homework 8.4                                                                432
  Homework 8.4 Answers                                                        433
  5. Equations with Square Roots                                              434
  5.1. Squaring Both Sides                                                    434
  Homework 8.5                                                                439
  Homework 8.5 Answers                                                        440
  6. Applications to Right Triangles                                          441
  6.1. Distance Formula                                                       441
  Homework 8.6                                                                443
  Homework 8.6 Answers                                                        444
  7. Rational Exponent                                                        445
  7.1. Rational Exponent Definition                                           445
  Homework 8.7                                                                449
  Homework 8.7 Answers                                                        451
  Practice Test 8                                                             452
  Practice Test 8 Answers                                                     453

Chapter 9. Quadratic Equations                                                455
  1. Complex Numbers                                                          455
  1.1. Imaginary Unit                                                         455
  1.2. The Set of Complex Numbers                                             456
  1.3. Adding and Multiplying Complex Numbers                                 457
  Homework 9.1                                                                459
  Homework 9.1 Answers                                                        460
  2. Square Root Property                                                     461
  2.1. Square Root Property                                                   461
  2.2. Solving Equations (nx + k)2 = r                                        463
  2.3. Complex Solutions                                                      464
  Homework 9.2                                                                466
  Homework 9.2 Answers                                                        467
  3. Completing the Square                                                    468
  3.1. Completing the Square for x 2 + bx                                     468
  3.2. Solving Quadratic Equations by Completing the Square                   470
  Homework 9.3                                                                473
  Homework 9.3 Answers                                                        474
  4. Quadratic Formula                                                        475
  4.2. Complex Solutions                                                      477
  Homework 9.4                                                                478
  Homework 9.4 Answers                                                        479
  Practice Final                                                              480
  Practice Final Answers                                                      482

Appendix A. To Instructors                                                    483
  1. Lecture Notes                                                            483
  1.1. Concepts                                                               483
  1.2. Linear Equations                                                       485
                                          12
CHAPTER 0. CONTENTS
  1.3.   Graphing                               489
  1.4.   Systems                                490
  1.5.   Polynomials                            492
  1.6.   Factoring                              493
  1.7.   Rational Expressions                   494
  1.8.   Radicals                               495
  1.9.   Quadratics                             497
Appendix B. To Developers                       499
  1. Contributing                               499
  1.1. Submitting Contributions                 499
  1.2. Document Hierarchy                       499
  2. Document Formatting Guidelines             500
  2.1. Mathematical Notions                     500
  2.2. Sets of Equations and Expressions        501
  2.3. Frames                                   502
  2.4. Homework and Answers                     504
  Homework B.2                                  505
  Homework B.2 Answers                          506
  3. To Do                                      507
  3.1. To Do Items                              507
  4. Working with the Source                    507
  4.1. Getting the Source                       507
  4.2. Building the Source with LATEX           507
  4.3. Branches                                 508
  5. Acknowledgments                            508
Index                                           509




                                           13
                                          CHAPTER 1


                                          Concepts


                        1. Expressions, Relations, and Substitution
   1.1. Expressions versus Relations.

  DEFINITION 1.1.1. An algebraic expression or simply expression is a symbolic description
  of a single number. We call this number the value of the expression.


  DEFINITION 1.1.2. Expressions are equivalent if they describe the same number, even
  though they may look very different. For example, expressions 2 · 3 and 5 + 1 are equiv-
  alent, because they both evaluate to 6.


  BASIC EXAMPLE 1.1.1. Here are some expressions:

   expression    what it stands for
       17        stands for the number 17
     10 + 7      is a different expression which also stands for the number 17
   (3 + 5) ÷ 2   stands for the number 4
        x        is a variable expression, and it stands for an unknown number
     2x + y      stands for a single unknown number; if we knew what expressions x
                 and y stand for, we would be able to compute the value of 2x + y


  DEFINITION 1.1.3. An equation is a statement about the equality of two expressions:
                                              A= B
  An equation has an expression on the left, another expression on the right, and an equality
  sign in the middle. An equation is true if the value of the expression on the left is the same
  as the value of the expression on the right, and false otherwise.


It is important to distinguish between equivalent expressions and equivalent equations. When we
say that equations are equivalent, it means that they have identical solutions (that is, the same
numbers make them true when we substitute them for the variables), even though equations
may look very different. The formal definition of equivalent equations is given later in the text.
                                               15
1. EXPRESSIONS, RELATIONS, AND SUBSTITUTION                            CHAPTER 1. CONCEPTS

  BASIC EXAMPLE 1.1.2. Here are some equations:

    equation     truth value
       4=4       true
   17 = 10 + 7 true
     3=1+1       false
     x +1=7      true if the value of x is 6, false otherwise
       x =6      this one is equivalent to the one above: it’s also true if the value of x is
                 6 and false otherwise


It is crucial to distinguish between expressions and equations. While intimately related to each
other, they are completely different types of objects. For example, we can add the same number
to both sides of any equation and obtain an equivalent equation, which is as true or false as the
original. But if we add an arbitrary non-zero number to an expression, its value will change
and we will end up with a different, unrelated expression.

  DEFINITION 1.1.4. Informally speaking, every equation corresponds to a more general
  object we call a binary relation, or just relation, which is a true-or-false statement about
  two numbers. There are many different types of relations besides equations, and they all
  differ from expressions in one key way: expressions denote numbers, whereas relations
  denote truth or falsehood.


  BASIC EXAMPLE 1.1.3. Besides equations, inequalities are some of the more useful relations
  for us:

    relation   truth value
     4<4       false, because 4 is not less than 4
   17 > 5 · 2 true, because 17 is greater than 10
     x>y       true if the value of x is greater than the value of y, false otherwise




                                               16
CHAPTER 1. CONCEPTS                     1. EXPRESSIONS, RELATIONS, AND SUBSTITUTION
    1.2. Algebraic Substitution. Algebraic substitution is the number one tool in our toolbox.
It can be described as follows: any time and for any reason, we can replace any expression
anywhere by any other expression which has the same value.

  BASIC EXAMPLE 1.2.1. We know that expressions 6 · 2, 7 + 5, and 12 all stand for the same
  number, so we can always swap one for the other in any expression, even if it is inside an
  equation. All of the following are perfectly equivalent statements:
                                     12x     =    y + 12
                                  (6 · 2)x   =    y + 12
                                  (6 · 2)x   =    y + (7 + 5)
  As we substitute, it is never a bad idea to guard the value of the inserted expression with
  parentheses. With practice, we learn to omit the substitution parentheses in simple cases,
  like when we substitute a single positive integer, but more complicated expressions must
  be substituted with parentheses so that the correct order of operations is preserved.


  THEOREM 1.2.1 (Algebraic Substitution). If the values of expressions A and B are the same,
  then we can substitute A by (B) in any expression, and obtain an equivalent expression.


  EXAMPLE 1.2.1. Suppose that the value of x is −4. Substitute −4 for x in the expression
  (3x − 5) + x.


                                ANSWER: (3(−4) − 5) + (−4)


  EXAMPLE 1.2.2. Suppose that the expression A has the same value as the expression B +5.
  Substitute B + 5 for A in the equation
                                        13A = 10 − A


  SOLUTION: This example demonstrates why the parentheses are indispensable in some
  situations. Without the parentheses, we could make a mistake by writing something like
  13B + 5 = 10 − B + 5, which would give us wrong values on both sides.


                             ANSWER: 13(B + 5) = 10 − (B + 5)




                                             17
1. EXPRESSIONS, RELATIONS, AND SUBSTITUTION                           CHAPTER 1. CONCEPTS
   1.3. Common Types of Expressions. There are four basic types of expressions which are
used heavily in this text: sums, products, terms, and factors.

  DEFINITION 1.3.1. A sum is an algebraic expression consisting of expressions being added,
  and those expressions are called terms. Because subtraction can be thought of as adding
  a negative number, the terms of the sum can be separated by either pluses or minuses.


  BASIC EXAMPLE 1.3.1. Here is a sum with 3 terms:
                                           A+ B + C
  Here is a sum with 4 terms, and its 3rd term −4 is negative:
                                         a + x − 4 + 7y
  Here is a sum with 2 terms (everything inside the parentheses is a single term):
                                       (x + y − 3z) + 14
  Incidentally, the first term in the sum above is itself a sum with 3 terms:
                                           x + y − 3z


  EXAMPLE 1.3.1. Identify all the terms in the expression
                                           5 y − 3x 2


                                ANSWER: 2 terms: 5 y, −3x 2


  EXAMPLE 1.3.2. Identify all the terms in the expression
                                      3(x + 1) + 5 + x y


  SOLUTION: When identifying terms of an expression, we do not look inside the paren-
  theses, so 3(x + 1) is a single term, even though another sum with two terms is locked up
  inside.


                             ANSWER: 3 terms: 3(x + 1), 5, x y




                                              18
CHAPTER 1. CONCEPTS                    1. EXPRESSIONS, RELATIONS, AND SUBSTITUTION

 DEFINITION 1.3.2. A product is an algebraic expression consisting of expressions being
 multiplied, and those expressions are called factors.


 BASIC EXAMPLE 1.3.2. Here is a product with 2 factors:
                                              4x
 Here is a product with 3 factors, and its third factor is itself a sum with 2 terms:
                                          AB(X + 7)
 In general, sums and products can go into each other like the Russian dolls:
                             ABC − 4(A + B)(X − Y) + AB(X + 7)
 What we have above is a sum with 3 terms, and the second term −4(A + B)(X −Y ) happens
 to be a product with three factors, and its second factor A + B happens to be a sum with
 two terms.


                                                                2
 EXAMPLE 1.3.3. Identify all the factors in the expression:       xy
                                                                7


 SOLUTION: Factors are easier to see if we make the multiplication visible:
                                         2
                                           ·x·y
                                         7
 We consider the rational number 2/7 as a single factor.


                               ANSWER: 3 factors: 2/7, x, y


                                                                              1
 EXAMPLE 1.3.4. Identify all the factors in the expression:     −10x(x + 1)
                                                                              z


 SOLUTION: Unlike the addition operation, multiplication is often invisible, so it may be
 helpful to rewrite the expression with the multiplication dots shown:

                                                            1
                                    (−10) · x · (x + 1) ·
                                                            z


                                                                 1
                           ANSWER: 4 factors: −10, x, x + 1,
                                                                 z

                                              19
1. EXPRESSIONS, RELATIONS, AND SUBSTITUTION                             CHAPTER 1. CONCEPTS

  DEFINITION 1.3.3. A numerical coefficient or just coefficient is a multiplicative factor in a
  product. It usually looks like a numerical constant, but in complicated cases may be any
  expression. In the latter case, the variables appearing in the coefficient are often called
  parameters, and must be clearly distinguished from the other variables.

  Traditionally, the coefficient is written on the left side of the product.


  BASIC EXAMPLE 1.3.3.


                                      product coefficient
                                       3x y 2      3
                                        −6a       −6
                                        3 3           3
                                          x
                                        4             4
                                        a4 b          1
                                        −x 2         −1




                                                20
CHAPTER 1. CONCEPTS                      1. EXPRESSIONS, RELATIONS, AND SUBSTITUTION
     Homework 1.1.

Determine the type and the meaning of the            14. Substitute y − 2 for h in the equation
following formal statements. For equations                            2h
and inequalities, determine whether they                                 = 5h
                                                                      3
are true or false. For expressions, deter-
mine their numerical value.

1. 7 = 6 − 1                                         Identify all the terms in the given expres-
                                                     sion.
     12 6
2.      =                                            15. 5x − 6
      4   2
3. (14 + 2) − 9                                      16. 1 + x + x y

4. (5 · 3) + 4                                       17. −(x + 7) − 1 + x

5. 14 < −1                                           18. (5x − 1) − (4 + x)

6. 5 · 3 = 9 + 6                                     19. 14x y 2
     3+5                                             20. −(3 + 2x) − A
7.
      4
                                                     21. (1 + A) − (1 − 2A) + (4A + 3)
8. 5 + 6 > 10
                                                                                      1
                                                     22. 3(a + x) − 5(x − 3)(2 y) +
                                                                                      7
Substitute the given expression for the vari-
able. Use the parentheses and do not sim-            Identify all the factors in the given expres-
plify the result.                                    sion.
9. Substitute 100 for x in the expression            23. −6x y 5
                    2x + 1
                                                     24. 14x 2 y 2 A6
10. Substitute 4 for x in the expression
                                                     25. 2x(u + 1)(u2 − 1)
                    5 − 4x
                                                     26. (A + B)2 (A − B)
11. Substitute −7 for y in the expression
                    y + y2                           27. a2 − b
                                                           1         1
12. Substitute −12 for y in the expression           28.     (a + 1)
                                                           4         x
                   y 2 (x + y)
                                                           1
                                                     29.     (ab)c
13. Substitute x + 1 for k in the equation                 2

                 k(k − 1) = 4k                       30. 11(x y 2 )(x + y)
                                                21
 1. EXPRESSIONS, RELATIONS, AND SUBSTITUTION                   CHAPTER 1. CONCEPTS
    Homework 1.1 Answers.

1. A false equation                        17. 3 terms: −(x + 7) , −1, x

3. An expression with the value 7          19. 1 term: 14x y 2

5. A false inequality
                                           21. 3 terms: 1 + A, −(1 − 2A), 4A + 3
7. An expression with the value 2
                                           23. 3 factors: −6, x, y 5
9. 2(100) + 1
                                           25. 4 factors: 2, x, u + 1, (u2 − 1)
11. (−7) + (−7)2
                                           27. 1 factor: a2 − b
13. (x + 1)((x + 1) − 1) = 4(x + 1)
                                                            1
15. 2 terms: 5x, −6                        29. 3 factors:     , ab, c
                                                            2




                                      22
CHAPTER 1. CONCEPTS                                                 2. REAL NUMBER AXIOMS
                                   2. Real Number Axioms


    2.1. Axioms. Axioms are the properties of numbers which are accepted without proof. In
a sense, the axioms serve as a kind of definition for numbers, but unlike traditional definitions,
which define new concepts in terms of old concepts, the axioms define the numbers and the
arithmetic operations by telling us what they can and cannot do. The true nature of a number
is never explained explicitly. We simply agree to call things numbers as long as they obey the
axioms.

The first nine axioms appear almost too simple, and indeed they are. Almost anyone familiar
with basic arithmetic knows these facts by heart.

  AXIOM 2.1.1 (Closure of Arithmetic Operations). Adding any two real numbers results in
  a real number. Multiplying any two real numbers results in a real number.


  BASIC EXAMPLE 2.1.1. Closure guarantees that addition and multiplication are defined for
  every pair of real numbers, and the result is a real number. When we substitute 5 for 2+3,
  or 6 for 2 · 3, we can say that the substitution is justified by the closure.


  AXIOM 2.1.2 (Commutativity of Addition). For any two real numbers X and Y ,
                                         X +Y =Y +X


  AXIOM 2.1.3 (Associativity of Addition). For any two real numbers X and Y ,
                                   X + (Y + Z) = (X + Y ) + Z


  BASIC EXAMPLE 2.1.2. When considered together, commutativity and associativity of ad-
  dition tell us that we can rearrange the terms of any sum, and perform the additions in
  any order we consider convenient. Because of that, when we work with expressions like
                                          a+b+c+d
  we are free to substitute them with equivalent expressions such as
                                          d+c+b+a
  or
                                        (a + c) + (b + d)


  AXIOM 2.1.4 (Commutativity of Multiplication). For any two real numbers X and Y ,
                                            XY = YX

                                               23
2. REAL NUMBER AXIOMS                                                   CHAPTER 1. CONCEPTS

  AXIOM 2.1.5 (Associativity of Multiplication). For any two real numbers X and Y ,
                                        X (Y Z) = (X Y )Z


  BASIC EXAMPLE 2.1.3. Just like with addition, commutativity and associativity of multi-
  plication allow us to compute values of products by multiplying factors in any order, so a
  product with 4 factors such as
                                       3 · x · y · (a − 1)
  is equivalent to any other product involving the same factors, regardless of the order of
  operations, for example
                                      (a − 1) · (3 · y · x)
  or
                                    (3 · y) · (x · (a − 1))


  AXIOM 2.1.6 (Additive Identity). There exists a real number 0, called zero, such that for
  all real numbers X ,
                                        X +0= X


  AXIOM 2.1.7 (Multiplicative Identity). There exists a real number 1, called one or unit,
  such that for all real numbers X ,
                                         X ·1= X


While it may seem silly that we have to explicitly demand the existence of numbers such as
zero and one, this is the natural way to proceed from the axioms. Note that other axioms apply
to all numbers, but do not require the existence of any specific number. But once we have the
identities and the closure of operations, we can immediately prove that there exists a number
equal to 1 + 1, and call it 2, and then also a number 2 + 1, which we call 3, and so on.

  AXIOM 2.1.8 (Additive Inverse). For each real number X , there exists a real number we call
  the additive inverse of X , or the opposite of X , written as −X , with the following property:
                                          X + (−X ) = 0


  BASIC EXAMPLE 2.1.4. The opposite of 5 is −5 because 5 + (−5) = 0.

  At the same time, the opposite of −5 is 5 because (−5) + 5 = 0, so we can say 5 and −5
  are opposites of each other. In general, it can be proven that every number has a unique
  opposite.

  The opposite of zero is zero itself, because 0 + 0 = 0.

                                                24
CHAPTER 1. CONCEPTS                                                    2. REAL NUMBER AXIOMS

  AXIOM 2.1.9 (Multiplicative Inverse). For each non-zero real number X , there exists a real
                                                                                          1
  number we call the multiplicative inverse of X , or the reciprocal of X , written as      , 1/X ,
                                                                                          X
  or X −1 , with the following property:
                                                1
                                                ‹
                                            X·     =1
                                                X


Note that zero is very special. It is the only real number which is its own opposite, and the
only real number with no reciprocal: there is no real number that would give us one when
multiplied by zero.

  BASIC EXAMPLE 2.1.5. The reciprocal of 2 is 1/2, which can also be written as 0.5. To
  check, simply multiply them, and make sure to get 1 as the result. By the same token, the
  reciprocal of 1/2 is 2.

  In practice, obtaining the reciprocal amounts to writing the number as a fraction, and
  turning that fraction “upside-down”. Recall that we can multiply fractions by multiplying
  numerators and multiplying denominators:
                                           A X     AX
                                             · =
                                           B Y     BY
  Recall also that we can write numbers like 2 as 2/1. It should be easy to see now that the
  reciprocal of any fraction A/B is the fraction B/A, as long as neither A nor B is zero. Here
  are some more reciprocal pairs:

                                                     2    1   29
                                  X   1 −1 4           −    −
                                                     3   17   31


                                  1             1    3           31
                                      1 −1               −17 −
                                  X             4    2           29


The last axiom deserves special attention. It is far less intuitive, and it takes a lot of practice to
apply it correctly, or even to notice that it can be applied.

  AXIOM 2.1.10 (Distributivity). For any three real numbers X , Y , and Z,
                                       X (Y + X ) = X Y + X Z




                                                    25
2. REAL NUMBER AXIOMS                                                       CHAPTER 1. CONCEPTS

 EXAMPLE 2.1.1. Use the distributivity to rewrite the expression without parentheses:
                                           4(x + 7)


 SOLUTION:
                                 4(x + 7)       =      4x + 4 · 7
                                                =      4x + 28


                                    ANSWER: 4x + 28


 EXAMPLE 2.1.2. Use the distributivity to rewrite the expression without parentheses:
                                      2(x + 2 y + 3z)


 SOLUTION: The distributive property can be shown to work for sums with more than two
 terms, so that each term gets multiplied by the number from the outside of the parentheses.
                        2(x + 2 y + 3z)     =        2 · x + 2 · 2 y + 2 · 3z
                                            =        2x + 4 y + 6z


                                  ANSWER: 2x + 4 y + 6z


 EXAMPLE 2.1.3. Use the distributivity to rewrite the expression as a product with two
 factors:
                                         4x + 4


 SOLUTION: The greatest common factor for these two terms is 4, so a single application
 of the distributive property gives a product of two factors: 4 and x + 1.
               4x + 4    =    4x + 4 · 1                     multiplicative identity
                         =    4(x + 1)                                  distributivity


                                    ANSWER: 4(x + 1)




                                                26
CHAPTER 1. CONCEPTS                                                  2. REAL NUMBER AXIOMS

  EXAMPLE 2.1.4. Use the distributivity to rewrite the expression as a product with two
  factors:
                                         ab + 3b


  SOLUTION: The greatest common factor for these two terms is b, so a single application
  of the distributive property gives a product of two factors: a + 3 and b.
                                        ab + 3b = (a + 3)b


                                       ANSWER: (a + 3)b



    2.2. Applying Axioms. Unlike the easy axioms, the distributivity of multiplication over
addition will play a major role in our study, and we will refer to it explicitly throughout the
text in order to justify algebraic substitutions. The rest of the axioms will still be in heavy use,
but they will be applied under the hood, so to speak.

Before we leave this section, let us have some fun and see how the familiar algebraic concepts
can be broken down into steps so tiny, that each step is justified by an axiom.

  EXAMPLE 2.2.1. Simplify the expression by appealing to the axioms:
                                          (x + 5) + (−x)


  SOLUTION:


   equivalent expressions axiom used for substitution
       (x + 5) + (−x)
       (5 + x) + (−x)     By commutativity of addition, x + 5 = 5 + x, so we were able
                          to substitute (5 + x) for (x + 5) without a change in value.
       5 + (x + (−x))     The whole expression was substituted by associativity of addi-
                          tion.
            5+0           x and −x are opposites and they must add up to zero, so we
                          were able to substitute zero for x + (−x).
              5           zero is the additive identity, so we were able to substitute 5 for
                          5 + 0.


                                           ANSWER: 5

                                                27
2. REAL NUMBER AXIOMS                                               CHAPTER 1. CONCEPTS

 EXAMPLE 2.2.2. Rewrite the expression without parentheses and then simplify by appeal-
 ing to the axioms:
                                    3(2 + x) + (−6)


 SOLUTION: This is just like the previous example, but we will try to justify the steps in a
 concise manner, rather than describing them in with sentences. Since we are going strictly
 one substitution at a time, the substitutions we make should be obvious.

                  equivalent expressions     axiom used for substitution
                      3(2 + x) + (−6)
                    (3 · 2 + 3x) + (−6)      distributivity
                      (6 + 3x) + (−6)        closure
                      (3x + 6) + (−6)        commutativity of +
                      3x + (6 + (−6))        associativity of +
                          3x + (0)           additive inverse
                             3x              additive identity


                                       ANSWER: 3x


  2.3. Inverses, Subtraction, Division.

 THEOREM 2.3.1. Each number has a unique opposite, and zero is the only number which
 is its own opposite. An opposite of an opposite of a number is the number itself. Formally,
 for any real number x,
                                         −(−x) = x




  PROOF. We appeal directly to the axioms:

            −(−x)     =    −(−x) + 0                               identity of +
                      =    −(−x) + (−x + x)                    additive inverse
                      =    (−(−x) + (−x)) + x                 associativity of +
                      =    0+ x                                additive inverse
                      =    x                                       identity of +

                                                                                               


                                             28
CHAPTER 1. CONCEPTS                                                2. REAL NUMBER AXIOMS

  THEOREM 2.3.2. Each non-zero number has a unique reciprocal. Only 1 and −1 are equal
  to their respective reciprocals:
                                       1               1
                                    1=           −1=
                                       1              −1
  A reciprocal of a reciprocal of a number is the number itself. Formally, for any non-zero
  real number x,
                                            1
                                                =x
                                         (1/x)



   PROOF. The proof is very similar to the previous one:
             1/(1/x)    =    1/(1/x) · 1                               identity of ·
                        =    1/(1/x) · (1/x · x)            multiplicative inverse
                        =    (1/(1/x) · 1/x) · x                  associativity of ·
                        =    1· x                           multiplicative inverse
                        =    x                                         identity of ·
                                                                                              

Axioms do not mention subtraction or division, but we can define them in terms of addition,
multiplication, and inverses.

  DEFINITION 2.3.1. Subtraction is an operation defined in terms of the addition and the
  opposite. For any two real numbers X and Y ,
                                      X − Y = X + (−Y )
  In other words, subtracting a number amounts to adding its opposite.


  DEFINITION 2.3.2. Division is an operation defined in terms of the multiplication and the
  reciprocal. For any two real numbers X and Y ,
                                                   1
                                       X ÷Y =X ·
                                                   Y
  In other words, dividing by a number amounts to multiplying by its reciprocal.
                                                   X
  Other notations commonly used for X ÷ Y are        and X /Y .
                                                   Y


Now that we finally have all of the arithmetic operations, let us go back for a moment to the
definitions of sum, product, term, and factor. Since subtraction is essentially addition, we are
fully justified in calling the following expression a sum with 3 terms:
                                           4x − 5 − Y
                                              29
2. REAL NUMBER AXIOMS                                                    CHAPTER 1. CONCEPTS
because it is equivalent to a sum of 4x, the opposite of 5, and the opposite of Y
                                        4x + (−5) + (−Y )
The same kind of reasoning applies to the division and the products. We can get away with
calling this a product with 5 factors:
                                                 1    1
                                       x · y ·z· ·
                                                 a b+c
even though we usually write down an equivalent fraction:
                                               x yz
                                             a(b + c)

                                                                                      x+y
  EXAMPLE 2.3.1 (Fraction Notation). Identify all the factors in the expression
                                                                                      x−y


  SOLUTION: The fraction notation is a division with a gotcha: unlike the regular division ÷
  which behaves similarly to the multiplication · in the order of operations, the fraction bar
  waits for the values of its numerator and denominator, making it the very last operation to
  be performed within the fraction. One way to make sense of this is by enclosing numerator
  and denominator in parentheses, so that the order of operations becomes unambiguous:
                                       x+y        (x + y)
                                               =
                                       x−y        (x − y)
  It may be helpful to remember that every fraction comes equipped with these parentheses,
  but most of the time they are invisible, so to speak. Now we can see that this expression,
  when thought of as a product, has 2 factors: (x + y) and 1/(x − y), which is the reciprocal
  of (x − y).


                                                                 1
                              ANSWER: 2 factors: (x + y),
                                                              (x − y)



    2.4. Conclusion. Two concepts stand out in this section, and will be alluded to throughout
the text. The distributive property is both indispensable and hard to grasp, and it is the only
axiom we will keep calling by name. The way we defined subtraction and especially division
will help us not to get lost while working with fractions and other complicated expressions.

The rest of the axioms are about to fade into shadows, but there is one last thing we want
to stress, as it is the kind of insight that is simple yet profound at the same time. Not only
the axioms can be used to break up everything we do into tiny, elementary, and fully justified
steps; we also must be able to justify everything we do with axioms alone. Every single algebraic
argument in this text is justifiable from the axioms and the logical principles, and nothing else
would be recognized as valid. The rigorous study of the properties of numbers which arise
directly from the axioms is called abstract algebra, and it is a fascinating, albeit a very difficult
                                                 30
CHAPTER 1. CONCEPTS                                                 2. REAL NUMBER AXIOMS
subject, consisting almost entirely of formal proofs, the simplest ones being similar to the step-
by-step simplification examples above.




                                               31
2. REAL NUMBER AXIOMS                                                   CHAPTER 1. CONCEPTS
   Homework 1.2.

Use the distributivity to rewrite the expres-        13. 6x + 12 + 18 y
sion without parentheses:
                                                     14. 15a + 45b + 30
1. 3(x + 16)
                                                     15. 28p + 7t + 7
2. 4(x + 6)
                                                     16. 24b + b + ab
3. 5(2 + 2a)

4. 8(1 + 2 y)
                                                     Simplify the expression by appealing to the
5. a(10 + x)                                         axioms:
6. b(x + y)                                          17. 2(x + 5) + 3
7. 5(6x + 9 + 4g)                                    18. 5 + 3(4 + a)
8. 6(2t + 3 + 4s)                                    19. (6 + x) + (−6)

                                                     20. g + (2 + (−g))
Use the distributivity to rewrite the expres-
                                                                1
                                                                  ‹
sion as a product with two factors:
                                                     21. x 7 +       + 8, given that x 6= 0
                                                                 x
9. 3a + 3b
                                                                      1
                                                                       ‹
                                                     22. 12 + a 4 +       , given that a 6= 0
10. 6 y + 6z                                                          a

11. 33x + 11                                         23. 5x + (6 + 3x)

12. 21a + 7                                          24. (2a + 9) + 8a




                                                32
CHAPTER 1. CONCEPTS                                                  2. REAL NUMBER AXIOMS
      Homework 1.2 Answers.

1. 3x + 48                                            19.    expression      axiom
                                                            (6 + x) + (−6)
3. 10 + 10a                                                 (x + 6) + (−6)   commutativity of +
                                                            x + (6 + (−6))   associativity of +
5. 10a + a x
                                                                x + (0)      additive inverse
                                                                   x         additive identity
7. 30x + 45 + 20g

                                                      21.       expression       axiom
9. 3(a + b)                                                      
                                                                      1
                                                                        ‹
                                                               x 7+       +8
                                                                      x ‹
11. 11(3x + 1)                                                          1
                                                            
                                                              x ·7+ x ·     +8   distributivity
                                                                        x
13. 6(x + 2 + 3 y)                                             (7x + (1)) + 8    multiplicative inverse
                                                                7x + (1 + 8)     associativity of +
15. 7(4p + t + 1)                                                  7x + 9        closure

17.      expression         axiom                     23.    expression      axiom
           2(x + 5)                                         5x + (6 + 3x)
      (2 · x + 2 · 5) + 3   distributivity                  5x + (3x + 6)    commutativity of +
       (2x + 10) + 3        closure                         (5x + 3x) + 6)   associativity of +
       2x + (10 + 3)        associativity of +               (5 + 3)x + 6    distributivity
           2x + 13          closure                             8x + 6       closure




                                                 33
3. THE NUMBER LINE AND SETS OF NUMBERS                                    CHAPTER 1. CONCEPTS
                          3. The Number Line and Sets of Numbers
   3.1. Roster.

  DEFINITION 3.1.1. A set of numbers is any collection of numbers. Any number that belongs
  to the set is called an element or a member of that set.


  DEFINITION 3.1.2 (Roster Notation). A set can be written down in the roster notation,
  which is a comma-separated list of its elements inside the curly brackets. A set with
  elements 1, 2, and 3 can be written as {1, 2, 3}. The order in which elements are listed is
  not important, so {1, 2, 3} is the same set as {3, 1, 2}. A particular number either belongs
  to a set or it does not, just like a particular cat can be either alive or dead, so writing {7, 7}
  is just as redundant as saying “this cat is alive and alive”.


  BASIC EXAMPLE 3.1.1. Very large and infinite sets can be stated in roster notation using
  the ellipsis. . . For example, the infinite set of positive odd numbers can be stated as
                                    {1, 3, 5, 7, 9, 11, 13, . . .}
  It is important to list enough elements for the pattern to become clear to the reader, and
  so the roster notation is too ambiguous for many interesting sets, like this one:
                                      {6, 28, 496, 8128, . . .}
  What is the next number in this sequence? If you cannot tell, then perhaps the roster
  notation is not the best way to describe this set.


  DEFINITION 3.1.3. A set with zero elements exists, and it is called the empty set. The
  traditional notation for the empty set is ∅, which is why it is probably not a good idea to
  strike through your zeroes: ;. Roster notation {} can also be used.

   3.2. Integers.

  DEFINITION 3.2.1. The set of positive integers is
                                        {1, 2, 3, 4, 5, . . .}


  DEFINITION 3.2.2. The set of non-negative integers is
                                      {0, 1, 2, 3, 4, 5, . . .}
  which is exactly the same as the set of positive integers with one additional element: zero,
  which is neither positive nor negative. The standard notation for this set is N, although
  some texts will use the same notation for positive integers.


Many texts use “whole numbers” and “natural numbers” to refer to either positive or non-
negative integers, but we will not use either term in order to avoid confusion.
                                                 34
CHAPTER 1. CONCEPTS                               3. THE NUMBER LINE AND SETS OF NUMBERS

  DEFINITION 3.2.3. The set of integers is
                             {0, 1, −1, 2, −2, 3, −3, 4, −4, 5, −5, . . .}
  which can also be written as
                           {. . . , −5, −4, −3, −2, −1, 0, 1, 2, 3, 4, 5, . . .}


One great way to visualize the integers is the number line, which is a graphical representation
of the collection of real numbers:

     -10 -9 -8 -7 -6 -5 -4 -3 -2 -1                 0    1     2    3    4    5    6   7   8   9 10

The number line extends infinitely far to the right and to the left, but we can only draw a finite
segment of it. Every point on the number line corresponds to a real number, and the picture
above highlights the integers.
The number line provides a convenient way to explain the addition of integers. Because addi-
tion is associative and commutative, the order in which numbers are added does not affect the
result. To compute a sum, we start at zero on the number line. For each positive term of the
sum we go that many units to the right on the number line, and for each negative term of the
sum we go that many units to the left. The point where we end up is the value of the sum.

  EXAMPLE 3.2.1. Find 5 + (−7)


  SOLUTION: 5 units to the right from zero take us to 5, and 7 units left take us to −2, so
  the value is −2.


                                            ANSWER: −2


  EXAMPLE 3.2.2. Find −10 − (−7)


  SOLUTION: By definition of subtraction, this expression is equivalent to
                                            −10 + (−(−7))
  But the opposite of an opposite is the number itself: −(−7) = 7, so this is equivalent to
                                                −10 + 7
  Starting at zero, we move 10 units to the left to reach −10, and then 7 units to the right,
  and end up at −3.


                                            ANSWER: −3

                                                   35
3. THE NUMBER LINE AND SETS OF NUMBERS                                     CHAPTER 1. CONCEPTS

 EXAMPLE 3.2.3. Find −3 + 2 − 4 − 5


 SOLUTION: By definition of subtraction, this expression is equivalent to
                                    (−3) + 2 + (−4) + (−5)
 Starting from 0 on the number line,

        •   3 units to the left take us to −3
        •   2 units to the right take us to −1
        •   4 units to the left take us to −5
        •   5 units to the left take us to −10


                                          ANSWER: −10

  3.3. Set Builder.

 DEFINITION 3.3.1 (Set-builder Notation). A set can be defined by the set-builder notation,
 which looks like this:              
                                       expression relation
 Recall that relation is basically a statement which can be true or false. Defined as above,
 the set will contain values of all expressions for which the relation is true.


 EXAMPLE 3.3.1. Write the set of even integers in set-builder notation.


 SOLUTION: In roster notation, the set of even integers looks like this:
                                  {. . . , −4, −2, 0, 2, 4, 6, 8, . . .}
 In set-builder, we can write             
                                           x x is even
 which reads “the set of numbers x such that x is even”. This may look like an arcane way
 to rewrite a simple English statement, but the real usefulness of set-builder is made clear
 when we start using nontrivial expressions. For example, and with a lot more rigor, we
 can express the same set as           
                                         2k k is an integer
 This reads “the set of all values of the expression 2k, where k is an integer”. So this set
 has 2, since 2 = 2 · 1; it has 4 since 4 = 2 · 2, and so on. Defined this way, the set has every
 even integer, and none of the odd ones.

                                              
                                ANSWER:           2k k is an integer


                                                    36
CHAPTER 1. CONCEPTS                          3. THE NUMBER LINE AND SETS OF NUMBERS
                                              
  EXAMPLE 3.3.2. List a few members of the set 3x x is a negative integer


  SOLUTION: Plug in a few negative integers into the expression to the left of the bar | to
  get elements such as 3(−1) = −3, 3(−2) = −6 and so on. State the answer in the roster
  notation.


                            ANSWER: {−3, −6, −9, −12, −15, . . .}


                                                           1
                                                      §                                ª
  EXAMPLE 3.3.3. List a few members of the set               k is a positive integer
                                                           k


  SOLUTION: Plug in a few positive integers into the expression to the left of the | to get
  elements such as 1/1 = 1, 1/2, 1/3 and so on. State the answer in the roster notation.


                                           1         1       1      1
                                 §                                              ª
                         ANSWER: 1,          ,         ,       ,      ,   ...
                                           2         3       4      5

   3.4. Rationals.

  DEFINITION 3.4.1. The set of rational numbers is the collection of all numbers which can
  be represented by a fraction a/b where a is an integer and b is a positive integer:
                                     1     −1      17
                                   §                          ª
                                        ,      ,        , ...
                                     2      3      13
  Entries above are listed in no particular order. It is possible, but not easy to come up with
  a pattern that would allow to list all rational numbers. The set-builder way of defining
  rationals completely removes the ambiguity:
                        na                                               o
                             a is an integer and b is a positive integer
                         b
  The standard notation for the set of all rational numbers is Q.


A number is rational as long as there is some way to write it as a fraction of integers. So zero
is a rational number because 0 = 0/1, and 0.5 is also a rational number because 0.5 = 1/2.
Every rational number corresponds to a point on the number line. For example, 1/2 is right in
between 0 and 1, and 1/3 is one third of the way from 0 to 1:

                  0                  1           1              2                   1
                                     3           2              3
                                                 37
3. THE NUMBER LINE AND SETS OF NUMBERS                                   CHAPTER 1. CONCEPTS
Rational numbers are what topologists call dense on the real line, meaning that infinitely many
can be found in every little interval. However, not all points on the number line correspond to
rational numbers.
   3.5. Reals.

  DEFINITION 3.5.1. The set of real numbers is the collection of all numbers with signs
  (positive or negative) and decimal representations. A decimal representation is a finite
  list of digits from 0 to 9 (the integer part), then a decimal point, then an infinite list of
  digits (the fractional part). It is traditional to omit an infinite tail of zeroes, and to avoid
  an infinite tail of nines:
                                         2.100000 . . . = 2.1
                                         17.99999 . . . = 18
  The standard notation for the set of all real numbers is R.


  THEOREM 3.5.1. Each real number corresponds to a unique point on the real number line,
  and each point on the real number line corresponds to a unique real number.



All rational numbers are real. 1/2 is a real number because it has a decimal representation
0.5, and 1/3 is a real number because it has a decimal representation 0.333333 . . . = 0.3.
A decimal representation of any integer fraction can be produced with long division. Some
divisions (like 1/3) seem to go on forever, but it can be shown that all rational numbers end
in simple repeating patterns.

Rewriting decimal representations as fractions of integers is straightforward when possible.

  EXAMPLE 3.5.1. Write the number 0.125 as a fraction of integers.


  SOLUTION: The last digit 5 is in the third position after the decimal point, which corre-
  sponds to thousandths, so we can represent this number as a fraction with denominator
  1000.


                                                      125
                                         ANSWER:
                                                     1000


  EXAMPLE 3.5.2. Write the number 1075.56 as a fraction of integers.


  SOLUTION: This number ends with hundredths, so it is equivalent to

                                                38
CHAPTER 1. CONCEPTS                           3. THE NUMBER LINE AND SETS OF NUMBERS

                                                    107556
                                        ANSWER:
                                                      100

                                                                               p
Probably the oldest known example of a real number that is not rational isp 2, which is the
length of the diagonal of a square with side of length 1. It can be written as 2p= 1.41421 . . .,
but unlike with rational numbers, the digits of the decimal representation of 2 do not have
a simple repeating pattern.

                                              p
                                               2
                                          1


                                                   1

Another famous example of a real number that is not rational is the ratio of a circle’s circum-
ference to its diameter, π = 3.14159 . . ., which is exactly the circumference (the length of the
border) of a circle with diameter 1.

                                                       π
                                               1




Real
p numbers are drawn on the number line in according to their decimal representations, so
 2 is between 1 and 2 while π is shortly after 3:
                                        p
                                         2                       π


                        0           1          2             3         4

  DEFINITION 3.5.2. Real numbers which are not rational are also known as irrational num-
  bers.


  EXAMPLE 3.5.3. Identify all the integers in the set
                                             1     9
                                §                                 ª
                                 −4, 0,        ,      ,    27.7
                                             2     3




                                               39
3. THE NUMBER LINE AND SETS OF NUMBERS                                CHAPTER 1. CONCEPTS

  SOLUTION:

         •   −4 is an integer                                                                Ø
         •   0 is an integer                                                                 Ø
         •   1/2 is rational, but not an integer
         •   9/3 is equal to 3, so it is an integer                                          Ø
         •   27.7 is not an integer

  We will state the answer as a subset consisting of integers, and note that we can put 3
  instead of 9/3 because sets contain numbers, not the expressions which happen to define
  the numbers.


                                       ANSWER: {−4, 0, 3}


  EXAMPLE 3.5.4. Identify all the non-negative integers in the set
                                   5   −10        0
                    §                                              ª
                      5, 5.5,        ,       ,      , −12, −0.17
                                   5     2       −6


  SOLUTION:

         •   5 is a non-negative integer                                                     Ø
         •   5.5 is not an integer
         •   5/5 is equal to 1, which is a non-negative integer                              Ø
         •   −10/2 is equal to −5, which is not non-negative
         •   0/(−6) is equal to 0, which is a non-negative integer                           Ø
         •   −12 is not non-negative
         •   −0.17 is neither non-negative nor an integer


                                        ANSWER: {5, 1, 0}

   3.6. Absolute Value.

  DEFINITION 3.6.1. The absolute value of a real number x, written |x|, is the distance from
  the number to zero on the real number line. Alternatively, the absolute value of a non-
  negative number is the number itself, and the absolute value of a negative number is its
  opposite.


Absolute value bars alter the order of operations like parentheses, but the distributive property
does not work on them. In general, we must know the value of the expression (that is, reduce
it down to a single number) before removing the bars and the negative sign. It is a grave but
                                                 40
CHAPTER 1. CONCEPTS                             3. THE NUMBER LINE AND SETS OF NUMBERS
common mistake to change pluses to minuses inside the absolute value bars before the value
of the expression is known.

  BASIC EXAMPLE 3.6.1.

  | − 17| = 17

  −|17| = −17

  |4 − 13| = | − 9| = 9

  |2(−5)(−3)| = | − 30| = 30

  2 − | − 5 − 3| = 2 − | − 8| = 2 − 8 = −6


  EXAMPLE 3.6.1. Find the value of the expression if x = 5
                                             |x − 7| + 2x


  SOLUTION:
          |x − 7| + 2x    =    |(5) − 7| + 2(5)
                          =    | − 2| + 10                  replaced 5 − 7 by its value −2
                          =    2 + 10                                   because | − 2| = 2
                          =    12


                                         ANSWER: 12




                                                  41
3. THE NUMBER LINE AND SETS OF NUMBERS                                CHAPTER 1. CONCEPTS
    3.7. Inequality Relations. The real number line provides an ordering of the set of real
numbers. If the number line is oriented so that 1 is to the right of 0, then given any two
distinct real numbers, the one on the right is greater than the one on the left.

  DEFINITION 3.7.1. We will use five inequality relations in this text:


               a < b a is (strictly) less than b              a           b


               a > b a is (strictly) greater than b           b           a


               a ≤ b a is less than or equal to b
               a ≥ b a is greater than or equal to b
               a 6= b a is not equal to b

  < and > are called strict inequalities, while ≤ and ≥ are called non-strict inequalities.

  Other ways to read a ≤ b are “a is at most b” and “b is at least a”.


  EXAMPLE 3.7.1. Determine whether the inequality is true or false:
                                            17 ≥ 17


  SOLUTION: 17 = 17, so the non-strict inequality holds.


                                        ANSWER: true


  EXAMPLE 3.7.2. Determine whether the inequality is true or false:
                                           −6 < −10


  SOLUTION: −6 is to the right of −10 on the real number line, so it is greater than −10,
  and the inequality is false.


                                        ANSWER: false


  EXAMPLE 3.7.3. Determine whether the inequality is true or false:
                                        3.1315 ≥ 3.14

                                              42
CHAPTER 1. CONCEPTS                       3. THE NUMBER LINE AND SETS OF NUMBERS

 SOLUTION: The number 3.1315 is just a tad to the right of 3.1400 on the number line, so
 it is greater, and the inequality holds.


                                     ANSWER: true


 EXAMPLE 3.7.4. Determine whether the inequality is true or false:
                                       2 1
                                         <
                                       6 3


 SOLUTION: To compare these fractions, we need to rewrite them with the same denomi-
 nator somehow. If we multiply 1/3 by 2/2, for example, its value will not change, but the
 denominator will:
                                    1 1 2 1·2 2
                                     = · =         =
                                    3 3 2 3·2 6
 So the inequality is equivalent to
                                         2 2
                                           <
                                         6 6
 which is false.


                                     ANSWER: false




                                           43
3. THE NUMBER LINE AND SETS OF NUMBERS                           CHAPTER 1. CONCEPTS
   Homework 1.3.

Find the value of the expression:           if z = −16

1. 4 − 9                                    20. Find the value of the expression
                                                               −k + 4 − 10
2. 17 − 24
                                            if k = −4
3. −2 − 6

4. −14 − 18                                 List at least five elements of the given set
                                            using the roster notation:
5. 5 − 7 − 13
                                                 
                                            21. 2k + 100 k is a non-negative integer
6. 6 + 8 − 33
                                                  
7. 7 − 7                                    22.    3(m − 1) m is a negative integer

                                                      x +1
                                                  §                               ª
8. (−7) − (−5)                              23.            x is a positive integer
                                                        x
9. (−8) − (−5) − 2                                
                                            24.       x 2 x is a non-negative integer
10. (−2) + (−5) + 5

11. 4 − 9 − (−4)
                                            25. Identify all the negative integers in the
                                            set
12. (−12) − (−12)
                                                                        −15
                                                §                                  ª
                                                 −7, 4, −7.5, −1, 0,         , 100
13. 8 + (−6) + 2                                                          3

14. 10 + (−12) − 3                          26. Identify all the non-negative integers
                                            in the set
15. −2 + 20 − 6 − 17                                          16         1
                                                     §                      ª
                                                       −2, 0,    , −100,
                                                               4         17
16. 4 + (−3) + (−2) + 1
                                            27. Identify all the integers in the set
17. Find the value of the expression               §
                                                          4           −12
                                                                               ª
                                                     12, , −10.1,          , π
                 −12 + x + 3                              3             2
if x = −9                                   28. Identify all the positive integers in the
                                            set
18. Find the value of the expression              §
                                                    4         60 −10 p 0
                                                                                ª
                                                      , −4,      ,     , 2,
                −10 + 14 + y                        6         6 −10           1
if y = −5

19. Find the value of the expression
                 −4 − z − 1
                                       44
CHAPTER 1. CONCEPTS                            3. THE NUMBER LINE AND SETS OF NUMBERS
Write the real number as a fraction of inte-         Determine whether the inequality is true or
gers.                                                false:

29. 0.4                                              35. 7 ≤ 7

30. 0.3                                              36. 9 > 5

31. −0.17                                            37. −10 < −1

32. −0.87                                            38. −5 ≥ 4

33. 3.1415                                           39. 0.1 < 0.1

34. −2.718                                           40. −9 ≥ −9
                                                           1 1
                                                     41.    >
                                                           8 4
                                                     42. −6 ≤ −10

                                                     43. π > 3
                                                         p
                                                     44. 2 ≤ 2




                                                45
3. THE NUMBER LINE AND SETS OF NUMBERS                     CHAPTER 1. CONCEPTS
       Homework 1.3 Answers.

1. −5                                       25. {−7, −1, −5}

3. −8                                       27. {12, −6}

5. −15                                             4
                                            29.
                                                  10
7. 0
                                                  −17
9. −5                                       31.
                                                  100
11. −1
                                                  31415
                                            33.
13. 4                                             10000

15. −5                                      35. true

17. −18                                     37. true

19. 11                                      39. false
21. {100, 102, 104, 106, 108, . . .}
                                            41. false
        3 4 5 6
    §                 ª
23. 2, , , , , . . .
        2 3 4 5                             43. true




                                       46
CHAPTER 1. CONCEPTS                                    4. PROPERTIES OF REAL NUMBERS
                               4. Properties of Real Numbers


    4.1. Basic Properties. The following properties of real numbers are well-known, but we
are going to highlight and prove them here, just to demonstrate the axiomatic approach we are
taking. As you go through the proofs, you can have some fun by covering up the comments,
and trying to name the axioms responsible for each step in the proof.

  THEOREM 4.1.1. For any real number A,
                                            0·A=0



   PROOF. We appeal directly to the axioms:
                 0A   =    0A + 0                               identity of +
                      =    0A + (1A − 1A)                    additive inverse
                      =    (0A + 1A) − 1A                   associativity of +
                      =    (0 + 1)A − 1A                        distributivity
                      =    1A − 1A                              identity of +
                      =    0                                 additive inverse
                                                                                           

  THEOREM 4.1.2. For all real numbers A and B,
                                    (−A)B = A(−B) = −(AB)
  and
                                     (−A)(−B) = AB
  As a consequence, multiplying numbers with opposite signs produces a negative result,
  while multiplying numbers with the same signs produces a positive result.



   PROOF. To show that (−A)B = −(AB), we appeal to the axioms, as well as the theorem
above:
              (−A)B    =    (−A)B + 0                              identity of +
                       =    (−A)B + (AB − AB)                   additive inverse
                       =    ((−A)B + AB) − AB                 associativity of +
                       =    (−A + A)B − AB                         distributivity
                       =    0B − AB                                identity of +
                       =    0 − AB                               theorem 4.1.1
                       =    −AB                                    identity of +
                                              47
4. PROPERTIES OF REAL NUMBERS                                         CHAPTER 1. CONCEPTS
Showing A(−B) = −(AB) is very similar. To show the last identity, we appeal to the statements
we’ve just proven:

                 (−A)(−B)     =    −(A(−B))
                              =    −(−(AB))
                              =    AB                          theorem 2.3.1

                                                                                                 


  EXAMPLE 4.1.1. Find (−5)(−7)



  SOLUTION: (−5)(−7) = 5 · 7 = 35


                                         ANSWER: 35


  EXAMPLE 4.1.2. Simplify (−4)(7x)



  SOLUTION: (−4)(7x) = −(4 · 7x) = −28x


                                        ANSWER: −28x


  EXAMPLE 4.1.3. Find (−2)(−3)(−5)



  SOLUTION: (−2)(−3)(−5) = (6)(−5) = −30


                                        ANSWER: −30


  THEOREM 4.1.3. For any real number A,
                                         (−1)A = −A
  This is a striking result so many of us take for granted, telling us that taking an opposite
  of a number is the same as multiplying it by the opposite of the multiplicative identity.


                                              48
CHAPTER 1. CONCEPTS                                        4. PROPERTIES OF REAL NUMBERS
   PROOF. We appeal to the axioms, as well as the facts we have proven earlier:
            (−1)A    =     (−1)A + 0                                 additive identity
                     =     (−1)A + (1A − (1A))                        additive inverse
                     =     ((−1)A + 1A) − (1A)                       associativity of +
                     =     ((−1) + 1)A − (1A)                            distributivity
                     =     0A − (1A)                                  additive inverse
                     =     0 − (1A)                                     theorem 4.1.1
                     =     −(1A)                                     additive identity
                     =     −A                                   multiplicative identity
                                                                                                

  EXAMPLE 4.1.4. Remove the parentheses and simplify:
                                           −(x − y + 2)


  SOLUTION: We will rewrite the opposite as a multiplication by −1 and apply the distribu-
  tive property. Recall also that subtracting a number is the same as adding its opposite.
         −(x − y + 2)     =     (−1)(x + (− y) + 2)
                          =     (−1)x + (−1)(− y) + (−1)2                    distributivity
                          =     −x + y + (−2)                        (−1)(− y) = 1 y = y
                          =     −x + y − 2


                                       ANSWER: −x + y − 2


Multiplying by −1 and then distributing may seem tedious, so in practice we simplify opposites
of sums using an even nicer property.

  THEOREM 4.1.4. For any collection of real numbers A, B, C, . . ., the opposite of their sum
  is a sum of their opposites:
                         −(A + B + C + . . .) = (−A) + (−B) + (−C) + . . .


  EXAMPLE 4.1.5. Remove the parentheses and simplify:
                                      −(x − 3 y + 5z − 1 − a)




                                                49
4. PROPERTIES OF REAL NUMBERS                                            CHAPTER 1. CONCEPTS

  SOLUTION:    As we remove these parentheses, we replace each term of the sum by its
  opposite:
                       −(x − 3 y + 5z − 1 − a) = −x + 3 y − 5z + 1 + a


                               ANSWER: −x + 3 y − 5z + 1 + a


  EXAMPLE 4.1.6. Remove the parentheses and simplify:
                                     −(−a + 2 − (5 − b))


  SOLUTION: We can remove the outer parentheses, and then the inner ones:
              −(−a + 2 − (5 − b))    =      a − 2 + (5 − b)
                                     =      a−2+5− b                     −2 + 5 = 3
                                     =      a+3− b
  Alternatively, we can remove the inner parentheses first, but then of course we will get
  the same result:
                        −(−a + 2 − (5 − b))        =      −(−a + 2 − 5 + b)
                                                   =      a−2+5− b
                                                   =      a+3− b


                                     ANSWER: a + 3 − b

   4.2. Positive Integer Exponent.

  DEFINITION 4.2.1. For any real number b and any positive integer n, the nth power of b
  is the product of n numbers b, written as
                                     b n = |b · b · {z
                                                    b · . . . · b}
                                              n times
                          n
  Within the expression b , b is called base and n is called exponent.


  BASIC EXAMPLE 4.2.1. Base −6, exponent 2:
                                    (−6)2 = (−6)(−6) = 36


  BASIC EXAMPLE 4.2.2. Base 5, exponent 3:
                                      53 = 5 · 5 · 5 = 125

                                                 50
CHAPTER 1. CONCEPTS                                    4. PROPERTIES OF REAL NUMBERS

  BASIC EXAMPLE 4.2.3.
                              (−3)4 = (−3)(−3)(−3)(−3) = 81


  BASIC EXAMPLE 4.2.4.
                          (−2)5 = (−2)(−2)(−2)(−2)(−2) = −32


Looking carefully at the examples above allows us to make a general statement about products
of negative numbers:

  THEOREM 4.2.1. A product of even number of negative factors is positive, while a product
  of odd number of of negative factors is negative.


  EXAMPLE 4.2.1. Evaluate (−1)1000


  SOLUTION:
                             (−1)1000 = (−1)(−1)(−1) . . . (−1)
                                        |        {z           }
                                             1000 times


  This is too long to write out by definition, but since (−1)(−1) = 1 and the signs cancel
  in pairs, it is possible to prove that (−1)n = 1 when n is even, and (−1)n = −1 when n is
  odd.


                                        ANSWER: 1


  BASIC EXAMPLE 4.2.5. The following is an example of a notational convention: the expo-
  nent always supersedes the sign in the order of operations.
                               −72 = −(72 ) = −(7 · 7) = −49
  Compare it with
                                  (−7)2 = (−7)(−7) = 49


  BASIC EXAMPLE 4.2.6. Unlike multiplication, the exponent does not distribute over addi-
  tion. You can easily check that
                                   (1 + 2)2 = 33 = 9
  is completely different from
                                  12 + 22 = 1 + 4 = 5




                                            51
4. PROPERTIES OF REAL NUMBERS                                CHAPTER 1. CONCEPTS
   Homework 1.4.

Simplify the expression:

1. (−7)(−3)
                                            Find the value of the expression:
2. (−1)(−17)
                                            13. (−4)2
3. (−2a)(−4b)
                                            14. (−5)2
4. (−6c)(−3d)
                                            15. 25
5. (−5)(−6)(−7)
                                            16. (−4)3
6. (−10)(−3)(−1)
                                            17. (−7)3

Remove the parentheses and simplify:        18. (−1)6

7. −(a − 2b − 6 + c)                        19. −34

8. −(−4 − 3a + 7b)                          20. (0.5)2

9. −(x + 5) − (5 − 3 y)                     21. (−0.6)2

10. −(6 − x) − (x + 20)                     22. −0.42

11. −(10 − (−4 + 2x))                       23. (−1)17

12. −(−7 − (19x − 5))                       24. (−1)54




                                       52
 CHAPTER 1. CONCEPTS                  4. PROPERTIES OF REAL NUMBERS
    Homework 1.4 Answers.

1. 21                            13. 16

3. 8a b                          15. 32

5. −210                          17. −343

7. −a + 2b + 6 − c               19. −81

9. −x − 10 + 3 y                 21. 0.36

11. −14 + 2x                     23. −1




                            53
5. ORDER OF OPERATIONS                                               CHAPTER 1. CONCEPTS
                                  5. Order of Operations

    5.1. Evaluating Sums and Products. The order of operations makes arithmetic expres-
sions unambiguous. We end up evaluating from inside out, so to speak, starting with the
innermost parentheses and absolute values.

Every expression in this text can be thought of as a sum of terms. Some of these terms will
be products of factors, and some of these factors will have exponents. We think of subtraction
as of adding the opposite, so our sums may have subtractions. We also think of division as of
multiplying by the reciprocal, so our products may have divisions.

  DEFINITION 5.1.1 (Order of Operations). To evaluate a sum of terms:

       (1) Find the value of each term.
       (2) Find the value of the sum by applying additions and subtractions from left to
           right.

  Some of the terms will be products of several factors. To evaluate a product of factors
  with exponents:

       (1) Find the value of each exponential expression by using a single factor on the left
           of the exponent as the base.
       (2) Find the value of the product by applying multiplications and divisions from left
           to right.

  At any point in this process we are free to remove or to insert parentheses by using the
  distributive property or any other axiom.


  EXAMPLE 5.1.1. Evaluate the expression
                                          5 + 4x − 2x 2
  if x = 3


  SOLUTION: This is a sum of 3 terms.
        5 + 4x − 2x 2   =   5 + 4(3) − 2(3)2
                        =   5 + 4(3) − 2(9)                      evaluate the exponent
                        =   5 + 12 − 18             all terms are evaluated, time to add
                        =   17 − 18
                        =   −1


                                        ANSWER: −1

                                               54
CHAPTER 1. CONCEPTS                                               5. ORDER OF OPERATIONS

 EXAMPLE 5.1.2. Evaluate the expression
                                       (3 − 5)(−8 + 14)


 SOLUTION: This is a product with 2 factors.
             (3 − 5)(−8 + 14)     =    (−2)(6)               evaluate each factor
                                  =    −12


                                        ANSWER: −12


 EXAMPLE 5.1.3. Evaluate the expression
                                      x 2 ÷ (x + 8)(x + 3)
 if x = −6


 SOLUTION: This is a product with 3 factors, the second factor being the reciprocal of
 (x + 8). Recall that multiplications and divisions are to be done left to right.
    x 2 ÷ (x + 8)(x + 3)   =    (−6)2 ÷ ((−6) + 8)((−6) + 3)           substitute −6 for x
                           =    (36) ÷ (2)(−3)                    all factors are evaluated
                           =    (18)(−3)                                       36/2 = 18
                           =    −54


                                        ANSWER: −54


 EXAMPLE 5.1.4. Evaluate the expression
                                             a − a2
                                             5+a
 if a = −3


 SOLUTION: It is important to remember that the fraction notation (Example 2.3.1) comes
 with invisible parentheses, and the division of numerator by denominator is last in the
 order of operations:
                                a − a2
                                       = (a − a2 ) ÷ (5 + a)
                                5+a




                                              55
5. ORDER OF OPERATIONS                                                   CHAPTER 1. CONCEPTS
 So we can think of this fraction as of a product of 2 factors, and we have to find the value
 of each factor before we can divide:
                a − a2         (−3) − (−3)2
                         =                                  substitute −3 for a
                 5+a             5 + (−3)
                                −3 − (9)
                           =
                                   2
                                −12
                           =
                                 2
                           =    −6


                                            ANSWER: −6


 EXAMPLE 5.1.5. Evaluate the expression
                                      | − 6 · 4| ÷ (−5 + 7)3 (−52 )


 SOLUTION: This can be seen as a product with 3 factors. We have to evaluate each factor
 before we can divide or multiply them. Note that we have to evaluate the base (−5 + 7)
 before we can exponentiate it.
    | − 6 · 4| ÷ (−5 + 7)3 · (−52 )     =    | − 24| ÷ (2)3 · (−25)
                                        =    (24) ÷ 23 · (−25)
                                        =    24 ÷ 8 · (−25)           all factors are evaluated
                                        =    (3)(−25)
                                        =    −75


                                            ANSWER: −75


 EXAMPLE 5.1.6. Evaluate the expression given that x = 2 and y = −1
                                               |x − y|
                                              −(x + y)2


 SOLUTION: We view this fraction as a product of 2 factors:
                                |x − y|
                                         = |x − y| ÷ (−(x + y)2 )
                               −(x + y)2



                                                   56
CHAPTER 1. CONCEPTS                                                  5. ORDER OF OPERATIONS
 We have to evaluate the numerator and the denominator before we can divide.
                             |x − y|           |2 − (−1)|
                                         =
                            −(x + y)2        −(2 + (−1))2
                                                      |3|
                                             =
                                                     −(1)2
                                                      3
                                             =
                                                     −1
                                             =       −3


                                      ANSWER: −3

  5.2. Like Terms.

 DEFINITION 5.2.1. Like terms, sometime called similar terms, are the terms which have all
 the same variable factors with the same corresponding exponents.


 EXAMPLE 5.2.1. Simplify the expression 3x y − 6x y


 SOLUTION: Both terms have the same variable factors x y, so we can factor them out
 using the distributivity:
                               3x y − 6x y       =    (3 − 6)x y
                                                 =    (−3)x y
                                                 =    −3x y


                                     ANSWER: −3x y


 EXAMPLE 5.2.2. Simplify the expression 5a − 2(a2 − 3a)


 SOLUTION: We begin by applying the distributive property to remove the parentheses:
                         5a − 2(a2 − 3a)     =       5a − 2a2 + 2 · 3a
                                             =       5a − 2a2 + 6a




                                             57
5. ORDER OF OPERATIONS                                              CHAPTER 1. CONCEPTS
 Here 5a and 6a are like terms, while −2a2 is different because it has exponent 2 instead
 of 1.
            5a − 2a2 + 6a    =    5a + 6a − 2a2                associativity of +
                             =    (5 + 6)a − 2a   2
                                                                   distributivity
                             =    11a − 2a   2




                                  ANSWER: 11a − 2a2


 EXAMPLE 5.2.3. Simplify the expression 5x − 1 − (3x − 1 + 10 y)


 SOLUTION: We begin by removing parentheses, using the fact that the opposite of a sum
 is the sum of opposites. After that we use commutativity and associativity to change the
 order of addition and put like terms next to each other, and then simplify them using
 distributivity.
      5x − 1 − (3x − 1 + 10 y)   =    5x − 1 − 3x + 1 − 10 y
                                 =    5x − 3x + 1 − 1 − 10 y           properties of +
                                 =    (5 − 3)x + (1 − 1) − 10 y           distributivity
                                 =    2x + 0 − 10 y
                                 =    2x − 10 y


                                  ANSWER: 2x − 10 y




                                             58
CHAPTER 1. CONCEPTS                                          5. ORDER OF OPERATIONS
      Homework 1.5.

Evaluate the expression:                      17. Evaluate the expression
                                                                −75 ÷ x 2
1. 3 + 8 ÷ | − 4|
                                              if x = −5
2. 8 ÷ 4 · 2
                                              18. Evaluate the expression
3. (−6 ÷ 6)3
                                                            45 ÷ 32 y( y − 1)
4. 5(−5 + 6) · 62                             if y = 3

5. 7 − 15 ÷ 3 + 6                             19. Evaluate the expression

6. (−9 − (2 − 5)) ÷ (−6)                                       6x ÷ 12x 3
                                              if x = −2
7. | − 7 − 5| ÷ (−2 − 2 − (−6))
                                              20. Evaluate the expression
8. 4 − 2 · |32 − 16|
                                                            −30 ÷ a(a + 4)2
      −10 − 6                                 if a = −6
9.            −5
       (−2)2

       2 + 4 · |7 + 22 |
10.                                           Simplify the expression by combining like
         4·2+5·3
                                              terms:
                                    
                            −18
11. (6 · 2 + 2 − (−6)) −5 +                   21. r − 9 + 10
                             6

              −13 − 2                         22. −4x + 2 − 4
12.
    2 − (−1) + (−6) − (−1 − (−3))
            3
                                              23. 4b + 6 + 1 + 7b
       |3 − (6 + 11)|      24                 24. −7x − 2 − 2x
13.                   +
              7         (1 − 3)2
                                              25. −8px + 5px
     −52 + (−5)2
14. 2
   |4 − 25 | − 2 · 3                          26. 2r 2 − r + 6r 2

      −9 · 2 − (3 − 6)                        27. t − 2t 2 − t 2 − 7t
15.
    1 − (−2 + 1) − (−3)
                                              28. −5x + y + 12x − y
           5 + 3 − 24 ÷ 6 · 2
                  2
16.
       (5 + 3(22 − 5)) + |22 − 5|2
                                              Simplify the expression by removing paren-
                                              theses and combining like terms:

                                              29. 9(b + 10) + 5b
                                         59
5. ORDER OF OPERATIONS                              CHAPTER 1. CONCEPTS
30. 4v − 7(1 − 8v)                  37. a(x + 3) − 4ax + 2a

31. −3(1 − 4x) − 4x                 38. 9x − 11x y − x(1 + 2 y)

32. −8x + 9(−9x + 9)                39. 5(1 − 6k) + 10(k − 8)

33. −10 − 4(n − 5)                  40. −7(4x − 6) + 2(10x − 10)

34. −6(5 − m) + 3m                  41. b(m + y) − m(b − y)

35. (8n2 − 3n) − (5 + 4n2 )         42. (2b − 8) − 2(b2 + 5)

36. (7x 2 − 3) − (5x 2 + 6x)




                               60
CHAPTER 1. CONCEPTS                                  5. ORDER OF OPERATIONS
       Homework 1.5 Answers.

1. 5                                23. 11b + 7

3. −1                               25. −3px

5. 8
                                    27. −6t − 3t 2
7. 6
                                    29. 14b + 90
9. −9
                                    31. −3 + 8x
11. −40
                                    33. 10 − 4n
13. 6
                                    35. 4n2 − 3n − 5
15. −3
                                    37. −3ax + 5a
17. −3

19. 8                               39. −20k − 75

21. r + 1                           41. b y + m y




                               61
6. PRIME NUMBERS                                                              CHAPTER 1. CONCEPTS
                                 6. Prime Numbers
  6.1. Integer Division and Prime Numbers.

 DEFINITION 6.1.1. A positive integer m is an integer divisor or just divisor of a positive
 integer n if dividing n by m leaves no remainder.


 BASIC EXAMPLE 6.1.1.

         •   4 is a divisor of 12 because 12/4 = 3
         •   5 is a divisor of 5 because 5/5 = 1
         •   3 is not a divisor of 5 because 5/3 leaves a remainder of 2
         •   2.5 is not a divisor of 5 because 2.5 is not an integer


 DEFINITION 6.1.2. We call a positive integer prime when it has exactly two positive integer
 divisors: one and itself.


 BASIC EXAMPLE 6.1.2.

         •   1 is not prime because it only has a single divisor, itself
         •   2 is prime because it has exactly two divisors: 1 and 2
         •   3 is also prime
         •   4 is not prime because it has three divisors: 1, 2, and 4
         •   5 is prime
         •   6 is not prime because it has four divisors: 1, 2, 3, and 6


 THEOREM 6.1.1. The set of prime numbers is infinite, and it starts like this:
  {2, 3, 5, 7, 11, 13, 17, 19, 23, 29, 31, 37, 41, 43, 47, 53, 59, 61, 67, 71, 73, 79, 83, 89, 97, . . .}


 DEFINITION 6.1.3. A positive integer with three or more positive integer divisors is called
 composite. Incidentally, 1 is neither prime nor composite.


 THEOREM 6.1.2 (Fundamental Theorem of Arithmetic). Every positive integer greater
 than 1 is either prime or is a product of prime factors, and this product is unique, up to
 the order of the factors.


 EXAMPLE 6.1.1. Write 10 as a product of prime factors.


                                            ANSWER: 2 · 5

                                                   62
CHAPTER 1. CONCEPTS                                                           6. PRIME NUMBERS

 EXAMPLE 6.1.2. Write 60 as a product of prime numbers.


 SOLUTION: A quick and greedy way to success is to try to express the number as a product
 of smaller numbers, and keep doing that until nothing is left but primes.


                                 60 = 6 · 10 = (2 · 3) · (2 · 5)

 The order is not important within the product, but a traditional way to state the answer
 is by using the exponential notation, and listing lower bases first.


                                     ANSWER: 22 · 3 · 5


 EXAMPLE 6.1.3. Write 231 as a product of prime numbers.


 SOLUTION: When it is not immediately clear how to factor, we try dividing by every prime,
 starting with the lowest one, as many times as possible. 231/2 leaves a remainder of 1,
 so 2 is not a factor. 231/3 = 77, so 3 is a factor, and then
                                   231 = 3 · 77 = 3 · 7 · 11


                                     ANSWER: 3 · 7 · 11


 EXAMPLE 6.1.4. Write 450 as a product of prime numbers.


 SOLUTION:
                       450 = 10 · 45 = (2 · 5)(5 · 9) = 2 · 5 · 5 · (3 · 3)


                                     ANSWER: 2 · 32 · 52




                                               63
6. PRIME NUMBERS                                                       CHAPTER 1. CONCEPTS

  EXAMPLE 6.1.5. Write 448 as a product of prime numbers.


  SOLUTION: Every even number has 2 as a factor, so we will keep dividing by 2 until we
  encounter an odd quotient:
                              448   =    2 · 224
                                    =    2 · 2 · 112
                                    =    2 · 2 · 2 · 56
                                    =    2 · 2 · 2 · (8 · 7)
                                    =    2 · 2 · 2 · (2 · 2 · 2) · 7


                                     ANSWER: 26 · 7

   6.2. Fractions of Integers in Lowest Terms.

  DEFINITION 6.2.1. A rational number is written in lowest terms if numerator and denom-
  inator have no common prime factors.


When numerator and denominator do have common factors, we can reduce the fraction to
lowest terms by canceling them.

  THEOREM 6.2.1. Common factors cancel in fractions. For any real number A and any two
  non-zero real numbers D and C,
                                      AC      A
                                           =
                                      DC      D


  EXAMPLE 6.2.1. Write 12/15 in lowest terms.


  SOLUTION: If we can detect a common factor, we can simplify by canceling it.

                                      12 3 · 4 4
                                        =     =
                                      15 3 · 5 5


                                                    4
                                        ANSWER:
                                                    5


  EXAMPLE 6.2.2. Write 60/315 in lowest terms.


                                            64
CHAPTER 1. CONCEPTS                                                            6. PRIME NUMBERS

 SOLUTION: Sometimes it is hard to identify common factors right away, but we can always
 completely factor both numerator and denominator, and then all of the common factors
 will become apparent.
                   60    =   6 · 10 = (2 · 3) · (2 · 5) = 2 · 2 · 3 · 5
                 315     =   5 · 63 = 5 · (7 · 9) = 5 · 7 · (3 · 3) = 3 · 3 · 5 · 7
 The common factors are 3 and 5:

                              60   2·2·3·5 2·2         4
                                 =             =     =
                              315 3 · 3 · 5 · 7 3 · 7 21


                                                       4
                                        ANSWER:
                                                      21


 EXAMPLE 6.2.3. Simplify the fraction assuming all variables are non-zero:
                                          4abx
                                          4b y


 SOLUTION: Common factors 4 and b cancel.


                                                      ax
                                        ANSWER:
                                                       y


 EXAMPLE 6.2.4. Simplify the fraction assuming all variables are non-zero:
                                              6 y3
                                             18 y 2


 SOLUTION:
                 6 y3         y3
                         =                             cancel common factor 6
                18 y 2       3 y2
                             y
                         =                            cancel common factor y 2
                             3


                                                      y
                                         ANSWER:
                                                      3

                                               65
6. PRIME NUMBERS                                                  CHAPTER 1. CONCEPTS

 EXAMPLE 6.2.5. Simplify the fraction assuming all variables are non-zero:
                                          60x 2
                                          40x


 SOLUTION:
             60x 2       60 · x
                     =                       cancel common variable factor x
             40x          40
                         3 · 20 · x
                     =                       20 is a common numerical factor
                          2 · 20
                         3x
                     =                                  rewrite in lowest terms
                          2


                                                  3
                                      ANSWER:       x
                                                  2




                                           66
CHAPTER 1. CONCEPTS                                                  6. PRIME NUMBERS
   Homework 1.6.

Write the number as a product of prime fac-        Rewrite the rational number in lowest
tors with exponents.                               terms.

1. 12                                                    21
                                                   23.
                                                         28
2. 14
                                                         21
                                                   24.
3. 16                                                    35
                                                         16
4. 18                                              25.
                                                         26
5. 19                                                    12
                                                   26.
6. 20                                                    48
                                                         12
7. 22                                              27.
                                                         40
8. 24                                                    75
                                                   28.
                                                         80
9. 26
                                                         52
10. 27                                             29.
                                                         13
11. 30                                                   110
                                                   30.
                                                          5
12. 36
                                                          4
                                                   31.
13. 40                                                   19

14. 42                                                   46
                                                   32.
                                                         48
15. 45
                                                         150
                                                   33.
16. 48                                                   450

17. 160                                                  72
                                                   34.
                                                         45
18. 180
                                                          27
                                                   35.
19. 625                                                  150
                                                          8
20. 225                                            36.
                                                         100
21. 196                                                  24
                                                   37.
22. 1000                                                 64

                                              67
6. PRIME NUMBERS                         CHAPTER 1. CONCEPTS
      25                Simplify the fraction assuming all variables
38.
      65                are non-zero:
       90                     18x 3
39.                     45.
      225                      6x
      253                     7u2
40.                     46.
       11                     28u
      160                   −6ab2
41.                     47.
      180                   −10b
      625                     −8x y
42.                     48.
      225                     12x y
       196
43.                           −x
      1000              49.
                              5x 3
      230
44.                            20
      460               50.
                              −60b
                              48x y 2
                        51.
                              30x 2 y

                            46x 2 y 2
                        52.
                            −26x 2




                   68
 CHAPTER 1. CONCEPTS                            6. PRIME NUMBERS
      Homework 1.6 Answers.

1. 22 · 3                                 4
                                   31.
                                         19
3. 24
                                         1
                                   33.
5. 19                                    3

7. 2 · 11                                 9
                                   35.
                                         50
9. 2 · 13
                                         3
11. 2 · 3 · 5                      37.
                                         8

13. 23 · 5                               2
                                   39.
                                         5
15. 32 · 5
                                         8
17. 25 · 5                         41.
                                         9
19. 54                                    49
                                   43.
                                         250
21. 22 · 72
    3                              45. 3x 2
23.
    4                                    3ab
                                   47.
       8                                  5
25.
      13
                                         −1
                                   49.
       3                                 5x 2
27.
      10
                                         8y
29. 4                              51.
                                         5x




                              69
7. FRACTIONS AND RATIONALS                                            CHAPTER 1. CONCEPTS
                                  7. Fractions and Rationals

    7.1. Fraction Notation. Recall that for any non-zero real number B there is a reciprocal,
which we denote by 1/B, such that B · (1/B) = 1. Recall also that division is multiplication by
the reciprocal:
                                              A      1
                                     A÷ B = = A·
                                              B      B

  THEOREM 7.1.1. For any two non-zero real numbers A and B, the reciprocal of a product
  is the product of reciprocals:
                                       1    1 1
                                         = ·
                                      AB    A B


  BASIC EXAMPLE 7.1.1. This theorem generalizes to larger products as well:
                                1 1       1          1            1
                                   ·    ·    =               =
                                7 10 −2 7 · 10 · (−2) −140
  This also also gives us a justification for multiplying fractions the way we do, by computing
  the product of numerators over the product of denominators:
                3 2                 1        1
                                     ‹       ‹
                  ·     =      3·      · 2·                       by definition of ÷
                7 5                 7        5
                                         1 1
                                             ‹
                        = (3 · 2) ·        ·                 reorder multiplication
                                         7 5
                                          1
                                            ‹
                        = (3 · 2) ·                                   apply theorem
                                         7·5
                                  1
                       =    6·                                           simplify
                                 35
                             6
                       =                                       by definition of ÷
                            35


  EXAMPLE 7.1.1. Identify all the factors in the expression by rewriting the fraction as a
  product:
                                             3b
                                             xy


  SOLUTION: Formally, this fraction is a product of (3b) and the reciprocal of (x y), which
  we can now separate into individual factors using the axioms and the basic properties:
                         3b                  1               1 1
                               = (3b) ·            = 3· b· ·
                         xy                (x y)             x y

                                              70
CHAPTER 1. CONCEPTS                                        7. FRACTIONS AND RATIONALS

                                                          1 1
                             ANSWER: 4 factors: 3 · x ·    ·
                                                          x y


 EXAMPLE 7.1.2. Identify all the factors in the expression by rewriting the fraction as a
 product:
                                       −5A(B − 1)
                                            3X


 SOLUTION: It may be useful to rewrite the expression with multiplications made visible
 so that factors stand out more:
                                  (−5) · A · (B − 1)
                                         3·X


                                                                 1 1
                       ANSWER: 5 factors: (−5) · A · (B − 1) ·    ·
                                                                 3 X


 THEOREM 7.1.2. To multiply fractions, multiply numerators to obtain the new numerator,
 and multiply denominators to obtain the new denominator. For any two real numbers A
 and B and any two non-zero real numbers C and D,
                                       A B     AB
                                        · =
                                      C D      CD


 EXAMPLE 7.1.3. Multiply fractions:
                                           x 4
                                            ·
                                           3 5


 SOLUTION:
                                      x 4  x · 4 4x
                                       · =      =
                                      3 5  3·5    15


                                                 4x
                                       ANSWER:
                                                 15




                                            71
7. FRACTIONS AND RATIONALS                                        CHAPTER 1. CONCEPTS

                                                           7
 EXAMPLE 7.1.4. Multiply and simplify the result:     8·
                                                           12


 SOLUTION: When multiplying (or dividing) a fraction by a number, it may be useful to
 represent that number as a fraction first:
                     7        8 7
                 8·     =       ·
                    12        1 12
                               8·7
                         =
                              1 · 12
                              4·2·7
                         =                            4 is a common factor
                               4·3
                              2·7
                         =
                               3
                              14
                         =
                               3


                                                 14
                                       ANSWER:
                                                  3


 EXAMPLE 7.1.5. Multiply and state the answer as a single fraction:
                                        3 x +1
                                          ·
                                        2   5


 SOLUTION: Recall that the fraction notation (Example 2.3.1) comes with invisible paren-
 theses, and so the sum x + 1 in the numerator becomes a single factor when we multiply
 numerators:
                                3 x +1          3 (x + 1)
                                  ·        =      ·
                                2    5          2    5
                                                 3(x + 1)
                                           =
                                                   2·5
                                                 3(x + 1)
                                           =
                                                    10


                                               3(x + 1)
                                    ANSWER:
                                                  10


                                           72
CHAPTER 1. CONCEPTS                                          7. FRACTIONS AND RATIONALS

 EXAMPLE 7.1.6. Find the area of a rectangular piece of fabric 5/4 feet long and 2/7 feet
 wide.


 SOLUTION: The area of a rectangle is the product of its length and width:
              5 2          5·2
                ·     =                         multiply the dimensions
              4 7          4·7
                              5
                        =                         cancel common factor 2
                             2·7
                             5
                        =
                             14


                                             5
                                  ANSWER:      square feet
                                            14


 THEOREM 7.1.3. To divide a fraction by a fraction, multiply it by the reciprocal of the
 divisor. For any real number A and any three non-zero real numbers B, C and D,
                                  A B       A D     AD
                                    ÷ = · =
                                  C D       C B     CB


 EXAMPLE 7.1.7. Divide fractions:
                                            6 2
                                             ÷
                                            5 5


 SOLUTION:
                                     6 2          6 5
                                      ÷      =     ·
                                     5 5          5 2
                                                  6·5
                                             =
                                                  5·2
                                                  6
                                             =
                                                  2
                                             =    3



                                       ANSWER: 3


                                            73
7. FRACTIONS AND RATIONALS                                         CHAPTER 1. CONCEPTS

                                     8    a+b
  EXAMPLE 7.1.8. Divide fractions       ÷     and state the answer as a single fraction.
                                    a−1    2


  SOLUTION:    Make the fraction notation parentheses visible in order to multiply sums
  correctly:
          8    a+b               8      (a + b)
             ÷           =            ÷                        fraction parentheses
         a−1    2             (a − 1)      2
                                 8       2
                         =           ·                  multiplication by reciprocal
                              (a − 1) (a + b)
                                   8·2
                         =
                              (a − 1)(a + b)
                                    16
                         =
                              (a − 1)(a + b)


                                                   16
                                 ANSWER:
                                             (a − 1)(a + b)



   7.2. Changing the Denominator. Before we start adding fractions, we need to figure out
a way to change a denominator without affecting the value of a fraction.

  EXAMPLE 7.2.1. Rewrite the fraction 7/10 so that it has the denominator 40.


  SOLUTION: The challenge is in keeping the value of the fraction the same, while changing
  its appearance. If we multiply a fraction by 1, its value will not change. So when we
  multiply it by 4/4, both numerator and denominator change, but the value of the fraction
  stays the same:
                                  7    7 4      7·4      28
                                    =     · =         =
                                 10 10 4 10 · 4 40


                                                  28
                                        ANSWER:
                                                  40




                                             74
CHAPTER 1. CONCEPTS                                     7. FRACTIONS AND RATIONALS

 EXAMPLE 7.2.2. Rewrite the fraction −11/3 so that it has the denominator 18.


 SOLUTION:
                               11    11 6   11 · 6    66
                           −      =−   · =−        =−
                                3    3 6    3·6       18


                                                66
                                    ANSWER: −
                                                18




                                          75
7. FRACTIONS AND RATIONALS                                            CHAPTER 1. CONCEPTS
      Homework 1.7.

Identify all the factors in the expression by              −12 −9
                                                     14.      ÷
rewriting the fraction as a product:                        7   5
      3x                                                   14
1.                                                   15.      ÷2
       5                                                    5
      6                                                    −13 −15
2.                                                   16.      ÷
      7a                                                    8   8

        x +2                                               5 5
3.                                                   17.    ÷
      2x(1 + a)                                            3 6

      4 yz                                                 −4 −13
4.                                                   18.      ·
      a+b                                                   5   8

      −4x(x + 3)                                            1   3
5.                                                   19.      ÷
       a b(5 + c)                                          10 2
                                                           5 5
         12u                                         20.    ·
6.                                                         3 3
   −5(x + 1)(x + 2)


                                                     21. Find the area of a square with the side
                                                     length of 3/4 feet.
Perform fraction multiplication or division,
simplify the result, and state the answer as         22. Find the area of a rectangular strip
a single fraction:                                   of land 21/5 meters long and 2/3 meters
      1 2                                            wide.
7.     ·
      3 5
      1 3
8.     ·
      8 8
           −2
9. 9 ·
            9
           5
10. 2 ·
           6
       6 11
11.     ·
       5 8
       −2 3
12.      ·
       3 4
                3
13. −2 ÷
                4
                                                76
CHAPTER 1. CONCEPTS                                           7. FRACTIONS AND RATIONALS
Rewrite the fraction so that it has the given              13
                                                     27.      with denominator 21
denominator:                                                7
      2                                                    9
23.     with denominator 12                          28.     with denominator 16
      3                                                    2
      −1                                                   −11
24.      with denominator 8                          29.       with denominator 600
      4                                                     6
      −3                                                   −5
25.      with denominator 28                         30.      with denominator 99
      14                                                    9
      7
26.     with denominator 20
      5




                                                77
 7. FRACTIONS AND RATIONALS                                              CHAPTER 1. CONCEPTS
      Homework 1.7 Answers.

                        1                               17. 2
1. 3 factors: 3 · x ·
                        5
                       1 1    1                                1
3. 4 factors: (x + 2) · · ·                             19.
                       2 x (1 + a)                            15

                                     1 1    1           21. 9/16 square feet
5. 6 factors: (−4) · x · (x + 3) ·    · ·
                                     a b (5 + c)
      2                                                        8
7.                                                      23.
     15                                                       12

9. −2                                                         −6
                                                        25.
      33                                                      28
11.
      20
                                                              39
    −8                                                  27.
13.                                                           21
     3
      7                                                       −1100
15.                                                     29.
      5                                                        600




                                                   78
CHAPTER 1. CONCEPTS                                               8. FRACTION ADDITION
                                 8. Fraction Addition
   8.1. Adding Fractions with a Common Denominator.

  THEOREM 8.1.1. A sum of fractions with a common denominator is the sum of numerators
  over the same denominator. For all real numbers A, B, and C 6= 0
                                   A B           A+ B
                                      +      =
                                   C C             C
                                      A B           A− B
                                       −       =
                                      C C            C


  PROOF. Express fractions as products, and then apply the distributive property to the com-
mon denominator:
                                A B              1      1
                                  +      = A· + B ·
                                C C             C       C
                                                          1
                                           =    (A + B)
                                                          C
                                                A+ B
                                           =
                                                 C
The subtraction case is similar.                                                          

                                           3 21
  EXAMPLE 8.1.1. Simplify the expression     +
                                           8   8


  SOLUTION:
                                   3 21 3 + 21 24
                                     +   =    =   =3
                                   8   8   8    8


                                        ANSWER: 3


                                           4   9
  EXAMPLE 8.1.2. Simplify the expression     −
                                           15 15


  SOLUTION:
                                   4   9   4 − 9 −5 −1
                                     −   =      =    =
                                   15 15    15    15   3


                                                    1
                                        ANSWER: −
                                                    3

                                               79
8. FRACTION ADDITION                                                  CHAPTER 1. CONCEPTS
  8.2. Least Common Multiple.

 DEFINITION 8.2.1 (Least Common Multiple). The least common multiple for a collection
 of positive integers a, b, c, . . . (LCM for short) is the smallest positive integer divisible
 by each of the a, b, c, . . . without a remainder. To find the LCM, we factor each integer
 completely, and then take the product with each prime factor to the highest degree we
 found.


 BASIC EXAMPLE 8.2.1.

                                        integers LCM
                                          4, 12   12
                                          4, 6    12
                                          8, 10   40
                                         2, 3, 5  30
                                         4, 6, 8  24


 EXAMPLE 8.2.1. Find the LCM for 16 and 20.


 SOLUTION: When it’s too hard to guess the LCM, we can factor each integer to expose the
 prime factors:
                                        16    =    24
                                        20    =    22 · 5
 The highest power of 2 is 4, and the highest power of 5 is 1, so the LCM is
                                      24 · 5 = 16 · 5 = 80


                                        ANSWER: 80


 EXAMPLE 8.2.2. Find the LCM for 34 and 40.


 SOLUTION: Factor each integer:
                                        34   =     2 · 17
                                        40   =     23 · 5
 So the LCM is 23 · 5 · 17 = 680


                                        ANSWER: 680

                                              80
CHAPTER 1. CONCEPTS                                                 8. FRACTION ADDITION

 EXAMPLE 8.2.3. Find the LCM for 121, 66, 8.


 SOLUTION: Factor each integer:
                                     121   =    112
                                     66    =    2 · 3 · 11
                                       8   =    23
 So the LCM is 23 · 3 · 112 = 2904


                                      ANSWER: 2904

  8.3. Adding Fractions of Integers.

 DEFINITION 8.3.1. The lowest common denominator for a collection of rational numbers
 (LCD for short) is the LCM of their denominators.


 THEOREM 8.3.1. Fractions with different denominators can be added by rewriting each
 fraction in an equivalent form using the LCD, and then adding as usual.


 EXAMPLE 8.3.1. Simplify and state as a fraction in lowest terms:
                                          5 3
                                            +
                                          2 4


 SOLUTION: The LCD is 4, so we rewrite the first fraction:
                                  5 5 2 10
                                     = · =
                                  2 2 2           4
 and then we can add:
                                 5 3           10 3
                                  +      =         +
                                 2 4            4     4
                                                  10 + 3
                                            =
                                                    4
                                                  13
                                            =
                                                   4


                                                     13
                                       ANSWER:
                                                      4

                                           81
8. FRACTION ADDITION                                                CHAPTER 1. CONCEPTS

 EXAMPLE 8.3.2. Simplify and state as a fraction in lowest terms:
                                          5 2
                                            −
                                          3 5


 SOLUTION: The LCD is 15, so we rewrite both fractions:
                                 5        5 5 25
                                    =       · =
                                 3        3 5 15
                                    2         2 3  6
                                         =     · =
                                    5         5 3 15
 and then we can subtract:
                                   5 2              25   6
                                    −        =         −
                                   3 5              15 15
                                                    25 − 6
                                             =
                                                     15
                                                    19
                                             =
                                                    15


                                                     19
                                        ANSWER:
                                                     15

  8.4. Mixed Numbers.

 DEFINITION 8.4.1. A rational number is written as a proper fraction when the absolute
 value of the numerator is less than the absolute value of the denominator. In other cases
 it is called an improper fraction.


 BASIC EXAMPLE 8.4.1. Some proper fractions: 17/19, −1/2, 10/20.

 Some improper fractions: 3/2, 7/7, −101/100.


 DEFINITION 8.4.2. A fraction is expressed as a mixed number when it is a sum of an integer
 and a proper fraction. Traditionally, the plus sign is not shown. For example, the number
 3.25, which is equal to
                                                 1
                                            3+
                                                 4
 looks like this in the mixed number notation:
                                             3 14


                                             82
CHAPTER 1. CONCEPTS                                             8. FRACTION ADDITION

 EXAMPLE 8.4.1. Rewrite 60/13 as a mixed number.


 SOLUTION: When we divide 60 by 13, the integer quotient is 4 and the remainder is 8, so
                                   60         8
                                       =4+
                                   13        13


                                                  8
                                        ANSWER: 4 13


 EXAMPLE 8.4.2. Rewrite 7 45 as an improper fraction.


 SOLUTION:
                                        4 35 4 35 + 4 39
                           7 45 = 7 +     =   + =    =
                                        5   5  5  5    5


                                                   39
                                         ANSWER:
                                                    5




                                             83
8. FRACTION ADDITION                                                  CHAPTER 1. CONCEPTS
      Homework 1.8.

Perform the summation and simplify.                       2 5
                                                    17.    −
                                                          5 4
      1 3
1.     +                                                       2
      8 8                                           18. −1 −
                                                               3
       4   5
2.       +                                                −5 15
      15 15                                         19.      −
                                                           7   8
      14 24
3.       −                                             3 9
       5   5                                        20.  +
                                                       2 7
      12 19
4.       −                                             5
                                                           
                                                             11
                                                                ‹
       7   7                                        21. − −
                                                       3      6
      17 10
5.      +
      6π 6π                                               13 4
                                                    22.     −
                                                          18 9
      10 22
6.       −
       3   3                                            1 11
                                                    23.   −
                                                        2    6
                                                               1
                                                              ‹
Find the LCM for given integers.                    24. −1 −
                                                               3
7. 3, 6
                                                        11 1
                                                    25.     −
8. 5, 15                                                 8    2
                                                          4      2
                                                                  ‹
9. 6, 9                                             26. − + −
                                                          3     15
10. 10, 25
                                                               5
                                                    27. −6 −
11. 2, 9, 10                                                   3

12. 3, 8, 15                                                15 5
                                                    28. −      +
                                                             8    3
13. 48, 84
                                                          3 7 1
                                                                   ‹
                                                    29.         −
14. 28, 42                                                2 9 9
                                                          2 6 9
                                                                   ‹
Perform the operations, simplify, state the         30.         +
                                                          3 5 5
answer as a single fraction in lowest terms:
                                                            5 7       13
                                                                 ‹
           15                                       31.       +     ÷
15. 2 +                                                     9 3        6
           8
                                                            7 11       5
                                                                  ‹
      3 5                                           32.       −      ÷
16.    +                                                    8    4     4
      5 4
                                               84
CHAPTER 1. CONCEPTS                                                  8. FRACTION ADDITION
Simplify the expression by combining like        Rewrite the mixed number as an improper
terms:                                           fraction:
      5   2                                      45. 1 32
33.     x+ x
      9   3
    11                                           46. 2 27
34.    a − 3a
     2                                                   5
                                                 47. −10 11
      5y 5y
35.      −                                       48. −9 49
       7   21
    13b 11b
36.      +
     15     45
         3 2 1
                  ‹
37. 5a −       − a
         4 3 3                                   Rewrite the fraction as a mixed number:
        5 1     9
                   ‹
                                                       22
38. b +      +    b                              49.
        3 6 10                                          5
      1            3                                   50
39.     (6a + 5b) + (4a − 9b)                    50.
      2            2                                    7
      2             1                                    110
40.     (9x − 4 y) − (3a − 2 y)                  51. −
      3             3                                    21
41. Evaluate the expression                              82
                                                 52. −
                      3                                  13
                 6a +
                      2
if a = −7/5

42. Evaluate the expression
                       4                         53. Find the perimeter of the shown shape
                3b +                             if side lengths are given in inches:
                      15
if b = −4/7

43. Evaluate the expression                                            9/14
               3       1
                        ‹
                   x+                                         5/14
               5       x                                                      2/7
if x = 2/3
                                                                       3/7
44. Evaluate the expression
              3       3     1
                       ‹
                  y−      ·                      54. Find the perimeter of a rectangular
             10       4     y                    strip of land 21/5 meters long and 2/3 me-
if y = 3/2                                       ters wide.

                                            85
 8. FRACTION ADDITION                                   CHAPTER 1. CONCEPTS
         Homework 1.8 Answers.

     1                                29. 1
1.
     2
                                            4
3. −2                                 31.
                                            3
      9
5.                                          11
     2π                               33.      x
                                             9
7. 6
                                            10 y
                                      35.
9. 18                                        21

11. 90                                      21    1
                                      37.      a−
                                             4    2
13. 336
                                      39. 9a − 11b
    31
15.
     8                                      −69
                                      41.
                                             10
         −17
17.
          20                                13
                                      43.
         −145                               10
19.
          56                                5
                                      45.
         7                                  3
21.
         2
                                                115
                                      47. −
             4                                   11
23. −
             3
                                      49. 4 52
         7
25.
         8                                   5
                                      51. −5 21
             23
27. −                                 53. 12/7 inches
              3




                                 86
CHAPTER 1. CONCEPTS                                                                  9. TRANSLATION
                                                    9. Translation

   9.1. Expressions. The order of operations in mathematical expressions makes them un-
ambiguous, and there are ways to carry some of that certainty into plain English. Here are
some of the common ways to describe arithmetic expressions:

                                 a+b            the sum of a and b
                                 a−b            the difference of a and b
                                  ab            the product of a and b
                                 a/b            the quotient of a and b
                                  a2            the square of a

To see the difficulty with translation, consider the phrase
                                      the sum of a and b times c
It could mean a + bc, or it could mean (a + b)c, and these expressions are not equivalent.
Sometimes the ambiguity can be removed by finding a different phrasing:

                           the sum of a and the product of b and c

definitely means a + bc, while

                                      the sum of a and b, times c

will probably be interpreted as (a + b)c, because of where the comma is placed in the sentence.

   9.2. Equations. Many English sentences can be translated into equations almost word for
word. Here are some of the most notorious and useful patterns:

  BASIC EXAMPLE 9.2.1.
                              a       is        2       units greater than       b
                              a       =                      b + 2


  BASIC EXAMPLE 9.2.2.
                                  a        is       5    units less than     b
                                  a        =                b − 5


  BASIC EXAMPLE 9.2.3.
                              a       is        6       times greater than       b
                              a       =                      6 · b


                                                           87
9. TRANSLATION                                                        CHAPTER 1. CONCEPTS

 BASIC EXAMPLE 9.2.4.
                              a   is   7   times less than     b
                              a   =           b / 7



 EXAMPLE 9.2.1. Describe the variables used to represent the given quantities, and write
 the statement as an algebraic relation:

                          One coffee costs as much as three donuts.



 SOLUTION: Whenever it is not quite clear what to write, it may be useful to rephrase a
 statement to make it sound like one of the patterns above. The price of one coffee is three
 times the price of a donut. If c is the price of one coffee in dollars and d is the price of
 one coffee in dollars, then c = 3d. We will state the answer by defining the variables and
 their units, and then stating the relation between them.



                                          ANSWER:
                           c is the price of one coffee in dollars
                           d is the price of one donut in dollars
                                           c = 3d


 EXAMPLE 9.2.2. Describe the variables used to represent the given quantities, and write
 the statement as an algebraic relation:

                     Alice is biking 2.5 miles per hour faster than Bob.



 SOLUTION: This statement compares speeds, so let a be Alice’s speed, and let b be Bob’s
 speed, and then the sentence can be rephrased as

                 Alice’s speed is 2.5 miles per hour greater than Bob’s speed.



                                           ANSWER:
                                  a is Alice’s speed in mph
                                  b is Bob’s speed in mph
                                          a = b + 2.5


                                             88
CHAPTER 1. CONCEPTS                                                         9. TRANSLATION

 EXAMPLE 9.2.3. Describe the variables used to represent the given quantities, and write
 the statement as an algebraic relation:

    The width of a FIBA regulation basketball court is 13 meters shorter than its length.


                                          ANSWER:
                            w is the width of a court in meters
                            l is the length of a court in meters
                                         w = l − 13


 EXAMPLE 9.2.4. Describe the variables used to represent the given quantities, and write
 the statement as an algebraic relation:

                        A tiger weighs 3.1 times less than a giraffe.


                                         ANSWER:
                              t is the weight of a tiger in kg
                             g is the weight of a giraffe in kg
                                               g
                                          t=
                                              3.1




                                             89
9. TRANSLATION                                                       CHAPTER 1. CONCEPTS
    9.3. Applications. We will solve applications by describing the variables used to represent
the quantities of interest, and stating appropriate equations. As you construct your own solu-
tions to applications, you may be using different variable names, and your equations may also
be somewhat different, depending on how you translate English sentences. The final numerical
answers, however, should always come out the same, regardless of translation.

  EXAMPLE 9.3.1. A brownie recipe is asking for 350 grams of sugar, and a pound cake recipe
  requires 270 more grams of sugar than a brownie recipe. How much sugar is needed for
  the pound cake?


  SOLUTION: Let b stand for the weight of sugar needed for the brownies, and p for the
  weight of sugar needed for the pound cake, both measured in grams. Then the phrase
  “pound cake recipe requires 270 more grams of sugar than a brownie recipe” translates as
  “p is 270 grams greater than b”, or
                                         p = b + 270
  Since b = 350, we get
                                     p = 350 + 270 = 620


                                        ANSWER:
       b and p are sugar amounts for brownies and pound cake respectively, in grams,
                                  equation: p = b + 270
                                     solution: p = 620


  EXAMPLE 9.3.2. The size of a compressed file is 1.74 MiB, while the size of the original
  uncompressed file is 5.5 times greater. What is the size of the uncompressed file?


  SOLUTION: We can restate the given informationas follows: the size of the uncompressed
  file is 5.5 times greater than the size of the compressed file:
                                           u = 5.5c


                                        ANSWER:
   u and c are the sizes of the uncompressed and the compressed file respectively, in MiB,
                                     equation: u = 5.5c
                                     solution: u = 9.57




                                              90
CHAPTER 1. CONCEPTS                                                            9. TRANSLATION
   Homework 1.9.

Describe the variables used to represent the          15. The upload speed in bytes per second
given quantities, and write the statements            is 10 times slower than twice the download
as algebraic relations.                               speed.

1. Suzy is 5 years older than Ron.                    16. Nancy’s wage in dollars per hour is 1.2
                                                      times greater than three times Phil’s wage.
2. A buffalo is 4 times heavier than a tiger.

3. The radius of Jupiter is 11 times greater
than the radius of the Earth.                         Solve applications by describing the vari-
                                                      ables used to represent the quantities of in-
4. A puppy is 6 pounds heavier than a kit-
                                                      terest, and stating appropriate equations.
ten.
                                                      17. Margo is 5 years old and Ivan is 34
5. The air is 30 ◦ F cooler than the water.
                                                      years older than Margo. How old is Ivan?
6. A donut is 2 dollars cheaper than a tea.
                                                      18. The temperature in Boston is −3 ◦ C
                                                      and the temperature at the South Pole is 57
7. The highway is twice as wide as the al-            ◦
                                                        C lower than the temperature in Boston.
ley.
                                                      What is the South Pole temperature?
8. In winter, the day is twice as short as the
                                                      19. Alice runs 1.9 times faster than Bob,
night.
                                                      who runs at 4.7 mph. How fast does Alice
9. Abby is 7 inches shorter than Kurt.                run?

10. A bicycle is 4 times faster than a run-           20. In February 2017 the US House of Rep-
ning person.                                          resentatives had 362 congressmen. There
                                                      were 286 fewer women in the House. How
11. Jeff’s pool is one and a half times               many congresswomen were there?
longer than Kate’s pool.
                                                      21. Tao’s first car was 3 times cheaper than
12. The postage this year is 2 cents more             his second car, and he bought his second
expensive than it was the last year.                  car for 7899 dollars. How much was Tao’s
                                                      first car?
13. The perimeter of a rectangle is 2 cm
greater than 4 times the length of the                22. The distance by car from Sacramento
shorter side.                                         to San Francisco is 88 miles. The distance
                                                      by car from Sacramento to Cool is 2.2 times
14. The price of a nail is 1.5 dollars                shorter than the distance to San Francisco.
cheaper than one tenth of the price of a              How long is the trip from Sacramento to
hammer.                                               Cool?




                                                 91
 9. TRANSLATION                                                           CHAPTER 1. CONCEPTS
     Homework 1.9 Answers.

1. s is Suzy’s age in years, r is Ron’s age in          15. u is the upload speed in bps, d is the
 years, s = r + 5                                        download speed in bps, u = (2u)/10

3. J is the radius of Jupiter in meters, E is
 the radius of the Earth in meters, J = 11E             17.
                                                         M and I are Margo’s and Ivan’s ages respec-
5. a is the temperature of the air in ◦ F, w             tively, in years,
 is the temperature of the water in ◦ F, a =             equation: I = M + 34
 w − 30                                                  solution: I = 39

7. h is the width of the highway in meters,
 a is the width of the alley in meters, h = 2a          19.
                                                         A and B are Alice’s and Bob’s speeds respec-
9. a is Abby’s height in inches, k is Kurt’s             tively, in mph,
 height in inches, a = k − 7                             equation: A = 1.9B
                                                         solution: A = 8.93
11. J is the length of Jeff’s pool in feet, K is
 the length of Kate’s pool in units of length,
 J = 1.5K                                               21.
                                                         f and s are the prices of the first and the
13. P is the perimeter of the rectangle in               second cars respectively, in dollars,
 cm, x is the length of the shorter side in              equation: f = s/3
 cm, P = 4x + 2                                          solution: f = 2633




                                                   92
CHAPTER 1. CONCEPTS                                                       9. PRACTICE TEST 1
                                          Practice Test 1




1. Identify the terms in the expression              11. Simplify the expression assuming that
            3 + 4x − 5(x + y)                        all variables are non-zero:
                                                                       8x 6x
2. Identify the factors in the expression                                 ÷
                                                                        ya 5y
            −3x( y − 1)( y + 1)
                                                     12. Find the LCM for 10, 16.
3. Use the distributivity to rewrite the ex-
                                                     13. Simplify the expression:
pression without parentheses:
              −3( y − 4x + 1)                                         3 5 3
                                                                       + ·
                                                                      4 6 2
4. Use the distributivity to rewrite the ex-
                                                     14. Evaluate the expression if
pression as a product with 2 factors:
              16x − 12 y − 2                                    a = −1   and b = 7
                                                                    a5
                                                                              ‹
                                                                             b
5. List at least four elements of the given                            ÷ 4−
set using the roster notation:                                      3       3
     
       3 − 2x x is a negative integer                15. Simplify the expression by combining
                                                     like terms:
6. Write the number 36 as a product of                        
                                                                    2
                                                                       ‹
                                                                           1
prime factors.                                               − 3x − y + (20x + 7 y)
                                                                    5      5
7. Rewrite the fraction 20/32 in lowest              16. Describe the variables used to represent
terms.                                               the given quantities, and write the state-
                                                     ments as algebraic relations.
8. Simplify the expression by combining
like terms:                                          The width of a rectangular room is 4 feet
           2(8 − y) − 3( y − 5)                      shorter than its length.

9. Simplify the expression:                          17. Solve the application by describing the
                14 − |5 − 7|                         variables used to represent the quantities of
                 30 − 3 · 23                         interest, and stating appropriate equations.

10. Evaluate the expression if x = −4                Alex runs 1.1 times faster than Beth, who
                  x −5                               runs at 2.8 miles per hour. Find how fast
                 1 − 2x                              Alex runs.




                                                93
 9. PRACTICE TEST 1                                            CHAPTER 1. CONCEPTS
       Practice Test 1 Answers.

1. 3 terms: 3, 4x, −5(x + y)                   12. 80

2. 4 factors: −3, x, ( y − 1), ( y + 1)
                                               13. 2
3. −3 y + 12x − 3
                                               14. −1/5
4. 2(8x − 6 y − 1)
                                                         9
5. {5, 7, 9, 11, . . .}                        15. x +     y
                                                         5
6. 22 · 32
                                               16. w and l and width and length respec-
7. 5/8                                          tively, in feet.
                                                               w= l −4
8. 31 − 5 y

9. 2                                           17. A and B are the respective speeds in
                                                miles per hour.
10. −1
                                                               A = 1.1 · B
    20
11.                                                            A = 3.08
    3a




                                          94
                                          CHAPTER 2


                                     Linear Equations


                                  1. Properties of Equations

   1.1. Equations and Solutions. Recall that an equation is a statement about equality of
two expressions, and it has to be either true or false. When an equation has no variables, we
can simplify the expressions and compare the numbers to find out whether the equations holds.
                                      3(4 − 5)    =     5−8
                                      12 − 15     =     −3
                                            −3    =     −3
So the equation above was true all along.

When an equation contains a variable, it will typically be true for some values of the variable,
and false for some others. Any value for which the equation is true will be known as its solution.

  DEFINITION 1.1.1. A number is called a solution for the equation involving one variable if
  substituting this number for the variable makes the equation true. The set of all solutions
  for the equation is called its solution set. To solve the equation means to find its solution
  set.


  EXAMPLE 1.1.1. Determine if −7 is a solution for the equation
                                           3 − x = 5x


  SOLUTION: Substitute (−7) for x everywhere in the equation and simplify both sides:
                                         3− x     =     5x
                                     3 − (−7)     =     5(−7)
                                         3+7      =     −35
                                            10    =     −35
  The equation is false, so −7 is not a solution.


                                         ANSWER: No

                                                 95
1. PROPERTIES OF EQUATIONS                                      CHAPTER 2. LINEAR EQUATIONS

  EXAMPLE 1.1.2. Determine if 6 is a solution for the equation
                                                 1
                                        w−4= w
                                                 3


  SOLUTION: Substitute 6 for w everywhere in the equation and simplify both sides:
                                                 1
                                    w−4 =          w
                                                 3
                                                        1
                                      (6) − 4      =      (6)
                                                        3
                                              2    =    2
  The equation is true, so 6 is a solution.


                                            ANSWER: Yes


  DEFINITION 1.1.2. Equations are equivalent if they have the same solutions sets.


We will typically solve equations by finding equivalent equations with known solution sets.
For example, simplifying an expression cannot change its value, so any algebraic simplification
produces an equivalent equation.

  EXAMPLE 1.1.3. Solve the equation x = 2(3 − 5)


  SOLUTION: The following equations are equivalent:
                                        x     =    2(3 − 5)
                                        x     =    2(−2)
                                        x     =    −4
  The last equation is true when x is −4, and false otherwise. We will state the answer as a
  set with one element, using the roster notation.


                                        ANSWER: {−4}




                                                  96
CHAPTER 2. LINEAR EQUATIONS                                        1. PROPERTIES OF EQUATIONS
  1.2. Addition Property for Equations.

 THEOREM 1.2.1. Adding the same number or expression to both sides of an equation
 produces an equivalent equation. Formally, for all expressions A, B, and C the following
 two equations are equivalent:
                                              A      =    B
                                              A+ C   =    B+C
 Since every number has an opposite, and subtracting a number means adding its oppo-
 site, we can also subtract the same number on both sides of an equation, and obtain an
 equivalent equation:
                                    A− C = B − C


 EXAMPLE 1.2.1. Solve the equation x − 3 = 10


 SOLUTION: We try to isolate x on the left side by canceling −3:
                   x −3          =       10
               x −3+3            =       10 + 3               added 3 to both sides
                         x       =       13                    combined like terms


                                               ANSWER: {13}


 EXAMPLE 1.2.2. Solve the equation 4x = 3x


 SOLUTION: We don’t know what number 3x stands for, but we know it is a number, and
 every number has an opposite, so we can add −3x to both sides, which is the same as
 subtracting 3x.
                   4x        =       3x
              4x − 3x        =       3x − 3x               added −3x to both sides
                     x       =       0                          combined like terms


                                                ANSWER: {0}




                                                     97
1. PROPERTIES OF EQUATIONS                                  CHAPTER 2. LINEAR EQUATIONS
   1.3. Multiplication Property for Equations.

  THEOREM 1.3.1. If C is an expression denoting a non-zero real number, then multiplying
  both sides of an equation by C produces an equivalent equation. Formally, for all expres-
  sions A and B, and all non-zero numbers C the following two equations are equivalent:
                                         A  = B
                                         AC = BC
  Since every non-zero number has a reciprocal, and dividing by a number means multi-
  plying by its reciprocal, we can also divide both sides by the same non-zero number, and
  obtain an equivalent equation:
                                           A        B
                                               =
                                          C         C


Multiplying both sides of an equation by zero almost guarantees a non-equivalent result. For
example, the equation x = 3 has a single solution, while the equation 0 · x = 0 · 3 is true for
every real number x, and so has infinitely many solutions.

  EXAMPLE 1.3.1. Solve the equation 5x = 75


  SOLUTION: We would like to get rid of 5 on the left so that x is isolated, so we divide
  both sides by 5, which is the same as multiplying both sides by 1/5:
                                         5x   =    75
                                     1             1
                                       (5x)   =      (75)
                                     5             5
                                          x   =    15


                                       ANSWER: {15}


                                     3    6
  EXAMPLE 1.3.2. Solve the equation − x =
                                     7    5


  SOLUTION: We would like to isolate x on the left side, so we need to cancel the coefficient
  −3/7. Multiply both sides by −7/3, which is the reciprocal of −3/7:
                                        3           6
                                      − x =
                                        7           5
                                  7   3               7 6
                                         ‹              ‹
                                −    − x      = −
                                  3   7               3 5

                                              98
CHAPTER 2. LINEAR EQUATIONS                                           1. PROPERTIES OF EQUATIONS
  On the left, reciprocals cancel. On the right we reduce the fraction to lowest terms by
  canceling the common factor 3.
                                                      7 · (2 · 3)
                                        x    =   −
                                                         3·5
                                                      14
                                        x    =   −
                                                      5


                                                  14
                                                §    ª
                                        ANSWER: −
                                                   5


  EXAMPLE 1.3.3. Solve the equation 6x − 4 = 20


  SOLUTION: Addition and multiplication properties may be exploited any number of times
  and in any order. Here we can isolate x on the left side if we add 4 first, and then divide
  both sides by 6.
                    6x − 4    =   20
                6x − 4 + 4    =   20 + 4                              addition property
                        6x    =   24                                combined like terms

                        6x         24
                              =                               multiplication property
                         6          6
                         x    =   4


                                            ANSWER: {4}



    1.4. Applications. We will solve applications by describing the variables used to represent
the quantities of interest, and stating appropriate equations. As you construct your own solu-
tions to applications, you may be using different variable names, and your equations may also
be somewhat different, depending on how you translate English sentences. The final numerical
answers, however, should always come out the same, regardless of translation.

  EXAMPLE 1.4.1. The profit of ACME Corporation in the second quarter was 8.84 billion
  dollars, which was 1.3 times greater than the profit in the first quarter. What was ACME’s
  profit in the first quarter?




                                                 99
1. PROPERTIES OF EQUATIONS                                       CHAPTER 2. LINEAR EQUATIONS

  SOLUTION: Let f and s be the profits in the first and second quarter respectively, measured
  in billions of dollars. Then we can translate the statement above as “s is 1.3 times greater
  than f ” or
                                            s = 1.3 f
  Since s = 8.84, we can substitute it and solve the resulting equation for f :
                                        8.84    =        1.3 f

                                        8.84             1.3 f
                                                =
                                         1.3              1.3
                                         6.8    =        f


                                          ANSWER:
    f and s are the first and the second quarter profits respectively, in billions of dollars,
                                      equation: s = 1.3 f
                                       solution: f = 6.8


  EXAMPLE 1.4.2. The width of a rectangle is 125 meters shorter than the length. Find the
  length if the width is 205 meters.


  SOLUTION: Let w and l be the width and the length respectively, measured in meters. Then
  the statement “The width of a rectangle is 125 meters shorter than the length” translates
  as “w is 125 meters less than l” or
                                        w = l − 125
  Since w = 205, we can substitute it and solve the resulting equation for l:
                                       205     =     l − 125
                                205 + 125      =     l − 125 + 125
                                       330     =     l


                                        ANSWER:
                w and l are the width and the length respectively, in meters,
                                   equation: w = l − 125
                                     solution: l = 330




                                               100
CHAPTER 2. LINEAR EQUATIONS                                    1. PROPERTIES OF EQUATIONS
      Homework 2.1.

Determine whether the given set is a possi-                    1    2
                                                     21. a −     =−
ble solution set for the given equation.                       6    3

1. {4},                        5x + 7 = 29                   x   2
                                                     22. −     =
                                                             6   9
2. {8},                         6 − x = −2
                                                               2    5
                                                     23. x −     =−
3. {−2},                       x +7=3− x                       3    6
                                           n              2        3
4. {4},                         −3 = 5 −             24. − + y = −
                                           2              3        4

5. {17},               5x − 10 = 5(x − 2)                  3
                                                     25.     x = 18
                                                           4
6. {−100},             5x − 10 = 5(x − 2)
                                                     26. −8.2x = 20.5
7. {0},                  x − 4x = 3x + 1
                           2
                                                     27. −6 = y + 25
8. {0},                   (1 − x) = x + 1
                                  2
                                                     28. −6 = x + 9
9. {−1, 9},                     |x − 4| = 5
                                                     29. x − 4 = −19
10. {−3},                        |3x| = −9
                                                     30. t − 7.4 = −12.9

                                                     31. −x = 28
Solve equations and check solutions.
                                                     32. −t = −8
11. 7x = 56
                                                     33. 12 = −7 + y
12. 12x = 72
                                                               1 8
                                                     34. x +    =
13. x + 21 = 10                                                3 3

14. x + 12 = −7                                           3      3
                                                     35. − y = −
                                                          5      5
15. 4.5 + x = −3.1
                                                          2       4
                                                     36. − x = −
16. −15x = −20                                            5      15
                                                     37. m − 2.8 = 6.3
17. 400 = −x
                                                     38. y − 5.3 = 8.7
18. −2x = −300
                                                           4
19. −7x = 49                                         39.     x = 16
                                                           5
       y
20.      = 11
       8
                                               101
1. PROPERTIES OF EQUATIONS                                    CHAPTER 2. LINEAR EQUATIONS
      3                                               number of US astronauts. Find the num-
40.     x = 27
      4                                               ber of ISS crew members from the United
                                                      States.
41. 2x + 3 = 13
                                                      51. According to NASA, the global land-
42. 3x − 1 = 26
                                                      ocean temperature index (average surface
                                                      temperature of Earth) in 2016 is 1.26 ◦ C
43. 2t + 9 = 43
                                                      greater than it was in 1920. Find the aver-
44. −5 y + 7 = −18                                    age temperature in 1920 if it was 0.99 ◦ C
                                                      in 2016.
45. 84 = 7x − 7
                                                      52. When Kyle went to Europe, his ticket
46. 50 = 9t + 1                                       to Berlin was 1.25 times cheaper than the
                                                      return ticket to New York. How much was
                                                      the return ticket if the ticket to Berlin cost
                                                      $480?
Solve applications by describing the vari-
ables used to represent the quantities of in-         53. Damian’s age is one year less than
terest, and stating appropriate equations.            twice the Colin’s age. How old is Colin, if
                                                      Damian is 17 years old?
47. The payout for playing a single num-
ber on a roulette table is 35 times greater           54. Mary spends 14 minutes on her morn-
than the bet. Cindy receives the payout of            ing commute, which is 5 minutes shorter
$3850. How much was Cindy’s bet?                      than three times the duration of her
                                                      evening commute. Find how much time
48. A croissant has 53 more calories than             she spends on her evening commute.
a donut. Find the amount of calories in one
donut if each croissant has 445 calories.             55. Bob says to Alice: if you give me 3 ap-
                                                      ples and then take half of my apples away,
49. A history textbook costs $90, which is            then I will be left with 13 apples. How
five times cheaper than an anatomy text-              many apples do I have now?
book. Find the price of the anatomy text.
                                                      56. Charlie says to Diane: if I give you 5 of
50. As of July 28, 2017 Russia contributed            my apples and then also half of the remain-
39 crew members for the International                 ing amount, then I will have 10 apples left.
Space Station, which is 12 fewer than the             How many apples do I have now?




                                                102
CHAPTER 2. LINEAR EQUATIONS                    1. PROPERTIES OF EQUATIONS
    Homework 2.1 Answers.

1. No                               39. {20}

3. Yes                              41. {5}

5. Yes                              43. {17}

7. No                               45. {13}

9. Yes                              47.
                                     b and p are the bet and the payout amounts
11. {8}                              respectively, in dollars,
                                     equation: p = 35b
13. {−11}                            solution: b = 110

15. {−7.6}                          49.
                                     h and a are the prices of the history text
17. {−400}                           and the anatomy text respectively, in dol-
                                     lars,
19. {−7}                                            a
                                     equation: h =
                                                    5
   § ª
      1                              solution: a = 450
21. −
      2
                                    51.
      1                              x and y are the 1920 and the 2016 tem-
   § ª
23. −                                peratures respectively, in ◦ C,
      6
                                     equation: y = x + 1.26
25. {24}                             solution: x = −0.27

27. {−31}                           53.
                                     D and C are the ages of Damian and Colin
29. {−15}                            respectively, in years,
                                     equation: D = (2C) − 1
31. {−28}                            solution: C = 9

33. {19}                            55.
                                     b and a are the amounts of apples Bob has
35. {1}                              before and after the exchange respectively,
                                     equation: (b + 3)/2 = a
37. {9.1}                            solution: b = 23




                              103
2. SOLVING LINEAR EQUATIONS                                   CHAPTER 2. LINEAR EQUATIONS
                                2. Solving Linear Equations
   2.1. Linear and Non-linear Equations.

  DEFINITION 2.1.1. A linear equation is an equation in which each term is either a numerical
  constant or the product of a numerical constant and the first power of a single variable.


  BASIC EXAMPLE 2.1.1. Here are some linear and some non-linear equations.

  A linear equation in one variable x:
                                         3 + x = 4x − 5
  A linear equation in two variables x and y:
                                             y = 4x − 17
  A linear equation in three variables x, y, and z:
                                       1
                                         x − 5 y + z = 14
                                       2
  The following equations are non-linear because they have non-linear terms, highlighted
  below. This one has a second power of a variable, when only first power is allowed:
                                             1 − x2 = 4x
  This one has a term with multiple variable factors, when at most one is allowed:
                                         5z = 3xy + 7x
  This one has an absolute value term, which is neither a numerical constant nor a variable
  product:
                                       |x − 4| = 2x + 7
  This equation is non-linear, but a few algebraic operations can show it is equivalent to a
  linear equation:
                    2(4 − x)    =    3(x + 5)
                      8 − 2x    =    3x + 15                      distributed
  This equation is also non-linear, but amazingly it is also equivalent to a linear equation:
                5x2 + 1    =   2x(1 + 2x) + x2
                5x2 + 1    =   2x + 4x2 + x2                                 distributed
                5x2 + 1    =   2x + 5x2                            combined like terms
          5x + 1 − 5x
            2          2
                           =   2x + 5x − 5x
                                         2      2
                                                           subtracted 5x 2 on both sides
                      1    =   2x                                  combined like terms




                                                104
CHAPTER 2. LINEAR EQUATIONS                                    2. SOLVING LINEAR EQUATIONS
    2.2. Isolating The Variable. Here we present a procedure for solving linear equations as
well as many types of non-linear equations, as long as they turn out to be equivalent to linear
equations. It is called isolating the variable because it boils down to finding an equivalent
(Definition 1.1.2) equation of the form
                                               x=c
where x is the unknown variable and c is a number. Here x is isolated on the left side in the
sense that it is there all alone (no other terms, no coefficient), and there is no x on the other
side. Even though it is traditional to isolate x on the left side, there is nothing wrong with
isolating it on the right: c = x.

  THEOREM 2.2.1 (Isolating The Variable). The following procedure will isolate the variable
  x in linear and some non-linear equations.

       (1) If the equation contains fractions, multiply both sides by the LCD. Cancel com-
           mon factors in each fraction to get rid of denominators.
       (2) Use the distributive property to get rid of parentheses, and then combine the like
           terms on both sides.
       (3) Use the addition property to cancel the variable terms on one side and the con-
           stant terms on the other side of the equation, and combine the like terms again.

  By now the equation should have assumed one of the three forms:

          • If the equation looks like ax = b then divide both sides of the equation by the
            coefficient a. Now it looks like x = c and c is the unique solution. The set of
            solutions can be written as {c}.
          • If the equation looks like 0 = 4 or something equally wrong, then it is false for
            all x. There are no solutions: the set of solutions is empty, or ∅.
          • If the equation looks like 0 = 0, then it is true for all x. Every real number is a
            solution: the set of solutions is the set of all reals, or R.


When a unique solution exists, it can be checked by substituting it for the variable in the original
equation and making sure the equation holds. When the solution set is R, it can be partially
checked by substituting a few random numbers for the variable and making sure the equation
holds for all of them.

  EXAMPLE 2.2.1. Solve the equation
                                       3(x − 3) + 1 = 4 + x


  SOLUTION: No fractions here, so we start by removing parentheses and combining the
  like terms:
                     3x − 9 + 1    =    4+ x                     distributivity
                         3x − 8    =    4+ x

                                               105
2. SOLVING LINEAR EQUATIONS                                             CHAPTER 2. LINEAR EQUATIONS
  Then use the addition property of the equation to cancel the terms with x on one side,
  and the constants on the other side, and combine the like terms again:
                                3x − 8 − x + 8        =       4+ x − x +8
                                            2x        =       12
  Finally, we divide both sides by the coefficient of the variable term and obtain a solution:
                                         2x          12
                                                =
                                          2           2
                                             x    =       6
  To check the answer, we substitute 6 for x in the original equation and simplify:
                                      3(x − 3) + 1        =        4+ x
                                      3(6 − 3) + 1        =        4+6
                                         3(3) + 1         =        10
                                                 10       =        10
  The equation holds, so 6 is a solution.


                                           ANSWER: {6}


  EXAMPLE 2.2.2. Solve the equation
                                    4x − (3 + x) = −3(1 − x)


  SOLUTION: No fractions here, so we remove the parentheses and combine the like terms:
                   4x − 3 − x     =     −3 + 3x                           distributivity
                       3x − 3     =     −3 + 3x
  Now we cancel variable terms on one side and constants on the other side, and combine
  the like terms again:
                           3x − 3 + 3 − 3x        =       −3 + 3x + 3 − 3x
                                            0     =       0
  This equation is true for all values of x. In other words, any real number makes the
  equation true when substituted for x.


                                            ANSWER: R




                                                 106
CHAPTER 2. LINEAR EQUATIONS                                   2. SOLVING LINEAR EQUATIONS

 EXAMPLE 2.2.3. Solve the equation
                                 2(7 + 5x) = 1 − (3 − 10x)


 SOLUTION: No fractions here, so we start by removing parentheses and combining the
 like terms:
                               14 + 10x      =     1 − 3 + 10x
                               14 + 10x      =     −2 + 10x
 After that we use the addition property to eliminate variable terms on one side and con-
 stants on the other side, and combine the like terms again:
                    14 + 10x − 10x − 14      =      −2 + 10x − 10x − 14
                                         0   =      −16
 This equation is false for all values of x. In other words, no real number can make the
 original equation true. There are no solutions.


                                         ANSWER: ∅


 EXAMPLE 2.2.4. Solve the equation
                                     1      2 1
                                       x +4= − x
                                     3      3 2


 SOLUTION: Here the LCD is 6, so we start by multiplying both sides by the LCD:
                                      1           2 1
                                           ‹         ‹
                                  6     x +4 =6    − x
                                      3           3 2
 As we distribute, the fractions cancel:
                                 1                 2     1
                              6· x +6·4 = 6· −6· x
                                 3                 3     2

                                  2x + 24    =     4 − 3x
 Now we eliminate variable terms on one side and constants on the other side, combine
 the like terms, and use the multiplication property to isolate the x variable:
        2x + 24 + 3x − 24    =     4 − 3x + 3x − 24
                       5x    =     −20
                       5x          −20
                             =                                 divided both sides by 5
                        5           5
                         x   =     −4

                                             107
2. SOLVING LINEAR EQUATIONS                               CHAPTER 2. LINEAR EQUATIONS
  To check the answer, we substitute −4 for x in the original equation and simplify:
                 1             2 1
                   x +4 =         − x
                 3             3 2
             1                  2 1
               · (−4) + 4   =    − · (−4)
             3                  3 2
                    −4          2
                       +4   =     +2
                    3           3
                −4 12           2 6
                   +        =    +                     find LCD to add fractions
                 3   3          3 3
                       8        8
                          =
                       3        3
  The equation holds, so −4 is a solution.


                                       ANSWER: {−4}


  EXAMPLE 2.2.5. Solve the equation
                                  41 5       2    1
                                              ‹
                                    =     x+     − x
                                  9   2      3    3


  SOLUTION: Here we have some fractions locked up inside the parentheses, so let us
  distribute before we try to figure out the LCD:
                                   41         5      2    1
                                                      ‹
                                        =         x+     − x
                                    9         2      3    3
                                 41          5   5 2 1
                                         =     x+ · − x
                                 9           2   2 3 3
                                41         5     10 1
                                      =      x+     − x
                                 9         2      6   3
  The LCD is 18, so we multiply both sides of the equation by 18 and simplify until all the
  denominators cancel:
            41             5      10 1
           ‹                            ‹
       18          = 18      x+      − x                 multiply both sides by LCD
            9              2       6   3
          18 · 41        18 · 5    18 · 10 18 · 1
                     =          x+        −       x      distributed on the right side
            9              2         6       3
              82     =   45x + 30 − 6x                            simplified fractions
              82     =   39x + 30                               combined like terms


                                             108
CHAPTER 2. LINEAR EQUATIONS                               2. SOLVING LINEAR EQUATIONS
 Now we finish solving by isolating the variable:
                                    82   =     39x + 30
                               82 − 30   =     39x + 30 − 30
                                    52   =     39x
                                    52         39x
                                         =
                                    39          39
                                     4
                                         = x
                                     3
 We can check the solution by substituting it into the original equation and making sure it
 holds:
                               41        5        2     1
                                                   ‹
                                     =        x+      − x
                                9        2        3     3
                               41        5 4 2           1 4
                                                   ‹
                                     =          +     − ·
                                9        2 3 3           3 3
                               41        5 6        4
                                             ‹
                                     =            −
                                9        2 3        9
                               41         30 4
                                     =      −
                                9         6   9
                               41              4
                                     =    5−
                                9              9
                               41         45 4
                                     =      −
                                9         9   9
                               41         41
                                     =
                                9         9


                                              4
                                             § ª
                                     ANSWER:
                                              3




                                             109
2. SOLVING LINEAR EQUATIONS                                  CHAPTER 2. LINEAR EQUATIONS
    2.3. Applications. We will solve applications by describing the variables used to represent
the quantities of interest, and stating appropriate equations. As you construct your own solu-
tions to applications, you may be using different variable names, and your equations may also
be somewhat different, depending on how you translate English sentences. The final numerical
answers, however, should always come out the same, regardless of translation.

  EXAMPLE 2.3.1. Alice works two jobs. Her full-time job pays four times more than her
  part-time job, and together they pay 3800 dollars per month. Find how much each job
  pays.


  SOLUTION: Let p be the amount the part-time job pays and let f be the amount the
  full-time job pays in dollars per month. Let us translate the given statements:
                      full-time job pays 4 times more than part-time job
                                             f = 4p
  And the statement ”together both jobs pay $3800” can be written as
                      full-time pay + part-time pay         =     total pay
                             f      +        p              =     3800
  Since f = 4p, we can substitute (4p) for f into the second equation and obtain a linear
  equation in one variable:
                     f +p    =    3800
                 (4p) + p    =    3800                    substitute (4p) for f
                       5p    =    3800                      combine like terms
                       5p         3800
                             =                          multiplication property
                       5            5
                         p   =    760
  Finally, we substitute 760 for p in the first equation to find that
                                    f = 4p = 4(760) = 3040
  To check the answer, we can verify that the two paychecks add up to the stated total:
                                        760 + 3040 = 3800


                                         ANSWER:
              p and f are part-time and full-time salaries respectively, in dollars
                                         f = 4p
                                      

                                         f + p = 3800
                                 solution: p = 760,    f = 3040


                                              110
CHAPTER 2. LINEAR EQUATIONS                                    2. SOLVING LINEAR EQUATIONS

 EXAMPLE 2.3.2. Rita bought rice and truffles a grocery store, and she paid $67.5 more for
 truffles than she paid for the rice. Find the price of each item if the total bill was $72.


 SOLUTION: Let r and t be the prices in dollars of the rice and the truffles respectively. It
 may be helpful to rephrase given statements to make them easier to translate:
                 she paid $67.5 more for truffles than she paid for the rice
              the price of truffles is 67.5 dollars greater than the price of rice
                                        t = r + 67.5
 And the other statement
                                     the total bill was $72
 can be translated as
                                         r + t = 72
 Since t = r + 67.5, we can substitute (r + 67.5) for t in the last equation, and then solve
 for r:
                        r+t      =    72
               r + (r + 67.5)    =    72
                   2r + 67.5     =    72                        combined like terms
            2r + 67.5 − 67.5     =    72 − 67.5                     addition property
                           2r    =    4.5                       combined like terms
                           2r         4.5
                                 =                            multiplication property
                           2           2
                            r    =    2.25
 Finally we use the first equation find that
                                      t     =   r + 67.5
                                      t     =   2.25 + 67.5
                                      t     =   69.75


                                         ANSWER:
              r and t are the prices of rice and truffles respectively, in dollars
                                          t = r + 67.5
                                      

                                          r + t = 72
                                solution: r = 2.25,     t = 69.75




                                                111
2. SOLVING LINEAR EQUATIONS                                 CHAPTER 2. LINEAR EQUATIONS

  EXAMPLE 2.3.3. Alice, Bob, and Charlie stand to inherit 12000 dollars, and the will stipu-
  lates that the inheritance is to be split in such a way that Bob gets two times more money
  than Alice, while Charlie should get 1600 dollars less than Alice. Find the inheritance
  amount for each person.


  SOLUTION: Let a, b, and c be the amounts in dollars inherited by Alice, Bob, and Charlie
  respectively. Bob gets twice as much as Alice, which we can write as
                                            b = 2a
  Charlie gets 1600 dollars less than Alice, which we can write as
                                         c = a − 1600
  Together, their amounts should add up to the total, so we can write this equation:
        Alice’s share + Bob’s share + Charlie’s share           =       total inheritance
              a       +      b      +        c                  =             12000
              a       +    (2a)     +  (a − 1600)               =             12000
  This is a linear equation in one variable, so we can solve it the usual way. There are no
  fractions here, so we start by removing parentheses and combining the like terms:
          a + 2a + a − 1600    =    12000                        remove parentheses
                  4a − 1600    =    12000                        combined like terms
                          4a   =    12000 + 1600            added 1600 on both sides
                          4a   =    13600                        combined like terms

                         4a         13600
                               =                              multiplication property
                         4            4
                           a   =    3400
  So Alice’s share is 3400 dollars, Bob’s share is 2a = 2 · 3400 = 6800 dollars, and Charlie’s
  share is (a − 1600) = 3400 − 1600 = 1800 dollars. We can see that the original equation
  holds, since the individual shares add up to the total:
                                3400 + 6800 + 1800 = 12000


                                        ANSWER:
   a, b, and c are inheritance amounts in dollars for Alice, Bob, and Charlie respectively,

                                   b = 2a
                                  

                                     c = a − 1600
                                     a + b + c = 12000
                                  

                         solution: a = 3400,    b = 6800,    c = 1800


                                               112
CHAPTER 2. LINEAR EQUATIONS                                  2. SOLVING LINEAR EQUATIONS

 EXAMPLE 2.3.4. Katy invested some money into bonds, and the amount of interest earned
 after one year was 25 times smaller than the original investment. Find the interest and
 original investment if the total amount in her account is $546.


 SOLUTION: Let P be the investment (also called principal) and let I be the interest, in
 dollars. The interest is 25 times smaller:
                                          I = P ÷ 25
 The total amount, which is the principal plus the interest, is $546:
                                          P + I = 546
 Substituting P ÷ 25 for I in the second equation gives us an equation in one variable we
 can solve:
                    P+I      =    546
                     P
                  P+         =    546                   multiply both sides by LCD
                    25
                     ‹
                   P
            25 P +           =    25(546)                    to get rid of fractions
                   25
                      25P
              25P +          =    25(546)                                distributed
                       25
                 25P + P     =    25(546)                         simplified fraction

                      26P    =    25(546)                     combined like terms

                      26P         25(546)
                             =                             multiplication property
                       26           26
                        P    =    525
 Finally, substituting 525 for P in the first equation gives us
                                        I = P ÷ 25 = 21


                                         ANSWER:
              P and I are the principal and the interest respectively, in dollars
                                         I = P ÷ 25
                                      

                                         P + I = 546
                                 solution: P = 525,     I = 21




                                             113
2. SOLVING LINEAR EQUATIONS                        CHAPTER 2. LINEAR EQUATIONS
      Homework 2.2.

Solve each equation:                             3           21
                                           22.     (1 + p) =
                                                 5           20
1. 4x = 6x − 28
                                           23. 2( y − 3.5) = −2(14 − y)
2. −2 = −2m + 12
                                                      9    11
                                           24. 2b +     =−
3. −2z − 10 = 5z + 18                                 5     5
         n                                     2x − 17 17 − 2x
4. 5 +     =4                              25.        =
         4                                        5        10
5. 0.2x − 0.1 = 0.6x − 2.1                      5 5        3
                                                            ‹
                                           26. − =      r−
                x                               8 4        2
6. −11 = −8 +
                2                                7    3
                                           27.     z + = −z
   x x x                                         5    5
7.  − = +1
   2 3 6
                                           28. −2(1 − 7p) = 8(p − 7)
8. 2 − (−3a − 8) = 1
                                           29. 7( y + 3) − 5 = 11 y − 4(1 + y)
9. −5(4x − 3) + 2 = −20x + 17
                                           30. −8n − 19 = −2(8n − 3) + 3n
10. 66 = 6(6 + 5x)
                                                 3(z − 5) 2z + 10
    1.37 − x                               31.           =
11.          − 1.29 = −1                            2        3
       4
                                                 45 3  7   19
                                           32.     + n= n−
12. −16n + 12 = 39 − 7n                          16 2  4   16

13. 3( y − 1) = y + 6                      33. 0.25(6t + 2) − 10 = 3t − 7.1

14. −(3 − 5n) = 12                         34. −76 = 5(1 + 3b) + 3(3b − 3)

15. −4x + 10 = −2(3x + 1)                        0.6x     6 − 0.15x
                                           35.        +2=
                                                 −12          3
16. 2(4x − 4) = −20 − 4x
                                               11 3      163
                                           36.    + r=
                    15 − 20x                    4   4     32
17. −(4x − 3) =
                        5
                                               1    29       4    2
                                                                   ‹
                                           37. n +     =2      n+
18. −(n + 8) + n = −8n + 2(4n − 4)             3     6       3    3

19. 0.4x + 0.2(10 − x) = 3                       1 2     3     7      83
                                                          ‹
                                           38. −      x−     − x =−
                                                 2 3     4     2      24
20. −6(8k + 4) = −8(6k + 3) − 2

      z+2
21.       = 2z + 3
       3
                                     114
CHAPTER 2. LINEAR EQUATIONS                                   2. SOLVING LINEAR EQUATIONS
Solve applications by describing the vari-            was invested and how much interest was
ables used to represent the quantities of in-         earned if the total amount in the account
terest, and stating appropriate equations.            is $1, 785

39. An éclair has five times as many calo-            45. A monthly pass for light rail is 1.2
ries as a latte, and together they have 264           times cheaper than the a bundle with 20
calories. Find how many calories each item            day passes. Find the price of the monthly
has.                                                  pass and the price of the bundle, if the bun-
                                                      dle costs $10 more than the monthly pass.
40. Cassie has 74 more stickers than
Margo, and together they have 400 stick-              46. Annie made $270 more in tips in June
ers. Find how many stickers each person               than she did in May. Find how much An-
has.                                                  nie made in tips in either month if the
                                                      amount in June was 1.5 times greater than
41. The width of a rectangular parking lot            the amount in May.
is 40 feet shorter than its length. Find the
dimensions of the lot if its perimeter is 640         47. A bread recipe calls for a mix of
feet.                                                 bleached flour, whole wheat flour, and rye
                                                      flour. The volume of whole wheat should
42. The width of a rectangular computer               be two times greater than the volume of
screen is 16/9 times greater than its height.         bleached, and the volume of rye should be
Find the height of the screen if the width is         1 cup less than the volume of bleached.
1920 pixels.                                          Find the volume in cups for each type of
                                                      flour if the recipe is asking for 15 cups of
43. Consider an astronomical triangle                 flour in total.
with sides a, b, and c, such that the length
of the side b is 3 light-years longer the             48. Three paper supply company employ-
length of a, and the length of c is 6 times           ees: Jim, Pam, and Leslie, made a bet about
shorter than the length of a. Find the                who can make the most sales in a month.
length of each side if the perimeter of the           Jim won the bet, with Pam making 6 fewer
triangle is 55 light-years.                           sales than Jim, and Leslie making 7 fewer
                                                      sales than Jim. Find how many sales each
44. The interest earned by John’s savings             employee has made, if the total amount of
account in 2017 is 50 times smaller than              sales for the month was 113.
his original investment. Find how much




                                                115
2. SOLVING LINEAR EQUATIONS                 CHAPTER 2. LINEAR EQUATIONS
    Homework 2.2 Answers.

1. {14}                             39.
                                     L and E are calorie amounts in a latte and
3. {−4}                              an éclair respectively
                                        E = 5L
                                     
5. {5}                                  E + L = 264

7. ∅                                 solution: L = 44,    E = 220

                                    41.
9. R
                                     w and l are width and length respectively,
                                     in feet
11. {0.21}
                                        w = l − 40
                                     

     9                                  2w + 2l = 640
    § ª
13.
     2                               solution: w = 140,    l = 180

15. {−6}                            43.
                                     a, b, and c are the side lengths in light-
17. R                                years,

                                      b = a+3
                                     
19. {5}
                                        c = a÷6
                                        a + b + c = 55
                                     
      7
   § ª
21. −
      5                              solution: a = 24,    b = 27,    c=4

23. ∅                               45.
                                     m and b the prices of the monthly pass and
25. {17/2}                           the bundle respectively, in dollars
                                        m = b ÷ 1.2
                                     
      1
   § ª
27. −                                   b = m + 10
      4
                                     solution: m = 50,    b = 60
29. ∅
                                    47.
31. {13}                             b, w, and r are the amounts of bleached,
                                     whole wheat, and rye flour respectively, in
33. {−1.6}                           cups,

                                      w = 2b
                                     

35. R                                   r = b−1
                                        b + w + r = 15
                                     
     3
    § ª
37.
     2                               solution: b = 4,    w = 8,     r =3

                              116
CHAPTER 2. LINEAR EQUATIONS                                            3. LINEAR FORMULAS
                                    3. Linear Formulas
  3.1. Isolating a Variable.

 DEFINITION 3.1.1 (Formula). While the word formula has no traditional formal definition,
 this will be our word of choice for equations in more than one variable.


 BASIC EXAMPLE 3.1.1. Some formulas useful in applications:
                                         P = 2l + 2w
 is a formula for the perimeter P of a rectangle with length l and width w, and
                                           V = lwh
 is a formula for the volume V of a box with length l, width w, and height h.


 DEFINITION 3.1.2. We say that a formula involving a variable y is solved for y if it is
 written in the form
                                     y = expression
 where one side of the equation is just y, and the expression on the other side does not
 contain y at all.


 BASIC EXAMPLE 3.1.2. Here are some formulas which are solved for a variable, and some
 that are not.
         E    =   mc 2                                                         solved for E
         x    =   10 y 3 − a b                                                 solved for x

                   4x 2 − 3x
         y    =              +1                                                solved for y
                       5a
         y    =   7(4x − yz)        not solved for y because y appears on the right side
        y2    =    x2 + 3              not solved for y because neither side is y by itself
    x + 40    =    y +4                not solved for x because neither side is x by itself


 EXAMPLE 3.1.1. Solve the formula for x:
                                      6 y − 7x − 10 = 0


 SOLUTION: We will isolate the variable x by following the same procedure (Theorem
 2.2.1) we used to solve linear equations. We just have to pretend that x is the only variable.
 There are no parentheses, so we use the addition property to isolate the terms with x on


                                             117
3. LINEAR FORMULAS                                                   CHAPTER 2. LINEAR EQUATIONS
  one side:
                       6 y − 7x − 10     =      0
           6 y − 7x − 10 − 6 y + 10      =      0 − 6 y + 10              addition property
                                −7x      =      −6 y + 10               combined like terms
  Finally, we divide by the coefficient of the term with x:
                                     −7x          −6 y + 10
                                             =
                                      −7             −7
                                                        −6 y + 10
                                           x     =
                                                           −7


                                                          −6 y + 10
                                    ANSWER: x =
                                                             −7


Note that the answer to the previous example is an equation, not a number. Note also that the
answer is not unique. In a solved formula such as
                                              −6 y + 10
                                          x=
                                                  −7
the right side can be stated in a variety of ways, all equivalent, and all correct:
                               −6 y + 10 10 − 6 y        6 y 10
                           x=              =           =     −     = ...
                                   −7          −7         7     7

  EXAMPLE 3.1.2. Solve the formula for z:
                                         5z − x = 3(4 − z)


  SOLUTION: z is locked up in parentheses on the right side, so use distributivity to get rid
  of these parentheses first:
                                       5z − x       =     3(4 − z)
                                       5z − x       =     12 − 3z
  Use the addition property to isolate terms with z on one side, and then combine like terms:
              5z − x + x + 3z   =      12 − 3z + x + 3z                  addition property
                          8z    =      12 + x                          combined like terms




                                                    118
CHAPTER 2. LINEAR EQUATIONS                                         3. LINEAR FORMULAS
 Finally, divide by the coefficient of the term with z:
                                        8z        12 + x
                                              =
                                         8           8
                                                   12 + x
                                        z   =
                                                     8


                                                    12 + x
                                   ANSWER: z =
                                                      8


 EXAMPLE 3.1.3. Solve the photon energy formula for h:
                                            hc
                                       E=
                                            λ


 SOLUTION: There is only one term with h, and it is already isolated on the right side, so
 we just need to use the multiplication property to get rid of the extra factors. Multiply
 both sides by the reciprocal of each factor on the right side besides h:
                         hc
                E =
                          λ
                λ          hc λ
                          ‹
            (E)      =
                c          λ c
              Eλ         hcλ
                    =                        cancel common factors on the right
               c         λc
              Eλ
                    =    h
               c


                                                      Eλ
                                     ANSWER: h =
                                                       c


 EXAMPLE 3.1.4. Solve the formula for X :
                                       aX = b(X − c)


 SOLUTION: The variable X is locked up inside the parentheses on the right side, so we
 distribute first:
                                     aX     =   bX − bc



                                             119
3. LINEAR FORMULAS                                            CHAPTER 2. LINEAR EQUATIONS
 Now we use the addition property to isolate the terms with X on the same side:
              aX − bX       =       bX − bc − bX                addition property
              aX − bX       =       −bc                       combined like terms
 We would like to combine the terms with X , but they are not, strictly speaking, similar,
 having different variable factors. We can, however, distribute and then multiply by the
 reciprocal of the resulting factor anyway:
                X (a − b)       =     −bc                          distributivity

                X (a − b)             −bc
                                =                        assuming (a − b) 6= 0
                  a−b                 a−b
                                      −bc
                        X       =
                                      a−b


                                                        −bc
                                          ANSWER: X =
                                                        a−b




                                                120
CHAPTER 2. LINEAR EQUATIONS                                           3. LINEAR FORMULAS
     Homework 2.3.

Solve the formula for the given variable           15. 4x − 5 y = 8                 for y
assuming that all variable expressions in-
volved are non-zero.                                      5
                                                   16. C = (F − 32)                 for F
                                                          9
1. a b = c                           for b
                                                   17. 4x − 7b = 4                  for b
       h
2. g =                               for h         18. 4x − 5 y = 8                 for x
       i
3. a + c = b                         for c               1     c
                                                   19.     +b=                      for b
                                                         a     a
4. x − f = g                         for x
                                                   20. lwh = V                      for w
5. P = n(p − c)                      for n
                                                             πr 2 h
                                                   21. V =                          for h
6. S = L − 2B                        for L                    3
                                                   22. r t = d                      for r
7. 2m + p = 4m + q                  for m
                                                   23. ax + b = c                   for a
8. q = 6(L − p)                      for L

     f                                             24. R = aT + b                   for T
9.     x=b                           for x
     g                                                   ym   c
                                                   25.      =                       for y
                                                          b   d
          3y
10. p =                              for y
           q                                       26. E = mc 2                    for m
                                                              4y
11. h = v t − 16t 2                  for v         27. c =                          for y
                                                             m+n
12. S = πrh + πr 2                   for h                rs
                                                   28.       =k                     for r
                                                         a−3
13. 3x + 2 y = 7                     for y
                                                   29. 4x − y = 5(x + y) − 1        for x
14. 5a − 7b = 4                      for a
                                                   30. 10(a + b) = 6(a − 2b)        for a




                                             121
3. LINEAR FORMULAS                          CHAPTER 2. LINEAR EQUATIONS
    Homework 2.3 Answers.
         c                                  4 − 4x
1. b =                            17. b =
         a                                    −7
3. c = b − a
                                            c 1
        P                         19. b =    −
5. n =                                      a a
       p−c
                                            3V
       p−q                        21. h =
7. m =                                      πr 2
        2
         bg                                 c−b
9. x =                            23. a =
          f                                  x

          h + 16t 2                         cb
11. v =                           25. y =
              t                             dm
             7 − 3x
13. y =                                     c(m + n)
                2                 27. y =
                                               4
             8 − 4x
15. y =                           29. x = 1 − 6 y
               −5




                            122
CHAPTER 2. LINEAR EQUATIONS                                                     4. PERCENT
                                          4. Percent
   4.1. Percent Units and Notation.

  DEFINITION 4.1.1. 1 percent, written also as 1%, is one hundredth of a unit, or 0.01 in
  decimal representation. The sentence “a part is k% of the whole” or
                                          p is k% of w
  can be written as
                                                       k
                                          p=              ·w
                                                      100

  where p is the part of the whole w, and k/100 is the decimal representation of k%.


  BASIC EXAMPLE 4.1.1. The decimal representation of k percent, which is k/100, may be
  useful in equations because it is easier to manipulate algebraically. To convert k% into a
  decimal form, simply divide k by 100. To convert the decimal into percent, multiply it by
  100. For example,
                            k percent decimal representation
                                3%                 0.03
                               35%                 0.35
                              150%                  1.5
                              0.6%                0.006


There are three easy question types which can be translated into the percent equation in terms
of the unknown variable x.

Finding the part, knowing the whole and the percentage:

                                  What is             k%    of w?
                                                       k
                                     x        =                ·       w
                                                      100
Finding the percentage, knowing the whole and the part:

                                   p is what % of w?
                                          x
                                   p =          · w
                                         100
Finding the whole, knowing the percentage and the part:

                              p is       k%   of what number?
                                      k
                              p =                 ·                x
                                     100
                                                  123
4. PERCENT                                                      CHAPTER 2. LINEAR EQUATIONS

 EXAMPLE 4.1.1. What is 14% of 50?


 SOLUTION: This translates as
                                           14
                                         x=   · 50 = 7
                                          100
 Alternatively, we can convert the percent into a decimal form and then solve a similar
 equation where the fraction is replaced by a decimal:
                                         x = 0.14 · 50 = 7


                                             ANSWER: 7


 EXAMPLE 4.1.2. The number 21 is what percent of 70?


 SOLUTION: This question translates as
                                                     x
                                             21 =       · 70
                                                    100
 We solve this equation for x:
                                       100            x       100
                              (21) ·           =          · 70 ·
                                        70            100        70
                                        30     =     x
 Note that x is already in percent.


                                         ANSWER: 30%


 EXAMPLE 4.1.3. The number 330 is 120% of what number?


 SOLUTION: This question translates as
                            120
                 330 =          ·x                             solve this equation for x
                            100
                 100          120      100
                                    ‹
         (330) ·       =          ·x ·
                 120          100      120
                   275    =      x
 So 330 is 120% of 275.


                                         ANSWER: 275

                                                   124
CHAPTER 2. LINEAR EQUATIONS                                                         4. PERCENT
   4.2. Percent Increase and Decrease.

  DEFINITION 4.2.1 (Percent Increase and Decrease). We say that a quantity A is obtained
  by increasing the quantity P by k% if
                                                     k
                                      A = P+            P
                                                   100
  Similarly, we say that A is obtained by decreasing P by k% if
                                                     k
                                      A = P−            P
                                                   100
  Note that in both cases the percentage is applied to the old quantity in order to obtain the
  new quantity, not the other way around.


In many cases we need to solve these equations for the old quantity P, and then it is more
convenient to use equivalent formulas, obtained by applying the distributive property to the
right sides:
                  A   =    P(1 + k/100)                      percent increase
                  A   =    P(1 − k/100)                      percent decrease

Percent increase and decrease formulas have many applications. Examples of percent increase
include

      • the sales tax, when the total price is the original price plus the tax, which is a percent-
        age of the original price
      • the price markup, like when the price of a popular item goes up by so many percent
      • the bank account or the investment amount increase due to accruing the interest,
        given the simple interest rate k

Examples of percent decrease include

      • the income tax, when the total income is the original amount minus the tax, which is
        a percentage of the original income
      • the price discount, like when a store puts up items for sale at k% off




                                              125
4. PERCENT                                                     CHAPTER 2. LINEAR EQUATIONS

 EXAMPLE 4.2.1. A department store puts up shirts on sale at 35% discount. Find the
 original price of a shirt before the discount if the new price is $26.


 SOLUTION: We will need to solve the percent decrease equation for P, which is the original
 price in dollars. The new price A = 26, and the percent decrease is k = 35:
                                         A       =    P(1 − k/100)
                                        26       =    P(1 − 35/100)
                                        26       =    P(1 − 0.35)
                                        26       =    0.65P
                               (26)/0.65         =    (0.65P)/0.65
                                        40       =    P


                                         ANSWER: $40


 EXAMPLE 4.2.2. A fast food chain raised the price of the beef burger from $5 to $6, and
 lowered the price of the chicken burger from $6 to $5. Find the percent increase for the
 price of the beef burger and the percent decrease for the price of the chicken burger.


 SOLUTION: For the beef burger, the old price is P = 5 and the new price is A = 6 dollars,
 so the percent increase equation is
                                                 k
                                    A = P+          ·P
                                                100
                                                    k
                                    6        =       5+ ·5
                                                  100
 We solve it for k, which is exactly the percent increase:
                                5k
               6−5 = 5+              −5               subtracted 5 on both sides
                                100
                           5k
                  1    =
                           100
                            k
                  1    =                                              lowest terms
                           20
                              1
                                ‹
              1 · 20   =        k · 20
                             20
                 20    =   k
 So the price of the beef burger went up by 20%.

                                                 126
CHAPTER 2. LINEAR EQUATIONS                                                      4. PERCENT
 The reader may be tempted to think that the price of the chicken burger went down by
 20%, but this is not the case. If the old price is P = 6 and the new price is A = 5 dollars,
 then the percent decrease equation we have to solve is
                               k
                A = P−             ·P
                              100
                                 k
                5     =    6−       ·6
                                100
                                6k
            5−6       =    6−       −6                        subtracted 6 on both sides
                                100
                               6k
              −1      =    −                                        combined like terms
                               100
                               3k
              −1      =    −                                                lowest terms
                               50
           50                 3k     50                                                  3
             ‹                 ‹     ‹
    (−1) −            =     −      −         multiplied both sides by reciprocal of −
           3                  50     3                                                  50
              50
                      =    k
              3
             16 23    =    k                      mixed numbers are great for answers
 So the price of the chicken burger went down by 16 32 %.


                     ANSWER: Up by 20% and down by 16 32 % respectively




                                            127
4. PERCENT                                                  CHAPTER 2. LINEAR EQUATIONS
   Homework 2.4.

1. What is 6% of 14?                                 much did the dealer pay for the van if the
                                                     price after the markup is $1189?
2. What is 9% of 15?
                                                     17. Alice invested $1230 into a savings
3. 4 is 25% of what number?                          account with 1.4% monthly interest rate.
                                                     What is the balance in the account after one
4. 15 is 5% of what number?                          month?
5. 20 is what percent of 50?                         18. Bob invested $6500 into a mutual fund
                                                     with 3.5% monthly interest rate. What
6. 16 is what percent of 64?
                                                     is the balance in the account after one
                                                     month?
7. 100 is what percent of 10?
                                                     19. Brent paid $5824 for a car, and this
8. What is 350% of 20?
                                                     price included the 4% sales tax. How much
9. 10.5 is 5% of what number?                        did the car cost before the sales tax was ap-
                                                     plied?
10. 1.8 is what percent of 30?
                                                     20. A tea importer’s total revenue is
11. What is 0.4% of 5000?                            $7689.60, and this figure includes the 8%
                                                     sales tax which needs to be sent to the
12. 2.2 is 5.5% of what number?                      state. Find the total revenue without the
                                                     sales tax applied.

                                                     21. Roger paid $238 for a webcam during
Solve the applications using the percent in-         a 15%-off sale. Find the original price of
crease and decrease formulas.                        the webcam.

13. Suzy earned $208 in tips after 20% in-           22. Sasha paid $68 for a painting frame
come tax. What was her pre-tax income?               during a 20% sale event. Find the original
                                                     price of the frame.
14. Tom’s pre-tax earnings were $1170.
How much money did Tom get after pay-                23. Colin took a $275 pay increase with
ing 30% income tax?                                  his promotion. What percent increase does
                                                     that represent if his old salary was $2250
15. A department store puts dresses up for           per month?
sale at 30% discount. If the discounted
price of a dress is $42, what was the origi-         24. Rosie took a $700 pay cut when she
nal price before the discount?                       took a job which allowed her to work from
                                                     home. What percent decrease does that
16. A used car dealer buys an old van and            represent if her new job pays her $2800 per
puts it up for sale with 45% markup. How             month?



                                               128
CHAPTER 2. LINEAR EQUATIONS                        4. PERCENT
    Homework 2.4 Answers.

1. 0.84                             13. $260

3. 16                               15. $60

5. 40%                              17. $1247.22

7. 1000%                            19. $5600

9. 210                              21. $280

11. 20                              23. 12 29 %




                              129
5. APPLICATIONS                                                    CHAPTER 2. LINEAR EQUATIONS
                                             5. Applications

   5.1. Distance and Work. This equation of motion at a constant speed is well known:
                                                   d = rt
Here d is the distance traveled, r is the constant speed, and t is the duration of the motion.
For example, a car moving at r = 45 miles per hour for t = 2 hours will cover the distance of
d = r t = 45·2 = 90 miles. Note that one has to be careful to make a choice of units and stick to
it throughout the equation. Here we went with miles for the distance, hours for the duration,
and miles per hour for the speed. If the duration was given in, say, minutes, we would have to
convert it to hours before using that number in the equation.

We can operate on units of measurement, the way physicists do, to make sure that our equation
makes sense. Substituting units for the quantities should produce an equation with the same
units on both sides. Note that the equations below are not legitimate algebraic equations,
and the square brackets indicate that we are looking at the relationship between the units of
measurement, rather than numbers:
                     [d]       =     [r t]

                                     miles
                   miles       =           · hours                    hours cancel
                                     hours
                   miles       =     miles
This idea generalizes readily for any kind of work done at a constant rate:
                                                   w = rt
In the case with motion, the work is simply the distance traveled. But we can also write this type
of equation whenever a variable changes at a constant rate over time, even if that variable is not
traditionally thought of as work. One great way to understand these problems is by thinking
about the units of the rate, because the units of its numerator will be the units of work.
                   [w]     =       [r t]

                                   units of work
        units of work      =                     · units of time        units of time cancel
                                   units of time
        units of work      =       units of work

  EXAMPLE 5.1.1. Ivan can grade 25 quizzes per hour. Describe the variables in the equation
  w = r t by stating their units of measurement, and find how many quizzes Ivan can grade
  in 4 hours.


  SOLUTION: The rate of work is measured in quizzes per hour, so we let w be the number
  of quizzes graded, let r be the rate of grading in quizzes per hour, and let t be the duration



                                                     130
CHAPTER 2. LINEAR EQUATIONS                                                5. APPLICATIONS
 of grading in hours. Given t = 4 hours, the number of quizzes Ivan can grade is
                                      w = 25 · 4 = 100


                                         ANSWER:
                                 w is the number of quizzes,
                           r is in quizzes per hour, t is in hours,
                                           w = 100


 EXAMPLE 5.1.2. Taylor makes 18 dollars per hour at her job. Describe the variables in the
 equation w = r t by stating their units of measurement, and find how many dollars Taylor
 earns in 3.5 hours.



 SOLUTION: The earning rate is measured in dollars per hour, so we let w be the dollars
 earned, let r = 18 be the rate in dollars per hour, and let t be the duration of work in
 hours. Note that in this application we do not refer to Taylor’s actual job as “work”. Given
 3.5 hours, Taylor will earn this many dollars:
                                      w = 18 · 3.5 = 63


                                         ANSWER:
                            w is the amount earned in dollars,
                           r is in dollars per hour, t is in hours,
                                            w = 63


 EXAMPLE 5.1.3. Uri can walk 7 km in 2 hours. Describe the variables in the equation
 w = r t by stating their units of measurement, and find how fast Uri walks.



 SOLUTION: We need to find Uri’s walking speed in km per hour, so we let w be the distance
 in km, let r be the speed in km per hour, and let t be the duration of the walk in hours.
 We can plug in what we know and solve the equation for r:
                                        w     =     rt
                                        7     =     r ·2
                                    (7)/2     =     (r · 2)/2
                                      3.5     =     r
 So Uri’s walking speed is 3.5 km per hour.


                                              131
5. APPLICATIONS                                                        CHAPTER 2. LINEAR EQUATIONS

                                            ANSWER:
                                  w is the distance walked in km,
                                 r is in km per hour, t is in hours,
                                               r = 3.5


   5.2. Applications of w = r t. Many problems related to distance and work can be solved
by writing an equation for the amount of work completed. For example, when two different
vehicles complete the same trip, one moving with speed r1 for t 1 units of time, the other moving
with speed r2 for t 2 units of time, we can write an equation
                                              r1 t 1 = r2 t 2
to express the fact that the distance covered by each vehicle was the same.

  EXAMPLE 5.2.1. Apu takes the express train from Springfield to Capital City, and a regular
  train on his way back from Capital City to Springfield. The express completes the trip in
  4 hours 30 minutes, while the regular train covers the same distance in 6 hours. Find the
  speed of each train if the express runs 15 miles per hour faster.


  SOLUTION: Let x be the speed of the express train and let r be the speed of the regular
  train in miles per hour. The express is 15 miles per hour faster than the regular train:
                                              x = r + 15
  The distance they cover is the same, so we can also write
                 express distance        x · 4.5      =         r ·6      regular distance
  We have to use 4.5 for the duration of the express trip because we must convert 4 hours
  30 minutes into hours for the equation. Now we can substitute (r +15) for x in the second
  equation and then solve for r:
                           x · 4.5   =   r ·6
                   (r + 15) · 4.5    =   r ·6
                r · 4.5 + 15 · 4.5   =   r ·6                                 distributivity
                    4.5r + 67.5      =   6r                        simplifying expressions
  Now we use the addition property to isolate the term with r on the right side:
            4.5r + 67.5 − 4.5r       =   6r − 4.5r                          addition property
                            67.5     =   1.5r                            combined like terms
                     (67.5)/1.5      =   (1.5r)/1.5                    multiplication property
                              45     =   r




                                                   132
CHAPTER 2. LINEAR EQUATIONS                                                          5. APPLICATIONS
  So the speed of the regular train is 45 miles per hour, and we can find the speed of the
  express from the first equation:
                                            x       =     r + 15
                                            x       =     45 + 15
                                            x       =     60


                                         ANSWER:
  x and r are the speeds of the express and the regular train respectively, in miles per hour
                                          x = r + 15
                                       

                                          4.5x = 6r
                                       solution: x = 60,           r = 45


Another common situation is when two workers each work towards a common goal, which is
to complete a known amount of work. Then we can write that the total work completed is
equal to the sum of individual work amounts:
               w1 + w2        =    w
            r1 t 1 + r2 t 2   =    w                    rewrite w1 and w2 in terms of rates

  EXAMPLE 5.2.2. Kay and July are opening the incoming mail together, with the goal of
  processing 400 envelopes. Find how much time they need to complete the task if Kay can
  process 15 envelopes per hour, and July can process 35 envelopes per hour.


  SOLUTION: We need to find t, which is the duration of time they both work together, in
  hours. The total amount of work w is 400 envelopes, so we can write that
                                  Kay’s work + July’s work = 400
                                    15 · t   +    35 · t   = 400
  This equation can be easily solved for t.
                                            50t         =      400
                                       (50t)/50         =      (400)/50
                                                t       =      8
  So it will take 8 hours of them working together to process 400 envelopes.


                                               ANSWER:
                                  t is the duration of work, in hours,
                                       equation: 15t + 35t = 400
                                             solution: t = 8

                                                    133
5. APPLICATIONS                                             CHAPTER 2. LINEAR EQUATIONS

  EXAMPLE 5.2.3. One morning Aristotle starts walking from Athens to Megara, and at the
  same time Socrates starts on a journey from Megara to Athens. Being younger, Aristotle
  is walking 1.8 times faster than Socrates. After a 5 hour journey, they meet somewhere
  in the middle of the way. Find the speed of each traveler if the total distance between
  Athens and Megara is 42 km.


  SOLUTION: Let a and s be the walking speeds of Aristotle and Socrates respectively, in
  km per hour. What we call work here is measured in km, so we are just talking about the
  distance traveled. Since Aristotle’s speed is 1.8 times greater than Socrates’ speed, we can
  write
                                            a = 1.8s
  And since they both walk for 5 hours and then meet in the middle of a 42 km long road,
  we can also write a distance equation:
                    A’s distance    +      S’s distance    = total distance
               A’s speed · duration + S’s speed · duration = total distance
                        a·5         +          s·5         =       42
  Now we can substitute 1.8s for a in this equation and solve for Socrates’ speed s:
              (1.8s) · 5 + (s) · 5   =   42
                         9s + 5s     =   42
                             14s     =   42                 combined like terms
                       (14s)/14      =   (42)/14
                                s    =   3
  Now that we know that Socrates was walking at 3 km per hour, we can get Aristotle’s
  speed from the first equation:
                                          a   =    1.8s
                                          a   =    1.8(3)
                                          a   =    5.4
  So Aristotle was walking at 5.4 km per hour.


                                         ANSWER:
        a and s are the speeds of Aristotle and Socrates respectively, in km per hour
                                         a = 1.8s
                                      

                                         5a + 5s = 42
                                     solution: a = 5.4,   s=3




                                               134
CHAPTER 2. LINEAR EQUATIONS                                                   5. APPLICATIONS
   Homework 2.5.

Solve applications of w = r t by describing          10. It takes Leela 56 days to inspect 14
the variables and their units of measure-            buildings. Find the rate of the building in-
ment.                                                spection in buildings per day.

1. The top speed of a bullet train is 210 km
per hour. Find how long it takes to travel
168 km at top speed.
                                                     Solve applications by describing the vari-
2. The average speed of a cheetah is 16              ables used to represent the quantities of in-
meters per second. Find how far the chee-            terest, and stating appropriate equations.
tah can run in 24 seconds at that speed.
                                                     11. Alice paddled for 3 hours upstream
3. A standard computer hard drive rotates            and then for 2 hours downstream. When
at 5400 revolutions per minute. Find the             going downstream, her speed was 10 mph
number of revolutions over 10 seconds.               greater than her speed when going up-
(Hint: Convert seconds into minutes.)                stream. The total distance traveled was 30
                                                     miles. How fast did Alice travel upstream
4. Morgan’s chicken coop produces on av-             and downstream?
erage 8 eggs per day. Find how many days
are needed to produce 52 eggs.                       12. Bob rode for 3 hours on the train and
                                                     then for 2 hours on the bus. The bus is 50
5. Eve typed up a 4050 word document                 km per hour slower than the train. The to-
over two and a half hours. Find Eve’s typ-           tal distance traveled by Bob was 225 km.
ing rate in words per minute. (Hint: Con-            Find the the speed of the train and the
vert hours into minutes.)                            speed of the bus.

6. Jeff’s household used 630 gallons of wa-          13. Ridwan can plant 4 trees per hour, and
ter over the three winter months. Find the           Sue can plant 5.5 trees per hour. Find how
rate of water consumption in gallons per             much time they need to plant 285 trees, if
month.                                               they are working together.

7. Jamie charges $80 per hour for her par-           14. Quinn runs at 14 km per hour and
alegal work. Find how long she has to work           walks at 6 km per hour. During his week-
in order to earn $2000.                              end exercise, Quinn walks and runs for an
                                                     equal amount of time. One weekend he ran
8. Ivan drinks 3.5 cups of coffee per day.           and walked for a total of 24 km. Find the
Find how long it takes for Ivan to drink 42          amount of time he walked.
cups of coffee.
                                                     15. A passenger and a freight train start
9. It takes Bongo two and a half hours to            toward each other at the same time from
mow the front lawn. Find how many times              two points 300 miles apart. If the rate of
he can mow the same lawn in 40 hours.                the passenger train exceeds the rate of the
                                                     freight train by 15 miles per hour, and they
                                                     meet after 4 hours, what must the rate of
                                                     each be?
                                               135
5. APPLICATIONS                                             CHAPTER 2. LINEAR EQUATIONS
16. A car and a truck are 276 miles apart            together, a 1200 gallon tank can be emp-
and start at the same time to travel toward          tied in 12 minutes. Find the rates in gallons
each other. The car is 5 miles per hour              per minute for both pumps.
faster than the truck. If they meet after 6
hours, find the rate of each.                        19. Two automobiles started at the same
                                                     time from a point, but traveled in oppo-
17. Lab 1 can process blood samples 3                site directions. Their rates were 25 and
times faster than Lab 2. Working together,           35 miles per hour respectively. After how
the two labs can process 84 samples over 7           many hours were they 180 miles apart?
days. Find the rate of work for each Lab, in
samples per day.                                     20. Two trains travel toward each other
                                                     from points which are 195 miles apart.
18. Pump 1 pumps water at the rate which             They travel at rate of 25 and 40 miles per
is 10 gallons per minute slower than the             hour respectively. If they start at the same
rate of Pump 2. With both pumps working              time, how soon will they meet?




                                               136
 CHAPTER 2. LINEAR EQUATIONS                                                    5. APPLICATIONS
     Homework 2.5 Answers.

1.                                                     solution: u = 2,    d = 12
 w is the distance traveled in km,
 r is in km per hour, t is in hours,
 t = 0.8 (48 minutes)                                 13.
                                                       t is the duration of work in hours,
3.                                                     equation: 4t + 5.5t = 285
 w is the number of revolutions,                       solution: t = 30
 r is in revolutions per minute, t is in min-
 utes,
                                                      15.
 w = 900
                                                       p and f are the speeds of the passenger
5.                                                     train and the freight train respectively, in
 w is the number of words,                             mph
 r is in words per minute, t is in minutes,               p = f + 15
                                                       
 r = 27                                                   4p + 4 f = 300
7.                                                     solution: p = 45,    f = 30
 w is wages earned in dollars,
 r is in dollars per hour, t is in hours,
 t = 25                                               17.
                                                       r1 and r2 are the rates for Lab 1 and Lab 2
9.                                                     respectively, in samples per day
 w is lawns mowed,                                     
                                                          r1 = 3r2
 r is in lawns mowed per hour, t is in hours,
                                                          7r1 + 7r2 = 84
 w = 16
                                                       solution: r1 = 9,   r2 = 3
11.
 u and d are the speeds of the upstream and
 the downstream travel respectively, in mph           19.
                                                       t is the duration of travel in hours,
    d = u + 10
 
                                                       equation: 25t + 35t = 180
    3u + 2d = 30                                       solution: t = 3




                                                137
6. LINEAR INEQUALITIES                                              CHAPTER 2. LINEAR EQUATIONS
                                       6. Linear Inequalities

    6.1. Solution Sets. Recall that we call an equation linear if every term is either a constant
or a product of a constant and the first power of a single variable.

  DEFINITION 6.1.1. A linear inequality is an inequality relation applied to two linear ex-
  pressions. There are four types of linear inequalities corresponding to the four types of
  inequality relations: x < y, x > y, x ≤ y, and x ≥ y.

  We say that a number is a solution for the given inequality in variable x if substituting
  that number for x makes the inequality true. The collection of all such numbers forms
  the solution set for the inequality.


  EXAMPLE 6.1.1. Determine whether 0, 10, and −5, are solutions for the inequality
                                            x + 3 < 4 − 2x


  SOLUTION: Let’s try x = 0:
                                        (0) + 3    <     4 − 2(0)
                                               3   <     4
  This is true, so 0 is a solution. Let’s try x = 10 next:
                                       (10) + 3    <     4 − 2(10)
                                             33    <     4 − 20
                                             33    <     − 16
  This is false, so 10 is not a solution. Finally we check x = −5:
                                       (−5) + 3 <        4 − 2(−5)
                                             −2    <     4 + 10
                                             −2    <     14
  This is true, so −5 is a solution.


                       ANSWER: 0 and −5 are solutions, and 10 is not


  EXAMPLE 6.1.2. Describe the set of solutions of the inequality x ≥ 6.


  SOLUTION: 6 is clearly a solution, and so is every number greater than 6. At the same
  time, no number less than 6 is a solution. So the set of solutions is very large: it includes
  6, 7, 7.5, and infinitely many other numbers. It would be unhelpful to state it as a roster:
                                       {6, 7, 7.5, 6.12, 100, . . .}

                                                   138
CHAPTER 2. LINEAR EQUATIONS                                             6. LINEAR INEQUALITIES
  But the set builder notation provides an unambiguous way of describing this set of num-
  bers:

                                                   
                                        ANSWER:        x x ≥6



    6.2. Interval Notation and Graphs. Inequality solutions in set builder notation are cum-
bersome, so here we introduce a different notation, which is well-suited for describing large
portions of the real line. But before we introduce the notation, let us look at what solutions of
linear inequalities look like graphically. In the following table we list the four types of inequal-
ities in one variable x, the corresponding graphs of their solution sets, and the new notation
describing these sets.

  inequality      shaded points solve the inequality            solution set in interval notation

     x<a                                                                    (−∞, a)
                                    a


     x≤a                                                                    (−∞, a]
                                    a


     x>a                                                                     (a, ∞)
                                    a


     x≥a                                                                     [a, ∞)
                                    a

Note that the graphs of solutions for x < a and x > a leave the point a empty, because a < a
and a > a are false, and so a does not belong in either solution set. For a similar reason the
graphs of x ≤ a and x ≥ a fill the point a and mark it as a solution.

The right column displays the description of the solution set in the interval notation.

  DEFINITION 6.2.1 (Interval Notation). The interval notation (a, b) describes the set of all
  real numbers between a and b. When we want to include the endpoints, we use square
  brackets: [a, b]. When we want the interval to extend indefinitely, we use the infinity
  sign: (a, ∞). The nuances are worked out in the following examples.




                                                  139
6. LINEAR INEQUALITIES                                       CHAPTER 2. LINEAR EQUATIONS

  BASIC EXAMPLE 6.2.1. Let us graph the solution set and describe it in the interval notation:
                                            x ≤ 17
  This inequality is true for 17 and all numbers below it:


                                              17

  The interval notation
                                           (−∞, 17]
  uses −∞ for the left endpoint meaning that the set extends indefinitely to the left on
  the real number line, and 17 for the right endpoint. On the left, the round parenthesis (
  indicates that −∞ does not solve the inequality, and so not in the solution set. On the
  right, the square bracket ] indicates that 17 does solve the inequality, and so is included
  in the solution set.


  BASIC EXAMPLE 6.2.2. Let’s graph the solution set and describe it in the interval notation:
                                            x > −4
  This inequality is true for all numbers above −4, false for −4 and all numbers below it:


                                              −4

  The interval notation
                                           (−4, ∞)
  uses −4 for the left endpoint and ∞ for the right endpoint, meaning that the set extends
  indefinitely to the right on the real number line. The round parentheses ( ) indicate that
  neither endpoint solves the inequality.


  BASIC EXAMPLE 6.2.3. Let’s graph the solution set and describe it in the interval notation:
                                            x < 28
  This inequality is true for all numbers below 28, false for 28 and all numbers above it:


                                              28

  The interval notation
                                         (−∞, 28)
  uses −∞ for the left endpoint, meaning that the set extends indefinitely to the right on
  the real number line, and 28 for the right endpoint. The round parentheses ( ) indicate
  that neither endpoint solves the inequality.

                                             140
CHAPTER 2. LINEAR EQUATIONS                                         6. LINEAR INEQUALITIES

  BASIC EXAMPLE 6.2.4. Let’s graph the solution set and describe it in the interval notation:
                                            20 ≤ x
  Note that we can also write this inequality in an equivalent form
                                             x ≥ 20
  Either way, this inequality is true for 20 and all the numbers above 20:


                                               20

  The interval notation
                                            [20, ∞)
  uses 20 for the left endpoint and ∞ for the right endpoint, meaning that the set extends
  indefinitely to the right on the real number line. The square bracket [ on the left indicates
  that 20 solves the inequality and is an element of the solution set. The round parenthesis
  ) on the right indicates that ∞ does not solve the inequality.


    6.3. Addition and Multiplication Properties. Much like we did with the equations, we
will solve inequalities by constructing equivalent inequalities with well-known solution sets.

  DEFINITION 6.3.1. Inequalities are equivalent if they have the same solution sets.


  THEOREM 6.3.1 (Addition Property for Inequalities). Adding the same number or expres-
  sion on both sides of an inequality produces an equivalent inequality.


In the following examples, we will describe each solution set by drawing a graph and stating
it in the interval notation.
  EXAMPLE 6.3.1. Solve the inequality
                                           x −4≤6


  SOLUTION:
                      x −4    ≤   6
                  x −4+4      ≤   6+4                      addition property
                          x   ≤   10                     combined like terms




                   ANSWER: (−∞, 10],
                                                           10


                                              141
6. LINEAR INEQUALITIES                                        CHAPTER 2. LINEAR EQUATIONS

  EXAMPLE 6.3.2. Solve the inequality:      4x + 3 > 3(x − 1)


  SOLUTION: Just as we did with equations, we will need to isolate the term with the
  variable on one side, so we start by getting rid of parentheses:
             4x + 3     >   3(x − 1)
             4x + 3     >   3x − 3                                         distributivity
         4x + 3 − 3     >   3x − 3 − 3              isolating constants on the right side
                 4x     >   3x − 6                                 combined like terms
           4x − 3x      >   3x − 6 − 3x        isolating variable terms on the left side
                   x    >   −6                                     combined like terms


                       ANSWER: (−6, ∞),
                                                             −6



  THEOREM 6.3.2 (Multiplication Property for Inequalities). Multiplying both sides of an
  inequality by the same positive number produces an equivalent inequality. Multiplying
  both sides of an inequality by the same negative number and then reversing the inequal-
  ity sign produces an equivalent inequality.

  Formally, the following three inequalities in x are all equivalent to each other for all real
  numbers a and all positive numbers m:
                               x < a, mx < ma, −mx > −ma
  Similarly, the following three inequalities in x are all equivalent to each other:
                               x ≤ a, mx ≤ ma, −mx ≥ −ma


  EXAMPLE 6.3.3. Solve the inequality:      15x ≥ 3


  SOLUTION: Dividing by 15 is the same as multiplying by its reciprocal 1/15, which is
  positive, so the inequality sign does not change:
                                           15x    ≥    3
                                       (15x)/15   ≥    3/15
                                              x   ≥    1/5


                    ANSWER: [1/5, ∞),                        1/5



                                              142
CHAPTER 2. LINEAR EQUATIONS                                        6. LINEAR INEQUALITIES

 EXAMPLE 6.3.4. Solve the inequality
                                    2(1 − 6x) < −14x + 5


 SOLUTION: First we get rid of parentheses and isolate the variable terms on one side, and
 then we use the multiplication property to isolate the variable:
        2(1 − 6x) <      − 14x + 5
          2 − 12x   <    − 14x + 5
    2 − 12x + 12x   <    − 14x + 5 + 12x                         added 12x to both sides
                2 <      − 2x + 5                                    combined like terms
             2−5 <       − 2x + 5 − 5                        subtracted 5 from both sides
               −3 <      − 2x                                        combined like terms
               −3       −2x
                    >                        dividing both sides by −2 reverses the sign
               −2       −2
              3/2 >     x                                    this is equivalent to x < 3/2


                 ANSWER: [−∞, 3/2),                        3/2




                                            143
6. LINEAR INEQUALITIES                                       CHAPTER 2. LINEAR EQUATIONS
      Homework 2.6.

Write an inequality corresponding to the             15. −47 ≥ 8 − 5x
shown solution set, and state it in the in-
                                                           6+ x
terval notation:                                     16.        ≤ −1
                                                            12
1.
                    −2                               17. −2(3 + k) < −44

2.                                                   18. −7n − 10 ≥ 60
                    1

3.                                                   19. 24 ≥ −6(m − 6)
                    5
                                                     20. −8(n − 5) ≥ 0
4.
                    −5
                                                     21. −r − 5(r − 6) < −18
5.
                    −3                               22. −60 ≥ −4(−6x − 3)
6.                                                   23. 4(2 y − 3) ≤ −44
                    4

                                                                   x
                                                     24. 2 > 9 −
                                                                   5
For each inequality, graph the solution set
and describe it using the interval notation.               4
                                                     25.     (3x + 4) ≤ 20
                                                           5
7. 2 + x < 3
                                                     26. 3(2 y − 3) > 21
8. 4 y ≥ 12                                                x
                                                     27.     +4≤1
       x                                                   3
9.       ≥ 10
      11                                                   2 x  4
                                                     28.    − <
          n                                                3 5  15
10. −2 ≤
         13
                                                     29. 24 + 4b < 4(1 + 6b)
       n
11. 8 + ≥ 6
       3                                             30. −8(2 − 2n) ≥ −16 + n
                x                                    31. −5v − 5 < −5(4v + 1)
12. 11 > 8 +
                2
                                                     32. −36 + 6x > −8(x + 2) + 4x
          a−2
13. 2 >
           5                                         33. 3(n + 3) + 7(8 − 8n) < 5n + 5 + 2
       v−9
14.        ≤2                                        34. −(4 − 5p) + 3 ≥ −2(8 − 5p)
        −4




                                               144
CHAPTER 2. LINEAR EQUATIONS                        6. LINEAR INEQUALITIES
    Homework 2.6 Answers.

1. x < −2, (−∞, −2)                 21. (8, ∞)

3. x ≥ 5, [5, ∞)
                                                       8
5. x > −3, (−3, ∞)

7. (−∞, 1)                          23. (−∞, −4]

                                                      −4
                      1

9. [110, ∞)
                                    25. (−∞, 7]

                     110
                                                       7
11. [−6, ∞)
                                    27. (−∞, −9]
                     −6

13. (−∞, 12)                                          −9



                     12             29. (1, ∞)

15. [11, ∞)
                                                       1

                     11
                                    31. (−∞, 0)
17. (19, ∞)
                                                       0
                     19

19. [2, ∞)                          33. (1, ∞)

                      2                                1




                              145
6. PRACTICE TEST 2                                          CHAPTER 2. LINEAR EQUATIONS
                                        Practice Test 2




1. Solve the equation:                               12. 384 is 96% of what number?
            20x − 7 = −5x − 2
                                                     13. The monthly cost of living in a met-
2. Solve the equation:                               ropolitan area went down from 1400 to
                                                     1330. What percent decrease does that
      7(x + 4) − 20 = 4(x + 2) + 3x                  represent?
3. Solve the equation:                               14. A savings account has $571.20 at the
              x        x −1                          end of the year, after the 2% simple interest
                −1=
              5          5                           is added. How much money was originally
                                                     invested into the account at the beginning
4. Matt makes 35 dollars per week less               of the year?
than Jane, and together they make 1245
dollars per week. How much money does                15. Train A, traveling 70 miles per hour,
each person make per week?                           leaves Westfall heading toward East River,
                                                     260 miles away. At the same time Train
5. The perimeter of a rectangular cloth is           B, traveling 60 miles per hour, leaves East
84 inches, and one side of the rectangle is          River heading toward Westfall. How soon
2.5 shorter than the long side. Find the di-         do the two trains meet?
mensions of the cloth.
                                                     16. A bus leaves the station at noon. Two
6. Solve the equation:                               hours later a car leaves the same station,
       x − 11 + 2(x − 2) = 6(5 − x)                  going 10 miles per hour faster than the bus,
                                                     and follows the same route. Find the speed
7. Solve the formula for D:                          of each vehicle if the car catches up to the
                AD      7                            bus 8 hours after the bus leaves the station.
                    =
                 3     2B
                                                     (Hint: Both vehicles end up covering the
8. Solve the formula for y:                          same distance, although the car travels
           −5 y + 4x − 12 = 18                       over a shorter period of time.)

9. Solve the formula for w:                          17. Solve the inequality:
          w + 6x = −4(2 y − w)                                   8(x + 1) ≥ 4x − 24

10. What is 14% of 280?                              18. Solve the inequality:
                                                                        1
11. 7 is what percent of 5?                                    7x − 3 > (15x + 4)
                                                                        2




                                               146
CHAPTER 2. LINEAR EQUATIONS                                                6. PRACTICE TEST 2
    Practice Test 2 Answers.

1. {1/5}                                            10. 39.2

2. R                                                11. 140%

3. ∅                                                12. 400

4.                                                  13. 5% decrease
 m and j are amounts for Matt and Jane re-
 spectively, in dollars per week                    14. $560

   m = j − 35
 
                                                    15.
   m + j = 1245                                      t is the duration of travel until trains meet,
solution: m = 605,     j = 640                       in hours,
                                                     equation: 70t + 60t = 260
5.                                                   solution: t = 2
 l and w are length and width of the cloth,
 in inches                                          16.
                                                     b and c are the speeds of the bus and the
    2l + 2w = 84
 
                                                     car respectively, in mph
    w = l ÷ 2.5
                                                        c = b + 10
                                                     
solution: l = 30,    w = 12                             b·8= c·6

6. {5}                                               solution: b = 30,    c = 40

          21                                        17. [−8, ∞)
7. D =
         2AB
                                                                          −8
       30 − 4x
8. y =
         −5                                         18. (−∞, −10)
         6x + 8 y
9. w =                                                                    −10
            3




                                              147
                                        CHAPTER 3


                                        Graphing


                         1. Reading and Constructing Graphs
 1.1. Cartesian Plane.

DEFINITION 1.1.1 (Ordered Pair). An ordered pair is a pair of numbers with a definite order
to them. The notation (a, b) denotes an ordered pair with number a first and number b
second. Here a is the x coordinate, or the first coordinate, or the first component of the
pair, and b is the y coordinate, or the second coordinate, or the second component of the
pair.

Two ordered pairs are equal if their first components are equal and their second compo-
nents are equal.


BASIC EXAMPLE 1.1.1. The pairs (10, 25) and (5 + 5, 52 ) are equal.

The pairs A = (10, 25) and B = (10, 10) are distinct even though their first components are
equal, since the second component of A is 25, and it is not equal to the second component
of B, which is 10.


DEFINITION 1.1.2. A Cartesian plane is an infinite flat plane equipped with a Cartesian
coordinate system. In this system, an ordered pair of real numbers (a, b) corresponds to
the point of intersection of two lines: the vertical line through the point a on the x-axis,
and the horizontal line through the point b on the y-axis.

                                        y
                                                      (a, b)
                                    b


                                                               x
                                                  a



Note that the distance from the point (a, b) to the x-axis is |b|, while the distance to the
y-axis is |a|.


                                            149
1. READING AND CONSTRUCTING GRAPHS                                                  CHAPTER 3. GRAPHING

 DEFINITION 1.1.3. We can graph a set of points on a coordinate plane by shading all points
 in that set with a specific color.


 BASIC EXAMPLE 1.1.2. There are two major visual styles for the coordinate plane. The
 illustration on the left shows the coordinate axes as thick black lines, and is typically used
 when the origin (0, 0) is visible on the graph. The illustration on the right is styled like a
 box and is often used when showing the axes is inconvenient.


      80
           y
      70
                                                                  40
      60

      50
                                                                  30
                                                              y
      40

      30
                                                                  20
      20

      10
                                               x
                                                                               75     100    125
                        50       100     150   200                                     x



 BASIC EXAMPLE 1.1.3. Finite sets of points look like collections of dots. The graph on the
 left shows the set containing points (−2, 0) and (1, 2):

                3                                     3                                      3
                    y                                     y                                      y
                2                                     2                                      2

                1                                     1                                      1
                                 x                                     x                                     x
  −3 −2 −1               1   2       3     −3 −2 −1               2        3        −3 −2 −1         1   2       3
         −1                                       −1                                       −1

               −2                                    −2                                     −2

               −3                                    −3                                     −3


 Infinite sets of points often look like lines and curves. The middle graph shows the set
 containing every point with x coordinate equal to 1.

 Even larger sets of points, usually defined by inequalities, often appear as solid shaded
 regions of the plane. The graph on the right shows the set of all points with x coordinate
 between 0 and 1 and y coordinate between 1 and 2.


                                                     150
CHAPTER 3. GRAPHING                             1. READING AND CONSTRUCTING GRAPHS

 EXAMPLE 1.1.1. Consult the given graph and find

     (1)   all points with x coordinate 20
     (2)   all points with y coordinate 30
     (3)   the point with the lowest y coordinate
     (4)   the point with the highest x coordinate


                      100
                            y
                       90

                       80

                       70

                       60

                       50

                       40

                       30

                       20

                       10
                                                                   x
                                10 20 30 40 50 60 70 80 90 100 110 120




 SOLUTION: To find the all points with x coordinate 20, we can follow the vertical line
 consisting of points with x coordinate equal to 20 and see where this line meets the graph.
 In our case, the only such point is (20, 90):


                      100
                            y
                       90

                       80

                       70

                       60

                       50

                       40

                       30

                       20

                       10
                                                                   x
                                10 20 30 40 50 60 70 80 90 100 110 120



                                              151
1. READING AND CONSTRUCTING GRAPHS                                   CHAPTER 3. GRAPHING
 To find the all points with y coordinate 30, we can follow the horizontal line consisting
 of points with y coordinate equal to 30 and see where this line meets the graph. In our
 case, we can see two such points, (40, 30) and (80, 30):


                      100
                            y
                       90

                       80

                       70

                       60

                       50

                       40

                       30

                       20

                       10
                                                                     x
                                10 20 30 40 50 60 70 80 90 100 110 120


 To find the point with the lowest y coordinate, we look at how low the graph goes, since
 points which appear to be down in the bottom have lower y coordinates. In our case, the
 graph goes as low as y = 10, and the only point this far down is (60, 10):


                      100
                            y
                       90

                       80

                       70

                       60

                       50

                       40

                       30

                       20

                       10
                                                                     x
                                10 20 30 40 50 60 70 80 90 100 110 120


 To find the point with the highest x coordinate, we look at how far to the right the graph
 goes, since points which appear to be on the right side have higher x coordinates. In our
 case, the graph goes as far to the right as x = 100, and the only point this far on the right
 is (100, 90):

                                             152
CHAPTER 3. GRAPHING                               1. READING AND CONSTRUCTING GRAPHS

                       100
                             y
                        90

                        80

                        70

                        60

                        50

                        40

                        30

                        20

                        10
                                                                                x
                                 10 20 30 40 50 60 70 80 90 100 110 120



                                            ANSWER:

     (1)   (20, 90)
     (2)   (40, 30) and (80, 30)
     (3)   (60, 10)
     (4)   (100, 90)


 EXAMPLE 1.1.2. Plot points A(1, 2), B(2, 1), C(−3, 1), and D(0, −4) on the same plane.


 SOLUTION: Note that A and B are different points: the first coordinate of A(1, 2) is 1, so
 A is located above 1 on the x-axis, whereas B(2, 1) is located above 2 on the x-axis.


                                                         5
                                                             y
                                                         4
                                                         3
                                                                     A(1, 2)
                                                         2
                                             C(−3, 1)                     B(2, 1)
                                                         1
                     ANSWER:                                                        x
                                         −5 −4 −3 −2 −1          1    2    3    4       5
                                                      −1
                                                        −2
                                                        −3
                                                             D(0, −4)
                                                        −4
                                                        −5


                                               153
1. READING AND CONSTRUCTING GRAPHS                                               CHAPTER 3. GRAPHING

 EXAMPLE 1.1.3. Plot points A(−40, 0.3), B(−20, 0.7), C(0, −0.6), and D(20, 0.2) on the
 same plane.


 SOLUTION: We may have to scale the coordinate axes in order to make the points spread
 out in a visually appealing fashion. In this case, the x coordinate ranges from −40 to 20,
 while the absolute value of the y coordinate never goes above 1.0.


                                                     0.8
                                                           y

                                                     0.7
                                     B(−20, 0.7)
                                                     0.6

                                                     0.5

                                                     0.4
                               A(−40, 0.3)
                                                     0.3

                                                     0.2

                                                     0.1

           ANSWER:                                                                          x
                      −50   −40   −30    −20   −10             10       20      30     40   50
                                                   −0.1

                                                   −0.2
                                                                             D(20, −0.2)
                                                   −0.3

                                                   −0.4

                                                   −0.5

                                                   −0.6
                                                           C(0, −0.6)
                                                   −0.7

                                                   −0.8




                                               154
CHAPTER 3. GRAPHING                              1. READING AND CONSTRUCTING GRAPHS
     Homework 3.1.

Plot all given points on the same coordinate         7.
plane.                                                         A(10, 15)
                                                               B(−5, −5)
1.
                                                               C(15, −10)
                A(1, 3)                                        D(−5, 15)
                B(−2, 4)
                C(−1, −1)                            8.
                                                              A(0, 0)
                D(2, 0)
                                                              B(0, 400)
2.                                                            C(200, 300)
                 A(0, 1)                                      D(400, 0)
                 B(2, −3)
                 C(1, −5)
                 D(−5, 0)

3.
                A(−1, −60)
                B(0, −20)
                C(1, 80)
                D(2, 90)

4.
                A(−120, 0)
                B(−20, −2)
                C(40, 2)
                D(100, −3)

5.
               A(−0.4, 0.4)
               B(0.1, −0.6)
               C(0.5, 0.1)
               D(0.7, 0)

6.
              A(0, −0.8)
              B(−0.5, −0.7)
              C(0.4, 0)
              D(−0.4, 0.5)
                                               155
1. READING AND CONSTRUCTING GRAPHS                                                          CHAPTER 3. GRAPHING
For each graph shown, find coordinates of                             11.
the points with required properties.                                                               5
                                                                                                       y
                                                                                                   4
9.
                        5                                                                          3
                                y
                        4                                                                          2

                        3                                                                          1
                                                                                                                           x
                        2
                                                                            −5 −4 −3 −2 −1                 1   2   3   4       5
                        1                                                                −1
                                                        x                                         −2
      −5 −4 −3 −2 −1                    1   2   3   4       5
                   −1                                                                             −3

                       −2                                                                         −4

                       −3                                                                         −5

                       −4
                                                                            (1)   all points with x coordinate zero
                       −5                                                   (2)   all points with y coordinate zero
                                                                            (3)   all points with x coordinate −3
      (1)   all points with     x       coordinate zero                     (4)   the point with the lowest y coor-
      (2)   all points with     y       coordinate zero                           dinate
      (3)   all points with     x       coordinate −4
      (4)   all points with     y       coordinate −1                 12.
                                                                                       14
                                                                                             y
                                                                                       13
10.
                            7                                                          12
                                    y                                                  11
                            6
                                                                                       10
                            5
                                                                                        9
                            4                                                           8
                            3                                                           7
                            2                                                           6
                            1                                                           5
                                                            x                           4
       −7 −6 −5 −4 −3 −2 −1             1 2 3 4 5 6 7                                   3
                         −1
                                                                                        2
                        −2
                                                                                        1
                        −3                                                                                                 x
                        −4                                                  −5−4−3−2−1
                                                                                    −1           1 2 3 4 5 6 7 8 9 10 11
                        −5                                                          −2
                        −6
                        −7                                                  (1)   all points with x coordinate zero
                                                                            (2)   all points with y coordinate zero
      (1)   all points with     x       coordinate zero                     (3)   all points with x coordinate 6
      (2)   all points with     y       coordinate zero                     (4)   the point with the highest y coor-
      (3)   all points with     x       coordinate −2                             dinate
      (4)   all points with     y       coordinate −6



                                                                156
CHAPTER 3. GRAPHING                                                  1. READING AND CONSTRUCTING GRAPHS
13.                                                                      15.
                               y                                               100
                       4                                                                 y
                                                                                90
                       3
                                                                                80
                       2
                                                                                70
                       1                                                        60
                                                       x
                                                                                50
      −4 −3 −2 −1                      1       2   3   4
                −1                                                              40
                                                                                30
                      −2
                                                                                20
                      −3
                                                                                10
                      −4                                                                                                        x
                                                                                             10 20 30 40 50 60 70 80 90 100

      (1)   all points with x coordinate zero
                                                                               (1) all points with x coordinate 40
      (2)   all points with y coordinate zero
                                                                               (2) all points with y coordinate 50
      (3)   all points with x coordinate 2
                                                                               (3) the point with the highest x coor-
      (4)   all points with the lowest y coor-
                                                                                   dinate
            dinate
                                                                               (4) the point with the lowest y coor-
14.                                                                                dinate
                           5
                                   y
                           4
                                                                         16.
                                                                               80
                                                                                     y
                           3
                                                                               70
                           2
                                                                               60
                           1
                                                           x                   50
      −5 −4 −3 −2 −1                       1   2   3   4       5
                   −1                                                          40

                        −2                                                     30

                        −3                                                     20
                        −4
                                                                               10
                        −5                                                                                                  x
                                                                                             25   50   75   100 125 150 175 200
      (1)   all points with x coordinate zero
      (2)   all points with y coordinate zero                                  (1) all points with x coordinate 125
      (3)   all points with x coordinate −2                                    (2) all points with y coordinate 40
      (4)   the point with the highest y coor-                                 (3) the point with the lowest y coor-
            dinate                                                                 dinate
                                                                               (4) the point with the highest y coor-
                                                                                   dinate



                                                                   157
 1. READING AND CONSTRUCTING GRAPHS                                                             CHAPTER 3. GRAPHING
     Homework 3.1 Answers.

1.                                                                        7.
                            y                                                                       y
             B
                    4                                                                          20
                                    A                                                           D         A

                    2                                                                          10

                                            D             x                                                            x
      −4   −2      C                    2             4                         −20     −10    B        10        20
                                                                                                              C
                  −2                                                                          −10


                  −4                                                                          −20


                                                                          9.
3.
                  100       y                             D
                                            C
                                                                                 (1)   (0, −2)
                                                                                 (2)   (4, 0)
                  50                                                             (3)   (−4, −4)
                                                                                 (4)   (2, −1)
                                                          x
                                                                          11.
      −2   −1           B               1             2


             A −50                                                               (1)   (0, 0)
                                                                                 (2)   (−4, 0) and (0, 0)
                                                                                 (3)   (−3, −3)
             −100                                                                (4)   (−2, −4)

                                                                          13.
5.
                        1       y
                                                                                 (1)   (0, 1)
                                                                                 (2)   (−3, 0), (−1, 0), (1, 0), (3, 0)
                  A 0.5                                                          (3)   (2, −1)
                                                                                 (4)   (−2, −1) and (2, −1)
                                                  C
                                                          D x
                                                                          15.
     −1    −0.5                                 0.5             1


                  −0.5              B                                            (1)   (40, 30), (40, 50), (40, 70)
                                                                                 (2)   (20, 50), (40, 50), (60, 50)
                                                                                 (3)   (60, 50)
                    −1                                                           (4)   (40, 30)
                                                                    158
CHAPTER 3. GRAPHING                                         2. GRAPHING LINEAR EQUATIONS
                                2. Graphing Linear Equations

   2.1. Solution Sets. Recall that equation is a particular kind of relation, which we infor-
mally defined as a true-or-false statement about numbers. Consider an equation in two vari-
ables x and y. It may be true for some ordered pairs (a, b) and false for others.

  DEFINITION 2.1.1. A solution to an equation in two variables x and y is any ordered pair
  (a, b) such that substituting a for x and b for y makes the equation true. The set of all
  such pairs is the solution set for that particular equation.


  EXAMPLE 2.1.1. Determine whether (−5, 4) is a solution for the equation 2x + 3 y = 3


  SOLUTION: The left side of the equation is
                                  2x + 3 y    =   2(−5) + 3(4)
                                              =   −10 + 12
                                              =   2
  while the right side is 3. The sides are unequal, the equation is false, so this is not a
  solution.


                                          ANSWER: No


  EXAMPLE 2.1.2. Determine whether (3, −2) a solution for the equation 2x 2 + y 3 = 4x + y


  SOLUTION: The left side of the equation is
                                 2x 2 + y 3   =   2(3)2 + (−2)3
                                              =   2(9) + (−8)
                                              =   10
  while the right side is
                                   4x + y     =   4(3) + (−2)
                                              =   10
  The sides are equal, the equation is true, so this is a solution.


                                          ANSWER: Yes




                                               159
2. GRAPHING LINEAR EQUATIONS                                                  CHAPTER 3. GRAPHING

 BASIC EXAMPLE 2.1.1. Some equations have empty solution sets, for example
                                     x + y = x + y + 17
 Some equations have solution sets which consist of all possible ordered pairs of numbers,
 for example
                                      x+y=x+y
 But most practically useful equations have solution sets which include some pairs, and
 exclude others.

 EXAMPLE 2.1.3. Describe the solution set for the equation y = x.


 SOLUTION: Any pair of the form (a, a), where a is a real number, will solve the equation.
 At the same time, any pair of distinct numbers (a, b) such that a 6= b will make the equation
 false, so the solution set consists of all pairs (a, a) where the components are equal to
 each other. For example, (1, 1) is a solution, and so is (3, 3), (−10, −10), and so on. As is
 typically the case with equations in two variables, the solution set is infinite.


                 ANSWER: The set of pairs (a, a), for each real number a.

  2.2. Graphing Solution Sets.

 DEFINITION 2.2.1. A graph of a relation, and in particular of an equation, consists of all
 points on the plain for which that particular relation is true.


 EXAMPLE 2.2.1. Graph the equation x = y.


 SOLUTION: The reader can refer to the example 2.1.3 for the description of the solution
 set, which consists of all the points (a, a), for each real number a. As we start plotting
 some of the solutions on the Cartesian plane, we realize they are all located on the so-
 called main diagonal.


                                                      3
                                                          y
                                                      2
                                                      1
                          ANSWER:                                     x
                                             −3 −2 −1         1   2       3
                                                    −1
                                                     −2
                                                     −3


                                             160
CHAPTER 3. GRAPHING                                      2. GRAPHING LINEAR EQUATIONS

 EXAMPLE 2.2.2. Graph the equation y = 5 − x 2 .


 SOLUTION: Whenever an equation is solved for y, we can attempt to sketch it by evalu-
 ating it for convenient values of x, and then connecting the dots, so to speak.

 If x = 0, then y = 5 − (0)2 = 5 − 0 = 5

 If x = 0.5 then y = 5 − (0.5)2 = 5 − 0.25 = 4.75

 If x = 1, then y = 5 − (1)2 = 5 − 1 = 4

 If x = 2, then y = 5 − (2)2 = 5 − 4 = 1

 If x = 3, then y = 5 − (3)2 = 5 − 9 = −4

 If x = −0.5, then y = 5 − (−0.5)2 = 5 − 0.25 = 4.75

 In fact, negative x will result in the same y values as their positive inverses, so we also
 get the points (−1, 4), (−2, 1) and (−3, −4). We can summarize the points we have found
 in a table of the corresponding x and y coordinates:
                       x −3 −2 −1 −0.5 0               0.5   1 2   3
                        y −4     1        4   4.75 5 4.75 4 1 −4

 We plot these points on the Cartesian plane:


                                                   y
                                               6

                                               4

                                               2
                                                                                x
              −3        −2           −1                  1         2        3
                                              −2

                                              −4

                                              −6

 At last, we draw a smooth curve through the dots.


                                               161
2. GRAPHING LINEAR EQUATIONS                                           CHAPTER 3. GRAPHING

                                           ANSWER:

                                                 y
                                             6

                                             4

                                             2
                                                                                 x
              −3         −2          −1                  1         2         3
                                            −2

                                            −4

                                            −6


   2.3. Graphing Lines.

  THEOREM 2.3.1. A graph of a linear equation in variables x and y
                                          Ax + B y = C
  where not both A and B are zero, is always a straight line. Conversely, every straight line
  on the Cartesian plane is the solution set of some linear equation in this form.


So the task of graphing a linear equation can be reduced to plotting just two solutions, and
then drawing a straight line through these points.

  EXAMPLE 2.3.1. Graph the equation y = 2x − 3


  SOLUTION: We can substitute any two numbers for x, but the best choices would make it
  easy to find the corresponding y, so we will stick to small integers.

  If x = 0, then y = 2(0) − 3 = −3

  If x = 1, then y = 2(1) − 3 = −1

  We plot the points (0, −3) and (1, −1) on the plane:




                                              162
CHAPTER 3. GRAPHING                                                2. GRAPHING LINEAR EQUATIONS

                                               3
                                                   y
                                               2
                                               1
                                                                   x
                                  −4 −3 −2 −1          1   2   3       4
                                            −1
                                              −2
                                              −3
                                              −4
                                              −5


 Finally, we draw a straight line through the points we found.


                                          ANSWER:
                                               3
                                                   y
                                               2
                                               1
                                                                   x
                                  −4 −3 −2 −1          1   2   3       4
                                            −1
                                              −2
                                              −3
                                              −4
                                              −5



 EXAMPLE 2.3.2. Graph the equation 3x − 5 y = 10


 SOLUTION: Guessing the solutions for this equation is inconvenient, so we solve it for y.
 The result is an equivalent equation which makes it really easy to find solution pairs.
                   3x − 5 y   =    10
                       −5 y   =    10 − 3x
                      −5 y          10 − 3x
                              =
                      −5              −5
                                          3
                          y   =    −2 +     x                 distributivity
                                          5
 It looks like we will have easier time if we substitute multiples of 5 for x.
                        3
 If x = 0, then y = −2 + (0) = −2
                        5

                                              163
2. GRAPHING LINEAR EQUATIONS                                               CHAPTER 3. GRAPHING
                        3
 If x = 5, then y = −2 + (5) = −2 + 3 = 1
                        5
 We plot the points (0, −2) and (5, 1) on the plane:

                                     3
                                         y
                                     2

                                     1
                                                                               x
                     −5 −4 −3 −2 −1          1    2    3   4   5   6   7   8       9
                                  −1

                                    −2

                                    −3

                                    −4


 Finally, we draw a straight line through the points we found.


                                         ANSWER:
                                     3
                                         y
                                     2

                                     1
                                                                               x
                    −5 −4 −3 −2 −1           1    2    3   4   5   6   7   8       9
                                 −1

                                    −2

                                    −3

                                    −4




                                                 164
CHAPTER 3. GRAPHING                                    2. GRAPHING LINEAR EQUATIONS
   Homework 3.2.

Graph the equation and highlight two dis-         10. y = 4
tinct points on the line.
                                                  11. x + y = −1
1. y = x + 2
                                                  12. x − y = −3
2. y = x − 2
                                                  13. x + 5 y = −15
3. y = 2x
                                                  14. 4x + y = 5
      1
4. y = x
      3                                           15. 8x − y = 5
5. y = −2x + 1                                    16. 3x + 4 y = 16
6. y = −x − 3                                     17. y + 3 = 0
        4                                         18. x − 2 = 0
7. y = − x − 3
        5
                                                  19. 2x + 3 y = 0
      3
8. y = x − 5
      2                                           20. −4x + 3 y = 0
9. x = −1




                                            165
 2. GRAPHING LINEAR EQUATIONS                                                                  CHAPTER 3. GRAPHING
      Homework 3.2 Answers.

1.                                                                      7.
                      6   y                                                                         6       y
                      5                                                                             5
                      4                                                                             4
                      3                                                                             3
                      2                                                                             2
                      1                                                                             1
                                                          x                                                                             x
     −6 −5 −4 −3 −2 −1        1   2       3       4   5       6               −6 −5 −4 −3 −2 −1                 1   2   3       4   5       6
                     −1                                                                       −1
                    −2                                                                         −2
                    −3                                                                         −3
                    −4                                                                         −4
                    −5                                                                         −5
                    −6                                                                         −6



3.                                                                      9.
                      5
                          y                                                                         3       y
                      4
                                                                                                    2
                      3

                      2
                                                                                                    1
                      1
                                                          x                                                                             x
     −5 −4 −3 −2 −1           1       2       3       4       5               −3    −2    −1                        1           2           3
                  −1
                                                                                               −1
                    −2

                    −3
                                                                                               −2
                    −4

                    −5                                                                         −3


5.                                                                      11.
                      5                                                                                 4
                          y                                                                                     y
                      4
                                                                                                        3
                      3
                                                                                                        2
                      2
                                                                                                        1
                      1
                                                          x                                                                                 x
     −5 −4 −3 −2 −1           1       2       3       4       5                −4   −3   −2    −1                   1       2       3           4
                  −1
                                                                                                    −1
                    −2
                                                                                                    −2
                    −3
                                                                                                    −3
                    −4

                    −5                                                                              −4

                                                                  166
CHAPTER 3. GRAPHING                                                                2. GRAPHING LINEAR EQUATIONS
13.                                                                        17.
                        6                                                                      5
                             y                                                                     y
                        5                                                                      4
                        4
                                                                                               3
                        3
                                                                                               2
                        2
                        1                                                                      1
                                                             x                                                         x
      −6 −5 −4 −3 −2 −1          1   2       3       4   5       6               −5 −4 −3 −2 −1        1   2   3   4       5
                      −1                                                                      −1
                       −2
                                                                                              −2
                       −3
                                                                                              −3
                       −4
                       −5                                                                     −4

                       −6                                                                     −5


15.                                                                        19.
                        7                                                                      5
                             y                                                                     y
                        6
                                                                                               4
                        5
                        4                                                                      3
                        3                                                                      2
                        2
                                                                                               1
                        1
                                                             x                                                         x
      −7 −6 −5 −4 −3 −2 −1       1   2   3       4   5   6       7               −5 −4 −3 −2 −1        1   2   3   4       5
                        −1
                                                                                              −1
                       −2
                       −3                                                                     −2
                       −4                                                                     −3
                       −5
                                                                                              −4
                       −6
                       −7                                                                     −5




                                                                     167
3. INTERCEPTS                                                         CHAPTER 3. GRAPHING
                                          3. Intercepts
   3.1. x and y-intercepts.

  DEFINITION 3.1.1. For any equation in variables x and y we define x-intercepts as the
  points where the graph of the equation meets the x-axis. In other words, x-intercepts are
  all the points (a, 0) which solve the given equation.

  Similarly, we define y-intercepts as the points where the graph of the equation meets
  the y-axis. In other words, y-intercepts are all the points (0, b) which solve the given
  equation.


  EXAMPLE 3.1.1. Find the intercepts for the pictured graph:

                                                   y
                                              5

                                                                x
                           −10       −5                     5    10

                                            −5




      ANSWER: There are three x-intercepts: (−6, 0), (−2, 0), and (5, 0). There is one
                                  y-intercept: (0, −3).

   3.2. Finding Intercepts.

  THEOREM 3.2.1. The most straightforward way to find x-intercepts from an equation is
  to replace y by 0 and then solve for x. Similarly, the standard way to find y-intercepts is
  to replace x by 0 in the equation and then solve for y.


  EXAMPLE 3.2.1. Find the intercepts for the equation 4x − 5 y = 40, then use them to plot
  the line.


  SOLUTION: To find x-intercepts, substitute 0 for y and solve for x:
                                     4x − 5(0)         =   40
                                              4x       =   40
                                               x       =   10


                                              168
CHAPTER 3. GRAPHING                                                           3. INTERCEPTS
 We found the x coordinate of the intercept, and the y coordinate is 0 by definition.

 To find y-intercepts, substitute 0 for x and solve for y:
                                       4(0) − 5 y     =    40
                                            −5 y      =    40
                                                y     =    −8
 We found the y coordinate of the intercept, and the x coordinate is zero by definition.
 Now we plot the x-intercept (10, 0) and the y-intercept (0, −8) on the plane, and draw a
 line through them.

                              15   y

                              10


                               5

                                                                     x
                        −5                  5         10        15   20

                              −5


                             −10


                             −15


                     ANSWER: x-intercept (10, 0), y-intercept (0, −8)


 EXAMPLE 3.2.2. Find the intercepts for the equation 2x = 17.


 SOLUTION: There is no y in this equation, so it is a vertical line, and the only intercept is
 an x-intercept. We can still follow the steps, though. To find the x-intercept, we solve for
 x and get x = 8.5. To find the y-intercept, we need to plug in 0 for x and solve for y, but
 the equation 0 = 17 has no solutions, so there are no y-intercepts.


                     ANSWER: x-intercept (8.5, 0), y-intercept: none

                                                169
3. INTERCEPTS                                                          CHAPTER 3. GRAPHING

  EXAMPLE 3.2.3. Find the intercepts for the equation −3 y + 6 = 0.


  SOLUTION: There is no x in this equation, so it’s a vertical line, and the only intercept is a
  y-intercept. To find the y-intercept, we solve for y and get y = 2. To find the x-intercept,
  we need to plug in 0 for y and solve for x, but the equation 6 = 0 has no solutions, so
  there are no x-intercepts.


                       ANSWER: x-intercept: none, y-intercept (0, 2)




                                              170
CHAPTER 3. GRAPHING                                                        3. INTERCEPTS
   Homework 3.3.

Find the coordinates of all x-intercepts and         15. x − 2 y = 4
all y-intercepts without graphing the line.
                                                     16. 3x − y = 9
1. 2x − 7 y = 14
                                                     17. y = 2x + 6
2. 3x + 5 y = 15
                                                     18. y = −3x + 5
3. 9x + 2 y = 36
                                                     19. 3x − 9 = 3 y
4. 10x + 3 y = 60
                                                     20. 4x − 5 y = 20
5. x − 3 = 0
                                                     21. −5x + 3 y = 180
6. 0 = 4 + y
                                                     22. 10x + 7 y = 280
7. y = 2x + 6
                                                     23. y = 30 − 3x
8. y = 3x − 12
                                                     24. 40 + y = 5x
9. 5x = 4 y
                                                     25. x + 5 = 1
10. −5x − 6 y = 120
                                                     26. y = 3
11. 4x + y = 10
                                                     27. −4x = 20 y + 80
12. 3x = 20 + 6 y
                                                     28. 60 − 20x = 3x

                                                     29. 3 y = −x
Find the intercepts, and plot the line with
intercepts clearly shown.                            30. 2 y − 4x = 0

13. 3x + 4 y = 12                                    31. y = −2

14. 2x − y = 6                                       32. x = 7




                                               171
 3. INTERCEPTS                                                            CHAPTER 3. GRAPHING
      Homework 3.3 Answers.

1. x-intercept (7, 0), y-intercept (0, −2)            17.
                                                                                y

                                                                            6
3. x-intercept (4, 0), y-intercept (0, 18)                                  5



5. x-intercept (3, 0), y-intercept: none                                                         x
                                                            −4 −3 −2                2        4

7. x-intercept (−3, 0), y-intercept (0, 6)
                                                                          −5

9. x-intercept (0, 0), y-intercept (0, 0)


11. x-intercept (2.5, 0), y-intercept (0, 10)

                                                      19.
                                                                                y
13.
                        y                                                   4
                    4
                    3                                                       2
                    2
                                                                                                 x
                                      x                     −4       −2             2   3    4
        −5                      4 5
                                                                          −2
                   −2                                                     −3
                                                                          −4
                   −4




15.                                                   21.
                        y                                                       y
                    4
                                                                           60
                                                                           50
                    2


                                      x                                                          x
        −5                      4 5                              −50−36                 50

                   −2
                                                                          −50

                   −4


                                                172
CHAPTER 3. GRAPHING                                        3. INTERCEPTS
23.                                    29.
                    y                                  y
               40
                                                   5
               30
               20


                             x                                     x
      −10               10                   −5                5

              −20


                                                  −5
              −40


25.                                    31.
                    y
                                                       y

                5                                  5




                             x
                                                                   x
       −5−4             5
                                             −5                5
                                                  −2

              −5
                                                  −5


27.
                    y

                5




                             x
      −20               20



              −4
              −5




                                 173
4. SLOPE                                                                CHAPTER 3. GRAPHING
                                              4. Slope

     4.1. Slope of a Line. The slope of a line can be understood as a real number which quan-
tifies the how steep the line is. One of the basic applications of the slope is the grade of the
road.
  DEFINITION 4.1.1. The grade of a straight segment of a road is the ratio of the vertical
  displacement, known as rise, and the horizontal displacement, known as run. It is tra-
  ditionally stated in percent, so 9% grade corresponds to the ratio 0.09. In the following
  illustration, the road with upward slope looks like a hypotenuse of the right triangle with
  the horizontal leg corresponding to the run, and the vertical leg corresponding to the rise.

                      rise                               road
            grade =                                                                rise
                      run
                                                          run


  EXAMPLE 4.1.1. Find the grade of the road if it rises 400 feet over the horizontal distance
  of 1500 feet.


  SOLUTION: The grade is 450/1500 = 0.3, or 30%


                                         ANSWER: 30%

The slope of a line on a coordinate plane is defined similarly, except that the rise carries a sign,
and positive slopes correspond to lines going upward and to the right, while negative slopes
correspond to lines going downward and to the right.

  DEFINITION 4.1.2. The slope of a segment with distinct endpoints (x 1 , y1 ) and (x 2 , y2 ) is
  denoted as m and can be computed using the slope formula, displayed below. The slope
  of a vertical segment is left undefined because in that case x 1 = x 2 , the denominator of
  the slope formula is zero, and the result of division is undefined.


                                         y2

                     y2 − y1                                                    y2 − y1
                                     y




               m=
                     x2 − x1             y1
                                                            x2 − x1
                                                  x1                       x2
                                                                  x



                                                174
CHAPTER 3. GRAPHING                                                                     4. SLOPE

  DEFINITION 4.1.3. The slope of a line is the slope of any segment with two distinct end-
  points on that line. The slope of a vertical line is left undefined.


The slope of a segment with endpoints (x 1 , y1 ) and (x 2 , y2 ) can be understood as the ratio of
rise and run, with rise represented by y2 − y1 and run represented by x 2 − x 1 . The slope can
also be understood as the change of y coordinate per unit change of x coordinate, or as the
rate of change of y with respect to x.
   4.2. Using Slope and Intercept to Plot.

  EXAMPLE 4.2.1. Plot the line with slope 1/2 and y-intercept (0, 1)


  SOLUTION: The slope 1/2 means that
                                            1 rise
                                              =
                                            2 run
  so the line rises 1 unit every time the x coordinate increases by 2 units. We can plot the
  y-intercept (0, 1), which is a point on the line, then count 2 units to the right, 1 unit up,
  and plot the point (2, 2). Having found two points, we can complete the graph by drawing
  a line through them.


                                            ANSWER:
                                               5
                                                   y
                                               4

                                               3

                                               2

                                               1
                                                                       x
                               −5 −4 −3 −2 −1          1   2   3   4       5
                                            −1

                                              −2

                                              −3

                                              −4

                                              −5




                                               175
4. SLOPE                                                                           CHAPTER 3. GRAPHING

  EXAMPLE 4.2.2. Plot the line with slope −3 and y-intercept (0, 4)


  SOLUTION: The slope −3 can be thought of as a fraction
                                            −3 rise
                                                 =
                                             1      run
  so the line falls 3 units every time the x coordinate increases by 1 unit. We can plot the
  y-intercept (0, 4), which is a point on the line, then count 1 unit to the right, 3 units down,
  and plot the point (1, 1). Having found two points, we can complete the graph by drawing
  a line through them.


                                            ANSWER:
                                               6   y
                                               5
                                               4
                                               3
                                               2
                                               1
                                                                           x
                              −6 −5 −4 −3 −2 −1        1   2   3   4   5       6
                                              −1
                                              −2
                                              −3
                                              −4
                                              −5
                                              −6




                                               176
CHAPTER 3. GRAPHING                                                              4. SLOPE
   Homework 3.4.

Plot the line using the slope and the inter-         21. (10, 18), (−11, −10)
cept.
                                                     22. (−3, 6), (−20, 13)
1. Slope 2/3, y-intercept (0, −1)
                                                     23. (−16, −14), (11, −14)
2. Slope 3/5, y-intercept (0, 1)
                                                     24. (13, 15), (2, 10)
3. Slope 0, y-intercept (0, 1)
                                                     25. (7, −14), (−8, −9)
4. Slope 0, y-intercept (0, −5)
                                                     26. (−18, −5), (14, −3)
5. Slope 3, y-intercept (0, 4)
                                                     27. (−5, 7), (−18, 14)
6. Slope −3, y-intercept (0, 2)
                                                     28. (19, 15), (5, 11)
7. Slope −2, y-intercept (0, −3)

8. Slope 2, y-intercept (0, 0)

9. Slope undefined, x-intercept (−3, 0)

10. Slope undefined, x-intercept (4, 0)

11. Slope −4/5, y-intercept (0, 6)

12. Slope −1/3, y-intercept (0, 5)

13. Slope 1/2, y-intercept (0, 0)

14. Slope 5/3, y-intercept (0, −2)



Find the slope of a segment with given end-
points.

15. (2, 10), (3, 15)

16. (3, 4), (7, 12)

17. (−2, 10), (−2, −15)

18. (1, 2), (−6, −14)

19. (−15, 10), (16, −7)

20. (13, −2), (7, 7)
                                               177
4. SLOPE                                                                                               CHAPTER 3. GRAPHING
Find the slope of the pictured line.                                               32.
                                                                                                           6    y
                                                                                                           5
29.
                        7                                                                                  4
                             y
                        6
                                                                                                           3
                        5
                                                                                                           2
                        4
                                                                                                           1
                        3
                                                                                                                                                        x
                        2
                                                                                         −6 −5 −4 −3 −2 −1          1       2       3       4   5           6
                        1                                                                                −1
                                                                     x
                                                                                                          −2
      −7 −6 −5 −4 −3 −2 −1       1       2   3   4       5       6       7
                        −1
                                                                                                          −3
                       −2
                                                                                                          −4
                       −3
                       −4                                                                                 −5

                       −5                                                                                 −6
                       −6
                       −7                                                          33.
                                                                                                           7
                                                                                                                y
                                                                                                           6
30.
                        5                                                                                  5
                             y
                                                                                                           4
                        4
                                                                                                           3
                        3                                                                                  2
                                                                                                           1
                        2                                                                                                                               x
                        1                                                                −7 −6 −5 −4 −3 −2 −1       1       2   3       4   5       6       7
                                                                                                           −1
                                                                     x                                    −2
      −5 −4 −3 −2 −1                 1       2       3       4           5                                −3
                   −1
                                                                                                          −4
                       −2                                                                                 −5
                                                                                                          −6
                       −3
                                                                                                          −7
                       −4

                       −5                                                          34.
                                                                                                           5
                                                                                                                y
31.                                                                                                        4
                         4
                             y                                                                             3
                         3                                                                                 2

                         2                                                                                 1
                                                                                                                                                        x
                         1                                                               −5 −4 −3 −2 −1                 1       2       3       4           5
                                                                                                      −1
                                                                     x
      −4 −3 −2 −1                        1       2           3           4                                −2
                −1                                                                                        −3

                       −2                                                                                 −4

                                                                                                          −5
                       −3

                       −4


                                                                             178
CHAPTER 3. GRAPHING                                                                                  4. SLOPE
35.                                                       36.
                    5                                                            6
                        y                                                            y
                    4                                                            5
                                                                                 4
                    3
                                                                                 3
                    2
                                                                                 2
                    1                                                            1
                                            x                                                                x
      −5 −4 −3 −2 −1        1   2   3   4       5               −6 −5 −4 −3 −2 −1        1   2   3   4   5       6
                   −1                                                           −1
                                                                               −2
                   −2
                                                                               −3
                   −3
                                                                               −4
                   −4                                                          −5
                   −5                                                          −6




                                                    179
 4. SLOPE                                                                                         CHAPTER 3. GRAPHING
      Homework 3.4 Answers.

1.                                                                          7.
                          6                                                                         5
                              y                                                                             y
                          5                                                                         4
                          4
                                                                                                    3
                          3
                                                                                                    2
                          2
                          1                                                                         1
                                                              x                                                                             x
     −6 −5 −4 −3 −2 −1            1       2   3   4       5       6               −5 −4 −3 −2 −1                    1   2       3       4       5
                     −1                                                                        −1
                         −2
                                                                                                   −2
                         −3
                                                                                                   −3
                         −4
                         −5                                                                        −4

                         −6                                                                        −5



3.                                                                          9.
                          4
                              y                                                                     5
                                                                                                            y
                          3
                                                                                                    4
                                                                                                    3
                          2
                                                                                                    2
                          1
                                                                                                    1
                                                              x                                                                             x
     −4   −3   −2   −1                1       2       3           4               −5 −4 −3 −2 −1                    1   2       3       4       5
                         −1                                                                    −1
                                                                                                  −2
                         −2
                                                                                                  −3
                         −3
                                                                                                  −4
                         −4                                                                       −5


5.                                                                          11.
                          6                                                                             7
                              y                                                                                 y
                          5                                                                             6

                          4                                                                             5
                                                                                                        4
                          3
                                                                                                        3
                          2
                                                                                                        2
                          1                                                                             1
                                                              x                                                                                 x
     −6 −5 −4 −3 −2 −1            1       2   3   4       5       6                −7 −6 −5 −4 −3 −2 −1             1   2   3       4   5   6       7
                     −1                                                                              −1
                                                                                                    −2
                         −2
                                                                                                    −3
                         −3
                                                                                                    −4
                         −4
                                                                                                    −5
                         −5                                                                         −6
                         −6                                                                         −7

                                                                      180
CHAPTER 3. GRAPHING                                                            4. SLOPE
13.                                                                  4
                             4                                 21.
                                 y                                   3
                             3
                                                               23. 0
                             2
                                                                         1
                             1                                 25. −
                                                                         3
                                                 x
      −4     −3   −2   −1            1   2   3       4
                                                                          7
                            −1                                 27. −
                                                                         13
                            −2
                                                                     3
                            −3                                 29.
                                                                     2
                            −4
                                                               31. undefined
15. 5
                                                                     5
17. undefined                                                  33.
                                                                     6
        17
19. −                                                          35. −1
        31




                                                         181
5. SLOPE-INTERCEPT FORM                                               CHAPTER 3. GRAPHING
                                    5. Slope-Intercept Form
   5.1. Slope-Intercept Form.

  DEFINITION 5.1.1. Every non-vertical line with slope m and y-intercept b is a solution set
  for an equation in the slope-intercept form
                                           y = mx + b
  If either m or b is zero, the corresponding term is not shown.


This definition can be thought of as a theorem with an easy proof.

  THEOREM 5.1.1. Given any two reals m and b, the slope of the line
                                           y = mx + b
  is m, and the y-intercept is b.


   PROOF. We can find the y-intercept of y = mx + b by making x zero and solving for y:
                                       y    =     mx + b
                                       y    =     m·0+ b
                                       y    =     b
To show that the slope is m, take any two points on the line, like (0, b) and (1, m + b), and use
the slope formula
                                    y2 − y1   (m + b) − b     m
                           slope =          =              =     =m
                                   x2 − x1       1−0          1
                                                                                               

  BASIC EXAMPLE 5.1.1. Some lines in slope-intercept form:

  y = 3x − 2    has slope 3 and y-intercept (0, −2)

  y = x +6     has slope 1 and y-intercept (0, 6)

  y = 17    has slope 0 and y-intercept (0, 17)

  y = −x     has slope −1 and y-intercept (0, 0)


  BASIC EXAMPLE 5.1.2. Some lines not in slope-intercept form:

  2 y = 3x + 4 is not solved for y

  y = −2(x + 5) has a product instead of a sum on the right


                                                182
CHAPTER 3. GRAPHING                                               5. SLOPE-INTERCEPT FORM

       4x − 5
  y=            has a fraction instead of a sum on the right
         2


  EXAMPLE 5.1.1. Find the slope and the y-intercept of the line
                                         y = −6x − 14


  SOLUTION: This equation is already in the slope-intercept form, so −6 is the slope and
  (0, −14) is the y-intercept.


                          ANSWER: slope −6, y-intercept (0, −14)


To put a linear equation in the slope-intercept form, one can solve for y, remove parentheses,
and state the right side in a simplified form.

  EXAMPLE 5.1.2. Find the slope, the y-intercept, and the equation in the slope-intercept
  form for the line
                                     x − 2y − 4 = 8


  SOLUTION: We will actually find the slope-intercept form first, which will provide us with
  everything else. We start by solving the equation of the line for the y variable.
                 x − 2y − 4    =   8                isolate the term with y on the left
         x − 2y − 4 + 4 − x    =   8+4− x           by adding 4 and −x on both sides
                       −2 y    =   −x + 12                         combined like terms
                                    −x + 12
                          y    =                               divided both sides by −2
                                      −2
                                    −x 12
                          y    =      +                 rewrote as a sum of two terms
                                    −2 −2
                                    1
                          y    =      x −6
                                    2


  This is the slope-intercept form, so 1/2 is the slope and (0, −6) is the y-intercept.


                           ANSWER: slope 1/2, y-intercept (0, −6)


                                              183
5. SLOPE-INTERCEPT FORM                                                 CHAPTER 3. GRAPHING

 EXAMPLE 5.1.3. If possible, find the slope, the y-intercept, and the equation in the slope-
 intercept form for the line
                                        −2 y + 3 = −5



 SOLUTION: Solve the equation for the y variable:
                                      −2 y + 3      =   −5
                                  −2 y + 3 − 3      =   −5 − 3
                                           −2 y     =   −8
                                             y      =   4


 There is no x, but we can pretend that its coefficient is zero:
                                           y = 0x + 4
 This is the slope-intercept form, so 0 is the slope and (0, 4) is the y-intercept.



                            ANSWER: slope 0, y-intercept (0, 4)



 EXAMPLE 5.1.4. If possible, find the slope, the y-intercept, and the equation in the slope-
 intercept form for the line
                                         2x = 3x + 5



 SOLUTION: There is no y, so it is not possible to solve this equation for y. If there are any
 solutions at all, this is a vertical line, and the slope is undefined. We solve this equation
 for x to find the x-intercept:
                  2x    =    3x + 5
             2x − 3x    =    3x + 5 − 3x
                  −x    =    5                                   not yet solved for x
                    x   =    −5                         multiplied both sides by −1
 This is a vertical with the x-intercept (−5, 0), so the slope is undefined and there are no
 y-intercepts.



                         ANSWER: slope undefined, no y intercepts



                                              184
CHAPTER 3. GRAPHING                                                               5. SLOPE-INTERCEPT FORM

 EXAMPLE 5.1.5. Determine the equation of the pictured line and state the answer in the
 slope-intercept form.


                                                    4
                                                        y

                                                    3

                                                    2

                                                    1

                                                                                  x
                            −4    −3   −2     −1            1       2     3           4
                                                   −1

                                                   −2

                                                   −3

                                                   −4




 SOLUTION: The y-intercept is (0, −2), so b = −2. To find the slope, take any two points
 on the line, like (0, −2) and (2, 1), and use the slope formula with (0, −2) for (x 1 , y1 ) and
 (2, 1) for (x 2 , y2 ):
                                        y2 − y1   1 − (−2) 3
                                 m=             =          =
                                       x2 − x2      2−0      2


                                                            3
                                       ANSWER: y =            x −2
                                                            2


 EXAMPLE 5.1.6. Determine the equation of the pictured line and state the answer in the
 slope-intercept form.


                                                        y
                                                   60


                                                   40


                                                   20

                                                                              x
                                 −10     −5                     5       8 10



                                                    185
5. SLOPE-INTERCEPT FORM                                                          CHAPTER 3. GRAPHING

  SOLUTION: The y-intercept is (0, 40), so b = 40. To find the slope, take any two points
  (x 1 , y1 ) and (x 2 , y2 ) on the line, like (0, 40) and (8, 0), and use the slope formula:
                                               y2 − y1    0 − 40
                                        m=             =         = −5
                                               x2 − x2     8−0


                                    ANSWER: y = −5x + 40



    5.2. Parallel Lines. More than 2000 years ago, Euclid provided an axiomatic definition
of parallel lines. The classical axioms of geometry imply that given a line and a point not on
the line, there exists a unique and distinct parallel line passing through that point, and that
parallel lines have no points in common. On a coordinate plane, we can provide a consistent
definition involving the slope.

  DEFINITION 5.2.1. Two distinct lines are parallel if their slopes are equal. Just as in clas-
  sical geometry, a line is not parallel to itself, even though it has the same slope as itself.
  This tradition, however, has very little bearing on anything we consider in this text.


                                                        y
                                                    6

                                                    5
                                                                m1 = m2
                                                    4
         Parallel lines never intersect:
                                                    3

                                                    2

                                                    1
                                                                                                   x
                                                            1     2   3   4    5       6   7   8



  EXAMPLE 5.2.1. Determine whether the two given lines are parallel.
                              y    =       4x − 5                             line 1
                       8x − 2 y    =       0                                  line 2


  SOLUTION: We will find the slopes and compare them. The first equation is in slope-
  intercept form, so the slope of the first line m1 = 4, and its y-intercept is (0, −5). To find



                                                186
CHAPTER 3. GRAPHING                                              5. SLOPE-INTERCEPT FORM
 the slope of the second line, we solve its equation for the y variable:
                                       8x − 2 y     =   0
                                  8x − 2 y − 8x     =   0 − 8x
                                            −2 y    =   −8x
                                            −2 y        −8x
                                                    =
                                            −2          −2
                                               y    =   4x


 This is now also in the slope-intercept form, so the slope of the second line m2 = 4 and its
 y-intercept is (0, 0). Since the slopes are equal and y-intercepts are different, we conclude
 that the lines are parallel. (If y-intercepts were the same, we would be looking at two
 equivalent equations for a single line.)


                                      ANSWER: parallel


 EXAMPLE 5.2.2. Determine whether the two given lines are parallel.
                          y   =     −2x + 4                      line 1
                      −3 y    =     6x − 12                      line 2


 SOLUTION: Divide the equation of the second line by −3 on both sides to solve it for y:
                                −3 y        6x − 12
                                        =
                                 −3             −3
                                                    6x   12
                                        y     =        −
                                                    −3 −3
                                        y     =    −2x + 4
 So the two equations are equivalent, and the solution sets are the same.


                                    ANSWER: a single line




                                              187
5. SLOPE-INTERCEPT FORM                                              CHAPTER 3. GRAPHING
    5.3. Perpendicular Lines. Another useful relationship between two lines is perpendicu-
larity. A geometrical definition requires a right angle between the lines (angle measure of 90◦
or π/2 radians), but on a coordinate plane we can provide a consistent definition involving the
slope.

  DEFINITION 5.3.1. Two lines are perpendicular (or orthogonal) if their slopes m1 and m2
  are negative reciprocals of each other:
                                            m1 m2 = −1
  Moreover, every vertical line is perpendicular to every horizontal line, even though the
  slope of a vertical line is undefined.


  THEOREM 5.3.1. If neither line is vertical, the definition can be restated in a form that
  shows the negative reciprocals explicitly. Two lines with non-zero slopes m1 and m2 are
  perpendicular if either
                                                  1
                                         m1 = −
                                                 m2
  or
                                                  1
                                         m2 = −
                                                 m1



                       Perpendicular lines intersect at the right angle:


                       y

                   6

                   5
                               m1 m2 = −1
                   4

                   3
                                                  90◦
                   2

                   1
                                                                           x
                           1      2    3      4         5   6   7    8



                                               188
CHAPTER 3. GRAPHING                                                    5. SLOPE-INTERCEPT FORM

 EXAMPLE 5.3.1. Determine whether the two given lines are parallel, perpendicular, or
 neither.
                              y       =       −3x + 8                    line 1
                       3y − x         =       4                          line 2



 SOLUTION: We will find the slopes and compare them. The first equation is in slope-
 intercept form, so m1 = −3. To find the slope of the second line, we solve its equation for
 the y variable:
             3y − x      =        4
         3y − x + x      =        x +4
                  3y     =        x +4
                 3y               x +4
                         =                          need to rewrite the right side as a sum
                  3                 3
                             x 4
                   y     =     +
                             3 3
 This is now in the slope-intercept form, so the slope m2 = 1/3. The slopes are not equal,
 so we check whether they are negative reciprocals of each other:
                                                  1
                                    m1 m2 = −3 · = −1
                                                  3
 They are, so the lines are perpendicular.



                                          ANSWER: perpendicular


 EXAMPLE 5.3.2. Determine whether the two given lines are parallel, perpendicular, or
 neither.
                          y       =       6                          line 1
                          x       =       −6                         line 2



 SOLUTION: The first line is horizontal and the second one is vertical, so they are perpen-
 dicular.


                                          ANSWER: perpendicular


                                                    189
5. SLOPE-INTERCEPT FORM                                                  CHAPTER 3. GRAPHING

 EXAMPLE 5.3.3. Determine whether the two given lines are parallel, perpendicular, or
 neither.
                          2x + y       =    4                       line 1
                          x + 2y       =    2                       line 2


 SOLUTION: We will find the slopes and compare them. The first equation is equivalent to
                                                y = −2x + 4
 so m1 = −2. To find the slope of the second line, we solve its equation for the y variable
 and put it in the slope-intercept form:
                x + 2y      =      2
            x + 2y − x      =      2− x
                    2y      =      −x + 2
                    2y             −x + 2
                            =
                     2               2
                                   −x 2
                      y     =         +                   rewrote right side as a sum
                                    2   2
                                  1
                      y     =  − x +1                     slope-intercept form
                                  2
 so m2 = −1/2. The slopes are not equal, so the lines are not parallel. Next we check
 whether they are negative reciprocals of each other:
                                                   1
                                                ‹
                                  m1 m2 = (−2) −      =1
                                                   2
 The result differs from −1, so the slopes are not negative reciprocals of each other, and
 the lines are not perpendicular.


                                           ANSWER: neither




                                                   190
CHAPTER 3. GRAPHING                                                  5. SLOPE-INTERCEPT FORM
   Homework 3.5.

Find the slope and the y intercept (if any)          Determine the equation of the pictured line
from the equation by solving for y and find-         and state the answer in the slope-intercept
ing an equivalent equation in the slope-             form.
intercept form. Do not graph the line.
                                                     17.
      8                                                                         4
                                                                                    y
1. y = x − 4
      5                                                                         3

        5                                                                       2
2. y = − x + 13
        7                                                                       1

3. y = −60x                                                                                                         x
                                                           −4   −3   −2   −1                1       2       3           4
4. y = −30x − 14                                                               −1

                                                                               −2
5. 5x = 7 y
                                                                               −3
6. 8x − 9 y = 0
                                                                               −4
7. 12x − 6 y = 9
                                                     18.
                                                                                6   y
8. 2x − 5 y = 8                                                                 5
                                                                                4
9. y − 3 = 5
                                                                                3
                                                                                2
10. y = 3
                                                                                1
                                                                                                                    x
11. x = 4                                                  −6 −5 −4 −3 −2 −1            1       2   3   4       5       6
                                                                           −1
12. 1 = 7 − x                                                                  −2
                                                                               −3
13. 3x + 4 y = 12                                                              −4
                                                                               −5
14. 4x + 2 y = 8                                                               −6

15. −4x + y = 7

16. 3x − y = 6




                                               191
5. SLOPE-INTERCEPT FORM                                                                      CHAPTER 3. GRAPHING
19.                                                                        22.
                        7                                                                         6
                             y                                                                        y
                        6                                                                         5
                        5
                                                                                                  4
                        4
                                                                                                  3
                        3
                                                                                                  2
                        2
                        1                                                                         1
                                                             x                                                                x
      −7 −6 −5 −4 −3 −2 −1       1   2   3       4   5   6       7               −6 −5 −4 −3 −2 −1        1   2   3   4   5       6
                        −1                                                                       −1
                       −2
                                                                                                −2
                       −3
                                                                                                −3
                       −4
                                                                                                −4
                       −5
                       −6                                                                       −5
                       −7                                                                       −6


20.                                                                        23.
                        6    y                                                                    6   y
                        5                                                                         5
                        4                                                                         4
                        3                                                                         3
                        2                                                                         2
                        1                                                                         1
                                                             x                                                                x
      −6 −5 −4 −3 −2 −1          1   2       3       4   5       6               −6 −5 −4 −3 −2 −1        1   2   3   4   5       6
                      −1                                                                         −1
                       −2                                                                       −2
                       −3                                                                       −3
                       −4                                                                       −4
                       −5                                                                       −5
                       −6                                                                       −6


21.                                                                        24.
                        6    y                                                                    6   y
                        5                                                                         5
                        4                                                                         4
                        3                                                                         3
                        2                                                                         2
                        1                                                                         1
                                                             x                                                                x
      −6 −5 −4 −3 −2 −1          1   2       3       4   5       6               −6 −5 −4 −3 −2 −1        1   2   3   4   5       6
                      −1                                                                         −1
                       −2                                                                       −2
                       −3                                                                       −3
                       −4                                                                       −4
                       −5                                                                       −5
                       −6                                                                       −6




                                                                     192
CHAPTER 3. GRAPHING                                         5. SLOPE-INTERCEPT FORM
For each pair of lines, determine whether             30.
they are parallel, perpendicular, or neither.
                                                            6x − 3 y       =           9
25.
                                3                            2x − y        =           3
                 y      =         x −3
                                4
                                                      31.
          3x − 4 y      =   −2

26.                                                                x       =       7

              8x − 2 y      =     7                          x −8          =       0

            3x + 12 y       =     9                   32.

27.
                                                               x       =       4
              6x − 4 y      =     5
                                                               y       =       4
            8x + 12 y       =     3
                                                      33.
28.
          3x − 4 y      =   12                              x − 4y         =       8
                                3                           4x + y         =       2
                 y      =         x +3
                                4
                                                      34.
29.
             2x − 4 y       =     6                         2x + 5 y       =           3

               x − 2y       =     3                         5x − 2 y       =           6




                                                193
 5. SLOPE-INTERCEPT FORM                                       CHAPTER 3. GRAPHING
    Homework 3.5 Answers.

1. Slope 8/5, y-intercept (0, −4)           19. y = −5

3. Slope −60, y-intercept (0, 0)                     1
                                            21. y = − x + 2
                                                     2
5. Slope 5/7, y-intercept (0, 0)
                                                      2
                                            23. y =     x −3
7. Slope 2, y-intercept (0, −3/2)                     5

9. Slope 0, y-intercept (0, 8)              25. parallel

11. Slope undefined, no y-intercept         27. perpendicular

13. Slope −3/4, y-intercept (0, 3)          29. neither (same line)

15. Slope 4, y-intercept (0, 7)             31. parallel

17. y = 2x − 1                              33. perpendicular




                                      194
CHAPTER 3. GRAPHING                                                       6. POINT-SLOPE FORM
                                      6. Point-Slope Form
   6.1. A Point and a Slope Determine a Line.

  DEFINITION 6.1.1. Every non-vertical line with slope m and passing through the point
  (x 1 , y1 ) is a solution set for an equation in the point-slope form
                                       y − y1 = m(x − x 1 )


This definition can be thought of as a theorem with an easy proof.

  THEOREM 6.1.1. Given any real m and any point (x 1 , y1 ), the line
                                       y − y1 = m(x − x 1 )
  has slope m and passes through the point (x 1 , y1 ).



    PROOF. Substitute x 1 for x and y1 for y into the equation of the line to check that (x 1 , y1 )
is a solution:
                                    y1 − y1    =     m(x 1 − x 1 )
                                          0    =     m·0
                                          0    =   0


The equation holds, so the line passes through the point (x 1 , y1 ). To find the slope, find the
slope-intercept form by solving for the y variable:
                                     y − y1    =    m(x − x 1 )
                                     y − y1    =    mx − mx 1
                                y − y1 + y1    =    mx − mx 1 + y1
                                          y    =    mx − mx 1 + y1
This does not look like the slope-intercept form, but rewriting the right side as a sum of two
terms helps:
                                    y = mx + (−mx 1 + y1 )
So −mx 1 + y1 is the y-intercept, while m is the slope.                                      




                                               195
6. POINT-SLOPE FORM                                                    CHAPTER 3. GRAPHING

  EXAMPLE 6.1.1. Find an equation in point-slope form for a line with slope 7, passing
  through the point (3, 4).


  SOLUTION: Substitute 7 for m and (3, 4) for (x 1 , y1 ) in the point-slope formula
                                       y − y1 = m(x − x 1 )
  to obtain the answer.


                                  ANSWER: y − 4 = 7(x − 3)


  EXAMPLE 6.1.2. Find an equation in point-slope form for a line with slope −3/2, passing
  through the point (−2, 5).


  SOLUTION: After we substitute −2 for x 1 and 5 for y1 in the point-slope formula we get
                                            3
                                 y − 5 = − (x − (−2))
                                            2
  Replacing −(−2) by +2 makes the answer look nicer, and is considered point-slope form.


                                                  3
                                 ANSWER: y − 5 = − (x + 2)
                                                  2


  EXAMPLE 6.1.3. Determine the slope and the coordinates of at least one point on the line
                                       y + 1 = −5(x − 8)


  SOLUTION: This equation is in the point-slope form, so m = −5 must be the slope, and
  (x 1 , y1 ) = (8, −1) must be a point on the line.


                               ANSWER: Slope −5, point (8, −1)


    6.2. Two Points Determine a Line. The very first Euclidean postulate states that two dis-
tinct points determine a line, so it is natural to ask for an equation of a line, given two points
on the coordinate plane.

  EXAMPLE 6.2.1. Find the slope and an equation in the point-slope form for the line passing
  through the points (1, −7) and (−4, 3).


                                              196
CHAPTER 3. GRAPHING                                                       6. POINT-SLOPE FORM

 SOLUTION: Let (x 1 , y1 ) = (1, −7), (x 2 , y2 ) = (−4, 3), and find the slope using the slope
 formula:
                                  y2 − y1      3 − (−7)     10
                            m=              =            =     = −2
                                  x2 − x1      (−4) − 1 −5

 Substitute −2 for m and (1, −7) for (x 1 , y1 ) in the point-slope formula to get the answer:
                                      y − (−7) = −2(x − 1)


                                 ANSWER: y + 7 = −2(x − 1)


 EXAMPLE 6.2.2. Find the slope and an equation in the slope-intercept form for the line
 passing through the points (−12, 3) and (−3, 6).


 SOLUTION: We will find an equation of this line in the point-slope form first, and then
 solve it for y to get the slope-intercept form. Let (x 1 , y1 ) = (−12, 3), (x 2 , y2 ) = (−3, 6),
 and find the slope using the slope formula:
                                 y2 − y1       6−3              3 1
                            m=           =                  = =
                                 x2 − x1    (−3) − (−12) 9 3

 Substitute 1/3 for m and (−12, 3) for (x 1 , y1 ) in the point-slope formula to get the equation
 in the point-slope form:
                                               1
                                    y − 3 = (x − (−12))
                                               3
 Solve the equation for y and put the right side in the slope-intercept form:
                               1
                   y −3 =        (x + 12)                          point-slope form
                               3
                                  1    1
                    y −3     =      x + · 12                          distributivity
                                  3    3
                                  1
                    y −3     =      x +4
                                  3
                                  1
                y −3+3       =      x +4+3                     add 3 to both sides
                                  3
                                  1
                        y    =      x +7                     slope-intercept form
                                  3


                                                      1
                                     ANSWER: y =        x +7
                                                      3

                                               197
6. POINT-SLOPE FORM                                                                    CHAPTER 3. GRAPHING
  6.3. Using Point and Slope to Plot.

  EXAMPLE 6.3.1. Plot the line passing through the point (−4, −5) with slope 2.


  SOLUTION: We can think of the slope 2 as
                                          rise 2
                                              =
                                          run 1
  The point (−4, −5) is on the line, and going 2 units up and 1 unit to the right gives us
  another point on the line (−3, −3).


                                           ANSWER:
                                              7
                                                   y
                                              6
                                              5
                                              4
                                              3
                                              2
                                              1
                                                                               x
                            −7 −6 −5 −4 −3 −2 −1       1   2   3   4   5   6       7
                                              −1
                                             −2
                                             −3
                                             −4
                                             −5
                                             −6
                                             −7




                                              198
CHAPTER 3. GRAPHING                                                     6. POINT-SLOPE FORM
   Homework 3.6.

Given a slope and a point on the line, state          16. (2, 3) and (4, 1)
the equation of the line in point-slope form.
                                                      17. (−3, 1) and (3, 5)
1. Slope −1, point (3, 6)
                                                      18. (−3, 4) and (3, 1)
2. Slope 1, point (2, 8)
                                                      19. (−3, 5) and (−1, −3)
3. Slope −3, point (−2, −5)
                                                      20. (−4, −1) and (1, 9)
4. Slope −2, point (−3, −1)
                                                      21. (5, 0) and (0, −2)
5. Slope 7/2, point (2, −4)
                                                      22. (−2, 0) and (0, 3)
6. Slope −4/5, point (−2, 6)

7. Slope −3/5, point (−4, −8)                         Graph the line with the given slope and
                                                      point.
8. Slope 1/3, point (4, 1)
                                                      23. Slope 3/4, point (1, 2)

                                                      24. Slope 2/5, point (−3, −4)
Determine the slope and the coordinates of
at least one point on the line.                       25. Slope −4/3, point (−2, 5)

9. y − 17 = −6(x + 1)                                 26. Slope −3/2, point (1, 0)

10. y + 4 = 3(x − 2)

           1                                          Graph the equation.
11. y − 7 = (x + 4)
           2
                                                                 2
                                                      27. y + 2 = (x − 1)
             2                                                   3
12. y − 1 = − (x − 13)
             3
                                                                 3
                                                      28. y − 1 = (x + 5)
13. y + 12 = −0.4(x + 0.1)                                       4

        1         1                                                1
                   ‹
14. y − = 9 x −                                       29. y − 1 = − (x − 3)
        3         4                                                4
                                                                   1
                                                      30. y − 1 = − (x − 3)
                                                                   2
Given two points on the line, find the slope                     1
and the equation of the line and state the            31. y + 4 = (x − 1)
                                                                 2
answer in the slope-intercept form.
                                                                 1
15. (6, 8) and (3, 5)                                 32. y + 2 = (x + 1)
                                                                 3
                                                199
6. POINT-SLOPE FORM                                                                  CHAPTER 3. GRAPHING
      Homework 3.6 Answers.

1. y − 6 = −1(x − 3)                                             25.
                                                                                        6   y
                                                                                        5
3. y + 5 = −3(x + 2)                                                                    4
                                                                                        3
          7                                                                             2
5. y + 4 = (x − 2)                                                                      1
          2                                                                                                         x
                                                                       −6 −5 −4 −3 −2 −1        1   2   3   4   5       6
                                                                                       −1
            3
7. y + 8 = − (x + 4)                                                                  −2
            5
                                                                                      −3
                                                                                      −4
9. Slope −6, point (−1, 17)
                                                                                      −5
                                                                                      −6
         1
11. Slope , point (−4, 7)
         2
                                                                 27.
                                                                                        6   y
13. Slope −0.4, point (−0.1, −12)                                                       5
                                                                                        4
                                                                                        3
15. Slope 1, y = x + 2
                                                                                        2
                                                                                        1
                  2                                                                                                 x
17. Slope 2/3, y = x + 3                                               −6 −5 −4 −3 −2 −1        1   2   3   4   5       6
                  3                                                                    −1
                                                                                      −2
19. Slope −4, y = −4x − 7                                                             −3
                                                                                      −4

                      2                                                               −5
21. Slope 2/5, y =      x −2                                                          −6
                      5

23.                                                              29.
                       6   y                                                            6   y
                       5                                                                5
                       4                                                                4
                       3                                                                3
                       2                                                                2
                       1                                                                1
                                                   x                                                                x
      −6 −5 −4 −3 −2 −1        1   2   3   4   5       6               −6 −5 −4 −3 −2 −1        1   2   3   4   5       6
                      −1                                                               −1
                     −2                                                               −2
                     −3                                                               −3
                     −4                                                               −4
                     −5                                                               −5
                     −6                                                               −6

                                                           200
CHAPTER 3. GRAPHING                                              6. POINT-SLOPE FORM
31.
                       6   y
                       5
                       4
                       3
                       2
                       1
                                                   x
      −6 −5 −4 −3 −2 −1        1   2   3   4   5       6
                      −1
                     −2
                     −3
                     −4
                     −5
                     −6




                                                           201
7. FUNCTION NOTATION                                                  CHAPTER 3. GRAPHING
                                     7. Function Notation


    7.1. Notation. In modern mathematics, a function is a certain kind of collection of ordered
pairs. In this text, however, we have no real need for this kind of formalism, so we will only
define a specific kind of a function, which is determined by an algebraic expression.

  DEFINITION 7.1.1 (Functions defined by expressions). If a formula with variables x and
  y is solved for y, then we can say that it defines a function. We can give this function any
  name we want, like f , and then f will denote a rule for taking a number x, called input
  or argument of the function f , and computing the number y, called output or value of the
  function f .


  BASIC EXAMPLE 7.1.1. Let the function F be defined by the equation
                                           y = x 2 − 2x
  When the argument x = 4, the value of F is
                                         (4)2 − 2(4) = 8
  When the argument x = −6, the value of F is
                                 (−6)2 − 2(−6) = 36 + 12 = 48
  In other words, finding the value of F amounts to substituting the argument into the
  formula which defines the function, and computing the result.


  DEFINITION 7.1.2. Given a function f , the notation f (x) denotes the value of f corre-
  sponding to the argument x. While this looks exactly like a product of variables f and
  x, the awkward placement of parentheses is usually enough to help the reader realize
  that f is not a variable, but a name of a function. The parentheses here are not the usual
  algebraic parentheses, and so they do not obey the distributive property. The only way to
  get rid of f (x) in an expression is to replace it by the corresponding value of f .

  f (x) reads “ f of x” or “the value of f at x”.


It is very traditional to create functions which share the name with the variable used to define
them. For example, a function defined by
                                            y = x +1
may also be defined by
                                          y(x) = x + 1
and then it is also called y. Even so, it is usually easy to determine the meaning of an expres-
sion, since function names do not appear in arithmetic expressions without the parentheses
and the argument.
                                               202
CHAPTER 3. GRAPHING                                               7. FUNCTION NOTATION

  EXAMPLE 7.1.1. Let y be a function defined by
                                        y(x) = x − 4
  Find y(15).


  SOLUTION: Substitute 15 for x in the expression x − 4 to find the value of y(15):
                                   y(15) = (15) − 4 = 11


                                        ANSWER: 11


  EXAMPLE 7.1.2. Let g be a function defined by
                                                |x + 6|
                                       g(x) =
                                                   x2
  Find g(−10).


  SOLUTION: We have to evaluate the expression inside the absolute value bars before we
  can remove them:
                                             |(−10) + 6|
                              g(−10) =
                                               (−10)2
                                                  | − 4|
                                           =
                                                   100
                                                   4
                                           =
                                                  100
                                           =      0.04


                                       ANSWER: 0.04


The absolute value is actually a traditional notation for a function with the following rule:
if the argument (or input) is non-negative, then the value of the function is the same as the
argument; if the argument is negative, then the value is the opposite of the argument.




                                            203
7. FUNCTION NOTATION                                                   CHAPTER 3. GRAPHING

 EXAMPLE 7.1.3. Let h be a function defined by
                                        h(x) = x 3 + 2
 Find h(4) + h(5).



 SOLUTION: We can find values h(4) and h(5) separately and then add them:
                               h(4)     =     (4)3 + 2 = 64 + 2 = 66
                               h(5)     =     (5)3 + 2 = 125 + 2 = 127
                        h(4) + h(5)     =     66 + 127 = 193



                                        ANSWER: 193


 EXAMPLE 7.1.4. Let f be a function defined by f (x) = 12 − 5x. Find f (0) − f (10).



 SOLUTION: Another way to simplify this kind of expression is to substitute f (0) and f (10)
 with expressions right away, but then we have to introduce the substitution parentheses:
                       f (0) − f (10)   =     (12 − 5(0)) − (12 − 5(10))
                                        =     12 − (−38)
                                        =     50



                                        ANSWER: 50



  7.2. Applications of Linear Functions.

 DEFINITION 7.2.1. A function f is linear if it can be defined by a linear expression
                                        f (x) = mx + b
 A graph of this linear function is the graph of the corresponding linear equation
                                            y = mx + b


 DEFINITION 7.2.2. A linear model is a linear function which is designed to model a rela-
 tionship between two quantities in an application.


                                               204
CHAPTER 3. GRAPHING                                               7. FUNCTION NOTATION

 EXAMPLE 7.2.1. A water pump fills a water tank with water at a constant rate. There
 are 14 gallons of water in the tank to begin with, and 30 seconds after the pump starts
 working the amount of water in the tank goes up to 24 gallons. Let x represent the
 number of seconds the pump was working, and let y(x) represent the amount of water
 in the tank at the corresponding time.

     (1) Find a linear model y(x) for the amount of water in the tank x seconds after the
         pump starts working.
     (2) Use the model to predict the amount of water in the tank 2 minutes after the
         pump starts working.
     (3) Use the model to predict the number of seconds needed to fill the tank with 100
         gallons of water.


 SOLUTION: When the pump turns on, the timer is at x = 0 and the number of gallons
 in the tank is y(x) = 14. After 30 seconds, x = 30 and y(x) = 24. It may be helpful to
 represent the two given data points as a table:
                                         x    y(x)
                                        0      14
                                        30     24

     (1) We need an equation of the line which goes through the points (0, 14) and (30, 24).
         The first point happens to be a y-intercept, but we still need to find the slope:
                                       24 − 14 10 1
                                  m=            =    =
                                        30 − 0    30 3
         Using the slope-intercept form y = mx + b we can write the linear model:
                                              1
                                      y(x) = x + 14
                                              3
     (2) The variable x measures time in seconds, so we need to convert 2 minutes into
         120 seconds before using our linear model:
                                    1
                          y(120) = · (120) + 14 = 40 + 14 = 54
                                    3
         So 2 minutes after the pump starts, there are 54 gallons of water in the tank.
     (3) Substitute 100 for y(x) in the equation of the model and solve for x:
                              1
                   100 =        x + 14
                              3
             100 − 14    =   13x + 14 − 14                  addition property
                              1
                   86    =      x                        combined like terms
                              3



                                             205
7. FUNCTION NOTATION                                                 CHAPTER 3. GRAPHING
          To finish solving for x we multiply both sides by 3, which is the reciprocal of 1/3:
                                 1
                                ‹
                (86) · 3 =         x ·3               multiplication property
                                 3
                  258    =    x
          So the pump has to work for 258 seconds before the tank has 100 gallons of
          water.


                                          ANSWER:

                  1
     (1) y(x) =     x + 14
                  3
     (2) 54 gallons
     (3) 258 seconds


 EXAMPLE 7.2.2. In year 2000, Brent’s private library contained 240 historical manuscripts,
 and by 2017 the collection grew to contain 461 historical manuscripts. Let x represent
 the year since 2000 and let y(x) represent the number of manuscripts in the library in
 that year.

     (1) Find a linear model y(x) for the number of manuscripts in a given year.
     (2) Use the model to predict the number of manuscripts in year 2024.
     (3) Use the model to predict the year when the library obtains the 1000th manu-
         script.


 SOLUTION: Since x represents the year since 2000, x and the year are related by
                                      year = 2000 + x
 It may be helpful to represent the two given data points as a table:
                                       Year    x    y(x)
                                      2000 0        240
                                      2017 17       461

     (1) We need an equation of the line which goes through the points (0, 240) and
         (17, 461). The first point happens to be a y-intercept, but we still need to find
         the slope:
                                    461 − 240 221
                               m=             =      = 13
                                      17 − 0     17
         Using the slope-intercept form y = mx + b we can write the linear model:
                                      y(x) = 13x + 240


                                              206
CHAPTER 3. GRAPHING                                             7. FUNCTION NOTATION
     (2) Year 2024 corresponds to x = 24:
                              y(24) = 13 · 24 + 240 = 552
         So the library will have 552 manuscripts in year 2024.
     (3) Substitute 1000 for y(x) in the equation of the model and solve for x:
                                1000     =   13x + 240
                                  760    =   13x
                                    6
                                 58 13   =   x
         x is somewhat over 58, so 1000th manuscript will be added sometime during the
         year 2058.


                                         ANSWER:

     (1) y(x) = 13x + 240
     (2) 552 manuscripts
     (3) year 2058




                                           207
7. FUNCTION NOTATION                                                   CHAPTER 3. GRAPHING
   Homework 3.7.

1. Find f (4) if f (x) = 3x + 1                           (1) Find a linear model T (x) for the
                                                              coffee temperature x minutes af-
2. Find g(2) if g(t) = t 3 + 1                                ter it was served.
                                                          (2) Use the model to predict the tem-
3. Find h(−6) if h(x) = 2 − 3x                                perature 8 minutes after the cof-
                                                              fee was served.
4. Find p(−3) if p(t) = t 2 + t
                                                          (3) Use the model to determine
                                                              the time when the temperature
5. Find F (−4) if F (s) = |1 + s|
                                                              reaches 74 ◦ F.
6. Find G(0) if G(x) = 2(3x − 7)
                                                      13. The population of a small town was
                           t −4                       32.6 thousand people in 1990 and it grew
7. Find H(−2) if H(t) =                               to 33.2 thousand people by year 2000. Let
                             t
                                                      x be the number of years since 1990, and
                           x2 − 1                     let y(x) be the population in thousands of
8. Find K(−5) if K(x) =
                             6                        people.

9. Find P(−1) if P(x) = x 4 − 4x 3
                                                          (1) Find a linear model P(x) for
10. Find Q(−3) if Q(s) = 2s3 − s2
                                                              the town population in the corre-
11. The temperature of the air was 41 ◦ F at                  sponding year.
sunrise, and 56 ◦ F 30 minutes after the sun-             (2) Use the model to predict the pop-
rise. Let x represent the number of min-                      ulation in year 2030.
utes since sunrise and let T (x) represent                (3) Use the model to determine the
the temperature at that time.                                 time when the population reaches
                                                              34.4 thousand people.
     (1) Find a linear model T (x) for the
         air temperature x minutes after              14. A racing car accelerates from zero to
         the sunrise.                                 60 km/s over 2.5 seconds. Let t be the
     (2) Use the model to predict the tem-            elapsed time in seconds and let v(t) be the
         perature 40 minutes after the sun-           speed of the car in km/s at that time.
         rise.
     (3) Use the model to determine
         the time when the temperature                    (1) Find a linear model v(t) for the
         reaches 46 ◦ F.                                      speed of the car t seconds after
                                                              start.
12. A cup of hot coffee is served at 97 ◦ F,              (2) Use the model to predict the
and 5 minutes later it cools down to 89 ◦ F.                  speed after 3 seconds.
Let x be the number of minutes since the                  (3) Use the model to determine the
coffee was served, and let T (x) be the tem-                  time when the speed reaches 108
perature at that time.                                        km/s.



                                                208
CHAPTER 3. GRAPHING                               7. FUNCTION NOTATION
       Homework 3.7 Answers.

1. 13                                11.
                                                         1
                                           (1) T (x) =     x + 41
3. 20                                                    2
                                           (2) 61 ◦ F
                                           (3) 10 minutes
5. 3
                                     13.
7. 3                                       (1) P(x) = 0.06x + 32.6
                                           (2) 35 thousand people
9. 5                                       (3) year 2020




                               209
7. PRACTICE TEST 3                                                       CHAPTER 3. GRAPHING
                                       Practice Test 3




1. Graph the equation and highlight two             10. Determine the equation of the pictured
distinct points on the line.                        line and state it in the slope-intercept form:
                   1
               y =− x +3                                                 4
                                                                             y
                   2
                                                                         3
2. Find the coordinates of all x-intercepts
                                                                         2
and all y-intercepts:
                                                                         1
               3x − y = 12
                                                                                             x

3. Find the coordinates of all x-intercepts         −4   −3   −2   −1            1   2   3       4
                                                                        −1
and all y-intercepts:
                     7    3                                             −2
               y=      x−
                     2    2                                             −3

4. Plot the line with slope 2 and y-                                    −4
intercept (0, −4).
                                                    11. Find an equation of the line with slope
5. Find the slope of the segment with end-          0.5 passing through the point (4, −6) and
points (−6, −3) and (−3, 2).                        state it in slope-intercept form.

6. Find an equivalent equation in slope-            12. Find an equation of the line passing
intercept form:                                     through the points (−1, 4) and (4, 14), and
              10x − 4 y = 8                         state it in slope-intercept form.

7. Find the slope of the line passing               13. Evaluate the function F (−4) if
through the points (3, 0) and (3, −4).                             F (x) = 1 − x − x 2

8. Determine whether the lines are paral-           14. Find an equation of the line with un-
lel, perpendicular, or neither:                     defined slope passing through the point
                                                    (−7, 7).
              y+x     =    4

            2y − 7    =    2x                       15. Find an equation of the line passing
                                                    through the point (5, 0) and parallel to the
9. Find an equation of the line with slope          line
−5 passing through the point (−2, 3) and                            x+ y =3
state it in point-slope form.                       State the answer in slope-intercept form.




                                              210
 CHAPTER 3. GRAPHING                                                                             7. PRACTICE TEST 3
      Practice Test 3 Answers.

1.                                                                        5. 5/3
                       7
                            y
                       6
                       5
                                                                                   5
                       4                                                  6. y =     x −2
                       3                                                           2
                       2
                       1
                                                            x
                                                                          7. undefined
     −7 −6 −5 −4 −3 −2 −1       1   2   3       4   5   6       7
                       −1
                      −2
                      −3
                                                                          8. perpendicular
                      −4
                      −5
                      −6
                      −7                                                  9. y − 3 = −5(x + 2)

2. x-intercept (4, 0), y-intercept (0, −12)
                                                                                   1    4
                 3                        3                               10. y = − x +
                     ‹                    ‹
3. x-intercept     , 0 , y-intercept 0, −                                          3    3
                 7                        2
4.
                       6    y                                             11. y = 0.5x − 8
                       5
                       4
                       3                                                  12. y = 2x + 6
                       2
                       1
                                                            x
                                                                          13. −11
     −6 −5 −4 −3 −2 −1          1   2       3       4   5       6
                     −1
                      −2
                      −3                                                  14. x = −7
                      −4
                      −5
                      −6                                                  15. y = −x + 5




                                                                    211
                                         CHAPTER 4


                                      Linear Systems


                             1. Graphing Systems of Equations

    1.1. Solution Sets. Recall that solutions to a linear equation in two variables x and y are
ordered pairs, which we can plot as points on the Cartesian plane. Recall also that the solution
set for any linear equation is a straight line.

  DEFINITION 1.1.1. A system of linear equations in two variables is two or more linear equa-
  tions in the same variables x and y, and it is written with a curly brace like this:
                                         x+y = 5
                                       

                                         x−y = 4
  An ordered pair is a solution for a system if it is a solution for each and every equation in
  the system.


                                                                               x+y = 5
                                                                           
  EXAMPLE 1.1.1. Determine whether (2, 3) is a solution for the system
                                                                               x−y = 4


  SOLUTION: Substitute 2 for x and 3 for y in the first equation:
                                           x+y      =   5
                                       (2) + (3)    =   5
                                               5    =   5
  So it is a solution for the first equation. Now we substitute 2 for x and 3 for y in the
  second equation:
                                           x−y      =   4
                                       (2) − (3)    =   4
                                             −1     =   4
  So it is not a solution for the second equation, and therefore not a solution for the system.


                                         ANSWER: No

                                              213
1. GRAPHING SYSTEMS OF EQUATIONS                                 CHAPTER 4. LINEAR SYSTEMS

  EXAMPLE 1.1.2. Determine whether (4.5, 0.5) is a solution for the system
                                     x+y = 5
                                  

                                     x−y = 4


  SOLUTION: Substitute 4.5 for x and 0.5 for y in the first equation:
                                            x+y      =   5
                                     (4.5) + (0.5)   =   5
                                                 5   =   5
  So it is a solution for the first equation. Now we substitute 4.5 for x and 0.5 for y in the
  second equation:
                                            x−y      =   4
                                     (4.5) − (0.5)   =   4
                                                 4   =   4
  So it is a solution for the second equation, and therefore a solution for the system.


                                        ANSWER: Yes


Knowing that solution sets of individual linear equations look like lines, we can try to figure
out the possibilities for the solution sets for systems of two equations. Ordered pairs which
satisfy both equations will look like points which belong to both graphs. So there are only
three major possibilities.

Two lines intersect at a          Two lines are parallel,             Two lines coincide, yield-
point, yielding a single so-      yielding an empty solu-             ing infinitely many solu-
lution.                           tion set.                           tions


           4                                 4                                  4
               y                                 y                                  y
           2                                 2                                  2
                       x                                 x                                    x
−4    −2           2       4       −4   −2           2       4        −4    −2            2       4
        −2                                −2                                  −2

         −4                                −4                                 −4




                                             214
CHAPTER 4. LINEAR SYSTEMS                                 1. GRAPHING SYSTEMS OF EQUATIONS
  1.2. Solving Systems by Graphing.

 EXAMPLE 1.2.1. Solve the system by graphing:
                                      x−y = 2
                                   

                                      x+y = 6


 SOLUTION: Let us plot both lines on the same grid. Here it will be convenient to use
 intercepts for graphing. The first line x − y = 2 has intercepts (2, 0) and (0, −2). The
 second line x + y = 6 has intercepts (6, 0) and (0, 6).


                                        8
                                            y

                                        6

                                        4

                                        2
                                                                      x
                             −4   −2             2        4       6       8
                                       −2

                                       −4

 The lines seem to intersect precisely at (4, 2). We can check the solution by making sure
 x = 4 and y = 2 satisfy both equations:
                                    (4) − (2)         =       2
                                    (4) + (2)         =       6


                                       ANSWER: (4, 2)


                                                          y = 10x − 70
                                                      
 EXAMPLE 1.2.2. Solve the system by graphing:
                                                          y = −20x + 80


 SOLUTION: Let us plot both lines on the same grid. Here it will be convenient to use the
 slope-intercept form for graphing. The first line y = 10x − 70 has y-intercept (0, −70)
 and slope 10, which we can plot as going up 10 units and 1 unit to the right. The second
 line y = −20x + 80 has y-intercept (0, 80) and slope −20, which we can plot by finding
 another point on the line 20 units down and 1 unit to the right.

                                                215
1. GRAPHING SYSTEMS OF EQUATIONS                                               CHAPTER 4. LINEAR SYSTEMS

                         100   y
                          80
                          60
                          40
                          20
                                                                                             x
                 −2    −1           1      2           3           4   5        6        7       8
                         −20
                        −40
                        −60
                        −80

 The lines seem to intersect precisely at (5, −20). We can check the solution by making
 sure x = 4 and y = 2 satisfy both equations:
                                   (−20)       =       10(5) − 70
                                   (−20)       =       −20(5) + 80


                                        ANSWER: (5, −20)


                                                                   2 y = 8x
                                                               
 EXAMPLE 1.2.3. Solve the system by graphing:
                                                                     y = 4x + 3


 SOLUTION: The first line in slope-intercept form is y = 4x, which is a line with slope 4
 passing through the origin. The second line has slope 4 and y-intercept 3.


                                                   5
                                                           y
                                                   4
                                                   3
                                                   2
                                                   1
                                                                                 x
                          −3       −2     −1                       1       2         3
                                               −1
                                               −2
                                               −3
                                               −4
                                               −5


                                                   216
CHAPTER 4. LINEAR SYSTEMS                               1. GRAPHING SYSTEMS OF EQUATIONS
 These lines have the same slope, but different y-intercepts, so they are parallel, there are
 no intersections, and the solution set is empty.


                                   ANSWER: no solutions


                                                        y+x = 4
                                                    
 EXAMPLE 1.2.4. Solve the system by graphing:
                                                          y = 4− x


 SOLUTION: Solving the first equation for y shows that it is equivalent to the other one:
                                         y+x      =      4
                                   y+x−x          =      4− x
                                              y   =      4− x
 Graphing either equation yields the same line:


                                          6   y

                                          4


                                          2

                                                                 x
                              −4    −2              2        4       6

                                         −2


                                         −4

 Every point satisfying the first equation also satisfies the second equation, so there are
 infinitely many solutions. Formally, we can write down the solution set in the set-builder
 notation, indicating that any ordered pair which solves one of the equations will also solve
 the entire system:                  
                                       (x, y) y = 4 − x
 Without going into specifics, we can also say that there are infinitely many solutions (as
 many as there are points on a line).


                            ANSWER: infinitely many solutions




                                              217
1. GRAPHING SYSTEMS OF EQUATIONS                              CHAPTER 4. LINEAR SYSTEMS
      Homework 4.1.

                                                            6x + y = −3
                                                        
Determine whether the order pair is a so-
lution for the given system of equations.         12.
                                                             x+y = 2
                 5x − 2 y = −3
             
                                                                  3
                                                      
1. (1, 4),                                             y = − x +1
                 7x − 3 y = −5
                                                      
                                                                  4
                                                  13.
                                                       y = −3 x + 2
                                                      
                      3x + 2 y = −5
                  
2. (−15, 20),                                                     4
                      4 y + 5x = 5                    
                                                       y = 2x − 4
                                                      
                 3x − 2 y = 0
             
3. (3, 2),                                        14.
                                                                1
                  x + 2 y = 15                         y =
                                                      
                                                                  x +2
                                                                2
                 3x − y = 5
             
                                                                5
                                                      
4. (2, 1),                                             y =       x +4
                  x+y = 3
                                                      
                                                                3
                                                  15.
                                                       y = −2 x − 3
                                                      
                      x − 3y = 1
                 
5. (−2, −1),                                                      3
                      x + 2y = 0
                                                                1
                                                      
                                                       y =
                                                                 x +4
           
                   1                                            2
            y = − x +3
                                                 16.
6. (4, 1),
                   2                                   y = 1x +1
                                                      
            y = 3x −2
                                                               2
                 4                                    
                                                       y = −x − 2
                                                      
                                                  17.
                                                                2
Solve the system by graphing.                          y =
                                                      
                                                                  x +3
                                                                3
         y = −x + 1
     
                                                        x + 4 y = −12
                                                      
7.                                                18.
         y = −5x − 3                                    2x + y = 4

         y = −3
     
                                                            2x + 3 y = −6
                                                        
8.                                                19.
         y = −x − 4                                          2x + y = 2

          x + 3 y = −9
     
                                                            3x + 2 y = 2
                                                        
9.                                                20.
         5x + 3 y = 3                                       3x + 2 y = −6

          y = 2x + 2
      
                                                            2x + y = −2
                                                        
10.                                               21.
          y = −x − 4                                        x + 3y = 9

           x−y = 4
      
                                                               x −3 = 3
                                                        
11.                                               22.
          2x + y = 2                                        5x + 2 y = 8
                                            218
CHAPTER 4. LINEAR SYSTEMS                                 1. GRAPHING SYSTEMS OF EQUATIONS
    Homework 4.1 Answers.

1. Yes                                                   11. (2, −2)
                                                                             5
                                                                                     y
                                                                             4
                                                                             3
3. No
                                                                             2
                                                                             1
                                                                                                                 x
                                                          −5 −4 −3 −2 −1                 1       2   3       4       5
                                                                       −1
5. No
                                                                         −2
                                                                         −3
                                                                         −4
                                                                         −5
7. (−1, 2)
               4
                   y
                                                         13. no solutions
               3
                                                                         5
               2                                                                 y
                                                                         4
               1                                                         3
                                           x
                                                                         2
 −4 −3 −2 −1           1       2       3       4                         1
           −1                                                                                            x

             −2                                           −5 −4 −3 −2 −1             1       2   3   4       5
                                                                       −1

             −3                                                         −2
                                                                        −3
             −4
                                                                        −4
                                                                        −5



9. (3, −4)                                               15. (−3, −1)
               5                                                             5
                   y                                                                 y
               4                                                             4
               3                                                             3
               2                                                             2
               1                                                             1
                                           x                                                                     x
 −5 −4 −3 −2 −1        1   2       3   4       5          −5 −4 −3 −2 −1                 1       2   3       4       5
              −1                                                       −1
             −2                                                          −2
             −3                                                          −3
             −4                                                          −4
             −5                                                          −5
                                                   219
1. GRAPHING SYSTEMS OF EQUATIONS                               CHAPTER 4. LINEAR SYSTEMS
17. (−3, 1)                                          21. (−3, 4)
               5                                                    5
                   y                                                    y
               4                                                    4
               3                                                    3
               2                                                    2
               1                                                    1
                                       x                                                    x
 −5 −4 −3 −2 −1        1   2   3   4       5          −5 −4 −3 −2 −1        1   2   3   4       5
              −1                                                   −1
              −2                                                   −2
              −3                                                   −3
              −4                                                   −4
              −5                                                   −5

19. (3, −4)
               5
                   y
               4
               3
               2
               1
                                       x
 −5 −4 −3 −2 −1        1   2   3   4       5
              −1
              −2
              −3
              −4
              −5




                                               220
CHAPTER 4. LINEAR SYSTEMS                                                      2. SUBSTITUTION
                                         2. Substitution

   2.1. Solving Systems using Substitution. We can solve systems of linear equations al-
gebraically by eliminating one of the variables using substitution, and then solving a linear
equation for the remaining variable.

                                         7x + y = 14
                                     
  EXAMPLE 2.1.1. Solve the system
                                         y − 3x = −6


  SOLUTION: Choose a variable to eliminate. Any variable will do, but the ones with low
  coefficients tend to produce the least tedium. We choose y in the first equation, since it
  has coefficient 1. Then we find an equivalent equation which is solved for y:
  (1)      7x + y     =   14                                              first equation
  (2)            y    =   14 − 7x            use this equation later, when x is found
  Now we can use algebraic substitution to eliminate y from the other equation, by replacing
  it with (14 − 7x). This yields a linear equation in one variable x, which we solve by usual
  means.
                       y − 3x   =    −6                              second equation
             (14 − 7x) − 3x     =    −6                     substitute (14 − 7x) for y
                     14 − 10x   =    −6                          combined like terms
              14 − 10x − 14     =    −6 − 14
                        −10x    =    −20
                       −10x          −20
                                =
                       −10           −10
                            x   =    2
  Finally, we substitute 2 for x in the equation (2), which we solved for y.
                                         y   =    14 − 7x
                                         y   =    14 − 7(2)
                                         y   =    14 − 14
                                         y   =    0
  So the only solution is the ordered pair (2, 0).


                                         ANSWER: (2, 0)




                                                 221
2. SUBSTITUTION                                                   CHAPTER 4. LINEAR SYSTEMS
This solution technique can be summarized as follows.

  THEOREM 2.1.1. To solve a linear system of two equations in two variables,

       (1) Solve either equation for either variable to get
            y = expression with x              or                x = expression with y
       (2) Substitute the expression into the other equation, producing a linear equation in
           one variable.
       (3) Solve the resulting equation to get
                 x = number                    or                     y = number
       (4) substitute the number you found into the equation from the first step to find the
           other coordinate.


When we are dealing with systems which have empty or infinite solution sets, the third step
of the procedure will result in an equation which is always false or always true respectively, as
the following examples illustrate.

                                          y = x +1
                                      
  EXAMPLE 2.1.2. Solve the system
                                          y = x −1


  SOLUTION: The first equation is already solved for y, so we substitute (x + 1) for y in the
  second equation:
                  (x + 1)   =    x −1
                x +1− x     =    x −1− x                   subtract x on both sides
                        1   =    −1
  This equation is always false, so no ordered pair (x, y) can ever satisfy the system.


                                      ANSWER: no solutions


                                          x + 3 y = −10
                                      
  EXAMPLE 2.1.3. Solve the system
                                              6 y = −2x − 20


  SOLUTION: We choose to eliminate x from the first equation because it has the most
  convenient coefficient.
                                      x + 3y   =     −10
                                           x   =     −3 y − 10

                                               222
CHAPTER 4. LINEAR SYSTEMS                                                2. SUBSTITUTION
 Substitute (−3 y − 10) for x into the other equation:
           6y    =    −2x − 20                               the other equation
           6y    =    −2(−3 y − 10) − 20            now we will solve this for y
           6y    =    6 y + 20 − 20
           6y    =    6y
 We can keep going and subtract 6 y on both sides to get an equivalent equation 0 = 0, but
 any time we get the same expression on both sides, we can instantly conclude that the
 linear equation is true for all numbers, which implies that the system has infinitely many
 solutions.


                           ANSWER: infinitely many solutions




                                           223
2. SUBSTITUTION                                 CHAPTER 4. LINEAR SYSTEMS
      Homework 4.2.

                                                     y = −6
                                          
Solve using substitution.
                                    11.
                                              3x − 6 y = 30
         y = −3x
     
1.
         y = 6x − 9                       
                                              6x − 4 y = −8
                                    12.
                                                     y = −6x + 2
         y = x +5
     
2.
         y = −2x − 4                      
                                              −2x − y = −5
                                    13.
                                               x − 8 y = −23
                x = 2y + 1
     
3.
         3x − 6 y = 2                     
                                              6x + 4 y = 16
                                    14.
                                              −2x + y = −3
               x = −3 y
     
4.
         x + 4 y = 10                     
                                               −6x + y = 20
                                    15.
                                              −3x − 3 y = −18
         y = −2x − 9
     
5.
         y = 2x − 1                       
                                              7x + 5 y = −13
                                    16.
                                               x − 4 y = −16
         y = −6x + 3
     
6.
         y = 6x + 3                       
                                              y − 2x = −6
                                    17.
                                              2y − x = 5
                 y = 6x − 6
     
7.
         −3x − 3 y = −24                  
                                              2x − y = 0
                                    18.
                                              y − 2x = −2
         −x + 3 y = 12
     
8.
                y = 6x + 21               
                                              2x + 3 y = −2
                                    19.
                                               2x − y = 9
             y = 3x − 1
     
9.          1
         x − y = 13
            3
                                          
                                                 y = 2x + 5
                                    20.
                                              −2 y = −4x − 10
             x − 3y = 7
      
10.
          −4x + 12 y = 28




                              224
 CHAPTER 4. LINEAR SYSTEMS                                     2. SUBSTITUTION
    Homework 4.2 Answers.

1. (1, −3)                           13. (1, 3)

3. ∅
                                     15. (−2, 8)
5. (−2, −5)
                                               17 16
                                                      ‹
7. (2, 6)                            17.         ,
                                                3 3
9. infinitely many solutions
                                               25 11
                                                          ‹
                                     19.          ,−
11. (−2, −6)                                    8    4




                               225
3. ELIMINATION                                                  CHAPTER 4. LINEAR SYSTEMS
                                         3. Elimination

    3.1. Equivalent Systems. Another algebraic technique for solving systems is known as
Gaussian elimination or simply elimination, or sometimes addition. It relies on two ways of
obtaining an equivalent system of equations. We do not need the full power of the Gaussian
elimination to solve a system of two equations in two variables, so we present a much simplified
version of the technique here.

  DEFINITION 3.1.1. Systems of linear equations are equivalent if they have the same solu-
  tion sets.


  THEOREM 3.1.1. Given a system of equations, we can replace either of the equations
  by an equivalent one to obtain an equivalent system. In particular, we can apply the
  multiplication property to either of the equations. Formally, for any real m 6= 0 these two
  systems are equivalent:
                           a = b                          =
                                                 
                                                      a           b
                           c = d                    m(c) = m(d)


  THEOREM 3.1.2. Given a system of equations, we can apply the addition property to either
  of the equations. In particular, we can use the fact that the two sides of the first equation
  are equal, and add them to the two sides of the other equation. Formally, these two
  systems are equivalent:
                            a = b                            =
                                                 
                                                       a          b
                            c = d                    c+a = d+b



    3.2. Eliminating Variable with Addition. In the following examples we will use proper-
ties of systems to eliminate one of the variables in one of the equations, solve for that variable,
then substitute the value we found into the other equation, and find the other coordinate of
the solution.
  EXAMPLE 3.2.1. Solve the system using elimination:
                                      2x + y = 7
                                   

                                      3x − y = 3


  SOLUTION: Notice that coefficients of the y variable are opposites of each other. If we
  add left sides of these equations, then y will cancel (will get eliminated) and we will be
  able to solve for x. So we are going to replace the second equation in the system by a sum
  of the given equations, by writing that the sum of left sides is equal to the sum of right
  sides:

                                               226
CHAPTER 4. LINEAR SYSTEMS                                                       3. ELIMINATION

                                           2x + y = 7
                             

                              (3x − y) + (2x + y) = 3 + 7
 Simplifying the second equation yields
             (3x − y) + (2x + y)      =      3+7
                 3x + 2x − y + y      =      10
                                 5x   =      10                  combined like terms
                                  x   =      2
 Now we can substitute 2 for x into the first equation and solve for y:
                                          2x + y        =   7
                                       2(2) + y         =   7
                                             4+ y       =   7
                                                  y     =   3
 So the only solution is the point (2, 3).


                                       ANSWER: (2, 3)


                                                                x − 4y = 6
                                                            
 EXAMPLE 3.2.2. Solve the system using elimination:
                                                                2x + y = 12


 SOLUTION: If we add equations now, neither variable will get eliminated. But we can
 start by applying the multiplication property so that coefficients for one of the variables
 become opposites of each other. As before, it is probably easier to eliminate the variable
 with smaller coefficients, so we will try to get rid of x first. Here we only need to multiply
 the first equation by −2 on both sides:
                                   −2(x − 4 y) = −2(6)
                                 

                                          2x + y = 12
 Simplifying the first equation yields an equivalent system:
                                      −2x + 8 y = −12
                                   

                                         2x + y = 12
 Now that the coefficients for x are opposites of each other, we can replace the second
 equation using the addition property for systems:
                                      −2x + 8 y = −12
                      

                         (2x + y) + (−2x + 8 y) = (12) + (−12)


                                                  227
3. ELIMINATION                                                          CHAPTER 4. LINEAR SYSTEMS
  Simplifying the second equation yields
                          (2x + y) + (−2x + 8 y)          =       (12) + (−12)
                                  2x − 2x + y + 8 y       =       0
                                                 9y       =       0
                                                      y   =       0
  Now we can substitute y = 0 into the other original equation x − 4 y = 6 and solve for x:
                                            x − 4y        =   6
                                           x − 4(0)       =   6
                                                 x        =   6
  So the only solution is the point (6, 0).


                                           ANSWER: (6, 0)


                                                                      2x − 5 y = 12
                                                              
  EXAMPLE 3.2.3. Solve the system using elimination:
                                                                      3x + 2 y = −1


  SOLUTION: This is, in a way, a worst case scenario, as far as coefficients are concerned.
  Before we can eliminate a variable using addition, we need to create a pair of opposite
  coefficients, and here we will have to multiply both equations. We will opt to eliminate
  x first because the coefficients seem a bit simpler. In order to do so, we will multiply the
  first equation by 3 on both sides, and the second equation by −2 on both sides:
                                      3(2x − 5 y) = 3(12)
                                 

                                    −2(3x + 2 y) = −2(−1)
  Simplifying both equations yields
                                           6x − 15 y = 36
                                       

                                           −6x − 4 y = 2
  Notice that now the coefficients for x are opposites of each other, so we can eliminate x
  using the addition property:
                                             6x − 15 y = 36
                         

                            (−6x − 4 y) + (6x − 15 y) = 2 + 36
  Simplifying the second equation yields
                                              −19 y = 38
  And solving it for y gives us
                                               y = −2


                                                 228
CHAPTER 4. LINEAR SYSTEMS                                                3. ELIMINATION
 Now we can substitute −2 for y into the original equation 2x − 5 y = 12 and solve for x:
                                       2x − 5 y     =   12
                                    2x − 5(−2)      =   12
                                        2x + 10     =   12
                                              2x    =   2
                                               x    =   1
 So the only solution is the point (1, −2).


                                     ANSWER: (1, −2)




                                              229
3. ELIMINATION                                  CHAPTER 4. LINEAR SYSTEMS
      Homework 4.3.

                                               2x − y = 5
                                          
Solve using elimination.
                                    11.
                                              5x + 2 y = −28
          4x + 2 y = 0
     
1.
         −4x − 9 y = −28                  
                                              −5x + 6 y = −17
                                    12.
                                                x − 2y = 5
         −7x + y = −10
     
2.
         −9x − y = −22                    
                                              10x + 6 y = 24
                                    13.
                                               −6x + y = 4
         −x − 2 y = −7
     
3.
          x + 2y = 7                      
                                                x + 3 y = −1
                                    14.
                                              10x + 6 y = −10
         −9x + 5 y = −22
     
4.
          9x − 5 y = 13                   
                                               2x + 4 y = 24
                                    15.
                                              4x − 12 y = 8
         −6x + 9 y = 3
     
5.
          6x − 9 y = −9                   
                                              −6x + 4 y = 12
                                    16.
                                              12x + 6 y = 18
         5x − 5 y = −15
     
6.
         5x − 5 y = −15                   
                                              8x − 5 y = −9
                                    17.
                                              3x + 5 y = −2
         4x − 6 y = −10
     
7.
         4x − 6 y = −14                   
                                              4x + 6 y = −1
                                    18.
                                               x − 3y = 2
         −3x + 3 y = −12
     
8.
         −3x + 9 y = −24                  
                                               x + 9y = 1
                                    19.
                                              2x − 6 y = 10
         −x − 5 y = 28
     
9.
         −x + 4 y = −17                   
                                              2x − 15 − 10 y = 40
                                    20.
                                                         28 = x − 4 y
           −10x − 5 y = 0
      
10.
          −10x − 10 y = −30




                              230
 CHAPTER 4. LINEAR SYSTEMS                              3. ELIMINATION
    Homework 4.3 Answers.

1. (−2, 4)                              11. (−2, −9)

3. Infinite number of solutions         13. (0, 4)

                                        15. (8, 2)
5. ∅
                                        17. (−1, 0.2)
7. ∅
                                                 1
                                                  ‹
                                        19. 4, −
9. (−3, −5)                                      3




                                  231
4. APPLICATIONS OF SYSTEMS                                        CHAPTER 4. LINEAR SYSTEMS
                                 4. Applications of Systems
   4.1. Constant Rate Problems.

  EXAMPLE 4.1.1. Suzy pays $111 for an order of 4 black printer cartridges and 7 color
  cartridges. When supplies start running low, she goes back to the same store and pays
  $126 for an order of 3 black and 10 color cartridges. Find the price of one black cartridge
  and the price of one color cartridge.


  SOLUTION: Let b and c be the prices of one black and one color cartridge respectively, in
  dollars. We can write an equation for the total price of each order. Since 4 black cartridges
  cost 4b dollars and 7 color cartridges cost 7c dollars,
                                            4b + 7c = 111
  Similarly, the second order can be expressed as
                                        3b + 10c = 126
  To solve this system by elimination, we can multiply both sides of the first equation by
  3, and both sides of the second equation by −4, so that we can add and eliminate the b
  variable:
                   3(4b + 7c) = 3(111)
             

                −4(3b + 10c) = −4(126)
                  12b + 21c = 333
             
                                                      simplified both equations
                −12b − 40c = −504
  Now the coefficients for b are opposites of each other, and after adding equations we get
      (12b + 21c) + (−12b − 40c)        =     333 + (−504)
                              −19c      =     −171                      canceled like terms
                                   c    =     9                  divided both sides by −19
  Now we substitute 9 for c in the original equation 3b + 10c = 126 and solve for b:
               3b + 10(9)    =    126
                  3b + 90    =    126
                        3b   =    36                    subtracted 90 on both sides
                         b   =    12                        divided both sides by 3


                                         ANSWER:
           b and c are prices of black and color cartridges respectively, in dollars
                                        4b + 7c = 111
                                     

                                        3b + 10c = 126
                                   solution: b = 12,       c=9

                                                  232
CHAPTER 4. LINEAR SYSTEMS                                           4. APPLICATIONS OF SYSTEMS
    4.2. Mixing Problems. Mixing problems may assume many forms. Traditional applica-
tions revolve around mixing appropriate volumes of various chemical solutions in order to get
a solution with a specific concentration. In other applications, we could be “mixing” currency
bills of two different denominations or different kinds of coffee or grain.

  EXAMPLE 4.2.1. A chemist receives an order for 10 liters of 48% isopropyl solution, that
  is, a solution consisting of 48 parts of isopropyl alcohol and 52 parts of water by volume.
  The stockroom only has some 15% solution and some 70% solution. How many liters of
  each must be mixed in order to fulfill the order?


  SOLUTION: As usual, we start by assigning variables to the quantities we need to find.
  Let x and y be the needed volumes of 15% and 70% solutions respectively, measured in
  liters.

  We will solve this problem by writing down and solving a system of two equations. The
  first one will say that the total volumes of the two solutions add up to 10 liters. The
  second one will say that the volumes of just the isopropyl in the two solutions add up
  to the volume of isopropyl in the final product. Before stating the equations, it may be
  helpful to construct the following table:

                                              15% solution 70% solution 48% solution
             total volume in liters                x                y              10
          volume of isopropyl in liters          0.15x            0.70 y        0.48 · 10

  Of course, mixing the solutions means that the corresponding volumes add up to the cor-
  rect totals:
                                         x + y = 10
                               

                                  0.15x + 0.7 y = 0.48 · 10
  We can solve this system using substitution. Solving the first equation for y yields
  (3)                                      y     =    10 − x
  We can substitute (10 − x) for y in the second equation to find x:
          0.15x + 0.7(10 − x)     =       0.48 · 10
              0.15x + 7 − 0.7x    =       4.8                                distributivity
                   −0.55x + 7     =       4.8                         combined like terms
                       −0.55x     =       4.8 − 7               subtracted 7 on both sides
                       −0.55x     =       −2.2                        combined like terms
                              x   =       4                    divided both sides by −0.55




                                                 233
4. APPLICATIONS OF SYSTEMS                                               CHAPTER 4. LINEAR SYSTEMS
  Now we can substitute 4 for x in the equation (3) and find y:
                                               y     =     10 − x
                                               y     =     10 − (4)
                                               y     =     6


                                       ANSWER:
            x and y are volumes of 15% and 70% solutions respectively, in liters
                                 x + y = 10
                              

                                 0.15x + 0.7 y = 0.48 · 10
                                       solution: x = 4,         y =6


  EXAMPLE 4.2.2. Charlie has in his coin jar a certain number of quarters and nickels. There
  are 77 coins in the jar, and together they are worth $6.05. Find how many quarters and
  how many nickels are in the jar.


  SOLUTION: Let q and n be quantities of quarters and nickels in the jar respectively. The
  total number of coins is 77, which we can translate as
                      q+n         =       77                          first equation
  Recall that a quarter is worth $0.25 and a nickel is worth $0.05. The total worth of coins
  is $6.05, which we can write as
                  0.25q + 0.05n           =     6.05                    second equation
  We can solve this system of two equations by substitution. First we solve the first equation
  for n, because it has a convenient coefficient 1:
  (4)             q+n     =       77
  (5)                 n   =       77 − q                        use this later to find n
  Now we can substitute (77 − q) for n in the second equation and solve for q:
        0.25q + 0.05(77 − q)          =       6.05
        0.25q + 3.85 − 0.05q          =       6.05                                     distributivity
                  0.2q + 3.85         =       6.05                           combined like terms
                          0.2q        =       6.05 − 3.85           subtracted 3.85 on both sides
                          0.2q        =       2.2                            combined like terms
                              q       =       11                        divided both sides by 0.2




                                                         234
CHAPTER 4. LINEAR SYSTEMS                                     4. APPLICATIONS OF SYSTEMS
 Finally we substitute 11 for q in the equation (5) and solve for n:
                                      n   =     77 − q
                                      n   =     77 − 11
                                      n   =     66


                                       ANSWER:
              q and n are the quantities of quarters and nickels respectively
                                  q + n = 77
                                

                                  0.25q + 0.05n = 6.05
                                solution: q = 11,    n = 66




                                              235
4. APPLICATIONS OF SYSTEMS                                     CHAPTER 4. LINEAR SYSTEMS
   Homework 4.4.

1. A collection of dimes and quarters is             the rest are five dollar bills. The total
worth $15.25. There are 103 coins in all.            amount of cash in the box is $50. Find the
How many of each is there?                           number of each type of bill in the cash box.

2. A purse contains $3.90 made up of                 9. A total of $9000 is invested, part of it at
dimes and quarters. If there are 21 coins in         10% and the rest at 12%. The total inter-
all, how many dimes and how many quar-               est after one year is $1030. How much was
ters are there?                                      invested at each rate?

3. There were 429 people at a play. Ad-              10. A total of $18000 is invested, part of
mission was $1 each for adults and 75                it at 6% and the rest at 9%. The total in-
cents each for children. The receipts were           terest after one year is $1248. How much
$372.50. How many children and how                   was invested at each rate?
many adults attended?
                                                     11. At a coin-operated laundromat, using
4. There were 200 tickets sold for a                 the washer 6 times and then the dryer 6
women’s basketball game. Tickets for stu-            times costs $13.50, while using the washer
dents were 50 cents each and for adults 75           4 times and then the dryer 8 times costs
cents each. The total amount of money col-           $12. Find the cost of one wash and the cost
lected was $132.50. How many of each                 of one drying cycle.
type of ticket was sold?
                                                     12. In June, Luke worked 150 hours at the
5. A total of $27000 is invested, part of it         regular rate and 10 hours at the overtime
at 12% and the rest at 13%. The total in-            rate, for the total paycheck of $4150. In
terest after one year is $3385. How much             July, Luke worked 145 hours at the regu-
was invested at each rate?                           lar rate and 16 hours at the overtime rate,
                                                     for the total paycheck of $4265. Find the
6. A total of $50000 is invested, part of it         regular and the overtime hourly rates.
at 5% and the rest at 7.5%. The total in-
terest after one year is $3250. How much             13. A tea that is 20% jasmine is blended
was invested at each rate?                           with a tea that is 15% jasmine. How many
                                                     pounds of each tea are used to make 5 lb
7. A coin bank contains nickels and dimes.           of tea that is 18% jasmine?
The number of dimes is 10 less than twice
the number of nickels. The total value of            14. How many pounds of coffee that is
all the coins is $2.75. Find the number of           40% java beans must be mixed with 80 lb
each type of coin in the bank.                       of coffee that is 30% java beans to make a
                                                     coffee blend that is 32% java beans?
8. A total of 26 bills are in a cash box.
Some of the bills are one dollar bills, and




                                               236
 CHAPTER 4. LINEAR SYSTEMS                                     4. APPLICATIONS OF SYSTEMS
    Homework 4.4 Answers.

                                                         d = 2n − 10
                                                     
1.
 d and q are quantities of dimes and quar-               0.1d + 0.05n = 2.75
 ters respectively
                                                     solution: n = 15,     d = 20
    0.1d + 0.25q = 15.25
 

    d + q = 103                                      9.
 solution: d = 70,    q = 33                          x and y are dollar amounts invested at
                                                      10% and 12% respectively
                                                        x + y = 9000
                                                      
3.
 a and c are quantities of adults and chil-             0.1x + 0.12 y = 1030
 dren respectively
                                                     solution: x = 2500,     y = 6500
   1a + 0.75c = 372.5
 

   a + c = 429                                       11.
                                                      w and d are the costs of one washer and
 solution: a = 203,    c = 226                        one dryer cycle respectively, in dollars
                                                         6w + 6d = 13.5
                                                      
5.
 x and y are dollar amounts invested at                  4w + 8d = 12
 12% and 13% respectively                            solution: w = 1.50,     d = 0.75
   x + y = 27000
 
                                                     13.
   0.12x + 0.13 y = 3385
                                                      x and y are weights of 20% and 15% teas
 solution: x = 12500,    y = 14500                    in pounds
                                                         x+ y =5
                                                      
7.
                                                         0.2x + 0.15 y = 0.18 · 5
 n and d are quantities of nickels and dimes
 respectively                                        solution: x = 3,    y =2




                                               237
5. MULTIVARIATE LINEAR INEQUALITIES                                CHAPTER 4. LINEAR SYSTEMS
                              5. Multivariate Linear Inequalities
   5.1. Solution Sets.

  DEFINITION 5.1.1. An ordered pair (a, b) is a solution for an inequality in two variables x
  and y if substituting a for x and b for y makes the inequality true. The solution set for an
  inequality in two variables is a collection of all ordered pairs which solve the inequality.


  EXAMPLE 5.1.1. Determine whether (4, −5) a solution for the inequality
                                             x + 1 < 2y


  SOLUTION: Substituting 4 for x and (−5) for y yields
                                            4+1 <      2(−5)
                                              5 <      − 10
  This is false, so (4, −5) is not a solution.


                                            ANSWER: no


  EXAMPLE 5.1.2. Determine whether (0, 12) a solution for the inequality
                                            −4x + 2 y ≥ 1


  SOLUTION: Substituting 0 for x and 12 for y yields
                                       −4 · 0 + 2 · 12 ≥       1
                                                   24 ≥        1
  This is true, so (0, 12) is a solution.


                                            ANSWER: yes


   5.2. Solving Inequalities.

  THEOREM 5.2.1. The solution set for a linear inequality in two variables x and y is a half
  of the coordinate plane. The equation of the line separating the solution set from the rest
  of the plane can be obtained by replacing the inequality sign by =.

  A strict inequality (< and >) solution set does not contain the points of the line, while a
  non-strict (≤ and ≥) inequality solution set does contain the points on the line.


                                                 238
CHAPTER 4. LINEAR SYSTEMS                              5. MULTIVARIATE LINEAR INEQUALITIES

 EXAMPLE 5.2.1. Graph the solution set for the inequality
                                           x+ y ≤2


 SOLUTION: This is a non-strict inequality, so the points on the line
                                           x+ y =2
 belong to the solution set. We plot the solid line first:


                                               4
                                                   y
                                               3
                                               2
                                               1
                                                                    x
                                 −4 −3 −2 −1           1   2    3       4
                                           −1
                                             −2
                                             −3
                                             −4


 The solution set also includes all points on one side of the line, but which side? One way
 to find out is by testing any one point not on the line. The point (0, 0), for example, is
 not on the line. If (0, 0) solves the inequality, then all points on the same side as (0, 0)
 (which is below the line in this case) belong to the solution set. And if (0, 0) does not
 solve the inequality, then the points on the other side of the line belong to the solution set.
 Substituting 0 for both x and y in the inequality yields
                                           0+0≤2
 This is true, so (0, 0) is a solution, and the entire side of the coordinate plane with (0, 0)
 consists of solutions, so we graph them all by shading that portion of the plane.


                                                            4
                                                                y
                                                            3
                                                            2
                                                            1
                       ANSWER:                                                      x
                                          −4 −3 −2 −1               1       2   3       4
                                                    −1
                                                           −2
                                                           −3
                                                           −4



                                              239
5. MULTIVARIATE LINEAR INEQUALITIES                                          CHAPTER 4. LINEAR SYSTEMS

  EXAMPLE 5.2.2. Graph the solution set for the inequality                   −2x + y > −1


  SOLUTION: This is a strict inequality, so the points on the line
                                          −2x + y = −1
  do not belong to the solution set. We are still going to plot the line first, but it will be
  dashed rather than solid, to indicate that the line itself is not a part of the solution set. The
  equation of this line in slope-intercept form is
                                            y = 2x − 1
  So the slope is 2 and the y-intercept is (0, −1).


                                                4
                                                    y
                                                3
                                                2
                                                1
                                                                     x
                                   −4 −3 −2 −1          1   2    3       4
                                             −1
                                               −2
                                               −3
                                               −4


  The origin (0, 0) is not on the line, so we can use it for locating the solution set. We
  substitute 0 for both x and y in the inequality to see whether it holds.
                                       −2 · 0 + 0 >          −1
                                                0 >          −1

  This is true, so (0, 0) is a solution, and the portion of the plane containing (0, 0) must be
  the solution set, so we shade accordingly.


                                                             4
                                                                 y
                                                             3
                                                             2
                                                             1
                         ANSWER:                                                     x
                                            −4 −3 −2 −1              1       2   3       4
                                                      −1
                                                            −2
                                                            −3
                                                            −4


                                                240
CHAPTER 4. LINEAR SYSTEMS                                   5. MULTIVARIATE LINEAR INEQUALITIES

 EXAMPLE 5.2.3. Graph the solution set for the inequality
                                            −2x ≥ 6


 SOLUTION: This is a non-strict inequality, so the points on the line
                                            −2x = 6
 belong to the solution set. This line equation is equivalent to
                                             x = −3
 which is a vertical line with x-intercept (−3, 0). We plot the solid line first:


                                                5
                                                    y
                                                4
                                                3
                                                2
                                                1
                                                                                 x
                                  −5 −4 −3 −2 −1        1    2       3       4       5
                                               −1
                                              −2
                                              −3
                                              −4
                                              −5


 The origin (0, 0) is not on the line, so we can use it for locating the solution set. With 0
 for both x and y in the inequality, we get
                                          −2 · 0 ≥           6
                                               0 ≥           6
 This is false, so the side of the coordinate plane to the right of the line is not a part of the
 solution set. Hence the points to the left of the line belong to the solution set, and we
 shade accordingly:


                                                                 5
                                                                         y
                                                                 4
                                                                 3
                                                                 2
                                                                 1
                        ANSWER:                                                                  x
                                           −5 −4 −3 −2 −1                    1       2   3   4       5
                                                        −1
                                                             −2
                                                             −3
                                                             −4
                                                             −5



                                               241
5. MULTIVARIATE LINEAR INEQUALITIES                                     CHAPTER 4. LINEAR SYSTEMS

  EXAMPLE 5.2.4. Graph the solution set for the inequality
                                             3x < 5 y




  SOLUTION: This is a strict inequality, so we will start by graphing a dashed line
                                             3x = 5 y
  The slope-intercept form of this equation is
                                                3
                                              y=  x
                                                5
  so the slope is 3/5 and the y-intercept is (0, 0). We can plot this line by starting at the
  y-intercept (0, 0), and then using the slope to locate another point on the line by going
  3 units up and 5 units to the right. This takes us to (5, 3), which is another point on this
  line.


                                                6   y
                                                5
                                                4
                                                3
                                                2
                                                1
                                                                            x
                             −6 −5 −4 −3 −2 −1          1   2   3   4   5       6
                                             −1
                                              −2
                                              −3
                                              −4
                                              −5
                                              −6


  Unlike in previous examples, the origin (0, 0) is on the line, so it is useless for locating the
  solution set. But we can pick any other point for that, as long as it is not on the line. Let
  us try (1, 0), which is just below the line. Substituting 1 for x and 0 for y in the inequality
  yields
                                          3·1 <         5·0
                                             3 <        0
  This is false, so (1, 0) is not a solution, and the portion of the plane below the line is not a
  part of the solution set. Hence the portion above the line must be the solution set, so we
  shade accordingly:



                                               242
CHAPTER 4. LINEAR SYSTEMS                          5. MULTIVARIATE LINEAR INEQUALITIES

                                   ANSWER:

                                      6    y
                                      5
                                      4
                                      3
                                      2
                                      1
                                                                    x
                      −6 −5 −4 −3 −2 −1        1    2   3   4   5       6
                                      −1
                                     −2
                                     −3
                                     −4
                                     −5
                                     −6




                                      243
5. MULTIVARIATE LINEAR INEQUALITIES                         CHAPTER 4. LINEAR SYSTEMS
   Homework 4.5.

Determine whether the given point is a so-         10. −3 y + 9x > 0
lution for the given inequality.
                                                   11. y ≥ x − 1
1. (2, 6)
                2x + y > 10                        12. y < x + 4

2. (8, −1)                                         13. y > x

               2x − y ≤ −11                        14. 2x ≤ y

3. (−3, 2)                                         15. x + y ≥ 3
                  x ≥ −1
                                                   16. x + y < −1
4. (7, −4)
                   y <6                            17. 2x − 6 y ≥ 6

5. (0, 5)                                          18. −4x − 8 y > 8
               3x + 3 > y − 3
                                                   19. y < −2
6. (0, 0)
                                                   20. x < 1
                14x < 17 y
                                                   21. 2x − 3 y < 12

                                                   22. 3x + 2 y ≥ −6
Graph the solution set for the given in-
equality.                                          23. −2x + 5 y < −10

7. y < x − 2                                       24. 5x + 4 y ≤ 20

8. y ≤ x + 5                                       25. 6 y ≥ 9x + 18

9. 4 y ≤ 6x                                        26. 5 y + 25x ≤ −25




                                             244
 CHAPTER 4. LINEAR SYSTEMS                                               5. MULTIVARIATE LINEAR INEQUALITIES
      Homework 4.5 Answers.

1. no                                                                    11.
                                                                                                     4
                                                                                                         y
                                                                                                     3

                                                                                                     2
3. no
                                                                                                     1
                                                                                                                                         x
                                                                               −4   −3    −2    −1           1           2           3       4
                                                                                                  −1
5. yes
                                                                                                    −2

                                                                                                    −3

                                                                                                    −4
7.
                       4
                           y
                       3                                                 13.
                                                                                                     3
                                                                                                         y
                       2
                                                                                                     2
                       1
                                                           x
                                                                                                     1
     −4   −3   −2   −1             1       2       3           4
                      −1                                                                                                                 x
                                                                               −3    −2        −1                1               2           3
                     −2
                                                                                                    −1
                     −3

                     −4                                                                             −2


                                                                                                    −3


9.                                                                       15.
                       6   y                                                                         5
                                                                                                         y
                       5                                                                             4
                       4
                                                                                                     3
                       3
                                                                                                     2
                       2
                       1                                                                             1
                                                           x                                                                             x
     −6 −5 −4 −3 −2 −1         1       2   3   4       5       6               −5 −4 −3 −2 −1                1       2       3       4       5
                     −1                                                                     −1
                     −2
                                                                                                    −2
                     −3
                                                                                                    −3
                     −4
                     −5                                                                             −4
                     −6                                                                             −5
                                                                   245
5. MULTIVARIATE LINEAR INEQUALITIES                                      CHAPTER 4. LINEAR SYSTEMS
17.                                                         23.
                        7                                                           7
                             y                                                           y
                        6                                                           6
                        5                                                           5
                        4                                                           4
                        3                                                           3
                        2                                                           2
                        1                                                           1
                                              x                                                                  x
      −7 −6 −5 −4 −3 −2 −1
                        −1       1 2 3 4 5 6 7                    −7 −6 −5 −4 −3 −2 −1
                                                                                    −1       1 2 3 4 5 6 7
                       −2                                                          −2
                       −3                                                          −3
                       −4                                                          −4
                       −5                                                          −5
                       −6                                                          −6
                       −7                                                          −7

19.                                                         25.
                        4
                             y                                                      6    y
                        3                                                           5
                                                                                    4
                        2
                                                                                    3
                        1                                                           2
                                              x                                     1
                                                                                                                 x
      −4   −3   −2   −1           1   2   3       4
                                                                  −6 −5 −4 −3 −2 −1          1   2   3   4   5       6
                       −1                                                         −1
                                                                                   −2
                       −2
                                                                                   −3
                       −3                                                          −4
                                                                                   −5
                       −4
                                                                                   −6
21.
                        7
                             y
                        6
                        5
                        4
                        3
                        2
                        1
                                              x
      −7 −6 −5 −4 −3 −2 −1
                        −1       1 2 3 4 5 6 7
                       −2
                       −3
                       −4
                       −5
                       −6
                       −7




                                                      246
CHAPTER 4. LINEAR SYSTEMS                                                 5. PRACTICE TEST 4
                                        Practice Test 4




1. Solve the system by graphing:                     6. A total of $2400 is invested into simple
                                                     interest accounts, part of it at 5% and the
               3y − x = 3
            
                                                     rest at 10%. The total interest after one
               2x − 4 = y                            year is $175. How much was invested at
                                                     each rate?
2. Solve the system:
              2x + y = 12
           
                                                     7. A pastry delivery is a mix of donuts and
               y − 3x = 2                            scones, 40 items in total. Each donut costs
                                                     $1.50, while each scone costs $2.50. Find
3. Solve the system:                                 how many of each type of pastry there are
                                                     if the total price of the order is $76.
            −2x + 3 y = −1
          

              2x + 5 y = 25                          8. How many liters of 20% saline solution
                                                     should be added to 40 liters of a 50% saline
4. Solve the system:                                 solution to make a 30% solution? How
           
              y = −3x − 4                            many liters of 30% solution will result?
              0 = 6x + 2 y                           9. Graph the solution set for inequality:
5. A party with 3 adults and 2 children
pays 45 dollars for their tickets at Acme                            y < 5x − 2
Theater. Another party, with 4 adults and
4 children, pays 68 dollars. Find the price          10. Graph the solution set for inequality:
of one adult ticket and one the price of one
child ticket.                                                 2 y + 6 ≥ −2(x + y − 3)




                                               247
 5. PRACTICE TEST 4                                                  CHAPTER 4. LINEAR SYSTEMS
    Practice Test 4 Answers.

1. (3, 2)                                                     d + s = 40
                                                         

                  5                                           1.5d + 2.5s = 76
                      y
                  4                                      solution: d = 24,         s = 16
                  3
                  2                                     8.
                  1                                      x and y are volumes of 20% and 30% so-
                                          x              lutions respectively, in liters
 −5 −4 −3 −2 −1           1   2   3   4       5
                                                            x + 40 = y
                                                         
              −1
              −2                                            0.2x + 0.5 · 40 = 0.3 · y
              −3
                                                         solution: x = 80,         y = 120
              −4
              −5                                        9.
                                                                               5
                                                                                       y
2. (2, 8)                                                                      4
                                                                               3
3. (5, 3)                                                                      2
                                                                               1
4. no solutions                                                                                                x
                                                              −5 −4 −3 −2 −1               1       2   3   4       5
                                                                           −1
5.
                                                                              −2
 a and c are prices of one adult and one
 child ticket respectively, in dollars                                        −3
                                                                              −4
   3x + 2 y = 45
 
                                                                              −5
   4x + 4 y = 68
 solution: a = 11,        c=6                           10.
                                                                                   4
                                                                                           y

6.                                                                                 3
 x and y are amounts invested at 5% and                                            2
 10% respectively, in dollars
                                                                                   1
   x + y = 2400
 
                                                                                                                   x
   0.05x + 0.1 y = 175                                         −4   −3   −2   −1               1       2   3           4
 solution: x = 1300,          y = 1100                                          −1

                                                                               −2
7.
                                                                               −3
 d and s are quantities of donuts and scones
 respectively                                                                  −4




                                                  248
                                         CHAPTER 5


                              Polynomial Expressions

                                    1. Integer Exponent


                                               2   y




                                               1




                                                                         x
                 −2            −1                             1              2
                                                     y=x

                                                     y = x2
                                              −1
                                                     y = x3



                                              −2

 1.1. Definition.

DEFINITION 1.1.1 (Integer Exponent). Recall that given a real number a and a positive
                                       b · . . . · b}. If n = 0 and b is any real number, then
integer n, notation b n means |b · b · {z
                                    n times

                                          b n = b0 = 1
If the exponent is negative and the base b 6= 0, then the reciprocal of the base is raised to
the corresponding power:                   ‹n
                                             1       1
                                    b =
                                     −n
                                                  = n
                                             b      b
In particular, b−1 becomes another way to express the reciprocal of b.


                                               249
1. INTEGER EXPONENT                                          CHAPTER 5. POLYNOMIAL EXPRESSIONS

  BASIC EXAMPLE 1.1.1. Some trivial uses of the definition:
                    x1      =   x                                 this is true for any real number x
                                 ‹3
                                 1     1
                   2   −3
                            =        =                                        reciprocal of 2 cubed
                                 2     8
              ‹−1
                3                   7
              −             =   −                   exponent −1 yields the reciprocal of the base
                7                   3
     (x 4 + 5x 2 + 13)0     =   1                 exponent zero always results in 1 for any base
                ‹−2             ‹2
                7                5     25
                            =        =              take the reciprocal of the base, then square it
                5                7     49


  THEOREM 1.1.1 (Exponent with Base ±1). The exponent with base 1 is always 1. If n is
  an integer, then
                                            1n = 1
  It is also true that (−1)n = 1 for all even n and (−1)n = −1 for all odd n, even if the
  exponent n is negative.



When we simplify a product or a quotient of exponential expressions, we will attempt to bring
it into the form with a single numerical coefficient, with each base appearing only once, and
with all the exponents positive. We will also defer our exploration of the negative case until
the end of this chapter, since polynomial expressions only make use of the non-negative integer
exponent. It is of note, though, that the theorems which follow work for the negative exponent
just as well.

   1.2. Products and Quotients of Exponents.

  THEOREM 1.2.1 (Product Rule for Exponents). A product of exponential expressions with
  the same base can be written as a single base with the sum of exponents. If m, n are
  integers and b is a real number, then
                                                b m b n = b m+n


  BASIC EXAMPLE 1.2.1. It is easy enough to see why the product rule works if we follow the
  definition to simplify a product with the same base raised to various powers.
                                        b3 b2      =    (bbb)(bb)
                                                   =    bbbbb
                                                   =    b5

                                                     250
CHAPTER 5. POLYNOMIAL EXPRESSIONS                                        1. INTEGER EXPONENT

 EXAMPLE 1.2.1. Simplify the expression:
                                          54 · 53 · 5


 SOLUTION:
                                54 · 53 · 5     =     54 · 53 · 51
                                                =     54+3+1
                                                =     58
                                                =     390625


                                     ANSWER: 390625


 EXAMPLE 1.2.2. Simplify the expression:
                                      (1 + x)5 (1 + x)7


 SOLUTION: Here the common base is (1 + x).
                            (1 + x)5 (1 + x)7         =     (1 + x)5+7
                                                      =     (1 + x)12
 Recall that exponent does not distribute over addition, so we cannot simplify this answer
 any more.


                                    ANSWER: (1 + x)12


 EXAMPLE 1.2.3. Simplify the expression:
                                        (a2 b6 )(ab3 )


 SOLUTION: Factors of a product can be multiplied in any order, so we bring similar bases
 next to each other, then apply the product rule:
                               (a2 b6 )(ab3 )     =       a2 a1 b6 b3
                                                  =       a2+1 b6+3
                                                  =       a3 b9


                                      ANSWER: a3 b9

                                                251
1. INTEGER EXPONENT                                     CHAPTER 5. POLYNOMIAL EXPRESSIONS

 EXAMPLE 1.2.4. Simplify the expression:
                                          (−4x 3 )(−3x 17 )


 SOLUTION:
                              (−4x 3 )(−3x 17 )    =    (−4)(−3)x 3 x 17
                                                   =    12x 3+17
                                                   =    12x 20


                                          ANSWER: 12x 20


 THEOREM 1.2.2 (Quotient Rule for Exponents). A quotient of exponential expressions
 with the same base can be written as a single base with the difference of exponents. If m
 and n are integers and b 6= 0 is a real number, then
                                          bm
                                             = b m−n
                                          bn


 BASIC EXAMPLE 1.2.2. It is easy to see why the quotient rule works if we follow the defi-
 nition.
             b6         b bb bb b
                  =                            four pairs of b factors cancel
             b4           b bb b
                      =     bb
                      =     b2


 BASIC EXAMPLE 1.2.3. The quotient rule is actually a direct consequence of definition of
 the negative exponent and the product rule. Recall that the fraction is in fact a product of
 the numerator and the reciprocal of the denominator:
            bm          m 1
                 =   (b  ) n            division is multiplication by reciprocal
            bn            b
                  =       b m b−n                  by definition of negative exponent
                  =       b m+(−n)                                         product rule
                  =       b m−n




                                                  252
CHAPTER 5. POLYNOMIAL EXPRESSIONS                                 1. INTEGER EXPONENT

 EXAMPLE 1.2.5. Simplify the expression:
                                             513
                                             510


 SOLUTION:
                                    513
                                            =      513−10
                                    510
                                            =      53
                                            =      125


                                     ANSWER: 125


 EXAMPLE 1.2.6. Simplify the expression:
                                           5x 6 y 8
                                           15x 2 y


 SOLUTION:
                 5x 6 y 8       1 x6 y8
                            =    ·                          lowest terms
                 15x 2 y        3 x2 y1
                                1 6−2 8−1
                            =     x y
                                3
                                1 4 7
                            =     x y
                                3


                                                   1 4 7
                                   ANSWER:           x y
                                                   3




                                            253
1. INTEGER EXPONENT                                  CHAPTER 5. POLYNOMIAL EXPRESSIONS
   1.3. Exponents of Sums. It is a common mistake is to assume that exponent distributes
over addition as well as over multiplication, but this is not the case. Students faced with an
expression such as
                                           (x + 2)2

are often tempted to rewrite it as x 2 + 22 or x 2 + 4, but this is not true in general, as anyone
can check by comparing the values of these expressions for a specific x. For example, if x = 1
then
                               (x + 2)2    =    (1 + 2)2 = 32 = 9
                                  x2 + 4   =    12 + 4 = 5
So these expressions are not equivalent. Instead, when faced with exponents of sums, we will
replace exponents by multiplication and then apply the distributivity of multiplication over
addition:
                           (x + 2)2    =    (x + 2)(x + 2)
                                       =    x · (x + 2) + 2 · (x + 2)
                                       =    x · x + x ·2+2· x +2·2
                                       =    x 2 + 2x + 2x + 4
                                       =    x 2 + 4x + 4

For now, though, we will leave answers like (x + 2)2 in exponential form.




                                               254
CHAPTER 5. POLYNOMIAL EXPRESSIONS                                  1. INTEGER EXPONENT
    Homework 5.1.

Simplify the given expression.            13. (x 2 y 7 )(x y 2 )

1. (−1)1982                               14. (n3 k10 )(n5 k4 )

2. (−1)1001                               15. (−x 4 y)(−6x 2 y 11 )

3. 00                                     16. (−5a3 b0 )(−20a5 b7 )

4. (1 + x 2 )0                                  1146
                                          17.
                                                1144
5. 23 · 24
                                                7108
     3
6. 3 · 3  3                               18.
                                                7105
7. (−4)(−4)2 (−4)3                              (−6)16 (−6)4
                                          19.
                                                  (−6)15
8. (−5)3 (−5)2
                                                   (−0.9)13
9. X 1 X 20 X 300                         20.
                                                (−0.9)2 (−0.9)8
10. Y 20 Y 0 Y 22
                                                60x 8 y 12
                                          21.
11. (x − y)6 (x − y)7                           −15x 5 y 11

12. (a + 2b)19 (a + 2b)                          8a6 y 5
                                          22.
                                                24x 5 y 3




                                    255
 1. INTEGER EXPONENT                 CHAPTER 5. POLYNOMIAL EXPRESSIONS
       Homework 5.1 Answers.

1. 1                                 13. x 3 y 9

3. 1
                                     15. 6x 6 y 12
5. 128
                                     17. 121
7. 4096
                                     19. −7776
9. X 321

11. (x − y)13                        21. −4x 3 y




                               256
CHAPTER 5. POLYNOMIAL EXPRESSIONS                                     2. PROPERTIES OF EXPONENT
                                2. Properties of Exponent
  2.1. Exponent of Exponent.

 THEOREM 2.1.1 (Power Rule for Exponents). When working with an exponent of an expo-
 nential expression, we can rewrite it as the same base with the product of the exponents.
 If m and n are integers and b is a real number, then
                                          (b m )n = b m·n


 BASIC EXAMPLE 2.1.1. It is easy to see why the power rule works if we follow the definition.
                                 (b2 )3     =     (bb)3
                                            =     (bb)(bb)(bb)
                                            =     bbbbbb
                                            =     b6


 EXAMPLE 2.1.1. Simplify the expression:         (23 )4


 SOLUTION:
                                      (23 )4      =       23·4
                                                  =       212
                                                  =       4096


                                          ANSWER: 4096


 EXAMPLE 2.1.2. Simplify the expression:
                                            ((x 5 )3 )7


 SOLUTION:
                                   ((x 5 )3 )7     =      (x 5·3 )7
                                                   =      (x 15 )7
                                                   =       x 15·7
                                                   =       x 105


                                          ANSWER: x 105

                                                 257
2. PROPERTIES OF EXPONENT                             CHAPTER 5. POLYNOMIAL EXPRESSIONS
   2.2. Distributivity.

  THEOREM 2.2.1 (Distributivity of Exponent over Multiplication). Integer exponent dis-
  tributes over multiplication. If n is an integer and a, b are real numbers, then
                                          (ab)n = a n b n
  This rule extends naturally to products with more than 2 factors:
                                        (x yz)n = x n y n z n


  BASIC EXAMPLE 2.2.1. It is easy to see why distributivity works if we follow the definition.
         (a b)4   =    (a b)(a b)(a b)(ab)
                  =    (aaaa)(b bb b)                changed the order of multiplication
                  =    a4 b4


  EXAMPLE 2.2.1. Simplify the expression:
                                              (−3x)4


  SOLUTION: The base of this exponential expression is a product with two factors: −3 and
  x, so we apply the distributive property:
                                    (−3x)4       =     (−3)4 x 4
                                                 =     81x 4


                                         ANSWER: 81x 4


  EXAMPLE 2.2.2. Simplify the expression:
                                             (−10abc)3


  SOLUTION:
                               (−10a bc)3       =     (−10)3 a3 b3 c 3
                                                =     −1000a3 b3 c 3


                                   ANSWER: −1000a3 b3 c 3




                                                258
CHAPTER 5. POLYNOMIAL EXPRESSIONS                                            2. PROPERTIES OF EXPONENT

 EXAMPLE 2.2.3. Simplify the expression:
                                                (x 3 y 4 )5


 SOLUTION: The base of the exponent has two factors: x 3 and y 4 , so we distribute the
 exponent first, and then apply the rule for exponentiating the exponent:
                                  (x 3 y 4 )5     =       (x 3 )5 ( y 4 )5
                                                  =       x 3·5 y 4·5
                                                  =       x 15 y 20


                                        ANSWER: x 15 y 20


 EXAMPLE 2.2.4. Simplify the expression:
                                        (−2ab2 )3 (a3 b)5


 SOLUTION: Distribute exponents over the products first, and simplify exponents of expo-
 nents:
                       (−2a b2 )3 (a3 b)5       =       (−2)3 a3 (b2 )3 · (a3 )5 b5
                                                =       −8a3 b2·3 · a3·5 b5
                                                =       −8a3 b6 · a15 b5

 The factors of this product can be multiplied in any order, so we bring similar bases next
 to each other, and then apply the product rule:
                           −8a3 b6 · a15 b5         =     −8 · a3 a15 · b6 b5
                                                    =     −8 · a3+15 · b6+5
                                                    =     −8a18 b11


                                      ANSWER: −8a18 b11




                                                  259
2. PROPERTIES OF EXPONENT                              CHAPTER 5. POLYNOMIAL EXPRESSIONS

  EXAMPLE 2.2.5. Simplify the expression:
                                             (4x y)17
                                             (4x y)14


  SOLUTION: Here the common base is (4x y), so we can apply the quotient rule, and
  then distribute the exponent over the product. Alternatively, we could distribute first, and
  cancel common factors later.
           (4x y)17
                       = (4x y)17−14
           (4x y)14
                       =    (4x y)3
                       =    43 x 3 y 3           distribute exponent over the product
                       =    64x y3   3




                                         ANSWER: 64x 3 y 3


  THEOREM 2.2.2 (Distributivity of Exponent in Fractions). Integer exponent distributes
  over division as well as over multiplication. If n is an integer, a is real, and b is a non-zero
  real, then                                a n a n
                                                 = n
                                             b       b

  This rule extends naturally to fractions with more than 2 factors:
                                         2x n     2n x n
                                            ‹
                                               = n n n
                                         abc     a b c


  BASIC EXAMPLE 2.2.2. It is easy to see why distributivity works if we follow the definition.
                                  a 3       aaa
                                          =
                                   b            b    b     b
                                              aaa
                                          =
                                              bbb
                                                  a3
                                            =
                                                  b3




                                                260
CHAPTER 5. POLYNOMIAL EXPRESSIONS                          2. PROPERTIES OF EXPONENT

 EXAMPLE 2.2.6. Simplify the expression:
                                            ‹2
                                            7
                                            c


 SOLUTION:
                                     ‹2
                                     7                72
                                             =
                                     c                c2
                                                      49
                                             =
                                                      c2


                                                  49
                                     ANSWER:
                                                  c2


 EXAMPLE 2.2.7. Simplify the expression:
                                                 ‹3
                                           −3d
                                       

                                            x4


 SOLUTION: The base of this exponential expression is a product of three factors: −3, d,
 and the reciprocal of x 4 , so we can apply the distributive property:
                     −3d 3           (−3)3 d 3
                          ‹
                                 =
                      x4               (x 4 )3
                                  −27d 3
                             =                              power rule
                                   x 4·3
                                  −27d 3
                             =
                                   x 12


                                                 −27d 3
                                   ANSWER:
                                                  x 12




                                           261
2. PROPERTIES OF EXPONENT                            CHAPTER 5. POLYNOMIAL EXPRESSIONS

  EXAMPLE 2.2.8. Simplify the expression:
                                             (−2bc 5 )3
                                              6b2 c 2


  SOLUTION: The base of the exponential expression in the numerator is a product of 3
  factors: −2, b, and c 5 . We have to distribute the exponent over this product before we
  can cancel common factors in the fraction:


         (−2bc 5 )3       (−2)3 b3 (c 5 )3
                      =                          distributed exponent over the product
           6b2 c 2           6b2 c 2
                          −8b3 c 5·3
                      =                                                    power rule
                           6b2 c 2
                              4 b3 c 15
                      =   −                                              lowest terms
                              3 b2 c 2
                           4
                      =   − b3−2 c 15−2                                  quotient rule
                           3
                           4
                      =   − bc 13
                           3


                                                   4
                                          ANSWER: − bc 13
                                                   3




                                               262
CHAPTER 5. POLYNOMIAL EXPRESSIONS                         2. PROPERTIES OF EXPONENT
      Homework 5.2.

Simplify the given expression.                  (m3 n2 )6
                                          12.
                                                (mn3 )4
1. (g 5 )8
                                          13. (32 )3
2. (h )9 10

                                          14. ((22 )2 )2
3. (−2k x)5

                                                3(2 )
                                                      3

4. (cat)     7                            15.

                                                x(3 )
                                                      4
5. 30 + 5x 0                              16.
                                                                   3
6. (2c 0 )4                               17. (x 5 y)(x y)2

7. (x y)7 ( yz)3                          18. (x y)5 (x 3 y)3
                                                                    2

8. (a2 b)4 (a5 b6 )                                           ‹4
                                                      x3
                                                
                                          19.
9. (3m3 c 3 )(−3m10 c 6 )2                          −2x y 5
                                                              ‹3
10. (a7 b4 )2 (ab2 c)9                               a5
                                                
                                          20.
                                                    −3a3 b
      (g h )
        7 2 2
11.
      (g 2 h)3




                                    263
 2. PROPERTIES OF EXPONENT           CHAPTER 5. POLYNOMIAL EXPRESSIONS
       Homework 5.2 Answers.

1. g 40                              11. g 8 h

3. −32k5 x 5                         13. 729

                                     15. 6561
5. 6
                                     17. x 21 y 9
       7   10 3
7. x y z
                                             x8
           23 15                     19.
9. 27m c                                   16 y 20




                               264
CHAPTER 5. POLYNOMIAL EXPRESSIONS                               3. INTRODUCTION TO POLYNOMIALS
                               3. Introduction to Polynomials


                                                    6   y
                                                    4
                                                    2
                                                                                      x
                  −3        −2         −1                         1            2          3
                                                   −2
                                                             y = x(x + 1)(x − 2)
                                                   −4
                                                   −6

  3.1. Definitions.

 DEFINITION 3.1.1. A monomial expression, or just monomial, is an algebraic expression
 which is a product of numerical constants and/or variables, with each variable optionally
 carrying a positive integer exponent.

 A simplified form of a monomial expression has a single numerical factor in the very front,
 called coefficient, and lists each variable base at most once.


 BASIC EXAMPLE 3.1.1. This is a monomial with variables x and y and constants −3 and 2:
                                          −3 · x 2 · y 5 · x · 2
 To simplify, rearrange the factors and use the properties of exponent to combine the bases:
                          −3 · x 2 · y 5 · x · 2    =       (−3)(2) · x 2 x 1 · y 5
                                                    =       −6 · x 2+1 · y 5
                                                    =       −6x 3 y 5
 In simplified form, this monomial has 3 factors and coefficient −6.

 These are not monomials:

 x + y, seen as a product, has a single factor (x + y) which is neither a numerical constant
 nor a variable. In general, no sum with more than one term is a monomial.

 2/x has the reciprocal of x as a factor, which is the same as x −1 , but reciprocals of variables
 and negative exponents are not allowed.

 (3x)2 is not a monomial expression, since its only factor is neither a variable nor a numer-
 ical constant. It is though equivalent to 9x 2 , which is a monomial with two factors and
 coefficient 9.

                                                    265
3. INTRODUCTION TO POLYNOMIALS                      CHAPTER 5. POLYNOMIAL EXPRESSIONS

 DEFINITION 3.1.2. The degree of a monomial is the sum of all the exponents of the vari-
 ables. Variables without visible exponents are considered to have exponent 1. Non-zero
 monomials with no variables have degree 0. The degree of the zero monomial is unde-
 fined. Descriptive names exist for monomials of low degrees, and learning the names of
 degrees 0 through 2 is particularly crucial for working with this text.

                                degree type of a monomial
                                  0          constant
                                  1            linear
                                  2         quadratic
                                  3            cubic
                                  4           quartic
                                  5           quintic


 BASIC EXAMPLE 3.1.2. Here are some monomials and their degrees.

 7x y, which is equivalent to 7x 1 y 1 , has degree 1 + 1 = 2, so it is a quadratic monomial
 with coefficient 7.

 −17 has degree 0, so it is a constant monomial with coefficient −17.

  x y 2 has degree 1 + 2 = 3, so it is a cubic monomial with coefficient 1.

 −x y 2 z 5 has degree 1 + 2 + 5 = 8 and coefficient −1.

 0 is the zero monomial. Its degree is traditionally left undefined in order to satisfy the
 following rule: the degree of a product of monomials is the sum of individual degrees. This
 rule works for any two non-zero monomials, but if we let the degree of the zero monomial
 to be 0, then any product involving it would also have degree 0, which is against the rule.


 DEFINITION 3.1.3. A polynomial expression, or just polynomial, is a sum of monomial ex-
 pressions. A polynomial in one variable, or univariate polynomial, uses at most one variable
 throughout the expression. A polynomial in two variables, or bivariate polynomial, uses at
 most two.


 DEFINITION 3.1.4 (Standard Form for Polynomials). When writing down a polynomial in
 one variable x, it is traditional to order monomials in the descending order of degree,
 starting with the highest one, so the standard way of writing a polynomial like
                    x − 3 + 2x 2 − 5x 7       is       − 5x 7 + 2x 2 + x − 3
 The coefficient of the highest degree term is called the leading coefficient of the polynomial.


                                              266
CHAPTER 5. POLYNOMIAL EXPRESSIONS                      3. INTRODUCTION TO POLYNOMIALS
There are no firm conventions for writing down a polynomial in two or more variables, or for
defining the leading coefficient in that case.

  DEFINITION 3.1.5 (Polynomial Names). The following names exist for polynomials with
  a low number of terms:

                          number of terms type of a polynomial
                                1               monomial
                                2               binomial
                                3               trinomial



  DEFINITION 3.1.6. The degree of a polynomial is the highest of all the monomial degrees.
  The degree of the zero polynomial is undefined. The descriptive names for low degrees
  are the same as for monomials.


  BASIC EXAMPLE 3.1.3. Here are some polynomials with their properties described:

              polynomial     number of terms degree      description
                    20             1           0     constant monomial
                   x −3            2           1       linear binomial
                x + x yz
                  3
                                   2           3       cubic binomial
                 2
               x − x −1            3           2    quadratic trinomial
            −x y + x y + x
              2 2       6  5
                                   3           7    7th degree trinomial


  BASIC EXAMPLE 3.1.4. These are not polynomials:
       1
  x−     has a term which is not a monomial.
       y

  4(x + 1) is not strictly speaking a polynomial expression, since it consists of a single term
  which is not a monomial. It is equivalent though to 4x + 4, which is a linear binomial, so
  later in the text we will refer to it as a polynomial in a fully factored form.


   3.2. Combining Similar Monomial Terms.

  DEFINITION 3.2.1. Monomial terms are like (used an an adjective) if their simplified forms
  consist of exactly the same variables raised to exactly the same powers. Sometimes like
  terms are also called similar terms.


                                              267
3. INTRODUCTION TO POLYNOMIALS                         CHAPTER 5. POLYNOMIAL EXPRESSIONS

 EXAMPLE 3.2.1. Simplify the polynomial by combining like terms.
                                             5x − 17x


 SOLUTION: We combine like terms by applying the distributive property to their common
 factors, which is all the variables.
                                  5x − 17x        =    (5 − 17)x
                                                  =    −12x


                                          ANSWER: −12x


 EXAMPLE 3.2.2. Simplify the polynomial by combining like terms.
                                           x y 2 + 13 y 2 x


 SOLUTION: These terms don’t look the same, but they are in fact similar, since we can
 rearrange the order of multiplication at will:
                              x y 2 + 13 y 2 x    =     x y 2 + 13x y 2
                                                  =     (1 + 13)x y 2
                                                  =     14x y 2


                                        ANSWER: 14x y 2


 EXAMPLE 3.2.3. Simplify the polynomial by combining like terms.
                                  3ab3 − 5 + a3 b + 4 − 6ab3


 SOLUTION: Note that the terms 3a b3 and a3 b are not similar. Even though they have the
 same variables, the corresponding exponents are different. We can rearrange the terms of
 the sum so as to put like terms next to each other, and then apply the distributive property:
      3a b3 − 5 + a3 b + 4 − 6ab3     =     3ab3 − 6ab3 − 5 + 4 + a3 b
                                      =     (3 − 6)ab3 − 1 + a3 b          −5 + 4 = −1
                                      =     −3ab3 − 1 + a3 b


                                 ANSWER: −3ab3 − 1 + a3 b

                                                 268
CHAPTER 5. POLYNOMIAL EXPRESSIONS                           3. INTRODUCTION TO POLYNOMIALS

 EXAMPLE 3.2.4. Simplify the polynomial by combining like terms.
                           0.3x − 0.6 + 2.3x 2 − 1.7x − x 2 + 0.6


 SOLUTION: Put like terms next to each other and distribute:
     0.3x − 0.6 + 2.3x 2 − 1.7x − x 2 + 0.6       =       2.3x 2 − x 2 + 0.3x − 1.7x − 0.6 + 0.6
                                                  =       (2.3 − 1)x 2 + (0.3 − 1.7)x + 0
                                                  =       1.3x 2 − 1.4x
 In answers, we will list the terms of polynomials in one variable in the descending order
 of degree.


                                  ANSWER: 1.3x 2 − 1.4x

  3.3. Evaluating Polynomials in Applications.

 EXAMPLE 3.3.1. Evaluate the expression x 2 − 2x − 6 for x = 5.


 SOLUTION:
                            x 2 − 2x − 6      =       (5)2 − 2 · (5) − 6
                                              =       25 − 10 − 6
                                              =       9


                                           ANSWER: 9


 EXAMPLE 3.3.2. Evaluate the expression −3x 2 + 2x y − 4 y 2 for x = 3 and y = −2.


 SOLUTION:
                 −3x 2 + 2x y − 4 y 2   =     −3 · (3)2 + 2(3)(−2) − 4(−2)2
                                        =     (−3)(9) + (−12) − 4(4)
                                        =     −27 − 12 − 16
                                        =     −55


                                        ANSWER: −55

                                              269
3. INTRODUCTION TO POLYNOMIALS                       CHAPTER 5. POLYNOMIAL EXPRESSIONS

 EXAMPLE 3.3.3. Professor calculates the class grade (out of 100 points) by computing the
 value of the following expression:
                                 0.1H + 0.1Q + 0.6T + 0.2F
 where H, Q, T , and F are the points accumulated for homework, quizzes, tests, and the
 final respectively. Find the class grade for a student with H = 85, Q = 90, T = 76, and
 F = 68.


 SOLUTION:
         0.1H + 0.1Q + 0.6T + 0.2F       =    0.1(85) + 0.1(90) + 0.6(76) + 0.2(68)
                                         =    8.5 + 9 + 45.6 + 13.6
                                         =    76.7


                                    ANSWER: 76.7 points


 EXAMPLE 3.3.4. The volume of a pyramid with a square base is given by the expression
                                            1 2
                                              a h
                                            3
 where a is the side of the square and h is the height of the pyramid. Find the volume
 of the Great Pyramid of Giza, with dimensions a = 230.4 meters and h = 146.5 meters.
 (Actually, the pyramid’s top deteriorated over time, so the actual height is a bit lower, but
 the historical height we are using here gives a better volume estimate.)


 SOLUTION:
                                1 2          1
                                  a h   =      (230.4)2 · 146.5
                                3            3
                                        =    2592276


                              ANSWER: 2592276 cubic meters




                                             270
CHAPTER 5. POLYNOMIAL EXPRESSIONS                        3. INTRODUCTION TO POLYNOMIALS
     Homework 5.3.

Determine whether the given expression is             16. − y 14
a polynomial expression.
                                                      17. 6x 3 − 3x 2 + 2x − 1
1. 7x − 3
                                                      18. 3x 8 + 12x 3 − 8
2. 2x + 9 − 7x
       5              2

                                                      19. −x y 2 z 3
         x +1
           2
3.
     x 2 − 5x + 1                                     20. m7 + x 4 y 4

4. −11                                                21. a + b + c + d + 3x

5. x 5 + x −5                                         22. ab − cd − x y
     1   2
6.     − 2+x
     x x
                                                      Combine the like terms. For polynomials in
7. (x + 1)(x 2 − 10)                                  one variable, state answers in the standard
                                                      form, with terms listed in the descending
     1    2
8.     x − x 10                                       order of degree.
     3    7
                                                      23. 6n2 − 5 + 5n2
    2            1
9. − x 3 + x 2 +
    7            7                                    24. 4x + 7x 2 + 3x
10. x 3 − x(x + 1)
                                                      25. 3x 4 − 2x + 2x + x 4

                                                      26. 9a5 + 3a2 − 2a5 − 3a2
For each given polynomial, describe its de-
gree and the number of terms in words. For            27. 10 y 2 + 2 y 3 − 3 y 3 − 4 y 2 − 6 y 2 − y 4
example, x + 1 is a linear binomial, while
x 2 + x y + y 2 is a quadratic trinomial. Use         28. 12b6 − b3 + 8b6 + 4b3 − b7 − 3b3
numbers for describing polynomials of high
degree and/or with too many terms. For                29. 14x 2 − 5x y + x y 2 − x y − 7x 2 + y 2 x
example, x 99 + x 3 + x 2 + x is a polynomial
of degree 99 with 4 terms.                            30. −a5 b2 + 4a2 b5 − 2a5 b2 − 6b5 a2
                                                                   1 2 2           5   1
11. −5                                                31. 9x 3 +     x − x + 7x 3 + x + x 2
                                                                   2    3          3   4
12. 6x                                                      1 4          12    3   1
                                                      32.     y − 2 y3 +    y − y − y4 + 6 y3
                                                            9            5     5   3
13. 5x y 2 − 3

14. x y + 1 − ab

               1
15. a2 −         bc
               2
                                                271
3. INTRODUCTION TO POLYNOMIALS                       CHAPTER 5. POLYNOMIAL EXPRESSIONS
Evaluate each polynomial if x = 3, y = −2,           42. The distance in meters traveled by a
and z = 0.1                                          projectile t seconds after the start of the
                                                     timer can be found as the value of the fol-
33. −3x + 9                                          lowing expression:

34. 12 − 10x                                                                     5 2
                                                                  3.5 + 1.5t +     t
                                                                                 2
35. 2x 2 − 3x + 6                                    Find the distance traveled by the projectile
                                                     10 seconds after the start of the timer.
36. −3x 2 − 4x + 5
              1 2                                    43. The amount of drug in the blood-
37. 2 y 4 −     y                                    stream, measured in mcg/mL, can be es-
              2
                                                     timated by the value of the expression
                  2
38. − y 3 +         y                                   −0.004t 4 + 0.004t 3 + 0.35t 2 + 0.6t
                  5
                                                     where t is the time in hours since the drug
39. x 2 − y 2 z                                      was administered. Find the amount of drug
                                                     in the bloodstream 5 hours after the injec-
40. x 4 y 3 z 2
                                                     tion.
41. A chemist finds the amount (measured
                                                     44. When the technology arm of Acme Co
in liters) of acid in a mixture of x liters
                                                     produces x computer monitors, the pro-
of solution A with y liters of solution B by
                                                     jected profit in dollars can be found using
computing the value of the expression
                                                     the expression
                        0.3x + 0.7 y
                                                                250x − 1.1x 2 − 4000
Find the amount of acid in a mixture of
x = 1.2 liters of solution A with y = 0.85           Find the profit corresponding to the pro-
liters of solution B.                                duction of 190 monitors.




                                               272
 CHAPTER 5. POLYNOMIAL EXPRESSIONS                          3. INTRODUCTION TO POLYNOMIALS
    Homework 5.3 Answers.

1. yes                                                  23. 11n2 − 5

3. no, because it is a fraction with a variable         25. 4x 4
 denominator
                                                        27. − y 4 − y 3
5. no, because it has negative exponents

7. no, even though it can be shown that this            29. 7x 2 − 6x y + 2x y 2
 product of binomials is equivalent to a cu-
 bic polynomial                                                       3 2
                                                        31. 16x 3 +     x +x
                                                                      4
9. yes, because rational coefficients are OK
                                                        33. 0
11. constant monomial
                                                        35. 15
13. cubic binomial
                                                        37. 30
15. quadratic binomial
                                                        39. 8.6
17. cubic polynomial with 4 terms

19. 6th degree monomial                                 41. 0.995 liters

21. linear polynomial with 5 terms                      43. 9.75 mcg/mL




                                                  273
4. SUMS OF POLYNOMIALS                               CHAPTER 5. POLYNOMIAL EXPRESSIONS
                                  4. Sums of Polynomials


We add polynomials by combining like terms.

  EXAMPLE 4.0.1. Add polynomials by combining like terms:
                               (4x 2 + 6x − 5) + (x 2 − 4x + 5)


  SOLUTION: Adding a sum amounts to adding each of its terms. To make like terms easier
  to see, we change the order of summation and put them next to each other:
             (4x 2 + 6x − 5) + (x 2 − 4x + 5)    =    4x 2 + 6x − 5 + x 2 − 4x + 5
                                                 =    4x 2 + x 2 + 6x − 4x − 5 + 5
                                                 =    (4 + 1)x 2 + (6 − 4)x + 0
                                                 =    5x 2 + 2x



                                     ANSWER: 5x 2 + 2x


  BASIC EXAMPLE 4.0.1. Subtracting a polynomial amounts to adding its opposite. To rewrite
  the opposite of a polynomial as a polynomial expression, we remove the parentheses and
  invert the sign of each term:
                      −(x 7 − x 4 a3 − x a6 + 7) = −x 7 + x 4 a3 + x a6 − 7


  EXAMPLE 4.0.2. Subtract polynomials by combining like terms:
                              (5x 2 − 7x − 6) − (2x 2 + 3x − 4)


  SOLUTION: The opposite of a sum is equal to the sum of opposites. This allows us to
  remove the parentheses on the second polynomial by taking the opposite of each term:
            (5x 2 − 7x − 6) − (2x 2 + 3x − 4)    =    5x 2 − 7x − 6 − 2x 2 − 3x + 4
                                                 =    5x 2 − 2x 2 − 7x − 3x − 6 + 4
                                                 =    (5 − 2)x 2 + (−7 − 3)x − 2
                                                 =    3x 2 − 10x − 2



                                  ANSWER: 3x 2 − 10x − 2


                                              274
CHAPTER 5. POLYNOMIAL EXPRESSIONS                              4. SUMS OF POLYNOMIALS

 EXAMPLE 4.0.3. Find the perimeter of the shape by adding lengths of all sides:

                                         4x + 1


                          2x − 5                         2x − 5


                                         4x + 1


 SOLUTION: The terms of a sum can be added in any order, so we can put like terms next
 to each other and apply the distributive property.
    (4x + 1) + (2x − 5) + (4x + 1) + (2x − 5)    =   4x + 2x + 4x + 2x + 1 − 5 + 1 − 5
                                                 =   (4 + 2 + 4 + 2)x − 8
                                                 =   12x − 8


                                    ANSWER: 12x − 8




                                           275
4. SUMS OF POLYNOMIALS                                CHAPTER 5. POLYNOMIAL EXPRESSIONS
      Homework 5.4.

Simplify the given expression and state the           Simplify the given expression and state the
answer as a polynomial expression. If a               answer as a polynomial expression. If a
polynomial is in one variable, list the terms         polynomial is in one variable, list the terms
of the answer in the decreasing order of de-          of the answer in the decreasing order of de-
gree.                                                 gree.

1. (3x + 7) + (x + 2)                                 17. (3 y + 1) − (5 y + 8)

2. (x + 10) + (12x + 1)                               18. (7 y − 3) − (11 y + 17)

3. (2t + 1) + (−8t + 7)                               19. (4x 2 + x − 7) − (3 − 8x 2 − 4x 3 )

4. (4t + 2) + (−11t − 3)                              20. (3a3 − 2a + 7) − (5a2 − 2a3 + 2a)

5. (7t 2 − 3t + 9) + (2t 2 + 4t − 6)                  21.
                                                          1 3          3      2             7
                                                                         ‹                    ‹
6. (8a2 + 4a − 7) + (6a2 − 4a − 1)                          y + 2 y2 −     − − y3 + 2 y2 +
                                                          5            10     5            1000
7. (3t 3 + 4t 2 − 1) + (−2t 2 − 4t + 5)
                                                                5 3 1    1     1 3 1   1
                                                                          ‹            ‹
                                                      22.         y − y−    − − y + y−
8. (4.9x 3 +3.2x 2 −5.1x)+(2.1x 2 −3.7x +4.5)                   8    4   3     2   4   3

     4 2 1               1 2 1         1              23. (0.9x 3 +0.2x −5)−(0.7x 4 −0.3x −0.1)
                   ‹                   ‹
9.     x + x −3 +          x − x−
     3      2            3       8     2
                                                      24.
10.                                                                                                     
                                                       0.07x 3 − 0.03x 2 + 0.01x − 0.02x 3 − 0.04x 2 − 1
    3 4 1 3 2         2 4 1 3 3 2
                 ‹              ‹
      y + y − y +3 +    y − y − y
    5    2   3        5    4   4
                                                      Find the perimeter of the shape by adding
11. (3x 2 +6x y − x y 2 −4)+(−x y − x y 2 +4)
                                                      lengths of all sides.
12. (7a2 b2 − 6ab) + (a4 + a2 b2 + 6ab)
                                                      25.
                                                                                7x

                                                             14 − 2x                          14 − 2x
Rewrite the opposites of polynomials as
polynomial expressions.                                                         7x

13. −(−5x + 7)                                        26.
                                                                            6 − 2x
14. −(x 2 + x)
                                                             3 + 3x                         3 + 3x
15. −(3a3 − a2 + 8b − 7ab)

16. −(−12z 6 − d 3 − zd)                                                    6 − 2x
                                                276
CHAPTER 5. POLYNOMIAL EXPRESSIONS                           4. SUMS OF POLYNOMIALS
27.                                        28.

                          5x 2 − 1                                   16x − 3
      3x − x 2                                   3x 2 − 1


                 17 − x                                       x 2 + 2x




                                     277
 4. SUMS OF POLYNOMIALS             CHAPTER 5. POLYNOMIAL EXPRESSIONS
      Homework 5.4 Answers.

1. 4x + 9                           15. −3a3 + a2 − 8b + 7ab

3. −6t + 8                          17. −2 y − 7

5. 9t 2 + t + 3                     19. 4x 3 + 12x 2 + x − 10

7. 3t 3 + 2t 2 − 4t + 4                   3 3    307
                                    21.     y −
                                          5     1000
     5 2 3    7
9.     x + x−
     3    8   2                     23. −0.7x 4 + 0.9x 3 + 0.5x − 4.9

11. 3x 2 + 5x y − 2x y 2            25. 10x + 28

13. 5x − 7                          27. 4x 2 + 2x + 16




                              278
CHAPTER 5. POLYNOMIAL EXPRESSIONS                                        5. PRODUCTS OF POLYNOMIALS
                                       5. Products of Polynomials

    5.1. Monomial Times Monomial. Since the order of multiplication does not affect the
value of a product, we can multiply monomials by multiplying their coefficients and adding
the variable powers using the product rule for exponent.

  EXAMPLE 5.1.1. Simplify the expression:
                                            (−5x 2 y 6 )(−2x y 3 z 4 )


  SOLUTION:
          (−5x 2 y 6 )(−2x y 3 z 4 )    =     (−5)(−2) · x 2 x · y 6 y 3 · z 4
                                        =     10 · x 2+1 · y 6+3 · z 4                 product rule
                                        =     10x 3 y 9 z 4


                                            ANSWER: 10x 3 y 9 z 4

   5.2. Monomial Times Polynomial.

  EXAMPLE 5.2.1. Simplify the expression:
                                             3x 2 (5x 2 − 4x − 1)


  SOLUTION: This can be done by distributing, and then simplifying the resulting monomial
  products:
       3x 2 (5x 2 − 4x − 1)      =     (3x 2 )(5x 2 ) − (3x 2 )(4x) − (3x 2 )(1)
                                 =     3 · 5 · x2 · x2 − 3 · 4 · x2 · x − 3 · 1 · x2
                                 =     15x 2+2 − 12x 2+1 − 3x 2                          product rule
                                 =     15x 4 − 12x 3 − 3x 2

  This is a univariate polynomial, so we state the answer in the standard form, by listing the
  terms in the descending order of their monomial degree:


                                       ANSWER: 15x 4 − 12x 3 − 3x 2




                                                      279
5. PRODUCTS OF POLYNOMIALS                               CHAPTER 5. POLYNOMIAL EXPRESSIONS

 EXAMPLE 5.2.2. Simplify the expression:
                                      −10x y 2 (x 2 − 3x y + 2 y 2 )


 SOLUTION: Notice that when we apply the distributive property, we have to compute the
 correct sign for each term:
     −10x y 2 (x 2 − 3x y + 2 y 2 )    =    −(10x y 2 )(x 2 ) + (10x y 2 )(3x y) − (10x y 2 )(2 y 2 )
                                       =    −10 · x 1 x 2 · y 2 + 30 · x 1 x 1 · y 2 y 1 − 20 · x · y 2 y 2
                                       =    −10x 1+2 y 2 + 30x 1+1 y 2+1 − 20x y 2+2
                                       =    −10x 3 y 2 + 30x 2 y 3 − 20x y 4

 This polynomial uses more than one variable, so there is no standard form, and the terms
 of the answer can be listed in any order:


                             ANSWER: −10x 3 y 2 + 30x 2 y 3 − 20x y 4

  5.3. Polynomial Times Polynomial.

 THEOREM 5.3.1. Multiplying two polynomials amounts to taking a sum of all monomials
 obtained by multiplying each term of the first polynomial by each term of the second poly-
 nomial. Since the product of any two monomials is a monomial, and a sum of monomials
 is a polynomial, it follows that the product of two polynomials is a polynomial.


 EXAMPLE 5.3.1. Simplify the expression:
                                           (3x − 2)(x + 5)


 SOLUTION: We start by distributing the multiplication over the first sum:
                          (3x − 2)(x + 5)        =     3x(x + 5) − 2(x + 5)
 Then we distribute again as we multiply binomials by monomials:
  3x(x + 5) − 2(x + 5)      =     (3x)(x) + (3x)(5) − (2)(x) − (2)(5)
                            =     3x 2 + 15x − 2x − 10                             simplified monomials
                            =     3x + (15 − 2)x − 10
                                      2
                                                                                    combined like terms
                            =     3x + 13x − 10
                                      2




                                      ANSWER: 3x 2 + 13x − 10

                                                  280
CHAPTER 5. POLYNOMIAL EXPRESSIONS                              5. PRODUCTS OF POLYNOMIALS

  EXAMPLE 5.3.2. Simplify the expression:
                                   (x + y − 1)(2x + 3 y + 5)


  SOLUTION: This is a product of two trinomials, and we need to multiply each term of the
  first one by each term of the second one. This results in a sum of 3 · 3 = 9 monomials,
  which we simplify by combining the like terms.
  (x + y − 1)(2x + 3 y + 5)    =     x(2x + 3 y + 5) + y(2x + 3 y + 5) − 1(2x + 3 y + 5)
                               =    (2x 2 + 3x y + 5x) + (2x y + 3 y 2 + 5 y) + (−2x − 3 y − 5)
                               =    2x 2 + 3 y 2 + (3x y + 2x y) + (5x − 2x) + (5 y − 3 y) − 5
                               =    2x 2 + 3 y 2 + (3 + 2)x y + (5 − 2)x + (5 − 3) y − 5
                               =    2x 2 + 3 y 2 + 5x y + 3x + 2 y − 5
  This is a polynomial in 2 variables, so there is no standard form, and we can list the terms
  of the answer in any order.


                           ANSWER: 2x 2 + 3 y 2 + 5x y + 3x + 2 y − 5


There are many alternative ways to annotate the process of multiplying polynomials, like the
table method, but of course they all produce the same result.

  EXAMPLE 5.3.3. Simplify the expression by using a table:
                                   (x + y − 1)(2x + 3 y + 5)


  SOLUTION: We will construct a table where the top row will list the terms of the first
  polynomial, and the left column will list the terms of the second polynomial. We will then
  fill out the middle of the table by multiplying the monomials from the corresponding row
  and column. We will complete the task by adding all of the monomials from the middle
  of the table and combining like terms.
                                           +x         +y     −1
                                   +2x    +2x   2
                                                    +2x y −2x
                                   +3 y   +3x y     +3 y 2   −3 y
                                   +5      +5x        +5 y   −5
     2x 2 + 2x y − 2x + 3x y + 3 y 2 − 3 y + 5x + 5 y − 5 = 2x 2 + 3 y 2 + 5x y + 3x + 2 y − 5


                           ANSWER: 2x 2 + 3 y 2 + 5x y + 3x + 2 y − 5

                                                281
5. PRODUCTS OF POLYNOMIALS                            CHAPTER 5. POLYNOMIAL EXPRESSIONS
    Homework 5.5.

Simplify the given expression and state the           17. (x + 3)(2x + 3)
answer as a polynomial expression. If a
polynomial is in one variable, list the terms         18. (3x + 1)(x + 5)
of the answer in the decreasing order of de-
gree.                                                 19. (x − 0.6)(x − 0.7)

1. (4x 4 )(8)                                         20. (1.1x + 10)(x − 1.2)

2. (−7)(6 y 7 )                                       21. (5x + 6 y 3 )(x + y)

3. (−x 5 )(x 8 )                                      22. (x 5 − 2)( y 2 − x y)

                                                            1          1
                                                                 ‹          ‹
4. (− y 3 )(−2 y 5 )                                  23.      −x        x −4
                                                            2          3
     1 2       2 3
         ‹         ‹
5.     t     − t                                                  2      3
                                                                   ‹          ‹
     5         3                                      24. 2x −             x +6
                                                                  3      2
       3 9         3
           ‹         ‹
6. − x         − x                                    25. (x + 1)(x 2 − x + 1)
       7           2

7. (−2x)(3x 2 y)                                      26. (x − 2)(x 2 + 2x + 4)

8. (−1.5a2 b2 )(−2b)                                  27. (x 2 − x + 3)(x + 1)

9. (3x)(5x)(−2x 7 )                                   28. (a2 + a − 7)(a + 2)

      2 2                                             29. (5x 2 − 3x − 1)(x 2 + 4)
         ‹
10.     x (−6x)(−5x 4 )
      3
                                                      30. (1 + 2a2 )(4a2 − 5a − 3)
11. 3x(x − y)
                                                      31. (x + 1)(x + 2)(x + 3)
12. −6 y(6 − 2x y)
                                                      32. 5x 2 (x + 4)(x − 4)
13. 3m(−m2 + 2m − 40)
                                                      33. (a + b)3
14. 12x(2x 3 − x + 3)
                                                      34. (x − 2 y)3
15. 0.5b (0.2b − b + 4)
          3        2
                                                      35. (c 2 − 2c + 3)(c 2 + 4c + 1)
16. 0.1c 4 (−4c 2 + 0.7c − 10)
                                                      36. (−2x 2 − x − 1)(x 2 − 5x + 1)




                                                282
 CHAPTER 5. POLYNOMIAL EXPRESSIONS                  5. PRODUCTS OF POLYNOMIALS
    Homework 5.5 Answers.

1. 32x 4                                   19. x 2 − 1.3x + 0.42

3. −x 13                                   21. 5x 2 + 5x y + 6x y 3 + 6 y 4

        2 5                                     1      25
5. −      t                                23. − x 2 +    x −2
       15                                       3      6

7. −6x 3 y                                 25. x 3 + 1

9. −30x 9                                  27. x 3 + 2x + 3

11. 3x 2 − 3x y                            29. 5x 4 − 3x 3 + 19x 2 − 12x − 4

13. −3m3 + 6m2 − 120m                      31. x 3 + 6x 2 + 11x + 6

15. 0.1b5 − 0.5b4 + 2b3                    33. a3 + 3a2 b + 3ab2 + b3

17. 2x 2 + 9x + 9                          35. c 4 + 2c 3 − 4c 2 + 10c + 3




                                     283
6. SPECIAL PRODUCTS                                   CHAPTER 5. POLYNOMIAL EXPRESSIONS
                                    6. Special Products
   6.1. Difference of Squares.

  THEOREM 6.1.1 (Difference of Squares Formula). For all real numbers A and B,
                                  (A + B)(A − B) = A2 − B 2



   PROOF.
        (A + B)(A − B)   =    A(A − B) + B(A − B)
                         =    A2 − AB + BA − B 2                   −AB and BA are like terms
                         =    A2 − B 2
                                                                                               

This is a nice computational shortcut which has many algebraic applications. In this text we
will use it for factoring polynomial expressions and for simplifying radical expressions.

  EXAMPLE 6.1.1. Simplify and state the result as a polynomial expression:
                                         (2x + 3)(2x − 3)


  SOLUTION: We will use the difference of squares formula (A + B)(A − B) = A2 − B 2 with
  A = 2x and B = 3.
                             (2x + 3)(2x − 3)       =     (2x)2 − (3)2
                                                    =     22 x 2 − 9
                                                    =     4x 2 − 9


                                         ANSWER: 4x 2 − 9


  EXAMPLE 6.1.2. Simplify and state the result as a polynomial expression:
                                    (x 3 − 5 y 6 )(x 3 + 5 y 6 )


  SOLUTION: We will use the difference of squares formula (A + B)(A − B) = A2 − B 2 with
  A = x 3 and B = 5 y 6 .




                                               284
CHAPTER 5. POLYNOMIAL EXPRESSIONS                                               6. SPECIAL PRODUCTS


                       (x 3 − 5 y 6 )(x 3 + 5 y 6 )   =   (x 3 )2 − (5 y 6 )2
                                                      =   x 3·2 − 52 y 6·2
                                                      =   x 6 − 25 y 12


                                     ANSWER: x 6 − 25 y 12


 EXAMPLE 6.1.3. Rewrite the difference of squares as a product of two binomials:
                                              36 − x 2


 SOLUTION: 36 = (6)2 and x 2 = (x)2 , so we can use the difference of squares formula
 (A + B)(A − B) = A2 − B 2 with A = 6 and B = x.
                                   36 − x 2 = (6 + x)(6 − x)


                                   ANSWER: (6 + x)(6 − x)


 EXAMPLE 6.1.4. Rewrite the difference of squares as a product of two binomials:
                                          4a2 − 100b6


 SOLUTION: 4a2 = (2a)2 and 100b6 = (10b3 )2 , so we can use the difference of squares
 formula (A + B)(A − B) = A2 − B 2 with A = 2a and B = 10b3 .
                       4a2 − 100b6        =     (2a)2 − (10b3 )2
                                          =     (2a + 10b3 )(2a − 10b3 )


                            ANSWER: (2a + 10b3 )(2a − 10b3 )




                                                285
6. SPECIAL PRODUCTS                                       CHAPTER 5. POLYNOMIAL EXPRESSIONS
   6.2. Square of a Binomial.

  THEOREM 6.2.1 (Square of a Binomial). For all real numbers A and B, the square of a
  binomial sum can be computed using the following formula:
                                 (A + B)2    =        (A + B)(A + B)
                                             =        A2 + 2AB + B 2
  The square of a binomial difference can be found using a similar formula:
                                 (A − B)2    =        (A − B)(A − B)
                                             =        A2 − 2AB + B 2



   PROOF.
        (A + B)(A + B)     =    A(A + B) + B(A + B)
                           =    A2 + AB + BA + B 2                AB and BA are like terms
                           =    A + 2AB + B
                                 2              2



       (A − B)(A − B)    =     A(A − B) − B(A − B)
                         =     A2 − AB − BA + B 2               −AB and −BA are like terms
                         =     A2 − 2AB + B 2
                                                                                               

  EXAMPLE 6.2.1. Simplify and state the result as a polynomial expression:
                                             (2a + 4)2


  SOLUTION: We will use the square of a binomial formula
                                     (A + B)2 = A2 + 2AB + B 2
  with A = 2a and B = 4:
                             (2a + 4)2   =      (2a)2 + 2(2a)(4) + (4)2
                                         =      22 a2 + 2 · 2 · 4 · a + 16
                                         =      4a2 + 16a + 16
  This is a polynomial in one variable, so we leave the answer in the standard form, listing
  the terms in the descending order of their monomial degrees.


                                     ANSWER: 4a2 + 16a + 16

                                                    286
CHAPTER 5. POLYNOMIAL EXPRESSIONS                                                   6. SPECIAL PRODUCTS

  EXAMPLE 6.2.2. Simplify and state the result as a polynomial expression:
                                                (7x − 6 y 5 )2


  SOLUTION: We will use the square of a binomial formula
                                       (A − B)2 = A2 − 2AB + B 2
  with A = 7x and B = 6 y 5 :
                       (7x − 6 y 5 )2      =     (7x)2 − 2(7x)(6 y 5 ) + (6 y 5 )2
                                           =     72 x 2 − 2 · 7 · 6 · x y 5 + 62 y 5·2
                                           =     49x 2 − 84x y 5 + 36 y 10


                                   ANSWER: 49x 2 − 84x y 5 + 36 y 10


There are other two sign patters for a binomial square, (−A + B)2 and (−A − B)2 , and they can
be reduced to the ones we already discussed.

  EXAMPLE 6.2.3. Simplify and state the result as a polynomial expression: (−x + 10)2


  SOLUTION:
            (−x + 10)2       =      (10 − x)2                                 same as (A − B)2
                             =      (10)2 − 2(10)(x) + (x)2
                             =      100 − 20x + x 2


                                       ANSWER: 100 − 20x + x 2


  EXAMPLE 6.2.4. Simplify and state the result as a polynomial expression: (−5 − x y)2


  SOLUTION:
         (−5 − x y)2     =       ((−5) + (−x y))2                                 same as (A + B)2
                         =       (−5)2 + 2(−5)(−x y) + (−x y)2                  all minuses cancel
                         =       25 + 10x y + x 2 y 2


                                     ANSWER: 25 + 10x y + x 2 y 2

                                                    287
6. SPECIAL PRODUCTS                                   CHAPTER 5. POLYNOMIAL EXPRESSIONS
    Homework 5.6.
                                                                       ‹2
Use appropriate special product formulas                        1 1
                                                            
to simplify the given expression and state            16.        + x
                                                                3 5
the answer as a polynomial expression. If a
                                                                            ‹2
polynomial is in one variable, list the terms                   1 4 6
                                                            
of the answer in the decreasing order of de-          17.         x − x
                                                                4    5
gree.
                                                                            ‹2
                                                                2    4
                                                            
1. (5a + 3)2                                          18.         y − y3
                                                                3    3
2. (1 + 4x)2                                          19. (−7 − 20x)2
3. (x + 10)(x − 10)                                   20. (−2x − 0.4 y)2
4. (9 − a)(9 + a)                                        
                                                            12 15 2
                                                                    ‹
                                                      21. −   +   c
                                                            5   2
5. (4x − 2 y)2
                                                                     ‹2
                                                            5 6
                                                         
6. (a2 − 3)2                                          22. − d + 6d 3
                                                            3
7. (2b + 5c)(2b − 5c)
                                                      23. ((2x + 1)(2x − 1))2
8. (p3 − 7)(p3 + 7)
                                                      24. ((0.5 y − 4x)(0.5 y + 4x))2
9. (2nb − 5b)2

10. (5 − 6x 2 y)2                                     Rewrite the difference of squares as a prod-
                                                      uct of two binomials.
11. (2x + x y 3 )2
                                                      25. 36x 2 − y 2
12. ( y 4 + x 2 )2
                                                      26. 4a2 − 9b2
13. (0.5x − 0.4)(0.5x + 0.4)
                                                      27. c 4 − d 10
14. (2.5x + 3.1 y)(2.5x − 3.1 y)
                                                      28. m6 − 16n8
       1 7 2
         ‹
15. 3 + x
       2                                              29. 25k2 s2 − 0.01k4 s2
                                                                        1 4 4
                                                      30. 49x 8 y 2 −     x y
                                                                        4




                                                288
 CHAPTER 5. POLYNOMIAL EXPRESSIONS                                 6. SPECIAL PRODUCTS
     Homework 5.6 Answers.

1. 25a2 + 30a + 9                                 1 8 3 5 36 2
                                           17.      x − x +    x
                                                 16    5    25
3. x 2 − 100
                                           19. 49 + 280x + 400x 2
5. 16x − 16x y + 4 y
          2               2

                                                 225 2        144
      2        2                           21.      c − 36c +
7. 4b − 25c                                       4            25

9. 4n2 b2 − 20nb2 + 25b2                   23. 16x 4 − 8x 2 + 1

11. 4x 2 + 4x 2 y 3 + x 2 y 6              25. (6x + y)(6x − y)

13. 0.25x 2 − 0.16
                                           27. (c 2 + d 5 )(c 2 − d 5 )
                   1 14
15. 9 + 3x 7 +       x                     29. (5ks + 0.1k2 s)(5ks − 0.1k2 s)
                   4




                                     289
7. QUOTIENTS OF POLYNOMIALS                              CHAPTER 5. POLYNOMIAL EXPRESSIONS
                                7. Quotients of Polynomials
  7.1. Monomial Divisor.

 THEOREM 7.1.1. A division of a polynomial by a monomial can be rewritten as a sum
 of monomial quotients with a common denominator. For all monomials A and B and all
 non-zero monomials C,
                                       A+ B    A B
                                            = +
                                         C     C C
 This result naturally generalizes to sums with more than two terms.



  PROOF.
                                    A+ B                        1
                                              =     (A + B) ·
                                     C                          C
                                                         1     1
                                              =     A·     +B·
                                                         C     C
                                                    A B
                                              =      +
                                                    C C
                                                                                              

 EXAMPLE 7.1.1. Simplify and state the result as a polynomial expression:
                                           x 4 − 2x 3 + 6x 2
                                                  2x


 SOLUTION: We rewrite the division as a sum of monomial quotients and then simplify the
 resulting fractions by canceling common factors:
            x 4 − 2x 3 + 6x 2         x 4 2x 3 6x 2
                                =        −    +
                   2x                 2x   2x   2x
                                      1 x4 x3     x2
                                =      ·  −   +3·                     lowest terms
                                      2 x   x     x
                                    1 3
                                =     x − x 2 + 3x
                                    2
 This is a polynomial in one variable, so we leave the answer in the standard form, listing
 the terms in the descending order of their monomial degrees.


                                                  1 3
                                    ANSWER:         x − x 2 + 3x
                                                  2



                                                  290
CHAPTER 5. POLYNOMIAL EXPRESSIONS                          7. QUOTIENTS OF POLYNOMIALS

  EXAMPLE 7.1.2. Simplify and state the result as a polynomial expression:
                                  −10a5 b6 + 6a3 b4 + 5ab2
                                           −5ab2


  SOLUTION: We rewrite the division as a sum of monomial quotients and then simplify
  the resulting fractions by canceling common factors. Notice that we compute the correct
  signs for each term just as we write the sum of quotients:
     −10a5 b6 + 6a3 b4 + 5a b2        10a5 b6 6a3 b4 5ab2
                                 =           −      −            cancel common factors
              −5a b2                   5ab2    5ab2   5ab2
                                              6
                                 =    2a4 b4 − a2 b2 − 1                in each fraction
                                              5


                                                 6
                                 ANSWER: 2a4 b4 − a2 b2 − 1
                                                 5


Unlike monomial products, not all monomial quotients are monomial. In general, a so-called
rational expression results, and we dedicate a whole chapter to them in this text. For now we
take a sneak peek at how this happens.

  EXAMPLE 7.1.3. Simplify and state the result as a sum of simplified monomial quotients:
                                      −p5 s − 7p2 s3 + p
                                            −p2 s2


  SOLUTION: We rewrite the division as a sum of monomial quotients and then cancel
  common factors in each term.
                       −p5 s − 7p2 s3 + p   p5 s    7p2 s3    p
                                          =       +        −
                             −p2 s2         p2 s2    p2 s2   p2 s2
                                                   p3        1
                                             =        + 7s − 2
                                                   s        ps
  The terms p3 /s and −1/(ps2 ) contain variable reciprocals, so they are not monomial, but
  proper rational expressions in simplified form.


                                             p3        1
                                   ANSWER:      + 7s − 2
                                             s        ps

                                             291
7. QUOTIENTS OF POLYNOMIALS                        CHAPTER 5. POLYNOMIAL EXPRESSIONS
    7.2. Polynomial Divisor. There exists a division algorithm for polynomials, which is useful
for rewriting them as products of polynomial factors.

  THEOREM 7.2.1. If A is a polynomial and B is a non-zero polynomial in one variable, and
  the degree of A is at least as big as the degree of B, then
                                            A        R
                                              =Q+
                                            B        B
  where Q and R are polynomials and the degree of R is less than the degree of B. If the
  polynomial remainder R happens to be zero, then A/B = Q is a polynomial quotient of
  two polynomials.


The polynomial long division algorithm will be explained in examples. In each case, we will
state the result in the form Q + R/B, just as in the theorem (7.2.1).

  EXAMPLE 7.2.1. Simplify using polynomial long division:
                                         x 2 + 5x + 6
                                             x +3


  SOLUTION: We begin by setting up the long division. It resembles the integer long division,
  but instead of columns with digits it has columns of monomial terms. Starting from the
  right and going to the left, the powers of x are 0, 1, 2, and so on, as many as the dividend
  monomial has.

      
  x +3      x 2 + 5x + 6

  Divide the leftmost monomial of the dividend by the leftmost monomial of the divisor:
  x 2 /x = x. Write the result in the appropriate column of the answer space.

                  x
      
  x +3      x 2 + 5x + 6

  Take the opposite of the monomial you just found and multiply it by the divisor x + 3:
                                   (−x)(x + 3) = −x 2 − 3x
  Write the result below the dividend in appropriate columns.

                  x
      
  x +3      x 2 + 5x + 6
          − x 2 − 3x




                                             292
CHAPTER 5. POLYNOMIAL EXPRESSIONS                        7. QUOTIENTS OF POLYNOMIALS
 Add the monomials above the line. No need to write zero for x 2 + (−x 2 ), since that place
 always cancels. Carry down the next monomial term from the dividend.

                    x
      
  x +3      x 2 + 5x + 6
          − x 2 − 3x
                  2x + 6

 Now we repeat the steps with 2x + 6 as the new dividend. Divide the leftmost monomial
 of the dividend by the leftmost monomial of the divisor: 2x/x = 2, and write the result
 in the answer space.

                    x +2
      
  x +3      x + 5x + 6
             2

          − x 2 − 3x
                  2x + 6

 Take the opposite of the monomial you just found and multiply it by the divisor x + 3:
                                  (−2)(x + 3) = −2x − 6
 Write the result below the dividend in appropriate columns.

                    x +2
      
  x +3      x 2 + 5x + 6
          − x 2 − 3x
                   2x + 6
                 − 2x − 6

 Add the monomials above the line.

                    x +2
      
  x +3      x + 5x + 6
             2

          − x 2 − 3x
                   2x + 6
                 − 2x − 6
                        0

 There’s nothing more to carry down, so we have reached the answer. The quotient x + 2 is
 written in the answer space. The remainder 0 is at the very bottom. We state the answer
 as a quotient without a remainder.


                                      ANSWER: x + 2


                                            293
7. QUOTIENTS OF POLYNOMIALS                      CHAPTER 5. POLYNOMIAL EXPRESSIONS

 EXAMPLE 7.2.2. Simplify using polynomial long division:
                                       x 3 − 2x 2 − 4
                                            x −3


 SOLUTION: We begin by setting up the long division. Notice the empty space between the
 terms −2x 2 and −4 of the dividend. This column is reserved for monomials of degree 1.
 Some people prefer to fill the zero entries below the dividend with visible zeroes, but we
 are just leaving them blank.

      
  x −3      x 3 − 2x 2   −4

 Divide the leftmost monomial of the dividend by the leftmost monomial of the divisor:
 x 3 /x = x 2 . Write the result in the appropriate column of the answer space.

                   x2
      
  x −3      x 3 − 2x 2   −4

 Take the opposite of the monomial you just found and multiply it by the divisor x − 3:
                                (−x 2 )(x − 3) = −x 3 + 3x 2
 Write the result below the dividend in appropriate columns.

                   x2
      
  x −3      x 3 − 2x 2   −4
          − x 3 + 3x 2

 Add the monomials above the line. Carry down the next monomial term from the dividend.
 The next term to be carried down is 0x, so we just carry down the empty space.

                   x2
      
  x −3      x 3 − 2x 2   −4
          − x 3 + 3x 2
                   x2

 Repeat the steps with 2x + 6 as the new dividend. Divide the leftmost monomial of the
 dividend by the leftmost monomial of the divisor: x 2 /x = x, and write the result in the
 appropriate column of the answer space.




                                           294
CHAPTER 5. POLYNOMIAL EXPRESSIONS                        7. QUOTIENTS OF POLYNOMIALS
                   x2 + x
      
  x −3      x 3 − 2x 2        −4
          − x 3 + 3x 2
                   x2

 Take the opposite of the monomial you just found and multiply it by the divisor x − 3:
                                   (−x)(x − 3) = −x 2 + 3x
 Write the result below the dividend in appropriate columns.

                   x2 + x
      
  x −3      x 3 − 2x 2        −4
          − x 3 + 3x 2
                   x2
                 − x 2 + 3x

 Add the monomials above the line. Carry down the next monomial term from the dividend.

                   x2 + x
      
  x −3      x 3 − 2x 2        −4
          − x 3 + 3x 2
                   x2
                 − x 2 + 3x
                         3x − 4

 Repeat the steps with 3x − 4 as the new dividend. Divide the leftmost monomial of the
 dividend by the leftmost monomial of the divisor: 3x/x = 3, and write the result in the
 answer space.

                   x2 + x + 3
      
  x −3      x 3 − 2x 2        −4
          − x 3 + 3x 2
                   x2
                 − x 2 + 3x
                         3x − 4

 Take the opposite of the monomial you just found and multiply it by the divisor x − 3:
                                   (−3)(x − 3) = −3x + 9
 Write the result below the dividend in appropriate columns.




                                            295
7. QUOTIENTS OF POLYNOMIALS                          CHAPTER 5. POLYNOMIAL EXPRESSIONS
                   x2 + x + 3
      
  x −3      x 3 − 2x 2        −4
          − x 3 + 3x 2
                   x2
                 − x 2 + 3x
                           3x − 4
                         − 3x + 9

  Add the monomials above the line.

                   x2 + x + 3
      
  x −3      x 3 − 2x 2        −4
          − x 3 + 3x 2
                   x2
                 − x 2 + 3x
                           3x − 4
                         − 3x + 9
                               5

  There’s nothing more to carry down, so we have reached the answer. The quotient x 2 +x+3
  is written in the answer space. The remainder 5 is at the very bottom. We state the answer
  in the form Q + R/B, where Q is the polynomial quotient, R is the polynomial remainder,
  and R is the original polynomial divisor.


                                                                 5
                                    ANSWER: x 2 + x + 3 +
                                                               x −3


Checking these answers is possible, but not easy. One way to do that is to multiply the answer
by the divisor, simplify, and make sure we get the original dividend back:
                          5                                                   5(x − 3)
                            ‹
         x2 + x + 3 +          (x − 3) = x 2 (x − 3) + x(x − 3) + 3(x − 3) +
                        x −3                                                    x −3
                                          =   x 3 − 3x 2 + x 2 − 3x + 3x − 9 + 5

                                          =   x 3 + (−3 + 1)x 2 + (−3 + 3)x − 4

                                          =   x 3 − 2x 2 − 4

We will present a few more examples of polynomial long division without explaining the steps.
The procedure is exactly the same in each case. Readers can do these on their own, and then
compare the solutions in order to check the answer and to detect where the first mistake was
made.
                                               296
CHAPTER 5. POLYNOMIAL EXPRESSIONS                             7. QUOTIENTS OF POLYNOMIALS

                                                                   6x 2 − 5x − 27
  EXAMPLE 7.2.3. Simplify using polynomial long division:
                                                                       2x − 5


  SOLUTION:                             3x + 5
                                2
                   2x − 5      6x − 5x − 27
                             − 6x 2 + 15x
                                       10x − 27
                                     − 10x + 25
                                            −2


                                                           −2
                                  ANSWER: 3x + 5 +
                                                          2x − 5


Checking the answer:
                     −2                                                      −2(2x − 5)
                          ‹
           3x + 5 +          (2x − 5)     =       3x(2x − 5) + 5(2x − 5) +
                    2x − 5                                                    (2x − 5)
                                          =       6x 2 − 15x + 10x − 25 + (−2)

                                          =       6x 2 − 5x − 27


                                                                   x 4 − 2x 3 + 3x − 4
  EXAMPLE 7.2.4. Simplify using polynomial long division:
                                                                          x2 − 5


  SOLUTION:                                       x 2 − 2x + 8
                            
                      x2 − 5      x 4 − 2x 3 + 3x 2         −4
                                − x4         + 5x 2
                                      − 2x 3 + 8x 2
                                        2x 3        − 10x
                                              8x 2 − 10x − 4
                                            − 8x 2      + 40
                                                      − 10x + 36


                                                          −10x + 36
                             ANSWER: x 2 − 2x + 8 +
                                                            x2 − 5


                                               297
7. QUOTIENTS OF POLYNOMIALS                           CHAPTER 5. POLYNOMIAL EXPRESSIONS
     Homework 5.7.

Simplify the given expression and state the           Simplify the expression using polynomial
result as a polynomial expression. If a poly-         long division. State the answer as a poly-
nomial is in one variable, list the terms of          nomial expression when the remainder is
the answer in the decreasing order of de-             zero; otherwise use the form Q + R/B,
gree.                                                 where Q is the polynomial quotient, B is
                                                      the polynomial divisor, and the degree of
     42x 6 − 24                                       the polynomial R is lower than the degree
1.
         4                                            of B.

     8a2 + 10a                                              x 2 + 2x − 15
2.                                                    11.
        16                                                       x +5

     x − 2x 2 + x 7                                         x 2 − 8x + 12
3.                                                    12.
          x                                                      x −6

     4 y 10 − 2 y 3 + y 2                                   4x 2 − 6x + 7
4.                                                    13.
              y2                                               −x − 3

   20t 3 − 25t 2 + 10t                                      3 y2 + 5 y + 4
5.                                                    14.
           −5t                                                 −y + 5

     14 y 5 + 28 y 4 − 70 y 3                               x 2 − 10x − 20
6.                                                    15.
             −14 y                                               x −5

   −12x 2 y 2 + 4x 4 y 2 + 16x y 3                          t 2 + 8t − 15
7.                                                    16.
               4x y                                              t +6

   9m2 n2 − 3m2 n − 6mn2                                    6 y 2 + 17 y + 8
8.                                                    17.
           −3mn                                                  2y + 5

   20a b2 c 3 − 14a2 b2 c 4                               10a2 + 19a + 10
9.                                                    18.
         10ab2 c 2                                            2a + 3

    17x 3 yz 5 + 16x 2 yz 6                               t 3 + 64
10.                                                   19.
           2x 2 yz                                          t +4

                                                            27 − x 3
                                                      20.
                                                             3− x
                                                            a3 − a2 + a − 1
                                                      21.
                                                                a2 + 1

                                                            8t 3 − 22t 2 − 5t + 12
                                                      22.
                                                                 2t 2 − 7t + 4


                                                298
 CHAPTER 5. POLYNOMIAL EXPRESSIONS                 7. QUOTIENTS OF POLYNOMIALS
    Homework 5.7 Answers.

1. 10.5x 6 − 6                                                  61
                                           13. −4x + 18 +
                                                               −x − 3
3. x 6 − 2x + 1
                                                         −45
                                           15. x − 5 +
                                                         x −5
5. −4t 2 + 5t − 2
                                                             3
7. −3x y + x y + 4 y
             3         2                   17. 3 y + 1 +
                                                           2y + 5

9. 2c − 1.4ac 2                            19. t 2 − 4t + 16

11. x − 3                                  21. a − 1




                                     299
8. NEGATIVE EXPONENT                                   CHAPTER 5. POLYNOMIAL EXPRESSIONS
                                       8. Negative Exponent

    8.1. Negative Integer Exponent. Recall that by definition of negative integer exponent,
if b is any non-zero real number and n is a positive integer, then
                                              ‹n
                                              1        1
                                      b =
                                       −n
                                                    = n
                                              b        b

Recall also that negative integer exponent obeys all of the exponential rules we’ve seen so far:

       (1)   The value of exponent with base 1 is 1
       (2)   The product rule and the quotient rule
       (3)   The power rule
       (4)   Distributivity over multiplication and division

In this section we will practice manipulating products and quotients involving negative ex-
ponent by rewriting them in the form where each variable base appears only once, and all
exponents are positive.

  BASIC EXAMPLE 8.1.1. We start with some basic examples. By definition, raising a base
  to a negative power amounts to raising the reciprocal of the base to the corresponding
  positive power:
              ‹2
              1     1
  4   −2
           =      =
              4     16
  Next, observe what happens to the sign of the base. The reciprocal of the negative number
  is negative, so the sign of the exponent has no influence on the sign of the base. What
  does make a difference is the oddness of the exponent. An odd exponent of any sign will
  preserve the sign of the base, and an even exponent will always produce a non-negative
  result:
             ‹3  ‹  ‹  ‹
                1         1      1     1        1
  (−5) = −
       −3
                    = −        −     −    =−
                5         5      5     5      125
             ‹4  ‹  ‹  ‹  ‹
                1         1      1     1     1      1
  (−3) = −
       −4
                    = −        −     −     −     =
                3         3      3     3     3     81
  Rewriting variable exponents is even easier, since we do not have to compute their values:
                   ‹2
                   1         1    a3
                             ‹
  a b 3 −2
             =a ·
                3
                       =a ·
                         3
                                =
                   b         b2   b2




                                                 300
CHAPTER 5. POLYNOMIAL EXPRESSIONS                                    8. NEGATIVE EXPONENT
In general, getting rid of the negative exponent within products and fractions is quite straight-
forward, thanks to the following fact.

  THEOREM 8.1.1. Within a product or a fraction, a factor in the numerator with negative
  exponent can be replaced by the same factor in the denominator with positive exponent.
  For all real numbers a, all non-zero real numbers b and X , and all integers n,
                                            aX −n    a
                                                  =
                                             b      bX n

  Conversely, a factor in the denominator with negative exponent can be replaced by the
  same factor in the numerator with positive exponent.
                                         a      aX n
                                             =
                                       bX −n     b



   PROOF.

First part:
                                     aX −n            a
                                               =         · (X −n )
                                      b               b
                                                            ‹n
                                                      a      1
                                               =         ·
                                                      b      X
                                                             1
                                                            ‹
                                                      a
                                               =         ·
                                                      b      Xn
                                                        a
                                               =
                                                      bX n
Second part:
                                                          1
                                                             ‹
                                     a             a
                                              =      ·
                                    bX −n          b     X −n
                                                        ‹−n
                                                   a     1
                                              =      ·
                                                   b     X
                                                   a
                                              =      · (X n )
                                                   b
                                                   aX n
                                              =
                                                    b
                                                                                               




                                                301
8. NEGATIVE EXPONENT                                            CHAPTER 5. POLYNOMIAL EXPRESSIONS

 EXAMPLE 8.1.1. Simplify and state the answer as an expression with positive exponents:
                                                        x −17 y 5
                                                       x −13 y −6


 SOLUTION: One way to deal with negative exponent factors is by replacing them with
 corresponding positive exponent factors, using the theorem 8.1.1:
                  x −17 y 5          x 13 y 5 y 6
                              =
                 x −13 y −6             x 17
                                     y 5+6
                              =                                     common factor x 13 canceled
                                      x4
                             y 11
                              =
                             x4
 Another way to deal with this is by applying the product and the quotient rules directly to
 the negative exponent. Here we are diving bases, so we will subtract the corresponding
 exponents:
     x −17 y 5
                   =     x −17−(−13) y 5−(−6)
    x −13 y −6
                   =     x −4 y 11

                     y 11
                   =                    got rid of the negative exponent for the answer
                     x4
 Both ways are correct and about as efficient, and choosing one over the other, or a mix of
 the two, is more of a personal preference.


                                                            y 11
                                                    ANSWER:
                                                            x4


 EXAMPLE 8.1.2. Simplify and state the answer as an expression with positive exponents:
                                                     (5x 7 x −4 )−2


 SOLUTION: We will simplify inside the parentheses first since we can. Alternatively, we
 could distribute the exponent over the product first.
                                      (5x 7 x −4 )−2      =     (5x 7+(−4) )−2

                                                          =     (5x 3 )−2


                                                         302
CHAPTER 5. POLYNOMIAL EXPRESSIONS                                            8. NEGATIVE EXPONENT
 Now we distribute exponent over the simplified product:
                        =      5−2 · (x 3 )−2
                               1
                        =         · x 3·(−2)                          product rule
                               52
                               1
                        =         · x −6
                               25
                                1
                        =
                               25x 6


                                                               1
                                           ANSWER:
                                                              25x 6


 EXAMPLE 8.1.3. Simplify and state the answer as an expression with positive exponents:
                                                −10x 7 y 20
                                                (5x 3 y 10 )2


 SOLUTION: We have to distribute the exponent over the product in the denominator
 before we can cancel anything:
               −10x 7 y 20               −10x 7 y 20
                                =
               (5x 3 y 10 )2           52 (x 3 )2 ( y 10 )2
                                       −10x 7 y 20
                                =                                         product rule
                                       52 x 3·2 y 10·2
                                       −10x 7 y 20
                                =
                                        25x 6 y 20
                                        2
                                =      − x 7−6 y 20−20                   quotient rule
                                        5
                                        2
                                =      − x1 y0
                                        5
                                        2
                                =      − x
                                        5


                                                     2
                                            ANSWER: − x
                                                     5



                                                    303
8. NEGATIVE EXPONENT                                CHAPTER 5. POLYNOMIAL EXPRESSIONS

  EXAMPLE 8.1.4. Simplify and state the answer as an expression with positive exponents:
                                          5 ‹−3
                                           x
                                          2 y2


  SOLUTION: Get rid of the negative exponent −3 by taking the reciprocal of its base:
                                 5 ‹−3         2 3
                                   x             2y
                                           =
                                  2 y2            x5
  And now distribute exponent over the fraction:
                                 2 3
                                  2y             23 ( y 2 )3
                                             =
                                    x5            (x 5 )3

                                                     8 y 2·3
                                                =
                                                      x 5·3
                                                     8 y6
                                                =
                                                     x 15


                                                    8 y6
                                        ANSWER:
                                                    x 15



    8.2. Scientific Notation. Scientific notation is a way of writing down extremely large and
extremely small numbers, and is useful in areas such as physics, engineering, and computer
science, to name a few. In fact, many calculators support some sort of scientific notation in or-
der to display very large and very small numbers on a tiny display only a dozen or so characters
wide.

A lot of quantities found in nature look bizarre, to say the least, when we express them in
familiar units suitable for things we do every day as humans. For example, the mass of our
Sun is approximately
                         1988550000000000000000000000000            kg
while the mass of an electron, which is one of the basic building blocks of the tangible world
around us, is approximately
                   0.000000000000000000000000000000910938356               kg
The difference between these two quantities is about 60 decimal places, and these are not even
the most extreme examples of ridiculously large/small numbers commonly occurring in nature.
Scientific notation allows writing these down in a concise form, making it easier to understand
the true magnitude of a number at a glance.
                                              304
CHAPTER 5. POLYNOMIAL EXPRESSIONS                                      8. NEGATIVE EXPONENT

 DEFINITION 8.2.1. A number is said to be in scientific notation if it is written in the form
                                        D.ddd . . . × 10n
 where D is a non-zero digit, ddd . . . are the rest of digits, × is a multiplication sign, and
 n is an integer exponent. The factor 100 , corresponding to n = 0, does not have to be
 shown.


 BASIC EXAMPLE 8.2.1. Let’s write some numbers in scientific notation. Here’s the mass of
 the Sun in kilograms:
                                     1.98855 × 1030
 The mass of an electron in kilograms:
                                      9.10938356 × 10−31
 Recall that multiplying a decimal representation by 10n amounts to shifting the decimal
 point by n digits. If n is positive, the decimal point shifts to the right, and if n is negative,
 the decimal point shifts to the left. Here are some other numbers in decimal notation,
 which we rewrite in scientific notation:
                                       31    =    3.1 × 101
                                      0.6    =    6 × 10−1
                                     402     =    4.02 × 102
                                   0.073     =    7.3 × 10−2
                              0.0000101      =    1.01 × 10−5
                               12345678      =    1.2345678 × 107
                                 1450.67     =    1.45067 × 103


 EXAMPLE 8.2.1. Simplify the expression and state the answer in scientific notation:
                                           16
                                         10000


 SOLUTION:
                                      16
                                              =      0.0016
                                    10000
                                              =      1.6 × 10−3


                                     ANSWER: 1.6 × 10−3




                                               305
8. NEGATIVE EXPONENT                                   CHAPTER 5. POLYNOMIAL EXPRESSIONS

 EXAMPLE 8.2.2. Simplify the expression and state the answer in scientific notation:
                                     (3.4 × 105 )(5 × 107 )


 SOLUTION: The order of multiplication in a product does not matter, so let’s multiply the
 numbers and combine the exponential expressions first:
           (3.4 × 105 )(5 × 107 )   =     (3.4)(5)(105 )(107 )
                                    =     17 · 105+7                     product rule
                                    =     17 · 1012
 This is not yet scientific notation; we still need to express 17 as 1.7 times a power of 10:
          17 · 1012   =    (1.7 × 101 )(1012 )            put 17 in scientific notation
                      =    1.7 × 101+12                          simplify powers of 10
                      =    1.7 × 1013


                                     ANSWER: 1.7 × 1013


 EXAMPLE 8.2.3. Simplify the expression and state the answer in scientific notation:
                                          7.08 × 10−18
                                           9.6 × 10−12


 SOLUTION:
      7.08 × 10−18          7.08 10−18
                       =        ·
      9.6 × 10−12            9.6 10−12
                       =    0.7375 · 10−18−(−12)                             quotient rule
                       =    0.7375 · 10   −6


                       =    7.375 · 10−1 · 10−6          put 0.7375 in scientific notation
                       =    7.375 · 10−1+(−6)                                product rule
                       =    7.375 × 10−7


                                    ANSWER: 7.375 × 10−7




                                                306
CHAPTER 5. POLYNOMIAL EXPRESSIONS                                             8. NEGATIVE EXPONENT
     Homework 5.8.

Simplify the given expression and state the         20. (b−5 )−7
answer as an expression with positive ex-
ponents.                                            21. (4a7 )−3

1. 3−3                                              22. (7x 4 )−2

2. 10−4                                             23. (−2x 2 y −1 )10

3. (−3)−6                                           24. (−3a−9 b4 )−3

4. (−2)−4                                                 15x −7
                                                    25.
                                                          10x −10
5. 8x −3
                                                          −12 y −5
                                                    26.
6. a b−5                                                   2 y −8
      4                                                            ‹−2
                                                              x4
                                                          
7.
     x −2                                           27.
                                                              3
     Q
8.                                                        
                                                              2
                                                                   ‹−3
    X −9                                            28.
    ‹−2                                                      y2
      b
9.                                                                    0
     4                                              29.    x4 + x2 + 1
      ‹−3
       −2                                           30. 17x 3 y −7
                                                                         1
10.
         y
                                                          −3a3 b−5
11. 3−5 · 38                                        31.
                                                          −6a7 b−8
12. 511 · 5−7                                              3x −2 y 4
                                                    32.
13. 2x −14 · x −20                                        −12x y −7

14. − y 7 · y −10                                   33. (x 2 y −3 )(x y −5 )−2

15. (5x −2 y −3 )(2x −4 y)                          34. (a−5 b4 )(a2 b−1 )−3
                                                                          −4
16. (3x     −5
                 y )(2x y )
                  −7    −2                                    −2x 5 y −6
                                                    35.
                                                                x −2
17. (a−5 )3
                                                                         ‹−3
                                                              3a−2 b4
                                                          
18. (b ) 10 −5                                      36.
                                                               2b−1
19. (2−3 )−2


                                              307
8. NEGATIVE EXPONENT                                CHAPTER 5. POLYNOMIAL EXPRESSIONS
Simplify the given expression and state the         54. 0.000000065
answer in decimal notation.
                                                    55. 50000200
                3
37. 4.93 × 10
                                                    56. 4000000000
38. 8.14 × 104

39. 8.93 × 10−3                                     Simplify the given expression and state the
                                                    answer in scientific notation.
40. 7.27 × 10−4
                                                    57. (3 × 105 )(2 × 108 )
41. 3.007 × 10−6
                                                    58. (3.2 × 107 )(2.2 × 10−4 )
42. 1.0204 × 10−5
                                                    59. (3.8 × 109 )(6.5 × 10−2 )
43. 5.033 × 105
                                                    60. (7.3 × 107 )(8.5 × 10−5 )
44. 9.99887 × 106
                                                          8.5 × 108
                                                    61.
                                                          3.4 × 10−5
Write the given number in scientific nota-                5.7 × 10−2
tion.                                               62.
                                                          2.4 × 105
45. 15                                                    7.0 × 103
                                                    63.
46. 123                                                   5.0 × 108
                                                          7.5 × 10−3
47. 0.7                                             64.
                                                          8.0 × 10−6
48. 0.04                                                   12
                                                    65.
                                                          4000
49. 37000000
                                                           13
50. 402000                                          66.
                                                          5000
51. 0.0584                                                658.7
                                                    67.
                                                           14
52. 0.000695
                                                           333
53. 0.0000007                                       68.
                                                          2500




                                              308
 CHAPTER 5. POLYNOMIAL EXPRESSIONS                           8. NEGATIVE EXPONENT
       Homework 5.8 Answers.

      1                                           b3
1.                                         31.
     27                                          2a4
      1                                    33. y 7
3.
     729
                                                  y 24
      8                                    35.
5.                                               16x 28
      x3
                                           37. 4930
           2
7. 4x
                                           39. 0.00893
  16
9. 2
   b                                       41. 0.000003007

11. 27                                     43. 503300
        2
13.                                        45. 1.5 × 10
       x 34

     10                                    47. 7 × 10−1
15. 6 2
    x y
                                           49. 3.7 × 107
        1
17.                                        51. 5.84 × 10−2
       a15

19. 64                                     53. 7.0 × 10−7

         1                                 55. 5.00002 × 107
21.
       64a21
                                           57. 6 × 1013
       1024x 20
23.                                        59. 2.47 × 108
         y 10

       3 3                                 61. 2.5 × 1013
25.      x
       2
                                           63. 1.4 × 10−5
       9
27.                                        65. 3.0 × 10−3
       x8

29. 1                                      67. 4.705 × 101




                                     309
8. PRACTICE TEST 5                                              CHAPTER 5. POLYNOMIAL EXPRESSIONS
                                                     Practice Test 5




1. Find an expression for the perimeter of                      7. Multiply and simplify by combining the
the shown shape, and simplify it by com-                        like terms:
bining the like terms:                                                    (3x − 6)(4 − x + 6x 2 )

                 4                                              8. Simplify the expression by performing
                                      x +1                      the polynomial long division:
                                                                          14x 3 + 7x 2 − 19x + 3
                                                                                  2x + 3
5x 2 − 3x                                                       9. Simplify the expression by performing
                                          x − 2x 2              the polynomial long division:
                                                                              6x 3 − 2x − 5
                                                                                   x −2
                  6x
                                                                10. Simplify the expression and state the
                                                                answer with positive exponents only:
2. Simplify the expression:
                                                                          (−5x −2 y)(−2x −3 y 2 )
            (3k5 m0 )2 · (−k4 m3 )
                                                                11. Simplify the expression and state the
3. Simplify the expression:                                     answer with positive exponents only:
                                                                              −1 2 −2
                         
                             a7 b2
                                     ‹3                                          x y
                     2
                ab                                                             −5x −2 y 4
                             a6 b2

4. Find the degree of the polynomial:                           12. Simplify the expression and state the
                                                                answer in decimal notation:
            x 10 − 6x 9 y 2 + 7x 6 y 6                                   (1.5 × 10−4 )(1.7 × 107 )
5. Describe the polynomial degree and the                       13. Simplify the expression and state the
number of terms in words:                                       answer in scientific notation:
                −4x 2 y + 7x y                                             (4 × 109 )(6 × 104 )

6. Describe the polynomial degree and the                       14. Simplify the expression and state the
number of terms in words:                                       answer in scientific notation:
                                                                                     0.9
                 ax + at − i t                                                      1800




                                                          310
 CHAPTER 5. POLYNOMIAL EXPRESSIONS                                8. PRACTICE TEST 5
    Practice Test 5 Answers.

1. 3x 2 + 5x + 5                                                   39
                                           9. 6x 2 + 12x + 22 +
                                                                  x −2
2. −9k14 m3
                                                 10 y 3
    4 2
3. a b                                     10.
                                                  x5
4. 11                                            25 y 4
                                           11.
5. cubic binomial                                 x2

                                           12. 2250
6. quadratic trinomial

7. 18x 3 − 39x 2 + 18x − 24                13. 2.4 × 1014

8. 7x 2 − 7x + 1                           14. 5.0 × 10−4




                                     311
                                          CHAPTER 6


                                  Factoring Polynomials


                                 1. Greatest Common Factor


                                 6   y

                                 4
                                          y = x 2 − 3x = x(x − 3)
                                 2

                                                                              x
                −2       −1                 1          2       3       4          5

                               −2


                               −4



   1.1. Factoring Polynomials. Factoring a polynomial means rewriting it in a product form,
with as many polynomial factors as possible. This is useful for solving polynomial equations, as
well as for providing an expression which offers hints about the behavior of the corresponding
graph. Some polynomials can only be written as a product with a single factor, for example
                                             (x + 1)
Other polynomials can be written with multiple factors:
                                     (x 2 − 3x) = (x)(x − 3)
In this section we start exploring basic strategies for finding these factorizations.

  DEFINITION 1.1.1. We call a polynomial irreducible over reals if it cannot be written as a
  product of two or more non-constant polynomials with real coefficients.

  We call a polynomial irreducible over integers or simply prime if it cannot be written as a
  product of two or more non-constant polynomials with integer coefficients.


                                                313
1. GREATEST COMMON FACTOR                             CHAPTER 6. FACTORING POLYNOMIALS

  THEOREM 1.1.1 (Polynomial Factorization). Every non-constant polynomial expression
  has a factorization: an equivalent form which is a product of polynomial expressions
  which are irreducible over reals. This factorization is unique if we disregard the order of
  multiplication and state polynomial factors in some kind of standard form.

  Stated in a modern way, all polynomial structures are unique factorization domains.


  THEOREM 1.1.2. All linear polynomials are irreducible over reals, and hence also prime.


  BASIC EXAMPLE 1.1.1. A quadratic polynomial
                                            a2 − b2
  can be written as a product of two linear factors
                                        (a + b)(a − b)
  which are irreducible. The only other equivalent polynomial products with two factors are
  (a − b)(a + b), (a − b)(b + a), and so on, differing only in the order of operations, which
  is what we mean when we say that the factorization is essentially unique.


In the sections up ahead we will not worry too much about doing a perfect job and obtaining
the optimal factorization, but eventually we will list the most notorious types of irreducible
polynomials in order to put together a comprehensive factoring strategy.

   1.2. Greatest Common Factor.

  DEFINITION 1.2.1. The greatest common factor, or GCF for short, for non-zero integers a
  and b is the largest positive integer c which is a factor of both a and b. In the absence
  of common prime factors, the GCF is defined to be 1. The definition extends naturally to
  larger collections of integers.

  Numbers with GCF 1 are called relatively prime.


  BASIC EXAMPLE 1.2.1. Some basic cases:
                  integers GCF
                     2, 3     1 no common prime factors
                   10, 30    10 the largest common integer factor is 10
                     4, 6     2 the only common factor is 2
                   6, 9, 15   3 the only common factor is 3
                  12, 18, 60  3 the largest common integer factor is 6

                                             314
CHAPTER 6. FACTORING POLYNOMIALS                                1. GREATEST COMMON FACTOR

 EXAMPLE 1.2.1. Find the GCF for 16 and 20.


 SOLUTION: For smaller numbers it’s easy enough to guess the GCF by simply staring at
 them, and you may have figured out already that the GCF in this example is 4. Regardless,
 here we will show how one can compute GCF even in difficult cases without guessing.
 First, we represent the numbers as products of primes:
                                      16       =      2·2·2·2
                                      20       =      2·2·5
 The GCF is a product of primes which is in common to all of these factorizations, in this
 case 2 · 2 = 4.


                                           ANSWER: 4


 EXAMPLE 1.2.2. Find the GCF for 15 and 28.


 SOLUTION: Represent the numbers as products of primes and detect all the ones they
 have in common:
                                       15      =        3·5
                                       28      =        2·2·7
 These factorizations have no primes in common, meaning 15 and 28 do not have a single
 common prime factor, so the GCF is 1.


                                           ANSWER: 1


 EXAMPLE 1.2.3. Find the GCF for 96 and 66.


 SOLUTION: Represent the numbers as products of primes and detect all the ones they
 have in common:
                                 96        =       2·2·2·2·2·3
                                 66        =       2 · 3 · 11
 The largest collection of prime factors in common is 2 · 3 = 6


                                           ANSWER: 6


 EXAMPLE 1.2.4. Find the GCF for 500 and 120.

                                                   315
1. GREATEST COMMON FACTOR                          CHAPTER 6. FACTORING POLYNOMIALS

 SOLUTION: Represent the numbers as products of primes and detect all the ones they
 have in common:
                                  500    =    2·2·5·5·5
                                  120    =    2·2·2·3·5
 The largest collection of prime factors in common is 2 · 2 · 5 = 20


                                        ANSWER: 20


 EXAMPLE 1.2.5. Find the GCF for 144, 60, and 90.


 SOLUTION: Represent the numbers as products of primes and detect all the ones they
 have in common:
                                 144    =    2·2·2·2·3·3
                                  60    =    2·2·3·5
                                  90    =    2·3·3·5
 The largest collection of prime factors in common is 2 · 3 = 6


                                        ANSWER: 6

  1.3. Monomial GCF.

 DEFINITION 1.3.1. The greatest common factor, or GCF for short, for non-zero monomials
 A and B is the monomial C such that its coefficient is the GCF of the coefficients of A and
 B, and the exponent of each variable factor of C is no greater than the exponent of the
 same variable in either A or B. The definition extends naturally to larger collections of
 monomials.

 Monomials with GCF 1 are called relatively prime.


 EXAMPLE 1.3.1. Find the GCF for 4a2 b and 2a4 b.


 SOLUTION: The GCF for coefficients is 2, the lowest exponent on a is 2, and the lowest
 exponent on b is 1, so GCF is 2a2 b.


                                       ANSWER: 2a2 b

                                             316
CHAPTER 6. FACTORING POLYNOMIALS                                 1. GREATEST COMMON FACTOR

 EXAMPLE 1.3.2. Find the GCF for 10x 3 y 5 and 15x 4 y 2 z 7 .


 SOLUTION: The GCF for coefficients is 5, the lowest exponent on x is 3, the lowest
 exponent on y is 2, and the lowest exponent on z is taken to be 0, since the first monomial
 does not have z at all. The overall GCF is 5x 3 y 2 .


                                       ANSWER: 5x 3 y 2


 EXAMPLE 1.3.3. Find the GCF for 14x 7 , 21x 5 , and 6x 4 .


 SOLUTION: The GCF for coefficients is 1, and the lowest exponent on x is 4, so GCF is x 4 .


                                         ANSWER: x 4


 EXAMPLE 1.3.4. Rewrite the expression by factoring out the GCF:
                                           12a2 − 6a


 SOLUTION: We determine that the GCF is 6a, and use the distributive property:
                                    12a2 − 6a = 6a(2a − 1)
 We can check the answer by multiplying the polynomials:
                          6a(2a − 1) = 6a · 2a − 6a · 1 = 12a2 − 6a


                                     ANSWER: 6a(2a − 1)


 EXAMPLE 1.3.5. Rewrite the expression by factoring out the GCF:
                                   80x 2 y + 40x y − 90x y 3


 SOLUTION: The GCF for coefficients is 10, the lowest exponent on x is 1, and the lowest
 exponent on y is 1, so the GCF is 10x y.
                       80x 2 y + 40x y − 90x y 3 = 10x y(8x + 4 − 9 y 2 )




                                              317
1. GREATEST COMMON FACTOR                         CHAPTER 6. FACTORING POLYNOMIALS
 We can check the answer by multiplying the polynomials:
             10x y(8x + 4 − 9 y 2 )   =   10x y · 8x + 10x y · 4 − 10x y · 9 y 2
                                      =   80x 2 y + 40x y − 90x y 3


                             ANSWER: 10x y(8x + 4 − 9 y 2 )




                                           318
CHAPTER 6. FACTORING POLYNOMIALS                     1. GREATEST COMMON FACTOR
    Homework 6.1.

Find the GCF for given expressions.         13. 2a2 − 2a − 8

1. 18, 24                                   14. 12x 2 + 6x − 30

2. 20, 32                                   15. 5t 2 + t

3. 12, 30, 60                               16. 3x 3 − x 2

4. 14, 42, 70                               17. −10 y 2 − 5 y

5. 14x, 7x y 2                              18. −4a2 + 3a3

6. 25 y 2 , 16 y 2 z                        19. 6a6 + 9a4 − 12a2

7. a2 b3 , 2ab4                             20. −10x 2 − 15x 4 − 9x 3

8. 6x 3 y 3 , 4x y 2                        21. 5x y 2 − 10x 2 y

9. 10x, 2x 2 , 14x 3                        22. 20a2 b2 + 15ab

10. 5 y 2 , 25 y 3 , y 5                    23. 12x 2 y + 6x y − 2x y 2

                                            24. −4a2 − 2ab + 8b2
Factor out the GCF.
                                            25. 3x 6 y 2 + 3x 5 y 3
11. 6x + 24
                                            26. a4 y 5 − 17a6 y
12. 10x + 40




                                      319
 1. GREATEST COMMON FACTOR            CHAPTER 6. FACTORING POLYNOMIALS
       Homework 6.1 Answers.

1. 6                                 15. t(5t + 1)

3. 6                                 17. 5 y(−2 y − 1)
5. 7x
                                     19. 3a2 (2a4 + 3a2 − 4)
7. a b3
                                     21. 5x y( y − 2x)
9. 2x

11. 6(x + 4)                         23. 2x y(6x + 3 − y)

13. 2(a2 − a − 4)                    25. 3x 5 y 2 (x + y)




                               320
CHAPTER 6. FACTORING POLYNOMIALS                                                           2. GROUPING
                                              2. Grouping

    2.1. Factoring by Grouping. An occasional polynomial with 4 terms can be factored using
a technique knows as grouping, which will be explained in examples.

     EXAMPLE 2.1.1. Factor by grouping: 6a + 3b + 10a2 + 5ab


     SOLUTION: Represent the given polynomial with 4 terms as a sum of two binomials:
                         6a + 3b + 10a2 + 5ab = (6a + 3b) + (10a2 + 5ab)
     For each binomial, factor out the GCF:
                        (6a + 3b) + (10a2 + 5ab) = 3(2a + b) + 5a(2a + b)
     Notice that now we have a sum of two terms, and there is a common factor (2a + b). The
     distributive property allows us to factor it out:
                             3(2a + b) + 5a(2a + b) = (3 + 5a)(2a + b)


                                      ANSWER: (3 + 5a)(2a + b)


Sometimes it may be hard to see how we got from
                                        3(2a + b) + 5a(2a + b)
to
                                        (3 + 5a)(2a + b)
with one application of the distributive property, but we can break this process down into
several steps. To see these expressions are equivalent, it may be helpful to create a temporary
variable T and set it equal to 2a + b. Then we can write
          3(2a + b) + 5a(2a + b)      =     3(T ) + 5a(T )                 substitute T for 2a + b
                                      =     (3 + 5a)T                                distributivity
                                      =     (3 + 5a)(2a + b)               substitute 2a + b for T


     EXAMPLE 2.1.2. Factor by grouping: x 5 − x 4 + x 3 − x 2


     SOLUTION: Represent the given polynomial with 4 terms as a sum of two binomials:
                              x 5 − x 4 + x 3 − x 2 = (x 5 − x 4 ) + (x 3 − x 2 )
     For each binomial, factor out the GCF:
                           (x 5 − x 4 ) + (x 3 − x 2 ) = x 4 (x − 1) + x 2 (x − 1)



                                                    321
2. GROUPING                                          CHAPTER 6. FACTORING POLYNOMIALS
 Notice that now we have a sum of two terms, and there is a common factor (x − 1). The
 distributive property allows us to factor it out:
                         x 4 (x − 1) + x 2 (x − 1) = (x 4 + x 2 )(x − 1)


                                ANSWER: (x 4 + x 2 )(x − 1)


 EXAMPLE 2.1.3. Factor by grouping: 5ab − b + 5a − 1


 SOLUTION: Represent the given polynomial with 4 terms as a sum of two binomials:
                          5ab − b + 5a − 1 = (5ab − b) + (5a − 1)
 For each binomial, factor out the GCF:
                       (5a b − b) + (5a − 1) = b(5a − 1) + 1(5a − 1)
 Notice that we explicitly factored out the GCF 1 in the second group. This is merely a
 precaution to make the application of the distributive property more clear:
                          b(5a − 1) + 1(5a − 1) = (b + 1)(5a − 1)


                                 ANSWER: (b + 1)(5a − 1)


 EXAMPLE 2.1.4. Factor by grouping: 16a3 − 12a2 − 12a + 9


 SOLUTION: Represent the given polynomial with 4 terms as a sum of two binomials:
                   16a3 − 12a2 − 12a + 9 = (16a3 − 12a2 ) + (−12a + 9)
 For each binomial, factor out the GCF:
                 (16a3 − 12a2 ) + (−12a + 9) = 4a2 (4a − 3) + 3(−4a + 3)
 We have a sum of two terms, but there is no common factor. Fortunately, (4a − 3) is the
 opposite of (−4a + 3), so we can obtain a common factor by factoring out −1:
                   4a2 (4a − 3) + 3(−4a + 3) = 4a2 (4a − 3) − 3(4a − 3)
 The distributive property allows us to factor out (4a − 3):
                        4a2 (4a − 3) − 3(4a − 3) = (4a2 − 3)(4a − 3)


                               ANSWER: (4a2 − 3)(4a − 3)




                                             322
CHAPTER 6. FACTORING POLYNOMIALS                                       2. GROUPING
   Homework 6.2.

Factor by grouping.                      13. 35x 3 − 10x 2 − 56x + 16

1. x 3 − 2x 2 + 5x − 10                  14. 14v 3 + 10v 2 − 7v − 5

2. z 3 − 3z 2 + 7z − 21                  15. 6x 3 − 48x 2 + 5x − 40

3. 9n3 + 6n2 + 3n + 2                    16. 28p3 + 21p2 + 20p + 15

4. 10x 3 + 25x 2 + 2x + 5                17. 15ab − 6a + 5b3 − 2b2

5. 4t 3 + 20t 2 + 3t + 15                18. 3mn − 8m + 15n − 40

6. 8a3 + 2a2 + 12a + 3                   19. 5mn + 2m − 25n − 10

7. 7x 3 − 5x 2 + 7x − 5                  20. 8x y + 56x − y − 7

8. 2t 3 − 12t 2 − t + 6                  21. 24x y − 30 y 3 − 20x + 25 y 2

9. 5x 3 + 6x 2 − 10x − 12                22. 56ab − 49a − 16b + 14

10. 10x 3 + 8x 2 − 5x − 4                23. 10x y + 25x + 12 y + 30

11. x 3 − 3x 2 + 6 − 2x                  24. 16x y − 6x 2 + 8 y − 3x

12. a3 + 8a2 + 2a + 16




                                   323
 2. GROUPING                       CHAPTER 6. FACTORING POLYNOMIALS
    Homework 6.2 Answers.

1. (x − 2)(x 2 + 5)               13. (5x 2 − 8)(7x − 2)

3. (3n + 2)(3n2 + 1)              15. (6x 2 + 5)(x − 8)

5. (t + 5)(4t 2 + 3)              17. (3a + b2 )(5b − 2)

7. (7x − 5)(x 2 + 1)              19. (m − 5)(5n + 2)

9. (x 2 − 2)(5x + 6)              21. (6 y − 5)(4x − 5 y 2 )

11. (x − 3)(x 2 − 2)              23. (5x + 6)(2 y + 5)




                            324
CHAPTER 6. FACTORING POLYNOMIALS                                 3. FACTORING X 2 + BX + C
                                 3. Factoring x 2 + bx + c
  3.1. Guessing the Coefficients.

 THEOREM 3.1.1. If a quadratic trinomial x 2 + bx + c with integer coefficients b and c
 factors over integers at all, then it can be written in the form
                               x 2 + bx + c = (x + m)(x + n)
 where m and n are integers such that mn = c and m + n = b.



  PROOF.
                        (x + m)(x + n)    =     x(x + n) + m(x + n)
                                          =     x 2 + x n + mx + mn
                                          =     x 2 + (m + n)x + mn
                                          =     x 2 + bx + c
                                                                                               

 EXAMPLE 3.1.1. Factor the trinomial x 2 + 5x + 6


 SOLUTION: We need to guess coefficients m and n such that mn = 6 and m + n = 5. One
 methodical way to do so is by checking all the possible ways to write 6 as a product of two
 integer factors:
                                     6    =    1·6
                                          =    2·3
                                          =    (−1)(−6)
                                          =    (−2)(−3)
 For each one of these, we can check whether m + n = 5. If it is, then we have found a
 factorization.

 If mn = 1 · 6 then m + n = 1 + 6 = 7 6= 5, no luck.

 If mn = 2 · 3 then m + n = 2 + 3 = 5, success, so x 2 + 5x + 6 = (x + 2)(x + 3).

 We can check the answer by multiplying the polynomials:
                         (x + 2)(x + 3)    =    x 2 + 3x + 2x + 2 · 3
                                           =    x 2 + 5x + 6


                                 ANSWER: (x + 2)(x + 3)

                                              325
3. FACTORING X 2 + BX + C                             CHAPTER 6. FACTORING POLYNOMIALS

  EXAMPLE 3.1.2. Factor the trinomial x 2 + 2x − 35


  SOLUTION: We need to guess coefficients m and n such that mn = −35 and m + n = 2.
                                     −35    =        1(−35)
                                            =        5(−7)
                                            =        7(−5)
                                            =        35(−1)
  If mn = 1(−35) then m + n = 1 − 35 = −34 6= 2, no luck.

  If mn = 5(−7) then m + n = 5 − 7 = −2 6= 2, no luck.

  If mn = 7(−5) then m + n = 7 − 5 = 2, success, so x 2 + 2x − 35 = (x + 7)(x − 5).

  Checking the answer: (x + 7)(x − 5) = x 2 − 5x + 7x − 35 = x 2 + 2x − 35.


                                  ANSWER: (x + 7)(x − 5)


  EXAMPLE 3.1.3. Factor the trinomial x 2 − 8x + 7


  SOLUTION: We need to guess coefficients m and n such that mn = 7 and m + n = −8.
                                     −7    =     1·7
                                           =     (−1)(−7)
  If mn = 1 · 7 then m + n = 1 + 7 = 8 6= −8, no luck.

  If mn = (−1)(−7) then m + n = −1 + (−7) = −8, success, so x 2 − 8x + 7 = (x − 1)(x − 7).


                                  ANSWER: (x − 1)(x − 7)


  EXAMPLE 3.1.4. Factor the trinomial h2 − 5h − 2


  SOLUTION: We need to guess coefficients m and n such that mn = −2 and m + n = −5.
                                      −2    =        1(−2)
                                            =        2(−1)
  If mn = 1(−2) then m + n = 1 − 2 = −1 6= −5, no luck.


                                               326
CHAPTER 6. FACTORING POLYNOMIALS                                  3. FACTORING X 2 + BX + C
  If mn = 2(−1) then m + n = 2 − 1 = 1 6= −5, no luck.

  There are essentially no other ways to have two integers with a product −2, we tried every
  possible one. Since both addition and multiplication are commutative, checking the prod-
  ucts (−1)2 and −2(1) would be redundant. We can conclude that the given polynomial is
  irreducible over integers, or prime for short.


                                       ANSWER: prime



    3.2. Variations of the Pattern. It is possible to spot this binomial pattern in other places
as well, like quadratics with leading coefficient −1, or the ones with a constant coefficient ±1,
or multivariate polynomials.

  EXAMPLE 3.2.1. Factor the trinomial −x 2 + 4x + 21


  SOLUTION: This is not a trinomial of the form x 2 + bx + c because the leading coefficient
  is −1 when it needs to be 1. But we can easily fix that by factoring out −1:
                             −x 2 + 4x + 21   =     −(x 2 − 4x − 21)
  Now we need to find two numbers m and n such that mn = −21 and m + n = −4. Let us
  try m = 3 and n = −7:
              −(x + 3)(x − 7)    =    −(x 2 − 7x + 3x − 21)
                                 =    −(x 2 − 4x − 21)                  looks right
                                 =    −x 2 + 4x + 21
  Note that there are other ways to write this factorization, which look different, but are
  essentially the same. They can be obtained by distributing the leading minus over either
  sum:
                    −(x + 3)(x − 7) = (−x − 3)(x − 7) = (x + 3)(−x + 7)


                                  ANSWER: −(x + 3)(x − 7)


  EXAMPLE 3.2.2. Factor the trinomial −6x 2 − x + 1


  SOLUTION: This is not really a polynomial of the form x 2 + bx + c, since the leading
  coefficient is −6 when it needs to be 1. We can, however, notice that the constant term is
  1, and if we expect an answer in the form
                                       (mx + 1)(nx + 1)

                                              327
3. FACTORING X 2 + BX + C                          CHAPTER 6. FACTORING POLYNOMIALS
  then we are looking for m and n such that mn = −6 and m + n = −1.

  If mn = 1(−6) then m + n = 1 − 6 = −5, no luck.

  If mn = 2(−3) then m + n = 2 − 3 = −1, success, so −6x 2 − x + 1 = (2x + 1)(−3x + 1).
  We can check the answer by multiplying the polynomials:
                (2x + 1)(−3x + 1)    =    (2x)(−3x) + (2x)1 + 1(−3x) + 1 · 1
                                     =    −6x 2 + 2x − 3x + 1
                                     =    −6x 2 − x + 1


                                ANSWER: (2x + 1)(−3x + 1)


  EXAMPLE 3.2.3. Factor the trinomial y 2 + 11 yz − 26z 2


  SOLUTION: This is a trinomial with two variables, but it uses the same pattern for coef-
  ficients as x 2 + bx + c, with the mixed term 11 yz playing the role of the linear term bx,
  and the −26z 2 term playing the role of the constant c. So we are looking for m and n such
  that mn = −26 and m + n = 11.

  If mn = 1(−26) then m + n = 1 − 26 = −25 6= 11, no luck.

  If mn = 2(−13) then m + n = 2 − 13 = −11 6= 11, no luck.

  If mn = 13(−2) then m+n = 13−2 = 11, success, so y 2 +11 yz −26z 2 = ( y +13z)( y −2z).


                                ANSWER: ( y + 13z)( y − 2z)



    3.3. Basic Factoring Strategy. By now we have seen 3 different techniques for factoring
a polynomial expression: factoring out the GCF, factoring by grouping, and factoring a simple
trinomial. Recall that the final factorization of any polynomial is essentially unique, so we
can apply these and other techniques incrementally and in any order we want. In most cases,
however, it is convenient to follow a simple strategy.

To take the advantage of all the factoring techniques we know so far,

    (1) Factor out the GCF, unless it’s 1.
    (2) If any of the remaining polynomial factors have four terms, try to factor them by
        grouping.
    (3) If any of the remaining polynomial factors are trinomials of the form x 2 + bx + c, try
        to factor them by guessing integer coefficients.

                                             328
CHAPTER 6. FACTORING POLYNOMIALS                                   3. FACTORING X 2 + BX + C

 EXAMPLE 3.3.1. Factor the trinomial x 3 + 3x 2 + 2x


 SOLUTION: Factor out the GCF first:
                               x 3 + 3x 2 + 2x = x(x 2 + 3x + 2)
 We attempt to factor the quadratic trinomial next. We need to replace it by an expression
 (x + n)(x + m) where mn = 2 and m + n = 3. The ways to write 2 as a product of two
 integers mn:
                                      2   =      1·2
                                          =      (−1)(−2)
 If mn = 1 · 2 then m + n = 1 + 2 = 3, success, so
                              x(x 2 + 3x + 2) = x(x + 2)(x + 3)


                                 ANSWER: x(x + 2)(x + 3)


 EXAMPLE 3.3.2. Factor the polynomial 10x 3 y − 2x 2 y + 30x y − 6 y


 SOLUTION: Every term of this polynomial has factors 2 and y, so we start by factoring
 out the GCF 2 y:
                10x 3 y − 2x 2 y + 30x y − 6 y    =    2 y(5x 3 − x 2 + 15x − 3)
 We attempt to factor the polynomial with 4 terms next. The only appropriate technique
 at our disposal is grouping. Notice that the GCF will have to be simply rewritten all the
 way until the answer. Nothing else we factor can possibly affect it.
                                                           
 2 y(5x 3 − x 2 + 15x − 3) = 2 y (5x 3 − x 2 ) + (15x − 3)            create binomial groups
                                                            
                            = 2 y x 2 (5x − 1) + 3(5x − 1)       find GCF for each binomial
                                                    
                            = 2 y (x 2 + 3)(5x − 1)                             distributivity



                               ANSWER: 2 y(x 2 + 3)(5x − 1)




                                              329
3. FACTORING X 2 + BX + C                 CHAPTER 6. FACTORING POLYNOMIALS
   Homework 6.3.

Factor the polynomial expression.         16. 3 y 2 + 9 y − 84

1. x 2 − 8x + 16                          17. x 3 − 6x 2 − 16x

2. a2 + 9a + 20                           18. x 3 − x 2 − 20x

3. x 2 − 11x + 10                         19. 5b2 + 35b − 150

4. y 2 − 8 y + 7                          20. x 4 + x 3 − 56x 2

5. t 2 + 9t + 14                          21. −b5 + b4 + 2b3

6. a2 + 14a + 40                          22. −2x 2 + 4x + 70

7. b2 + 5b + 4                            23. a2 + 5a + 9

8. z 2 + 8z + 7                           24. x 2 − 7x + 18

9. d 2 + 7d + 10                          25. −3x 3 + 63x 2 + 300x

10. x 2 + 8x + 15                         26. 2x 2 − 42x − 144

11. a2 − 2a − 15                          27. x 2 + 7x y + 10 y 2

12. b2 − b − 42                           28. x 2 − 2x y − 3 y 2

13. x 2 − 4x − 45                         29. b2 + 7bc + 7c 2

14. x 2 + 22x + 121                       30. 5x 7 − 20x 6 − 25x 5

15. 2x 2 + 14x − 36




                                    330
 CHAPTER 6. FACTORING POLYNOMIALS                       3. FACTORING X 2 + BX + C
    Homework 6.3 Answers.

1. (x − 4)(x − 4)                         17. x(x + 2)(x − 8)

3. (x − 1)(x − 10)                        19. 5(b − 3)(b + 10)
5. (t + 7)(t + 2)
                                          21. −b3 (b − 2)(b + 1)
7. (b + 1)(b + 4)
                                          23. prime
9. (d + 2)(d + 5)
                                          25. −3x(x − 25)(x + 4)
11. (a − 5)(a + 3)

13. (x + 5)(x − 9)                        27. (x + 5 y)(x + 2 y)

15. 2(x − 2)(x + 9)                       29. prime




                                    331
4. FACTORING AX 2 + BX + C                            CHAPTER 6. FACTORING POLYNOMIALS
                                  4. Factoring ax 2 + bx + c
   4.1. Guessing the Coefficients.

  THEOREM 4.1.1. If a quadratic trinomial ax 2 + bx + c with integer coefficients a, b and c
  factors over integers at all, then it can be written in the form
                               a x 2 + bx + c = (g x + m)(hx + n)
  where g, h, m, n are integers such that gh = a, mn = c, and gn + mh = b.



   PROOF.
                     (g x + m)(hx + n)      =   g x(hx + n) + m(hx + n)
                                            =   g xhx + g x n + mhx + mn
                                            =   (gh)x 2 + (gn + mh)x + mn
                                            =   ax 2 + bx + c
                                                                                               

When we try to factor
                                            ax 2 + bx + c
we would like to make sure we checked every possible combination of integer coefficients g,
h, m, n such that gh = a and mn = c, but we want to avoid redundant checking of equivalent
combinations of coefficients. To do so, we will use a specific pattern. We will also assume that
a is positive: if it is not, we can easily make it positive by factoring out −1, for example
                               −3x 2 + 5x + 2 = −(3x 2 − 5x − 2)

  THEOREM 4.1.2. In order to check whether a trinomial with integer coefficients
                                           ax 2 + bx + c
  with a > 0, factors over integers as
                                         (g x + m)(hx + n)
  it is sufficient to examine every pair of positive integers g and h such that gh = a and
  g ≤ h, and for each one of these, every possible integer pair m and n with mn = c.


There are many other ways to check all combinations of four coefficients systematically, and
none of them are particularly better than the one presented in this text. It is important to
understand that experienced human algebraists rarely go through a list of possibilities to factor
trinomials. Instead, after doing dozens and dozens of basic examples, they train the brain to
recognize these patterns instantly, and combinations of coefficients which just feel right often
turn out to be the solutions. The reader should expect to learn (eventually) to try only a few
combinations “at random”, check whether the factorization works each time, and “stumble” on
the correct answer after only a few attempts.
                                                332
CHAPTER 6. FACTORING POLYNOMIALS                                    4. FACTORING AX 2 + BX + C
  4.2. Factoring Trinomials.

 EXAMPLE 4.2.1. Factor the trinomial 2x 2 + 7x + 3


 SOLUTION: We need to guess 4 coefficients g, h, m, n such that gh = 2, mn = 3, and
 g n + mh = 7. We will do so by going over all of the ways to represent coefficients 2 and
 3 as products of two integers. The combinations are presented in the table that follows.
 Note that for g and h we only try 1 and 2 respectively, because 1 · 2 is the only way to get
 gh = 2 with positive g ≤ h. But for m and n we try every possible assignment which gives
 us mn = 7. The row with the correct value of gn + mh = 7 is highlighted.


                                  g h       m      n gn + mh
                                  1    2  1  3               5
                                  1    2  3  1               7
                                  1    2 −1 −3              −5
                                  1    2 −3 −1              −7

 The second row yields the correct coefficients, so the answer is
                             (g x + m)(hx + n) = (x + 3)(2x + 1)
 The answer can be checked by multiplying and combining like terms:
                     (x + 3)(2x + 1)    =       x(2x) + x · 1 + 3(2x) + 3 · 1
                                        =       2x 2 + x + 6x + 3
                                        =       2x 2 + 7x + 3


                                 ANSWER: (x + 3)(2x + 1)


 EXAMPLE 4.2.2. Factor the trinomial 8x 2 + 22x + 15


 SOLUTION: We need to guess 4 coefficients g, h, m, n such that gh = 8, mn = 15, and
 g n + mh = 22. We will do so by going over all of the ways to represent coefficients 8 and
 15 as products of two integers. The combinations are presented in the table that follows.
 Note that for g · h we only try two pairs: 1 · 8 and 2 · 4, because these are the only ways to
 get gh = 8 with positive g ≤ h. But for m and n we try every possible assignment which
 gives us mn = 15.




                                                333
4. FACTORING AX 2 + BX + C                           CHAPTER 6. FACTORING POLYNOMIALS

                                 g h      m         n gn + mh
                                 1   8   1 15             23
                                 1   8   3  5             29
                                 1   8   5  3             43
                                 1   8  15  1            122
                                 1   8 −1 −15            −23
                                 1   8 −3 −5             −29
                                 1   8 −5 −3             −43
                                 1   8 −15 −1           −122
                                 2   4   1 15             34
                                 2   4   3  5             22
                                 2   4   5  3             26
                                 2   4  15  1             62
                                 2   4 −1 −15            −34
                                 2   4 −3 −5             −22
                                 2   4 −5 −3             −26
                                 2   4 −15 −1            −62

  The highlighted row with gn + mh = 22 provides the coefficients for the answer
                            (g x + m)(hx + n) = (2x + 3)(4x + 5)


                                 ANSWER: (2x + 3)(4x + 5)


  EXAMPLE 4.2.3. Factor the trinomial 5x 2 − 27x + 10


  SOLUTION: We need to guess 4 coefficients g, h, m, n such that gh = 5, mn = 10, and
  g n + mh = −27. We will do so by going over all of the ways to represent coefficients 5 and
  10 as products of two integers. The combinations are presented in the table that follows.
  Note that for g · h we only try one pair: 1 · 5, because this is the only way to get gh = 5
  with positive g ≤ h. But for m and n we try every possible assignment which gives us
  mn = 10.




                                              334
CHAPTER 6. FACTORING POLYNOMIALS                             4. FACTORING AX 2 + BX + C

                               g h     m         n gn + mh
                               1   5   1 10            15
                               1   5   2  5            15
                               1   5   5  2            27
                               1   5  10  1            52
                               1   5 −1 −10           −15
                               1   5 −2 −5            −15
                               1   5 −5 −2            −27
                               1   5 −10 −1           −15

 The highlighted row gn + mh = −27 shows the correct coefficients, so the answer is
                          (g x + m)(hx + n) = (5x − 2)(x − 5)


                               ANSWER: (5x − 2)(x − 5)




                                           335
4. FACTORING AX 2 + BX + C         CHAPTER 6. FACTORING POLYNOMIALS
   Homework 6.4.

1. 2x 2 − 13x − 7                  14. 49x 2 − 42x + 9

2. 2x 2 + 7x − 4                   15. −20 y 2 + 25 y − 5

3. 9x 2 − 9x + 2                   16. −10a2 + 8a + 18

4. 5a2 − 19a − 4                   17. 6z 2 + 21z + 15

5. 3x 2 + 19x + 6                  18. 12a2 + 68a − 24

6. 35x 2 + 34x + 8                 19. 16t 2 − 23t + 7

7. 8t 2 + 6t − 9                   20. 9t 2 − 14t + 5

8. −4x 2 − 12x − 5                 21. 9x 2 + 18x + 5

9. 1 − 4x + 3x 2                   22. 16x 2 + 32x + 7

10. 9 − 21x − 18x 2                23. 18x 3 + 33x 2 + 9x

11. 2x 2 − 6x − 17                 24. 6a3 − 4a2 − 10a

12. 15 + c − 2c 2                  25. 14x 4 − 19x 3 − 3x 2

13. 25x 2 − 40x + 16               26. 70 y 4 − 68 y 3 + 16 y 2




                             336
 CHAPTER 6. FACTORING POLYNOMIALS                      4. FACTORING AX 2 + BX + C
    Homework 6.4 Answers.

1. (2x + 1)(x − 7)                        15. −5( y − 1)(4 y − 1)

3. (3x − 1)(3x − 2)                       17. 3(x + 1)(2x + 5)
5. (3x + 1)(x + 6)
                                          19. (16t − 7)(t − 1)
7. (2t + 3)(4t − 3)
                                          21. (3x + 1)(3x + 5)
9. (3x − 1)(x − 1)

11. prime                                 23. 3x(3x + 1)(2x + 3)

13. (5x − 4)2                             25. x 2 (2x − 3)(7x + 1)




                                    337
5. FACTORING SPECIAL PRODUCTS                               CHAPTER 6. FACTORING POLYNOMIALS
                                5. Factoring Special Products

   5.1. Difference of Squares. Recall the difference of squares formula:
                                  A2 − B 2       =      (A + B)(A − B)
for any two reals A and B.

  EXAMPLE 5.1.1. Factor the special product expression 25x 2 − 9 y 2


  SOLUTION: This can actually be done by guessing coefficients for a factored form of the
  trinomial 25x 2 + 0x y − 9 y 2 , but being able to detect a difference of squares saves a lot of
  time and effort:
                             25x 2 − 9 y 2       =     (5x)2 − (3 y)2
                                                 =     (5x + 3 y)(5x − 3 y)


                                 ANSWER: (5x + 3 y)(5x − 3 y)


  EXAMPLE 5.1.2. Factor the special product expression 200x 6 − 18


  SOLUTION: 200 and 18 are not squares of integers, but may be a special product will
  become evident if we factor out GCF first:
                          200x 6 − 18        =       2(100x 6 − 9)
                                                                          
                                             =       2 (10x 3 )2 − (3)2
                                                                             
                                             =       2 (10x 3 + 3)(10x 3 − 3)


                               ANSWER: 2(10x 3 + 3)(10x 3 − 3)


It is worthy of note that a sum of squares is irreducible over reals.

  THEOREM 5.1.1. If X and Y are monomials with relatively prime coefficients and no vari-
  ables in common, then the binomial X 2 + Y 2 is irreducible over reals.


  BASIC EXAMPLE 5.1.1. It may be tempting to write that x 2 +4 is equivalent to (x +2)(x −2)
  or perhaps (x +2)2 , but neither statement is true. Just like any other sum of squares, x 2 +4
  is prime.

                                                     338
CHAPTER 6. FACTORING POLYNOMIALS                           5. FACTORING SPECIAL PRODUCTS

 EXAMPLE 5.1.3. Factor the expression 27x 7 + 3x 5 .


 SOLUTION: Factor out GCF first.
                                 27x 7 + 3x 5 = 3x 5 (9x 2 + 1)
 (9x 2 + 1) is a sum of squares, which is prime, so we cannot factor the polynomial any
 further.


                                  ANSWER: 3x 5 (9x 2 + 1)


 EXAMPLE 5.1.4. Factor the expression a4 − b4


 SOLUTION: Factor as a difference of squares:
                              a4 − b4      =   (a2 )2 − (b2 )2
                                           =   (a2 + b2 )(a2 − b2 )
 The factor (a2 + b2 ) is a sum of squares, which is prime. The factor (a2 − b2 ) though is a
 difference of squares again, so we can factor it further:
                    (a2 + b2 )(a2 − b2 )   =     (a2 + b2 ) ((a + b)(a − b))


                             ANSWER: (a2 + b2 )(a + b)(a − b)




                                               339
5. FACTORING SPECIAL PRODUCTS                              CHAPTER 6. FACTORING POLYNOMIALS
   5.2. Square of a Binomial. Recall the binomial square formulas:
                                 A2 + 2AB + B 2        =    (A + B)2
                                 A2 − 2AB + B 2        =    (A − B)2
for any two reals A and B.

  EXAMPLE 5.2.1. Factor the special product expression 64x 2 + 16x + 1


  SOLUTION: This can be done by guessing the coefficients, but detecting the square of a
  binomial saves some time. Here we notice that 64x 2 = (8x)2 and 1 = (1)2 , while 16x is
  indeed 2(8)(x).
                        64x 2 + 16x + 1      =    (8x)2 + 2(8)(x) + (1)2
                                             =    (8x + 1)2


                                      ANSWER: (8x + 1)2


  EXAMPLE 5.2.2. Factor the special product expression 25 y 2 − 60 yz + 36z 2


  SOLUTION: This can be done by guessing the coefficients, but detecting the square of a
  binomial saves some time. Here we notice that 25 y 2 = (5 y)2 , and 36z 2 = (6z)2 , while
  −60 yz is indeed −2(5 y)(6x).
                    25 y 2 − 60 yz + 36z 2   =     (5 y)2 − 2(5 y)(6x) + (6z)2
                                             =     (5 y − 6z)2


                                     ANSWER: (5 y − 6z)2




                                                 340
CHAPTER 6. FACTORING POLYNOMIALS              5. FACTORING SPECIAL PRODUCTS
   Homework 6.5.

Factor the given expression.             16. 8 y 2 − 98

1. x 2 − 14x + 49                        17. x 2 − 5x y + 9 y 2

2. x 2 − 10x + 25                        18. a2 − 9ab − 10b2

3. 4x 2 + 8x + 4                         19. 5x 4 + 125x 2

4. 9 y 2 + 12 y + 4                      20. ab2 + 9a

5. x 2 − 100                             21. 25 y 2 − 4

6. 25x 2 − 1                             22. 80x 2 − 45

7. 3n3 + 60n2 + 300n                     23. 81x 4 − 625

8. x 5 + 24x 4 + 144x 3                  24. x 4 y 4 − 1

9. 20a2 + 100a + 125                     25. x 8 − 256

10. 27x 2 + 36x + 12                     26. y 16 − 1

11. 2x 2 + 28x + 98                      27. x 2 + 16x y + 64 y 2

12. 16 y 2 − 24 y + 9                    28. 81 y 2 − 18 yz + z 2

13. 6x 2 − 24                            29. 32x 2 − 80x y + 50 y 2

14. 5 y 2 − 5                            30. −36c 2 − 96cd − 64d 2

15. 200t 2 − 8




                                   341
 5. FACTORING SPECIAL PRODUCTS          CHAPTER 6. FACTORING POLYNOMIALS
    Homework 6.5 Answers.

1. (x − 7)2                            17. prime

3. 4(x + 1)2                           19. 5x 2 (x 2 + 25)
5. (x + 10)(x − 10)
                                       21. (5 y + 2)(5 y − 2)
7. 3n(n + 10)      2

                                       23. (9x 2 + 25)(3x + 5)(3x − 5)
9. 5(2x + 5)   2

                                       25. (x 4 + 16)(x 2 + 4)(x + 2)(x − 2)
11. 2(x + 7)   2



13. 6(x + 2)(x − 2)                    27. (x + 8 y)2

15. 8(5t 2 + 1)(5t 2 − 1)              29. 2(4x − 5 y 2 )




                                 342
CHAPTER 6. FACTORING POLYNOMIALS                            6. GENERAL FACTORING STRATEGY
                                 6. General Factoring Strategy

    6.1. Irreducible Polynomials. In this section we will combine all of the polynomial fac-
toring techniques presented so far. We will also describe which polynomials are irreducible.
Knowing which polynomials cannot be factored any further will gives us an assurance that our
answers are in fact in the fully factored form.

     (1) By theorem 1.1.2, all linear polynomials are prime.
     (2) Some prime trinomials such as x 2 + x + 1 can be detected by trying out every feasible
         combination of coefficients in factorizations. See theorems 3.1.1 and 4.1.1.
     (3) By theorem 5.1.1, sums of relatively prime monomial squares X 2 + Y 2 are irreducible
         over reals and therefore also prime.

  BASIC EXAMPLE 6.1.1. In this text we will usually recognize a fully factored polynomial as
  a product of linear and irreducible quadratic factors, something like
                                       5x 7 (x + 1)3 (x 2 + 4)
  We use exponential notation for brevity, but if we think of this as a product of polynomial
  factors, then x 7 as a linear factor x raised to the 7th power, (x + 1)3 is a linear factor
  x + 1 raised to the 3rd power, and x 2 + 4 is an irreducible quadratic factor raised to the
  1st power.



    6.2. Factoring Strategy. By now we have seen several different techniques for factoring a
polynomial expression: factoring out the GCF, factoring by grouping, factoring a trinomial by
guessing the coefficients, and the special product shortcuts. Recall that the final factorization
of any polynomial is essentially unique, so we can apply these techniques incrementally and in
any order we want. In most cases, however, it is convenient to try them in a specific order.

To take the advantage of all the factoring techniques we know so far,

     (1) Factor out the GCF, unless it’s 1.
     (2) If a polynomial factor has four terms, try to factor it by grouping.
     (3) If a polynomial factor has two terms, try to factor it as a difference of squares A2 − B 2 .
     (4) If a polynomial factor is a trinomial of the form x 2 + bx + c, try to factor it by guessing
         two integer coefficients for a product of two binomials.
     (5) If a polynomial factor is a trinomial of the form ax 2 + bx + c, try to factor it as a
         binomial square, and if that fails, try to guess four integer coefficients for a product
         of two binomials.

One may have to apply these techniques repeatedly, until only prime polynomial factors remain.




                                                343
6. GENERAL FACTORING STRATEGY                         CHAPTER 6. FACTORING POLYNOMIALS

 EXAMPLE 6.2.1. Factor the polynomial completely:
                                             x2 + x


 SOLUTION: x 2 + x = x(x + 1), which is a product of a linear monomial and a linear
 binomial, so every factor is irreducible.


                                     ANSWER: x(x + 1)


 EXAMPLE 6.2.2. Factor the polynomial completely:
                                      3x 3 − 3x 2 + x − 1


 SOLUTION: Factoring by grouping yields
                       3x 3 − 3x 2 + x − 1   =      3x 2 (x − 1) + 1(x − 1)
                                             =      (3x 2 + 1)(x − 1)
 The factor 3x 2 +1 is an irreducible quadratic binomial, and x−1 is linear, so also irreducible
 over reals.


                                 ANSWER: (3x 2 + 1)(x − 1)


 EXAMPLE 6.2.3. Factor the polynomial completely:
                                     −m6 + 2m5 + 35m4


 SOLUTION: The GCF is m4 , and we also notice that factoring out −1 will create an easy
 trinomial pattern:
                       −m6 + 2m5 + 35m4        =      −m4 (m2 − 2m − 35)
                                               =      −m4 (m + 5)(m − 7)
 We think of m4 as a linear factor m raised to 4th power, so every factor is linear, and hence
 prime, and we are done.


                               ANSWER: −m4 (m + 5)(m − 7)




                                              344
CHAPTER 6. FACTORING POLYNOMIALS                               6. GENERAL FACTORING STRATEGY

 EXAMPLE 6.2.4. Factor the polynomial completely:
                                   6x 3 y 5 − 21x 4 y 5 + 3x 3 y 6


 SOLUTION: We start by factoring out the GCF:
                    6x 3 y 5 − 21x 4 y 5 + 3x 3 y 6     =    3x 3 y 5 (2 − 7x + y)
 This is a product of a monomial and a linear trinomial, so we are done.


                                 ANSWER: 3x 3 y 5 (2 − 7x + y)


 EXAMPLE 6.2.5. Factor the polynomial completely:
                                       a3 − 5a2 − 4a + 20


 SOLUTION: The GCF is 1, and there are four terms, so let’s try grouping:
                       a3 − 5a2 − 4a + 20        =       a2 (a − 5) − 4(a − 5)
                                                 =       (a2 − 4)(a − 5)
 a2 − 4 is not linear, so we should check whether we can factor it more, and in fact we can
 as a difference of squares:
                       (a2 − 4)(a − 5)       =        (a2 − 22 )(a − 5)
                                             =        ((a + 2)(a − 2)) (a − 5)


                               ANSWER: (a + 2)(a − 2)(a − 5)




                                                 345
6. GENERAL FACTORING STRATEGY                   CHAPTER 6. FACTORING POLYNOMIALS
    Homework 6.6.

Factor the given polynomial completely.         16. z 5 − 2z 4 + 5z 3 − 10z 2

1. 5 y 2 − 80                                   17. 5 y 5 − 80 y

2. 10 y 2 − 490                                 18. 7z 4 − 112

3. 18 y 4 − 12 y 3                              19. x 2 − 5x y + 8 y 2

4. x 2 + x − 12                                 20. 25z 2 + 10z x + x 2

5. 6x 3 − 9x 2 + 2x − 3                         21. 16t 2 + 28t − 30

6. x 2 + 5x + x y + 5 y                         22. 36x 2 + 24x − 45

7. 6x 3 + x 2 − 5x                              23. −6x 2 − 7x + 1

8. 25 y 3 − 30 y 2 + 9 y                        24. 16x 4 + y 4

9. 3x 4 + 6x 2                                  25. 4w3 + 20w2 − 4w − 20

10. 4 y 2 + 36                                  26. u4 + 3u3 − 16u2 − 48u

11. 4z 2 w + 13zw + 10w                         27. 4x 2 + y 2 − 4x y

12. 5x 2 − 30x + 10                             28. 11a4 − 11b4

13. x 2 y 4 + 4x y 4 − 32 y 4                   29. −360 y 2 + x 2 + 2x y

14. −60 + 52 y − 8 y 2                          30. 3x 3 + 17x 2 y − 6x y 2

15. x 4 + 6x 3 − 6x 2 − 36x




                                          346
 CHAPTER 6. FACTORING POLYNOMIALS               6. GENERAL FACTORING STRATEGY
     Homework 6.6 Answers.

1. 5( y + 4)( y − 4)                      17. 5 y( y 2 + 4)( y + 2)( y − 2)

3. 6 y 3 (3 y − 2)                        19. prime
5. (2x − 3)(3x 2 + 1)
                                          21. 2(2t + 5)(4t − 3)
7. x(6x − 5)(x + 1)
                                          23. prime
9. 3x 2 (x 2 + 2)
                                          25. 4(w + 5)(w + 1)(w − 1)
11. w(4z + 5)(z + 2)

13. y 4 (x − 4)(x + 8)                    27. (2x − y)2

15. x(x 2 − 6)(x + 6)                     29. (x + 20 y)(x − 18 y)




                                    347
7. SOLVING EQUATIONS BY FACTORING                    CHAPTER 6. FACTORING POLYNOMIALS
                             7. Solving Equations by Factoring
   7.1. Zero Product Property.


  THEOREM 7.1.1. If a product of several numbers is zero, then one of these numbers must
  be zero. Conversely, if one of the factors in a product is zero, then the value of the product
  is zero.

  Formally, for all real numbers a and b, ab = 0 if and only if a = 0 or b = 0. This statement
  generalizes naturally to products with any number of factors.



 THEOREM 7.1.2 (Zero Product in Equations). If P and Q are polynomials, then the solution
 set of the equation PQ = 0 consists of all solutions of P = 0 together with all solutions of
 Q = 0.



  EXAMPLE 7.1.1. Solve the equation
                                        x 2 − 3x + 2 = 0



  SOLUTION: Factoring the expression on the left yields
                                       (x − 1)(x − 2) = 0
  Here (x − 1) and (x − 2) are the polynomial factors P and Q respectively in the theorem
  7.1.2.

  Solving the equation x − 1 = 0 gives us x = 1.

  Solving the equation x − 2 = 0 gives us x = 2.

  So the solutions to the quadratic equation are 1 and 2.

  Let’s check the answers. If x = 1, then
                                x 2 − 3x + 2 = 12 − 3 · 1 + 2 = 0
  If x = 2, then
                                x 2 − 3x + 2 = 22 − 3 · 2 + 2 = 0

  Multiple solutions are common for quadratic equations, so we will state answers using the
  roster notation.



                                        ANSWER: {1, 2}

                                              348
CHAPTER 6. FACTORING POLYNOMIALS                           7. SOLVING EQUATIONS BY FACTORING
    7.2. Equation Solving Strategy. We are already familiar with the procedure for solving
linear polynomial equations, and now we can tackle all the rest of them. Faced with a poly-
nomial equation, we will first try to determine whether it is linear or not. If it is linear, then
we will solve it by isolating the variable. If it is a quadratic or a higher degree equation, then
we will arrange it so that all the non-zero terms are on the same side, factor the polynomial
completely, and take advantage of the zero product property.

  EXAMPLE 7.2.1. Solve the equation
                                            2x 2 + x = 1


  SOLUTION: This equation appears to be quadratic, so we will add −1 to both sides in
  order to make the right side zero, then factor by guessing the coefficients.
                                                2x 2 + x    =    1
                                          2x + x − 1
                                            2
                                                            =    0
                                     (2x − 1)(x + 1)        =    0
  The left side is a product, and the right side is zero, so we can apply the zero product
  property. Solving the equation 2x − 1 = 0 gives us
                                          2x − 1       =   0
                                             2x        =   1
                                                 x     =   0.5
  Solving the equation x + 1 = 0 gives us x = −1.


                                      ANSWER: {0.5, −1}


  EXAMPLE 7.2.2. Solve the equation
                                           8x 2 − 18 = 0


  SOLUTION: All the non-zero terms of this quadratic equation are already on the left side,
  so let’s try factoring it completely. Instead of factoring out 2 we opt to multiply both sides
  by 0.5, canceling it altogether.
                     8x 2 − 18   =    0
                       2
                0.5(8x − 18)     =    0.5(0)
                           2
                      4x − 9     =    0                    this is a difference of squares
                   (2x)2 − 32    =    0
             (2x + 3)(2x − 3)    =    0


                                                     349
7. SOLVING EQUATIONS BY FACTORING                    CHAPTER 6. FACTORING POLYNOMIALS
  The left side is a product, and the right side is zero, so we can apply the zero product
  property. Solving the equation 2x + 3 = 0 gives us x = −1.5.

  Solving the equation 2x − 3 = 0 gives us x = 1.5.


                                   ANSWER: {1.5, −1.5}


  EXAMPLE 7.2.3. Solve the equation
                                        14x 2 = −16x


  SOLUTION: We will add 16x to both sides to make the right side zero, and try to factor.
  Notice that we opt to divide both sides of the equation by the common factor 2 rather than
  to factor it out, because dividing both sides by a non-zero number makes an equivalent
  equation.
                       14x 2   =    −16x
                14x 2 + 16x    =    0                     divide both sides by 2

                14x 2 + 16x         0
                               =
                     2              2
                14x 2 16x
                     +         =    0                              distributivity
                 2     2
                   7x 2 + 8x   =    0                              the GCF is x
                  x(7x + 8)    =    0                              distributivity

  Notice we are not allowed to divide both sides of the equation by x. Unlike the non-zero
  quantity 2, x is an unknown quantity which may turn out to be zero.

  The expression x(7x + 8) has two polynomial factors: x and 7x + 8. By the zero product
  property x = 0 is a solution, and solving 7x + 8 = 0 gives us
                                     7x + 8     =    0
                                         7x     =    −8
                                           x    =    −8/7


                                    ANSWER: {0, −8/7}




                                               350
CHAPTER 6. FACTORING POLYNOMIALS                               7. SOLVING EQUATIONS BY FACTORING

 EXAMPLE 7.2.4. Solve the equation
                                           4 y 3 = 7 y 2 + 15 y


 SOLUTION: This looks like a cubic equation, so we will subtract (7 y 2 +15 y) on both sides,
 and then factor completely.
                        4 y3      =    7 y 2 + 15 y
         4 y 3 − 7 y 2 − 15 y     =    0                                           the GCF is y
         y(4 y 2 − 7 y − 15)      =    0                         factor by guessing coefficients
         y(4 y + 5)( y − 3)       =    0

 By the zero product property, y = 0 is a solution;

 solving 4 y + 5 = 0 yields y = −1.25;

 and solving y − 3 = 0 yields y = 3.

 As always the case with any equation, we can check the answers by substituting them and
 checking that the equation holds.

 If y = 0 then
                                      4 · 03       =     y · 02 + 15 · 0
                                           0       =     0
 If y = 3 then
                                      4 · 33       =     7 · 32 + 15 · 3
                                       108         =     63 + 45
                                       108         =     108
 Finally, if y = −1.25 then
                           4(−1.25)3           =       7(−1.25)2 + 15(−1.25)
                                −7.8125        =       10.9375 − 18.75
                                −7.8125        =       −7.8125




                                      ANSWER: {−1.25, 0, 3}




                                                       351
7. SOLVING EQUATIONS BY FACTORING                  CHAPTER 6. FACTORING POLYNOMIALS
   Homework 6.7.

Solve the given equation by using the zero         15. y 2 + 4 y − 21 = 0
product property.
                                                   16. y 2 = 7 y + 18
1. (x + 3)(x − 10) = 0
                                                   17. x 2 − 11x + 18 = 0
2. (x − 2)(x + 9) = 0
                                                   18. x 2 − 8x + 15 = 0
3. (x − 1)(x − 8) = 0
                                                   19. y 2 − 6.5 y = 0
4. (x + 4)(x + 3) = 0
                                                   20. 4z 2 − 12z = 0
5. 3x(x + 4) = 0
                                                         2 2 1
                                                   21.     x + x =0
6. 2 y( y − 6) = 0                                       3    3

7. (2x + 3)(4x − 5) = 0                            22. 3z 2 + 4z = 0

8. (5x − 10)(6x + 10) = 0                          23. 2x 2 = 72
      
           1
             ‹                                     24. 6 y 2 − 54 = 0
9. x x +
    2
               =0
           2
                                                   25. 10x + x 2 + 25 = 0
              2
               ‹
       3
10. 4 y y −       =0                               26. 6x + 9 + x 2 = 0
              3

11. x(4x + 0.8)(x − 0.1)2 = 0                      27. (x + 1)(x − 7) = −16

12. y 2 (3 y − 6)( y + 1.7) = 0                    28. (x + 2)(7 − x) = 18

                                                   29. (3x + 5)(x + 3) = 7

Solve the given equation by factoring.             30. (5x + 4)(x − 1) = 2

13. x 2 + 7x + 6 = 0                               31. 81x 2 − 6 = 19

14. x 2 + 6x + 5 = 0                               32. 14x 2 − 3 = 42x − 3




                                             352
 CHAPTER 6. FACTORING POLYNOMIALS          7. SOLVING EQUATIONS BY FACTORING
    Homework 6.7 Answers.

1. {−3, 10}                               17. {2, 9}

3. {1, 8}                                 19. {0, 6.5}

                                               1
                                             §      ª
5. {−4, 0}                                21. − , 0
                                               2
    3 5
  §     ª
7. − ,                                    23. {−6, 6}
    2 4
                                          25. {−5}
    1
  §      ª
9. − , 0
    2                                     27. {3}
11. {−0.2, 0, 0.1}                           §
                                                    2
                                                      ª
                                          29. −4, −
                                                    3
13. {−6, −1}
                                               5 5
                                             §      ª
                                          31. − ,
15. {−7, 3}                                    9 9




                                    353
8. FACTORING APPLICATIONS                              CHAPTER 6. FACTORING POLYNOMIALS
                                  8. Factoring Applications

    8.1. Applications to Areas. Recall that the area of a rectangle with width w and length l
is the product of its dimensions.




                     A= w·l                 h




                                                               w

Recall also that the area of a triangle with base b and height h is one half of their product.




                           bh                                      h
                      A=
                           2



                                                               b

  EXAMPLE 8.1.1. The length of a rectangular carpet is 1 foot greater than the width. Find
  the dimensions of the carpet if its surface area is 12 square feet.


  SOLUTION: Let w and l be the width and the length in feet, respectively. We can translate
  “the length is 1 foot greater than the width” as
                                            l = w+1
  We can also write the equation for the area:
                                             wl = 12
  This is not a linear system, but we can solve it by substitution. The first equation is already
  solved for l, so we substitute (w + 1) for l in the area equation and solve for w:
                                                wl    =   12
                                       w(w + 1)       =   12
                                         w2 + w       =   12

                                                354
CHAPTER 6. FACTORING POLYNOMIALS                                 8. FACTORING APPLICATIONS
 This is a quadratic equation, which we will solve by factoring.
                    w2 + w     =   12                put non-zero terms on one side
               w2 + w − 12     =   0                  trinomial pattern x 2 + bx + c
            (w − 3)(w + 4)     =   0
 By the zero product property, the solutions are w = 3 and w = −4. The width cannot be
 negative, so the solution −4 is not applicable. The only useful solution is 3. To find the
 length, we substitute 3 for w in the equation solved for l:
                                        l    =   w+1
                                        l    =   (3) + 1
                                        l    =   4


                                       ANSWER:
                     w and l are width and length respectively, in feet
                                         l = w+1
                                      

                                         wl = 12
                                   solution: w = 3,      l =4


 EXAMPLE 8.1.2. The height of a triangular sail is 1 meter less than the twice the length of
 its base. Find the dimensions of the sail if its surface area is 14 square meters.


 SOLUTION: Let b and h be the base and the height of the sail in meters. The sentence
 “the height of a triangular sail is 1 meter less than the twice the length of its base” can be
 translated as
                                           h = 2b − 1

 We can also write the equation for the area:
                                            bh/2 = 14

 The first equation is already solved for h, so we can substitute (2b − 1) for h in the area
 equation and solve for b:
                        bh
                             = 14                              area equation
                        2
                b(2b − 1)
                             =     14                               substitution
                   2
                b(2b − 1)    =     28                 multiplied both sides by 2



                                              355
8. FACTORING APPLICATIONS                                      CHAPTER 6. FACTORING POLYNOMIALS
  With the last step, we multiplied both sides by 2 to get rid of the fraction. The result looks
  like a quadratic equation, so we solve it by factoring:
                        2b2 − b    =   28                              distributed on the left side
                    2
               2b − b − 28         =   0                      trinomial pattern ax 2 + bx + c
             (2b + 7)(b − 4)       =   0                     factored by guessing coefficients

  By the zero product property, either
                                           2b + 7        =     0
                                                2b       =     −7
                                                 b       =     −7/2
  or
                                                b−4       =        0
                                                     b    =        4
  The negative solution b = −7/2 is not applicable because it cannot denote a length, so the
  only useful solution is b = 4. To find the height, substitute 4 for b in the equation solved
  for h:
                                            h    =       2b − 1
                                            h    =       2(4) − 1
                                            h    =       7


                                           ANSWER:
                        b and h are base and height respectively, in meters
                                            h = 2b − 1
                                          

                                            bh/2 = 14
                                       solution: b = 4,            h=7


  EXAMPLE 8.1.3. The height of a triangular tile is 40% greater than its base. Find the
  dimensions of the tile if its area is 17.5 square inches.


  SOLUTION: Let b and h be the base and the height of the triangle in inches. The sentence
  “the height of a triangular tile is 40% greater than its base” is a statement about percent
  increase, and can be translated as
                h       =   b + 0.4b                     see definition 4.2.1 in chapter 2
                h       =   1.4b                                          combined like terms



                                                     356
CHAPTER 6. FACTORING POLYNOMIALS                                     8. FACTORING APPLICATIONS
 And the statement about the area can be translated as
                                        bh
                                              = 17.5
                                         2
 Using the first equation, which is solved for h, we can substitute 1.4b for h into the second
 equation, and solve for b:
                        bh
                             = 17.5
                        2
                 b(1.4b)
                               =       17.5                  substituted 1.4b for h
                    2

 To get rid of the fraction, we multiply both sides by 2:
                 b(1.4b)       =   35
                   1.4b2       =   35                              simplify left side
                       b   2
                               =   25                      divide both sides by 1.4

 This looks like a quadratic equation, so we solve it by factoring:
                     b2 − 25       =     0                subtracted 25 on both sides
              (b + 5)(b − 5)       =     0                difference of squares b2 − 52

 By the zero product property, the solutions are −5 and 5. Negative length does not make
 sense, so the negative solution is not applicable, and the only useful solution is b = 5. We
 can find the other dimension using the equation solved for h:
                                              h   =   1.4b
                                              h   =   1.4 · 5
                                              h   =   7


                                        ANSWER:
                     b and h are base and height respectively, in inches
                                        h = b + 0.4b
                                     

                                        bh/2 = 17.5
                                       solution: b = 5,      h=7




                                                  357
8. FACTORING APPLICATIONS                             CHAPTER 6. FACTORING POLYNOMIALS
   Homework 6.8.

1. The width of a rectangular rug is 4 times          5. The width of a rectangular laptop
less than its length. Find the dimensions of          screen is 7 inches less than its length. Find
the rug if its area is 36 square feet.                the dimensions of the screen if its area is
                                                      144 square inches.
2. The length of a rectangular room is 6
yards greater than its width. Find the di-            6. The width of a rectangular mural is 10%
mensions of the room if its area is 55 square         greater than its length. Find the dimen-
yards.                                                sions of the mural if its area is 110 square
                                                      meters.
3. The base of a triangle is 2 meters longer
than twice its height. Find the base and              7. Dmitriy buys a gold earring shaped like
the height if the area of the triangle is 12          a thin flat triangle. Find the dimensions of
square meters.                                        the earring if the base of the triangle is 25%
                                                      shorter than its height, and the area of the
4. The base of a triangle is 2 units longer           triangle is 24 mm2 .
than the height. Find the base and the
height if the area of the triangle is 12              8. The height of a triangular piece of fab-
square units.                                         ric is 3 times greater than its base. Find the
                                                      dimensions of the triangle if its area is 13.5
                                                      square inches.




                                                358
 CHAPTER 6. FACTORING POLYNOMIALS                                 8. FACTORING APPLICATIONS
    Homework 6.8 Answers.

1.                                                    5.
 w and l are width and length respectively,            w and l are width and length respectively,
 in feet                                               in inches
    w = l/4                                               w= l −7
                                                      

    wl = 36                                               wl = 144
 solution: w = 3,   l = 12                             solution: w = 9,   l = 16

3.                                                    7.
 b and h are base and height respectively, in          b and h are base and height respectively, in
 meters                                                mm
    b = 2h + 2                                            b = h − 0.25h
                                                      

    bh/2 = 12                                             bh/2 = 24
 solution: b = 8,   h=3                                solution: b = 6,   h=8




                                                359
8. PRACTICE TEST 6                                  CHAPTER 6. FACTORING POLYNOMIALS
                                       Practice Test 6




Factor each of the following expressions            Solve each of the following equations using
completely:                                         factoring:

1. 48x 7 y 3 + 42x 4 y 4                            11. x 2 + 100 = 20x

2. 6x 4 − 42x 3 − 6x 2                              12. 25x 2 − 9 = 0

3. x 2 − 10x + 24                                   13. 3x 2 − 3x = 36

4. 18x 2 + 63x 2 a − 24x − 84x a                    14. 5x 3 = 45x

5. 5x 2 − 17x + 6                                   15. 18x 2 − 3x = 6

6. 4 y 2 + 10 y − 6                                 16. x(2x − 1) = 3

7. 1 − 16x 4                                        17. x 3 − 9x 2 + 2x − 18 = 0

                                                    18. The length of a rectangular carpet is 5
Solve each of the following equations using         times greater than the width, and the area
the zero product property:                          of the carpet is 45 square feet. Find the di-
                                                    mensions of the carpet.
8. x(x + 4) = 0
                                                    19. A piece of stained glass is shaped like a
9. 10(x − 6)(3x + 1) = 0                            triangle, with its height 4 cm shorter than
                                                    its base. Find the base and the height of
10. −7x 2 (x 2 + 1)(x − 1) = 0                      the triangle if its area is 48 cm2 .




                                              360
 CHAPTER 6. FACTORING POLYNOMIALS                               8. PRACTICE TEST 6
     Practice Test 6 Answers.

1. 6x 4 y 3 (8x 3 + 7 y)                  14. {0, 3, −3}

2. 6x 2 (x 2 − 7x − 1)                    15. {2/3, −1/2}
3. (x − 4)(x − 6)
                                          16. {3/2, −1}
4. 3x(3x − 4)(2 + 7a)
                                          17. {9}
5. (5x − 2)(x − 3)
                                          18.
6. 2(2 y − 1)( y + 3)                      l and w are length and width respectively,
                                           in feet
7. (1 + 4x 2 )(1 + 2x)(1 − 2x)
                                              lw = 45
                                           

8. {0, −4}                                    l = 5w
                                           solution: l = 15,   w=3
9. {6, −1/3}

10. {0, 1}                                19.
                                           b and h are base and height respectively, in
11. {10}                                   cm
                                              bh/2 = 48
                                           
12. {−3/5, 3/5}
                                              h= b−4
13. {4, −3}                                solution: b = 12,   h=8




                                    361
                                       CHAPTER 7


                                 Rational Expressions


                           1. Simplifying Rational Expressions



                                           4
                                               y
                                           3

                                           2                 1
                                                        y=
                                                             x
                                           1
                                                                     x
                      −4    −3    −2   −1           1    2       3       4
                                         −1

                                         −2

                                         −3

                                         −4


 1.1. Definition.

DEFINITION 1.1.1. A rational expression is either a polynomial expression or a quotient of
a polynomial and a non-zero polynomial.


BASIC EXAMPLE 1.1.1. Here are some rational expressions:

                    5− x          1        1 + x2            x 2 + 2x + 1     x + yw
      x2 − x
                     4            x         x −2                 x +1         x y − w5

On the other hand
                                              1 1
                                                +
                                              x x
is not a rational expression, since it is neither a polynomial nor a quotient of polynomials.

                                            363
1. SIMPLIFYING RATIONAL EXPRESSIONS                     CHAPTER 7. RATIONAL EXPRESSIONS
   1.2. Variable Restrictions. Unlike polynomials, which can always be evaluated, rational
expressions will occasionally fail to have a defined value. As the most basic example, the
expression 1/x can be evaluated for every real number x except for x = 0. Because of that,
the process of simplifying rational expressions may well produce expressions which are not
equivalent, but almost equivalent, with only a finite number of disagreements.

  BASIC EXAMPLE 1.2.1. Expressions
                             (x + 2)(x − 1)
                                                 and       x +2
                                  x −1
  are not equivalent even though they agree on the value for almost all real numbers x. But
  when x = 1,
                                      x +2=1+2=3
  whereas
                           (x + 2)(x − 1) (1 + 2)(1 − 1) 3 · 0
                                          =                =
                                x −1            1−1            0
  which is undefined.


In this text we will simplify rational expressions by canceling common polynomial factors, even
though in many cases it will result in non-equivalent expressions. Our excuse will be that we
can find exactly which inputs lead to disagreements.

  EXAMPLE 1.2.1. Find all variable values which make the given expression undefined:
                                           x +1
                                           x −5


  SOLUTION: A rational expression is a quotient of two polynomials. The numerator is
  always defined, and the denominator is always defined. The only operation which may
  possibly fail is the division: the fraction bar. It will fail just in case if the denominator is
  equal to zero. So we can make the denominator equal to zero, and then solve for x. We
  ignore the numerator completely.
                                          x −5     =    0
                                              x    =    5
  The only number which makes this expression undefined is x = 5.


                                          ANSWER: {5}


  EXAMPLE 1.2.2. Find all variable values which make the given expression undefined:
                                           x +4
                                          x 2 − 16

                                               364
CHAPTER 7. RATIONAL EXPRESSIONS                  1. SIMPLIFYING RATIONAL EXPRESSIONS

 SOLUTION: We can make the denominator equal to zero, and then solve for x. We ignore
 the numerator completely, even though it happens to have the factor (x + 4), which can
 also be found in the denominator.
                                         x 2 − 16     =   0
                                  (x + 4)(x − 4)      =   0
 By the zero product property, x is either −4 or 4.


                                    ANSWER: {−4, 4}


 EXAMPLE 1.2.3. Find all variable values which make the given expression undefined:
                                          1 1
                                            +
                                          x 7
                                          x 2
                                            −
                                          6 9


 SOLUTION: There are two variable denominators in this expression, and if either one
 happens to be 0, then the whole expression is undefined. One variable denominator is
 x from 1/x up on top, so x = 0 will make the expression undefined. The other variable
 denominator is
                                           x 2
                                             −
                                           6 9
 so we make it equal to zero and solve for x:
                                      x 2
                                        −      = 0
                                      6 9
                                           x          2
                                               =
                                           6          9
                                                      4
                                           x   =
                                                      3


                                    ANSWER: {0, 4/3}




                                           365
1. SIMPLIFYING RATIONAL EXPRESSIONS                       CHAPTER 7. RATIONAL EXPRESSIONS
    1.3. Canceling Common Polynomial Factors. We can simplify rational expressions by
fully factoring both numerator and denominator, and then canceling common polynomial fac-
tors. In answers, we will write simplified rational expressions as quotients of polynomial fac-
torizations. This way we can be sure nothing else can be canceled.

  EXAMPLE 1.3.1. Simplify and state the answer as a polynomial:
                                             a2 − 1
                                             a+1


  SOLUTION:
                                 a2 − 1             (a + 1)(a − 1)
                                            =
                                 a+1                   (a + 1)
                                            =       a−1


                                       ANSWER: a − 1


  EXAMPLE 1.3.2. Simplify and state the answer as a quotient of polynomial factorizations:
                                          6x − 3
                                       4x − 4x + 1
                                          2




  SOLUTION:
                               6x − 3                    3(2x − 1)
                                                =
                            4x 2 − 4x + 1             (2x − 1)(2x − 1)
                                                        3
                                                =
                                                      2x − 1


                                                        3
                                      ANSWER:
                                                      2x − 1


  EXAMPLE 1.3.3. Simplify and state the answer as a quotient of polynomial factorizations:
                                            4a2 − 5ab
                                            30b − 24a


  SOLUTION:
                                4a2 − 5ab              a(4a − 5b)
                                                =
                                30b − 24a              6(5b − 4a)

                                                366
CHAPTER 7. RATIONAL EXPRESSIONS                       1. SIMPLIFYING RATIONAL EXPRESSIONS
 The factors (4a − 5b) and (5b − 4a) are not identical, so they cannot be canceled outright.
 But they are opposites of each other, so the quotient is equivalent to −1 whenever defined.
 Algebraically, we can make them look exactly the same if we factor out −1.
     a(4a − 5b)          a(−1)(−4a + 5b)
                    =
     6(5b − 4a)             6(5b − 4a)
                         a(−1)(5b − 4a)
                    =                             now they are identical and can be canceled
                           6(5b − 4a)
                         −a
                    =
                         6


                                                       −a
                                         ANSWER:
                                                       6


 EXAMPLE 1.3.4. Simplify and state the answer as a quotient of polynomial factorizations:
                                        x 3 + 2x 2 + x + 2
                                              x4 − 1


 SOLUTION: The numerator has 4 terms, so we will attempt grouping; the denominator
 is a difference of squares. We factor polynomials completely before rewriting the rational
 expression.
                         x 3 + 2x 2 + x + 2   =      x 2 (x + 2) + 1(x + 2)
                                              =      (x 2 + 1)(x + 2)
  x 2 + 1 is an irreducible quadratic, and x + 2 is linear, so the numerator is fully factored.
                                                                                 2
           x 4 − 1 = (x 2 + 1)(x 2 − 1)                 difference of squares x 2 − 12
                   =    (x 2 + 1)(x + 1)(x − 1)              difference of squares x 2 − 12
 x 2 + 1 is an irreducible quadratic, and the other two factors are linear, so the denominator
 is fully factored. Now we can rewrite the rational expression in a fully factored form and
 cancel common polynomial factors:
                        x 3 + 2x 2 + x + 2               (x 2 + 1)(x + 2)
                                              =
                              x4 − 1                (x 2 + 1)(x + 1)(x − 1)
                                                       (x + 2)
                                              =
                                                    (x + 1)(x − 1)


                                                  x +2
                                   ANSWER:
                                              (x + 1)(x − 1)

                                              367
1. SIMPLIFYING RATIONAL EXPRESSIONS                   CHAPTER 7. RATIONAL EXPRESSIONS
      Homework 7.1.

Find all variable values which make the                   21k
                                                    14.
given expression undefined.                               24k3

   3k2 + 30k                                            32x 3 y
1.                                                  15.
    k + 10                                               8x y 2

        15n2                                               90x 2
2.                                                  16.
      10n + 25                                            20x 2 y
         27p                                              18m − 24
3.                                                  17.
      18p2 − 36p                                            60
        x + 10                                             20
4.                                                  18.
      8x 2 + 80x                                          4p + 2
      10m2 + 8m                                               x +1
5.                                                  19.
        10m                                               x 2 + 8x + 7
      10x + 16                                              32x 2
6.                                                  20.
       6x + 20                                            28x 2 + 28
      b2 + 12b + 32                                       n2 + 4n − 12
7.                                                  21.
       b2 − 4b − 32                                       n2 − 7n + 10
      10 y 2 + 30 y                                       b2 + 14b + 48
8.                                                  22.
       35 y 2 − 5 y                                       b2 + 15b + 56
      w2 − 1                                                   9v + 54
9.                                                  23.
      w2 + 1                                              v2   − 4v − 60

       16x 2 + 25                                         30x − 90
10.                                                 24.
       16x 2 − 25                                         50x + 40

                                                          12x 2 − 42x
                                                    25.
                                                          30x 2 − 42x
Simplify the given expression and state the
answer as a quotient of polynomial factor-                6a − 10a2
izations.                                           26.
                                                          10a + 4a2
    21x 2                                                 k2 − 12k + 32
11.                                                 27.
    18x                                                      k2 − 64
       12n
12.                                                        9p + 18
       4n2                                          28.
                                                          p2 + 4p + 4
       24a
13.
       40a2
                                              368
CHAPTER 7. RATIONAL EXPRESSIONS         1. SIMPLIFYING RATIONAL EXPRESSIONS
    9n2 + 89n − 10                           n2 − 2n + 1
29.                                      33.
       9n + 90                                 6n + 6

    3x 2 − 29x + 40                            7a2 − 26a − 45
30.                                      34.
    5x 2 − 30x − 80                            6a2 − 34a + 20

      2x 2 − 10x + 8                              56x − 48
31.                                      35.
      3x 2 − 7x + 4                            24x 2 + 56x + 32

      7n2 − 32n + 16                           4k3 − 2k2 − 2k
32.                                      36.
         4n − 16                               9k3 − 18k2 + 9k




                                  369
 1. SIMPLIFYING RATIONAL EXPRESSIONS               CHAPTER 7. RATIONAL EXPRESSIONS
      Homework 7.1 Answers.

1. {−10}                                           n+6
                                             21.
                                                   n−5
3. {0, 2}
                                                       9
                                             23.
5. {0}                                              v − 10

7. {−4, 8}                                         2x − 7
                                             25.
                                                   5x − 7
9. ∅
                                                   k−4
    7x                                       27.
11.                                                k+8
     6
                                                   9n − 1
    3                                        29.
13.                                                  9
    5a
                                                   2(x − 4)
    4x 2                                     31.
15.                                                 3x − 4
     y
                                                   (n − 1)2
    3m − 4                                   33.
17.                                                6(n + 1)
     10
        1                                              7x − 6
19.                                          35.
      x +7                                         (3x + 4)(x + 1)




                                       370
CHAPTER 7. RATIONAL EXPRESSIONS                         2. PRODUCTS OF RATIONAL EXPRESSIONS
                           2. Products of Rational Expressions

    2.1. Multiplying Rational Expressions. We can multiply rational expressions just like we
would multiply fractions: the numerator of the product is the product of numerators, and
the denominator of the product is the product of denominators. Rather than to carry out the
multiplication of polynomials, we will factor them, so that we can cancel common polynomial
factors and leave the answer in a simplified form.

  EXAMPLE 2.1.1. Simplify and state the answer as a quotient of polynomial factorizations:
                                               25x 2 24 y 4
                                                     ·
                                               9 y 8 55x 7


  SOLUTION: We begin by multiplying across and then we cancel common integer and
  variable factors.
        25x 2 24 y 4   25 · 24 · x 2 y 4
               ·     =
         9 y 8 55x 7   9 · 55 · y 8 x 7
                             (5 · 5)(2 · 2 · 2 · 3)
                       =                                       cancel variables, factor integers
                            (3 · 3)(5 · 11) · x 5 y 4
                             5·2·2·2
                       =                                      canceled common prime factors
                            3 · 11 · x 5 y 4
                              40
                       =
                            33x 5 y 4


                                                          40
                                        ANSWER:
                                                        33x 5 y 4


  EXAMPLE 2.1.2. Simplify and state the answer as a quotient of polynomial factorizations:
                                        x2 − 4     x +1
                                               · 2
                                        x + 3 x + 3x + 2


  SOLUTION: We can multiply across, then factor all the polynomials, and cancel the com-
  mon polynomial factors. Note that the invisible parentheses of the fraction notation be-
  come visible once we rewrite the expression as a single fraction.
                      x2 − 4     x +1                        (x 2 − 4)(x + 1)
                             · 2                    =
                      x + 3 x + 3x + 2                    (x + 3)(x 2 + 3x + 2)



                                                   371
2. PRODUCTS OF RATIONAL EXPRESSIONS                     CHAPTER 7. RATIONAL EXPRESSIONS
 The difference of squares x 2 − 4 factors as
                                   x 2 − 4 = (x + 2)(x − 2)
 while the trinomial x 2 + 3x + 2 factors as
                                x 2 + 3x + 2 = (x + 1)(x + 2)
 Now we can rewrite the original fraction in a fully factored form, and cancel common
 polynomial factors.
     (x 2 − 4)(x + 1)         (x + 2)(x − 2)(x + 1)
                          =
  (x + 3)(x 2 + 3x + 2)       (x + 3)(x + 1)(x + 2)
                              (x − 2)
                          =                              common polynomial factors canceled
                              (x + 3)


                                                      x −2
                                        ANSWER:
                                                      x +3



  2.2. Dividing Rational Expressions.

 EXAMPLE 2.2.1. Simplify and state the answer as a quotient of polynomial factorizations:
                                        14 21
                                           ÷
                                        25 10


 SOLUTION: Recall that division amounts to multiplying by the reciprocal of the divisor:
                                14 21 14 10 14 · 10
                                   ÷     =    ·      =
                                25 10 25 21 25 · 21
 Instead of finding these large products, we opt to factor the integers completely so that it
 is easier to cancel all the common factors.
                                 14 · 10        (2 · 7)(2 · 5)
                                           =
                                 25 · 21        (5 · 5)(3 · 7)
                                                  2·2
                                           =
                                                  5·3
                                                   4
                                           =
                                                  15


                                        ANSWER: 4/15


                                                372
CHAPTER 7. RATIONAL EXPRESSIONS                   2. PRODUCTS OF RATIONAL EXPRESSIONS

 EXAMPLE 2.2.2. Simplify and state the answer as a polynomial or a quotient of polynomial
 factorizations:
                                  x 2 + 8x + 16 x + 4
                                               ÷
                                         x          x2


 SOLUTION: First we rewrite the division as a multiplication by the reciprocal, and then
 multiply fractions as usual:
   x 2 + 8x + 16 x + 4           x 2 + 8x + 16     x2
                ÷           =                  ·                         when we multiply sums
          x        x2                   x        x +4
                                 (x 2 + 8x + 16)x 2
                            =                              fraction parentheses become visible
                                      x(x + 4)
 Now we can factor the polynomials completely and cancel all common factors. x 2 +8x +16
 is a square of a binomial, and factors as (x + 4)2 , so we can write
      (x 2 + 8x + 16)x 2         (x + 4)2 x 2
                            =
           x(x + 4)               x(x + 4)
                            =    (x + 4)x             common factors x and (x + 4) cancel
 For the sake of consistency, we will state polynomial answers in the standard form.


                                      ANSWER: x 2 + 4x


 EXAMPLE 2.2.3. Simplify and state the answer as a polynomial or a quotient of polynomial
 factorizations:
                                    3x 2 − 75 5 − x
                                             ÷
                                      5+ x        12


 SOLUTION: First we rewrite the division as a multiplication by the reciprocal, and then
 multiply fractions as usual:
                           3x 2 − 75 5 − x              3x 2 − 75 12
                                    ÷            =               ·
                            5+ x      12                 5+ x      5− x
                                                         (3x 2 − 75)12
                                                 =
                                                        (5 + x)(5 − x)
 3x 2 − 75 has GCF 3 and then factors as a difference of squares:
                                3x 2 − 75   =     3(x 2 − 25)
                                            =     3(x 2 − 52 )
                                            =     3(x + 5)(x − 5)

                                                373
2. PRODUCTS OF RATIONAL EXPRESSIONS                   CHAPTER 7. RATIONAL EXPRESSIONS
 So in a fully factored form, the rational expression looks like this:
       (3x 2 − 75)12          3(x + 5)(x − 5)12
                         =
      (5 + x)(5 − x)            (5 + x)(5 − x)
                             36(x − 5)
                         =                           common factor (x + 5) canceled
                               5− x
 Note that we canceled x + 5 with 5 + x because they are equivalent, but we can not cancel
 x −5 with 5− x as easily because they are opposites, and dividing a number by its opposite
 results in −1.
           36(x − 5)          36(x − 5)
                      =
             5− x           (−1)(−5 + x)
                              36(x − 5)
                       =
                             (−1)(x − 5)
                             36
                       =                           common factor (x − 5) canceled
                             −1
                       =     −36


                                       ANSWER: −36




                                             374
CHAPTER 7. RATIONAL EXPRESSIONS                 2. PRODUCTS OF RATIONAL EXPRESSIONS
      Homework 7.2.

Simplify and state the answer as a polyno-                    8k        1
                                                    15.             ÷
mial or a quotient of polynomial factoriza-               24k2 − 40k 15k − 25
tions.
                                                              p−8         1
        2                                           16.                ÷
      8x 9                                                p2 − 12p + 32 p − 10
1.       ·
       9 2
                                                                       6
   20n 7                                            17. (n − 8) ·
2.    ·                                                             10n − 80
    3n 5n
                                                           3x − 6
   5x 2 5                                           18.            · (x + 3)
3.     ÷                                                  12x − 24
    4    6
                                                        x 2 − 7x + 10 x 2 − x − 20
      10p   8                                       19.              ÷
4.        ÷                                                  x −2         x + 10
       5    10
                                                        21v 2 + 16v − 16 35v − 20
      7(m − 6) 5m(7m − 5)                           20.                 ÷
5.            ·                                              3v + 4        v−9
       m−6      7(7m − 5)
                                                            b+2
          7r       (r − 6)
                         2                          21.              · (5b − 3)
6.               ·                                        40b2 − 24b
      7r(r + 10)     r −6
                                                          2n2 − 12n − 54     1
         7          n−2                             22.                  ·
7.             ÷                                               n+7         2n + 6
      10(n + 3) (n + 3)(n − 2)
                                                          x 2 + 11x + 24 x 2 + 5x − 24
                                                    23.                  ÷
      6x(x + 4)     6x(x − 6)                               6x 3 + 18x 2   6x 3 + 6x 2
8.              ÷
        x −3      (x − 3)(x − 6)
                                                           n−7      n2 − 13n + 42
                                                    24.           ÷
      25n + 25      4                                     6n − 12      12 − 6n
9.             ·
         5       30n + 30
                                                             k−7       7k2 − 28k
       v−1       4                                  25.              ·
10.        · 2                                            k2 − k − 12 8k2 − 56k
        4   v − 11v + 10
                                                          x 2 − 12x + 32 7x 2 + 14x
              9         b−5                         26.                 ·
11.                 ÷ 2                                    x 2 − 6x − 16 7x 2 + 21x
       b2   − b − 12 b − b − 12
                                                              n−7       9n + 54
     x − 10     7                                   27.               ÷
12.         ÷                                             n2 − 2n − 35 10n + 50
    35x + 21 35x + 21
                                                          27a + 36 6a + 8
    x 2 − 6x − 7 x + 5                              28.            ÷
13.             ·                                          9a + 63   2
        x +5      x −7
                                                          x2 − 1    x2 − 4   x2 + x − 2
     1    8a + 80                                   29.          · 2       ÷
14.     ·                                                 2x − 4 x − x − 2    3x − 6
    a−6      8
                                                        x 2 + 3x − 10 8x + 20 2x 2 − x − 3
                                                    30.               ÷        ·
                                                         x 2 + 6x + 5   6x + 15 2x 2 + x − 6
                                              375
 2. PRODUCTS OF RATIONAL EXPRESSIONS           CHAPTER 7. RATIONAL EXPRESSIONS
         Homework 7.2 Answers.

1. 4x 2                                        3
                                         17.
                                               5
     3x 2
3.                                              x + 10
      2                                  19.
                                                 x +4
5. 5m
                                                b+2
                                         21.
      7                                          8
7.
     10                                         x +1
                                         23.
     2                                          x −3
9.
     3                                            7
                                         25.
                                               8(k + 3)
          9
11.
         b−5                                     10
                                         27.
                                               9(n + 6)
13. x + 1
                                               3
15. 5                                    29.
                                               2




                                   376
CHAPTER 7. RATIONAL EXPRESSIONS                        3. LCD FOR RATIONAL EXPRESSIONS
                             3. LCD for Rational Expressions
  3.1. LCM for Polynomials.

 DEFINITION 3.1.1. A common multiple for a collection of polynomials P, Q, R, . . . with
 integer coefficients is a polynomial which is divisible by P, Q, R, . . . without a remainder,
 using the polynomial long division.

 The least common multiple (or LCM) for a collection of polynomials with integer coeffi-
 cients is a common multiple of the least possible polynomial degree and with coefficients
 as low as possible.

 The lowest common denominator (or LCD) for a collection of rational expressions is the
 LCM of their denominators.


 THEOREM 3.1.1. To find the least common multiple for a collection of polynomials with
 integer coefficients, we can factor each polynomial completely, and then take the product
 with each prime factor and each irreducible polynomial factor to the highest degree we
 found. Note that the integer factor of the polynomial LCM is the LCM for the collection
 of individual integer factors.


 EXAMPLE 3.1.1. Find the LCM for given polynomial expressions and state it in a fully
 factored form.
                                    x 4 , x, x 7


 SOLUTION: These polynomials are already fully factored. The only factor here is x, and
 its highest degree is 7, so LCM is x 7 .


                                           ANSWER:


 EXAMPLE 3.1.2. Find the LCM for given polynomial expressions and state it in a fully
 factored form.
                                4m2 , 6mn, 2n2


 SOLUTION: These polynomials are already fully factored. The LCM for integer factors 4,
 6, and 2 is 12, the highest degree of m is 2, and the highest degree of n is also 2. So the
 LCM is 12m2 n2 .


                                      ANSWER: 12m2 n2

                                             377
3. LCD FOR RATIONAL EXPRESSIONS                       CHAPTER 7. RATIONAL EXPRESSIONS

  EXAMPLE 3.1.3. Find the LCM for given polynomial expressions and state it in a fully
  factored form.
                          25x 2 , 5x 2 − 5, 10x 2 − 20x + 10


  SOLUTION: We factor the polynomials first:
                                   25x 2                          already factored

                    5x 2 − 5   =   5(x 2 − 1)                                 GCF
                               =   5(x + 1)(x − 1)            difference of squares

          10x 2 − 20x + 10     =   10(x 2 − 2x + 1)                           GCF
                               =   10(x − 1)2              special product pattern
  The LCM for the integer factors 5, 25, and 10 is 50. The highest degree of (x − 1) is 2,
  the highest degree of x is 2, and the highest degree of (x + 1) is 1. Hence the polynomial
  LC M is 50x 2 (x + 1)(x − 1)2 .


                               ANSWER: 50x 2 (x + 1)(x − 1)2

   3.2. LCD for Rational Expressions.

  EXAMPLE 3.2.1. Find the LCD for rational expressions
                                         1      1
                                            ,
                                        2x     x3


  SOLUTION: These denominators are already fully factored. The highest degree of the
  factor 2 is 1, and the highest degree of the factor x is 3, so LCD = 21 · x 3 .


                                       ANSWER: 2x 3


  EXAMPLE 3.2.2. Find the LCD for rational expressions
                                 1           1         1
                                      ,            ,
                              25x 2 y    20x 3 y 2   2x w7


  SOLUTION: In the examples above we found the integer factor for the polynomial LCM
  by taking the LCM of individual integer factors. In more complicated situations we may
  have to find integer factorizations in order to compute the LCM.


                                                378
CHAPTER 7. RATIONAL EXPRESSIONS                                     3. LCD FOR RATIONAL EXPRESSIONS
 We factor the denominators first:
                  25x 2 y      =       52 · x 2 y
                 20x 3 y 2     =       22 · 5 · x 3 y 2
                                       2x w7                                already factored
 The highest degree of 2 is 2, the highest degree of 5 is 2, the highest degree of x is 3, the
 highest degree of y is 2, and the highest degree of w is 7, so LCD = 22 · 52 · x 3 y 2 w7


                                           ANSWER: 100x 3 y 2 w7


 EXAMPLE 3.2.3. Find the LCD for rational expressions
                                   x −5          7
                                          ,     2
                                  4x − 4     6x − 6


 SOLUTION: Recall that for the purpose of finding the LCD, we are free to ignore the
 numerators completely. We factor the denominators first:
                                       4x − 4       =      22 (x − 1)

                                   6x 2 − 6         =      2 · 3(x 2 − 1)
                                                    =      6(x + 1)(x − 1)
 The highest degree of 2 is 2, the highest degree of 3 is 1, the highest degree of (x − 1) is
 1, and the highest degree of (x + 1) is 1, so LCD = 22 · 3 · (x + 1)(x − 1).


                                       ANSWER: 12(x + 1)(x − 1)


 EXAMPLE 3.2.4. Find the LCD for rational expressions
                                       x4                         19x
                                              ,
                                   x2 − x − 6              x 2 y − 6x y + 9 y


 SOLUTION: Recall that for the purpose of finding the LCD, we are free to ignore the
 numerators completely. We factor the denominators first:
                x2 − x − 6         =      (x − 3)(x + 2)

          x 2 y − 6x y + 9 y       =      y(x 2 − 6x + 9)                                      GCF
                                   =      y(x − 3)(x − 3)                   special product pattern
                                   =      y(x − 3)2

                                                          379
3. LCD FOR RATIONAL EXPRESSIONS                         CHAPTER 7. RATIONAL EXPRESSIONS
  The highest degree of y is 1, the highest degree of (x − 3) is 2, and the highest degree of
  (x + 2) is 1, so LCD = y(x − 3)2 (x + 2)


                                 ANSWER: y(x − 3)2 (x + 2)


  EXAMPLE 3.2.5. Find the LCD for rational expressions
                                      1             1
                                             ,
                                 x − 6x + 9
                                  2              3x − x 2


  SOLUTION: We factor the denominators first:
                                  x 2 − 6x + 9     =   (x − 3)2
                                      3x − x 2     =   x(3 − x)
  The highest degree of x is 1, and the highest degree of (x −3) is 2. (3− x) is not equivalent
  to (3 − x), but we can make it equivalent by factoring out −1. In the interest of keeping
  the overall degree of the LCD as low as possible, we treat the factor (3 − x) as if it was
  (x − 3) with degree 1. Hence the LCD is x(x − 3)2 .


                                     ANSWER: x(x − 3)2




                                                 380
CHAPTER 7. RATIONAL EXPRESSIONS                                 3. LCD FOR RATIONAL EXPRESSIONS
   Homework 7.3.

Find the least common multiple for each                      Find the least common denominator for
collection of polynomial expressions and                     each collection of rational expressions and
state it in a fully factored form:                           state it in a fully factored form:

1. 2a3 ,     6a4 b2 ,   4a3 b5                                      1              1                  1
                                                             11.       ,                 ,
                                                                   a+1          (a − 1)2         a2   −1
2. 5x 2 y,     25x 3 y 5 z
                                                                     1             1                  1
                                                             12.        ,                ,
3. x 2 − 3x,        x − 3,         x                               x −2         (x + 2)2         x2   −4

4. 4x − 8,      x − 2,         4                                     2n − 1                   n+1
                                                             13.               ,
                                                                   2n2 + n − 1          2n2   + 3n − 2
5. x + 2,      x −4
                                                                          4                     m
                                                             14.                 ,
6. x,      x − 7,     x +1                                         m2   − 2m − 3         2m2   + 3m + 1

7. x 2 − 25,        x +5                                             1            2             3
                                                             15.        ,            ,
                                                                   t −3         t +3         t2 − 9
8. x 2 − 9,     x 2 − 6x + 9
                                                                    5                   10
                                                             16.       ,
9. x + 3x + 2,
     2
                        x + 5x + 6
                           2
                                                                   a−5          a2   − 10a + 25

10. x 2 −7x +10,           x 2 −2x −15,   x 2 + x −6                5x                  2x
                                                             17.          ,
                                                                   x2 − 9        x 2 + 11x + 24
                                                                    2x                 4x
                                                             18.        ,
                                                                   x2−4          x2   + 5x + 6
                                                                          x +3                          x −1
                                                             19.                      ,
                                                                   6x 3 − 24x 2 + 18x           4x 5 − 24x 4 + 20x 3
                                                                              1                           1
                                                             20.                         ,
                                                                   9 y3   − 9 y 2 − 18 y       6 y5   − 24 y 4 + 24 y 3




                                                       381
3. LCD FOR RATIONAL EXPRESSIONS            CHAPTER 7. RATIONAL EXPRESSIONS
    Homework 7.3 Answers.

1. 12a4 b5                              11. (a + 1)(a − 1)2

3. x(x − 3)                             13. (2n − 1)(n + 1)(n + 2)

5. (x + 2)(x − 4)                       15. (t + 3)(t − 3)

7. (x + 5)(x − 5)                       17. (x + 3)(x − 3)(x + 8)

9. (x + 1)(x + 2)(x + 3)                19. 12x 3 (x − 1)(x − 3)(x − 5)




                                  382
CHAPTER 7. RATIONAL EXPRESSIONS                      4. SUMS OF RATIONAL EXPRESSIONS
                            4. Sums of Rational Expressions

    4.1. Sums with a Common Denominator. When simplifying a sum with a common de-
nominator, we take the sum of numerators over the common denominator, and then simplify
the resulting rational expression.

  EXAMPLE 4.1.1. Simplify and state the answer as a quotient of polynomial factorizations:
                                       4x        4
                                            +
                                      x2 − 1 x2 − 1


  SOLUTION: The denominators are the same, so we can add the numerators over the
  common denominator, then factor the numerator, and cancel common polynomial factors,
  if any.
                          4x         4          4x + 4
                          2
                                + 2        =
                         x −1 x −1               x2 − 1
                                                      4(x + 1)
                                              =
                                                   (x + 1)(x − 1)
                                                     4
                                              =
                                                   x −1


                                                    4
                                     ANSWER:
                                                  x −1


  EXAMPLE 4.1.2. Simplify and state the answer as a quotient of polynomial factorizations:
                                     4x − 1 2x − 9
                                            −
                                      x +4     x +4


  SOLUTION: The denominators are the same, so we can subtract the numerators over the
  common denominator, combine like terms, factor the numerator, and cancel any common
  polynomial factors we find. Notice that we are subtructing a sum, so we have to make the
  fraction parentheses visible.
        4x − 1 2x − 9          4x − 1 − (2x − 9)
              −            =                             subtract the entire numerator
         x +4   x +4                 x +4
                               4x − 1 − 2x + 9
                           =                                 add the sum of opposites
                                    x +4
                               2x + 8
                           =                                        combine like terms
                                x +4


                                            383
4. SUMS OF RATIONAL EXPRESSIONS                        CHAPTER 7. RATIONAL EXPRESSIONS
  Now we will factor polynomials completely and cancel identical polynomial factors:
                                  2x + 8            2(x + 4)
                                             =
                                   x +4              x +4
                                             =     2


                                        ANSWER: 2


   4.2. General Sums. The most economical way to add rational expressions with different
denominators is by rewriting them all with the LCD.

  EXAMPLE 4.2.1. Simplify and state the answer as a quotient of polynomial factorizations:
                                         3      4
                                            −
                                       10x 15x 2


  SOLUTION: LCD = 30x 2 , so before we can subtract, we need to modify each fraction to
  have this denominator.
                          3      4           3 3x        4    2
                             −         =        ·    −      ·
                         10x 15x 2         10x 3x 15x 2 2
                                              9x      8
                                         =       2
                                                   −
                                             30x     30x 2
  Now that the denominators are the same, we can subtract the numerators:
                                9x      8           9x − 8
                                   2
                                     −     2
                                               =
                               30x     30x           30x 2
  Both the numerator and the denominator are fully factored, so we are done.


                                                   9x − 8
                                     ANSWER:
                                                    30x 2




                                             384
CHAPTER 7. RATIONAL EXPRESSIONS                        4. SUMS OF RATIONAL EXPRESSIONS

 EXAMPLE 4.2.2. Simplify and state the answer as a quotient of polynomial factorizations:
                                          x    1
                                            +
                                        x −1 x


 SOLUTION: LCD = x(x − 1), so before we can add, we need to modify each fraction to
 have this denominator.
            x    1        x   x 1 x −1
               +     =       · + ·                  when multiplying sums
         x −1 x         x −1 x x x −1
                            x2        x −1
                      =           +                  show fraction parentheses
                        x(x − 1) x(x − 1)
 Now that the denominators are the same, we can add the numerators:
                             x2     x −1                x 2 + (x − 1)
                                  +                =
                          x(x − 1) x(x − 1)                x(x − 1)
                                                      x2 + x − 1
                                                 =
                                                       x(x − 1)
 The polynomial in the numerator is prime, so everything is fully factored, and we are done.


                                                x2 + x − 1
                                   ANSWER:
                                                 x(x − 1)


 EXAMPLE 4.2.3. Simplify and state the answer as a quotient of polynomial factorizations:
                                     2             x
                                           −
                               x 2 + 3x + 2 x 2 + 5x + 6


 SOLUTION: To find the LCD, we factor the denominators:
                             x 2 + 3x + 2   =     (x + 1)(x + 2)
                             x 2 + 5x + 6   =     (x + 2)(x + 3)
 So LCD = (x + 1)(x + 2)(x + 3), and before we can subtract fractions, we need to rewrite
 them with this denominator. To do so, we need to multiply the first fraction by
                                          x +3
                                          x +3
 and the second fraction by
                                          x +1
                                          x +1



                                            385
4. SUMS OF RATIONAL EXPRESSIONS                    CHAPTER 7. RATIONAL EXPRESSIONS
  The result looks like this:
           2                x                2              x
                   − 2            =                  −
      x + 3x + 2 x + 5x + 6
        2                              (x + 1)(x + 2) (x + 2)(x + 3)
                                             2         x +3       x         x +1
                                  =                  ·      −             ·
                                       (x + 1)(x + 2) x + 3 (x + 2)(x + 3) x + 1
                                             2(x + 3)              x(x + 1)
                                  =                         −
                                       (x + 1)(x + 2)(x + 3) (x + 2)(x + 3)(x + 1)

  Now that the denominators are equivalent, we can subtract the numerators:
                                       2(x + 3) − x(x + 1)
                                 =
                                      (x + 1)(x + 2)(x + 3)

  Finally, we will simplify the numerator by combining like terms, factoring it, and then
  canceling common polynomial factors, if any:
                                         2x + 6 − x 2 − x
                                 =
                                      (x + 1)(x + 2)(x + 3)
                                          −x 2 + x + 6
                                 =
                                      (x + 1)(x + 2)(x + 3)

  If we factor out −1 in the numerator, then we can detect an easy trinomial pattern and
  factor it as a product of two binomials:
                                          −(x 2 − x − 6)
                                 =
                                      (x + 1)(x + 2)(x + 3)
                                         −(x − 3)(x + 2)
                                 =
                                      (x + 1)(x + 2)(x + 3)
                                         −(x − 3)
                                 =
                                      (x + 1)(x + 3)


                                              −x + 3
                                ANSWER:
                                           (x + 1)(x + 3)




                                           386
CHAPTER 7. RATIONAL EXPRESSIONS                        4. SUMS OF RATIONAL EXPRESSIONS

 EXAMPLE 4.2.4. Simplify and state the answer as a quotient of polynomial factorizations:
                                       4a      7a
                                           +
                                      a−2 2−a


 SOLUTION: Since a − 2 and 2 − a are opposites of each other, we treat them as the same
 polynomial factor, and the LCD is (a − 2). Before we can add, though, we need to factor
 out −1 in one of the denominators, so that they become equivalent.
                           4a      7a           4a        7a
                               +          =         +
                          a−2 2−a              a − 2 −1(a − 2)
 Because −1 is its own reciprocal, we can move the factor −1 into the numerator, making
 the denominators the same, and allowing us to perform the addition:
                          4a      7a                   4a   −1(7a)
                              +                  =        +
                         a − 2 −1(a − 2)              a−2    a−2
                                                      4a + (−7a)
                                                 =
                                                         a−2
                                                      −3a
                                                 =
                                                      a−2
 Everything is fully factored, so we are done.


                                                     −3a
                                     ANSWER:
                                                     a−2




                                            387
4. SUMS OF RATIONAL EXPRESSIONS                       CHAPTER 7. RATIONAL EXPRESSIONS
      Homework 7.4.

Perform the summation, simplify, and state                  2    2
                                                    15.       +
the answer as a quotient of polynomial fac-               x −1 x +1
torizations.
                                                           2z   3z
    2   4                                           16.       −
1.    +                                                   z−1 z+1
   a+3 a+3
                                                            2    3
          2                                         17.        +
        x    6x − 8                                       x − 5 4x
2.         −
      x −2    x −2
                                                           8x    3
                                                    18.       −
      t + 4t 2t − 7
      2
                                                          x2−4 x +2
3.          +
       t −1   t −1                                           4x      x
                                                    19.           +
        a2 + 3a      4                                    x 2 − 25 x + 5
4.              − 2
      a + 5a − 6 a + 5a − 6
       2
                                                            t      5
                                                    20.        −
         2x 2 + 3   x 2 − 5x + 9                          t − 3 4t − 12
5.                −
      x 2 − 6x + 5 x 2 − 6x + 5                             2      4
                                                    21.        +
      3   4                                               x + 3 (x + 3)2
6.      + 2
      x x                                                        2      4
                                                    22.              −
      5   5                                               5x 2   + 5x 3x + 3
7.      −
      6r 8r                                                 3a      9a
                                                    23.          +
       6     3                                            4a − 20 6a − 30
8.       2
           + 2
      xy    x y                                             x   x
                                                    24.       +
                                                          x −5 5− x
       8     5
9.       3
           + 2                                             t     y
      9t    6t                                      25.       −
                                                          y−t   y+t
       x +5 x −3
10.        +
         8   12                                             x    x −5
                                                    26.        +
                                                          x −5     x
       a+2 a−4
11.       −
        2   4                                              2x       4
                                                    27.        − 2
                                                          x2− 1 x + 2x − 3
       2a − 1 5a + 1
12.          +
        3a2    9a                                              x −1        x +5
                                                    28.                + 2
                                                          x2   + 3x + 2 x + 4x + 3
       x − 1 2x + 3
13.         −
        4x     x                                                x +1        x +6
                                                    29.                 + 2
                                                          x2   − 2x − 35 x + 7x + 10
       a+2 a−4
14.       −
        2   4                                             3x + 2    x
                                                    30.          +
                                                          3x + 6 4 − x 2

                                              388
CHAPTER 7. RATIONAL EXPRESSIONS           4. SUMS OF RATIONAL EXPRESSIONS
    4 − a2 a − 2                               2x       3
31. 2     −                             34.        − 2
    a −9 3−a                                  x2− 1 x + 5x + 4

       4y  2    2                                    x              7
32.       − −                           35.                 −
       2
      y −1 y  y +1                            x 2 + 15x + 56 x 2 + 13x + 42

          x          2                         2x     5
33.             − 2                     36.       + 2
      x + 5x + 6 x + 3x + 2
       2                                      x2−9 x + x −6




                                  389
 4. SUMS OF RATIONAL EXPRESSIONS               CHAPTER 7. RATIONAL EXPRESSIONS
      Homework 7.4 Answers.

      6                                        2(x + 5)
1.                                       21.
     a+3                                       (x + 3)2
3. t + 7
                                                 9a
   x +6                                  23.
5.                                             4(a − 5)
   x −5
    5                                          t 2 + 2t y − y 2
7.                                       25.
   24r                                         ( y + t)( y − t)

     15t + 16
9.                                                2(x + 2)
       18t 3                             27.
                                               (x + 1)(x + 3)
      a+8
11.
       4                                          2(x − 4)
                                         29.
      −7x − 13                                 (x − 7)(x + 2)
13.
        4x
                                                   a−2
       4x                                31.
15.                                            (a + 3)(a − 3)
      x2−1
      11x − 15                                     x −3
17.                                      33.
      4x(x − 5)                                (x + 3)(x + 1)

         x(x − 1)                                  x −8
19.                                      35.
      (x + 5)(x − 5)                           (x + 8)(x + 6)




                                   390
CHAPTER 7. RATIONAL EXPRESSIONS                                           5. COMPLEX FRACTIONS
                                      5. Complex Fractions
   5.1. Definition.

  DEFINITION 5.1.1. Within this text, complex fractions are all the expressions which can be
  built up by combining polynomials with arithmetic operations +, −, ·, and ÷

  Many other sources use a narrow definition which only admits fractions of fractions
  and/or mixed numbers.

  The reader should also be careful not to confuse complex fractions with fractions which
  involve complex numbers, which we briefly mention later in the text.


  BASIC EXAMPLE 5.1.1. Here are some traditional complex fractions:


                                          1 x        1 34
                                           ÷ ,
                                          2 5        6 73

  And here are even more complex fractions which are built up as quotients of sums of
  rational expressions:

                                       3 1       x     y
                                        −          +
                             x         y 2       y     z          1
                                  ,        ,             ,
                           1            ab       1     b     1+       1
                             −1                    −                      1
                           x                     a     z          1+
                                                                          2

   5.2. Simplifying Complex Fractions.

  THEOREM 5.2.1. For every complex fraction we can find a rational expression which is
  almost equivalent. That is, the value of the complex fraction will be equal to the value of
  a corresponding rational expression whenever both values are defined.



    PROOF. By induction of arithmetic expressions. The most basic type of complex fraction
is a polynomial, which is a rational expression. A sum, a product, or a quotient of rational
expressions can be rewritten as a rational expression in simplified form, and they will have the
same values, except at finitely many points where one of them is undefined.                   

We can simplify every complex fraction by rewriting every sum as a rational expression, and
then simplifying products and quotients of rational expressions by canceling common polyno-
mial factors. These steps may have to be repeated several times. For answers, we will factor
the rational expressions and state them as quotients of polynomial factorizations.
                                               391
5. COMPLEX FRACTIONS                                 CHAPTER 7. RATIONAL EXPRESSIONS

 EXAMPLE 5.2.1. Simplify and state the answer as a quotient of polynomial factorizations:
                                           x
                                             1
                                         1+
                                             x


 SOLUTION: We start by rewriting the sum in the denominator as a rational expression. In
 order to do that, we have to find the LCD for 1 and 1/x and then add fraction as usual.
                  x             x                                    1
                        =                          the LCD for 1 and is x
                    1        x 1                                     x
               1+               +
                    x        x x
                                 x
                        = 
                               x +1
                                     ‹

                                 x

                          x +1
 Now we can divide x by          as usual:
                            x
              x              x +1
                     =   x÷                division is multiplication by the reciprocal
            x +1
                ‹
                               x
              x
                                x
                     =   x·
                              x +1
                           x2
                     =
                         x +1


                                                    x2
                                      ANSWER:
                                                  x +1


 EXAMPLE 5.2.2. Simplify and state the answer as a quotient of polynomial factorizations:
                                         1 x
                                           +
                                         2 4
                                         1 x
                                           −
                                         4 2


 SOLUTION: To avoid rewriting a giant fraction, we will simplify this expression in three
 stages. First we will simplify the sums, and then divide the resulting rational expressions.




                                            392
CHAPTER 7. RATIONAL EXPRESSIONS                                        5. COMPLEX FRACTIONS
 Simplify the numerator:
                   1 x                 2 x
                     +          =       +                       the LCD is 4
                   2 4                 4 4
                                       2+ x
                                =
                                        4

 Simplify the denominator:
                   1 x                1 2x
                    −      =            −                        the LCD is 4
                   4 2                4   4
                                      1 − 2x
                                =
                                         4

 Divide numerator by denominator:
        2 + x 1 − 2x        2+ x       4
             ÷          =         ·                      multiply by reciprocal of divisor
          4      4            4     1 − 2x
                                     (2 + x)4
                            =                                  show fraction parentheses
                                    4(1 − 2x)
                                     2+ x
                            =                                  canceled common factor 4
                                    1 − 2x


                                                       2+ x
                                         ANSWER:
                                                      1 − 2x


 EXAMPLE 5.2.3. Simplify and state the answer as a quotient of polynomial factorizations:
                                             2
                                        1+
                                               4
                                           3+
                                               x


                                                                                4
 SOLUTION: We start by simplifying the sum of rational expressions 3 +
                                                                                x
                        4             3x 4
                   3+           =        +                      the LCD is x
                        x              x   x
                                      3x + 4
                                =
                                        x




                                                393
5. COMPLEX FRACTIONS                                      CHAPTER 7. RATIONAL EXPRESSIONS
                                                                     3x + 4
 Now we can compute the quotient of rational expressions 2 and              in the usual way:
                                                                       x
                                  3x + 4         x        2x
                               2÷        =2·          =
                                     x         3x + 4 3x + 4
                                                           2x
 Finally, we simplify the sum of rational expressions 1 +        in the usual way:
                                                          3x + 4
                    2x          3x + 4      2x
              1+            =           +                    the LCD is 3x + 4
                 3x + 4         3x + 4 3x + 4
                                   3x + 4 + (2x)
                           =
                                      3x + 4
                                   5x + 4
                           =
                                   3x + 4


                                                     5x + 4
                                       ANSWER:
                                                     3x + 4


 EXAMPLE 5.2.4. Simplify and state the answer as a quotient of polynomial factorizations:
                                            x −1 + x −2
                                            x −3 − x −2


 SOLUTION: We can convert negative exponents into rational expressions and then simplify
 the resulting complex fraction just as before:
                                                 1     1
                                                     + 2
                                    x +x
                                     −1     −2
                                                 x x
                                               =
                                     −3
                                    x −x    −2   1     1
                                                   3
                                                     − 2
                                                 x     x

 Simplify the numerator:
                 1    1             x   1
                   + 2         =      +                        the LCD is x 2
                  x x               x2 x2
                                    x +1
                               =
                                      x2

 Simplify the denominator:
                  1     1            1     x
                    3
                      − 2 =            3
                                         − 3                   the LCD is x 3
                  x    x             x    x
                                     1− x
                               =
                                      x3

                                               394
CHAPTER 7. RATIONAL EXPRESSIONS                         5. COMPLEX FRACTIONS
 Divide numerator by denominator:
               x +1 1− x            x +1    x3
                    ÷         =          ·
                 x2   x3              x2   1− x
                                    (x + 1)x 3           x3
                              =                             =x
                                    x 2 (1 − x)          x2
                                    (x + 1)x
                              =
                                      1− x


                                             x(x + 1)
                                ANSWER:
                                              1− x




                                         395
5. COMPLEX FRACTIONS                                     CHAPTER 7. RATIONAL EXPRESSIONS
     Homework 7.5.

Simplify and state the answer as a quotient of              −5
                                                               −3
polynomial factorizations:                                 b−5
                                                       10.
                                                            10
      1                                                        +6
     1+                                                    b−5
      x
1.                                                           x   1
      1                                                        −
   1− 2                                                    x +1 x
     x                                                 11.
                                                             x   1
   1                                                           +
      −1                                                   x +1 x
   y2
2.                                                          2a   3
       1                                                       −
   1+                                                      a−1 a
       y                                               12.
                                                            −6
                                                               −4
     a−2                                                   a−1
3.
     4
       −a                                                    3
                                                            ‹
     a
                                                             x
                                                       13.  ‹
   25                                                        9
      −a
4.
    a                                                       x2
   5+a                                                      x 
   1     1                                                   3x − 2 ‹
       −                                               14. 
   a2 a                                                         x
5.
   1     1                                                     2
                                                             9x − 4
     2
       +
   a     a
                                                            1     1
     1 1                                                        −
        +                                                  a−h a
                                                       15.
     b 2‹                                                     h
6. 
        4
      2
     b −1                                                    1     1
                                                                 −
                                                           x +h x
                                                       16.
        4                                                      h
     2−
      x +2
7.                                                         x −1 + y −1
       10                                              17.  2
   5−
                                                             x − y2
                                                                     
      x +2
                                                               xy
        12
     4+
      2x − 3                                                 x − x −1
8.
        15                                             18.
   5+                                                        x + x −1
      2x − 3
                                                                   2
     3                                                       x −1+
          +2                                                     x −4
   2a − 3                                              19.
9.                                                                 6
    −6                                                     x +3+
          −4                                                     x −4
   2a − 3




                                                 396
CHAPTER 7. RATIONAL EXPRESSIONS                     5. COMPLEX FRACTIONS
           18                               x −1 x +1
   x −5−                                        −
          x +2                              x +1 x −1
20.                                     22.
            6                               x −1 x +1
    x +7+                                       +
          x +2                              x +1 x −1
    2   5
      −
    b b+3
21.
    3   3
      +
    b b+3




                                  397
 5. COMPLEX FRACTIONS                        CHAPTER 7. RATIONAL EXPRESSIONS
         Homework 7.5 Answers.
        x                                     x
1.                                     13.
      x −1                                    3
      −a
3.                                              1
     a+2                               15.
                                             a(a − h)
     −a + 1
5.
      a+1                                      1
                                       17.
     2                                        x−y
7.
     5
                                              x −2
     1                                 19.
9. −                                          x +2
     2
         x2 − x − 1                          −b + 2
11.                                    21.
         x2 + x + 1                          2b + 3




                                 398
CHAPTER 7. RATIONAL EXPRESSIONS               6. EQUATIONS WITH RATIONAL EXPRESSIONS
                          6. Equations with Rational Expressions
   6.1. Extraneous Solutions.

  DEFINITION 6.1.1 (Extraneous Solution). Some techniques for solving equations will yield
  numbers which are not solutions. This is a reasonable compromise as long as these tech-
  niques never miss a legitimate solution. In this case, we can solve an equation by making a
  list of all possible solutions, and then checking each one. The ones that make the equation
  true are the proper solutions, while the rest are called extraneous solutions. It is important
  to understand that strictly speaking they are not solutions at all.


When solving equations involving rational expressions, we will often simplify them by cancel-
ing polynomial factors with their reciprocals, for example
                                    (x + 1)(x − 2)    x +1
                                                    =
                                        x(x − 2)         x
Recall that these expressions are not exactly equivalent, since the one on the left is not defined
for x = 2. In fact, if x = 2, then the reciprocal of x − 2 is not defined, and nothing can
be canceled. So when we do this within an equation, we make a tacit assumption that the
polynomial factor x − 2 6= 0, which is equivalent to x 6= 2. In other words, 2 can not be a
solution.

  THEOREM 6.1.1. An equation involving rational expressions can be solved by multiplying
  both sides by the polynomial LCD, canceling common polynomial factors, and then solving
  the resulting polynomial equation. Some or all solutions of the polynomial equation may
  be extraneous. To find out which ones, it is sufficient to determine which values of the
  variable make the expressions within the original equation undefined: if they come up as
  solutions, they must be extraneous, while everything else will work.

  A more generic (and somewhat more tedious) way to detect an extraneous solution is to
  use it in the original equation and show that the equation fails to be satisfied.




                                              399
6. EQUATIONS WITH RATIONAL EXPRESSIONS                CHAPTER 7. RATIONAL EXPRESSIONS

  EXAMPLE 6.1.1. Solve the equation
                                             x    x −2
                                      x−        =
                                           x −1   x −1


  SOLUTION: Looking at denominators, if x = 1 then x − 1 = 0, and expressions involving
  denominator x − 1 will have undefined values, so 1 can not be a solution.

  Our technique is to multiply both sides of the equation by the LCD, which is x − 1, and
  then simplify fractions:
                       x         x −2
                 x−         =                       original equation in factored form
                     x −1        x −1
                                   x −2
                                       ‹
            x 
        x−         (x − 1) =              (x − 1)         both sides multiplied by LCD
           x −1                    x −1

  As we distribute, we end up multiplying each term of the equation by LCD, which amounts
  to multiplying the numerator of each term by LCD:
                   x(x − 1)       (x − 2)(x − 1)
     x(x − 1) −               =                                              distributivity
                   (x − 1)           (x − 1)
             x(x − 1) − x     =   x −2              common polynomial factors canceled

  This looks like a quadratic equation, so we will solve it by factoring:
               x2 − x − x     =   x −2                           combine like terms
                     2
                    x − 2x    =   x −2           subtract x and add 2 on both sides
              x − 3x + 2
               2
                              =   0                                           factor
           (x − 1)(x − 2)     =   0
  By the zero product property, solutions are 1 and 2. Solutions of this quadratic equation
  are the possible solutions of the original equation involving rational expressions, but some
  of them may be extraneous. We figured out early on that 1 makes expressions undefined,
  so it is extraneous, and the only solution is 2.

  Alternatively, we can check the answers in the usual way and discard the ones that do not
  work. When we substitute 1 for x in the original equation, we end up dividing by zero,
  which is not allowed:
                                         (1)         (1) − 2
                                 (1) −           =
                                       (1) − 1       (1) − 1


                                         ANSWER: {2}



                                              400
CHAPTER 7. RATIONAL EXPRESSIONS                6. EQUATIONS WITH RATIONAL EXPRESSIONS

 EXAMPLE 6.1.2. Solve the equation
                                             1           1
                                                     = 2
                                     2x 2   + 3x + 1  x −1


 SOLUTION: Factor denominators and find the LCD:
                             2x 2 + 3x + 1     =     (2x + 1)(x + 1)
                                      2
                                     x −1      =     (x + 1)(x − 1)
 Recall that LCD is the product of prime polynomial factors, each with the highest exponent
 we found, and each of these factors has the highest degree 1, so
                                 LCD = (x + 1)(x − 1)(2x + 1)
 If either of these factors becomes zero, some expressions within the original equation
 become undefined. We can make a list of forbidden solutions by solving the equation
                                  (x + 1)(x − 1)(2x + 1) = 0
 By the zero product property,

        • If x + 1 = 0 then x = −1
        • If x − 1 = 0 then x = 1
        • If 2x + 1 = 0 then x = −1/2

 So if any of these numbers come up as solutions, we will know they are extraneous. Now
 we can multiply both sides of the equation by the LCD and get rid of fractions by canceling
 common polynomial factors:
               1                     1
                         =                         original equation in factored form
        (2x + 1)(x + 1)        (x + 1)(x − 1)

 Multiply both sides by LCD to get rid of fractions:
          1                                                 1
                  ‹                                                ‹
                     (x + 1)(x − 1)(2x + 1) =                          (x + 1)(x − 1)(2x + 1)
   (2x + 1)(x + 1)                                    (x + 1)(x − 1)

 Note that denominators do not change, and what we do amounts to multiplying the nu-
 merator of each term in the equation by LCD:
    (x + 1)(x − 1)(2x + 1)            (x + 1)(x − 1)(2x + 1)
                                 =                                     cancel common factors
       (2x + 1)(x + 1)                    (x + 1)(x − 1)
                       x −1      =    2x + 1                          this is a linear equation

                         −2      =    x                           x is isolated on the right




                                               401
6. EQUATIONS WITH RATIONAL EXPRESSIONS                   CHAPTER 7. RATIONAL EXPRESSIONS
  The solution −2 of the linear equation is not among the forbidden solutions, so it should
  work. Alternatively, it can be checked in the original equation involving rational expres-
  sions:
                                      1                     1
                                                   =
                             2(−2) + 3(−2) + 1
                                   2                    (−2)2 − 1
                                         1                  1
                                                     =
                                 2 · 4 + (−6) + 1          4−1
                                           1               1
                                                     =
                                         8−6+1             3
                                                1          1
                                                     =
                                                3          3

  The equation holds true, so −2 is the only solution.


                                         ANSWER: {−2}


  EXAMPLE 6.1.3. Solve the equation
                                               24
                                         x+        = −9
                                              x −2


  SOLUTION: The LCD is x − 2, so the only forbidden solution is x = 2. We multiply both
  sides by the LCD, distribute as needed, and then simplify resulting rational expressions:
                 24
                    ‹
            x+         (x − 2) = (−9) (x − 2)          both sides multiplied by LCD
                x −2
                     24(x − 2)
        x(x − 2) +               =    −9x + 18                             distributivity
                       x −2
                 x 2 − 2x + 24   =    −9x + 18                   factor (x − 2) canceled
  This is a quadratic equation, which we can solve by moving all the non-zero terms to one
  side, factoring, and applying the zero product property:
     x 2 − 2x + 24 + 9x − 18     =   −9x + 18 + 9x − 18        added 9x − 18 to both sides

                 x 2 + 7x + 6    =   0                                combined like terms

              (x + 1)(x + 6)     =   0
  By the zero product property, solutions of this quadratic equation are −1 and −6. The
  only forbidden solution is 2, so both −1 and −6 should work. Alternatively, we can check
  whether they are solutions for the original equation.


                                               402
CHAPTER 7. RATIONAL EXPRESSIONS             6. EQUATIONS WITH RATIONAL EXPRESSIONS
 If x = −1 then the left side of the original equation evaluates to
         24                      24
     x+         = (−1) +
        x −2                  (−1) − 2
                            24
                =    −1 +
                            −3
                =    −1 + (−8)

                =    −9                  this is the right side, so the equation holds true

 If x = −6 then the left side of the original equation evaluates to
         24                      24
     x+         = (−6) +
        x −2                  (−6) − 2
                            24
                =    −6 +
                            −8
                =    −6 + (−3)

                =    −9                  this is the right side, so the equation holds true


                                    ANSWER: {−6, −1}




                                            403
6. EQUATIONS WITH RATIONAL EXPRESSIONS     CHAPTER 7. RATIONAL EXPRESSIONS
      Homework 7.6.

Solve the equation.                             4x       4     1
                                         16.         −       =
                                               2x − 6 5x − 15 2
      x 6
1.     − =0                                     −3      x
      6 x                                17.        =
                                               x +3   x +3
      3  4 1
2.      = −                                      3      x
      x  x 5                             18.         =
                                               2x − 6 2x − 6
         4
3. a +     = −5                                4− x    12
         a                               19.        =
                                               1− x   3− x
         3
4. y +     = −4                                 7   1  3
         y                               20.       + =
                                               3− x 2 4− x
      y +3     6
5.         =                                     7  1  y −2
      y −3   y −3                        21.       − =
                                               y −3 2  y −4
      x   18
6.      =                                       2    6
      2    x                             22.       −     =1
                                               3− x 8− x
          1 1
7. 3x −    − =0                                  1   1    3x + 8
          2 x                            23.       −     = 2
                                               x +2 2− x  x −4
                 4
8. x + 1 =                                      x +2  1  3x − 3
               x +1                      24.         − =
                                               3x − 1 x  3x 2 − x
          20     5x
9. x +        =      −2                        x +1 x −1 5
         x −4   x −4                     25.       −    =
                                               x −1 x +1 6
       x2 + 6 x − 2
10.          +      = 2x                       x −1 x +2 3
       x −1    x −1                      26.       +    =
                                               x −3 x +3 4
            6     2x
11. x +        =                                 3     2x + 1
          x −3   x −3                    27.         +        =1
                                               2x + 1 2x − 1
       x −4   12
12.         =    +1                            3x − 5 5x − 1 x − 4
       x −1 3− x                         28.         +      −      =2
                                               5x − 5 7x − 7 1 − x
        2x     4x + 5   3
13.          =        −                          3    5      5x
       3x − 4 6x − 1 3x − 4              29.       +     = 2
                                               x −3 x +2  x − x −6
        6x + 5   2       3x
14.            −       = 2                       2    1       x
         2
       2x − 2x 1 − x 2  x −1             30.       +     = 2
                                               x −2 x +4  x + 2x − 8
        3m      7     3
15.          −      =                            5   30
       2m − 5 3m + 1 2                   31.       − 2   =1
                                               y −3 y −9

                                  404
CHAPTER 7. RATIONAL EXPRESSIONS   6. EQUATIONS WITH RATIONAL EXPRESSIONS
        1    1      1                           4    2t     12
32.       +     = 2                     34.       +     = 2
      x +3 x −3  x −9                         t −3 t −3  t − 6t + 9
        5   3x       4
33.       +     = 2
      x −2 x −2  x − 4x + 4




                                  405
6. EQUATIONS WITH RATIONAL EXPRESSIONS         CHAPTER 7. RATIONAL EXPRESSIONS
    Homework 7.6 Answers.

1. {−6, 6}                               19. {−5, 0}

                                                   16
                                               §        ª
3. {−4, −1}                              21.          ,5
                                                    3
5. ∅
                                         23. {−8}
    1 2
  §     ª
7. − ,                                           1
                                             §     ª
    2 3                                  25. − , 5
                                                 5
9. {3}
                                                1
                                             § ª
                                         27.
11. {2}                                        10

                                         29. ∅
13. {−1}
                                         31. {2}
15. {−5}
                                                 7
                                            §      ª
                                         33. −2,
17. ∅                                            3




                                  406
CHAPTER 7. RATIONAL EXPRESSIONS                                                 6. PRACTICE TEST 7
                                            Practice Test 7




Find all variable values which make the                         1         1
                                                       9.           + 2
given expression undefined:                                  4x 2− 9 4x + 12x + 9
         x +3
1.                                                         2   2
     x 2 + 5x + 6                                            −
                                                           x 3x
                                                       10.
             y5                                            1   5
2.                                                           −
     y 3 − 10 y 2 + 25 y                                   x 6x

                                                              1
                                                               1+
Find the lowest common denominator for                        x
                                                       11.
the given expressions and state it in a fully                 1
                                                           1− 2
factored form:                                               x
       1         2x            4
3.                                                                  1
       6        x +1         2x + 2                    12.
                                                                        1
                                                              1+
        1              2           3                                1+      1
4.                                                                          x
        x2           2
                    x −1       x + 2x + 1
                                2


        1            1         1
5.
       6x 5         4x 7      8x 6
           7                y           1              Solve the equation:
6.
       10x + 5x
          2                50x 2        2
                                      4x − 1                  5 1  1
                                                       13.     − =
                                                              x 3  x
Simplify the expression and state the an-                             1        3
swer in a fully factored form:                         14. 2 −             =
                                                                    x2 + x   x +1
     x 2 + x − 6 x 2 + 10x + 21                                 x       2          5
7.              ÷                                      15.         + 2         =
     x2 − x − 2       x +1                                    x + 2 x + 5x + 6   x +3
     x + 3 1 − 2x                                               x    3       7x
8.        −                                            16.        +     = 2
     x − 4 8 − 2x                                             x −5 x +2  x − 3x − 10




                                                 407
 6. PRACTICE TEST 7                         CHAPTER 7. RATIONAL EXPRESSIONS
     Practice Test 7 Answers.

1. {−2, −3}                                       4x
                                      9.
                                           (2x − 3)(2x + 3)2
2. {0, 5}
                                      10. 8
3. 6(x + 1)                                    x
                                      11.
                                             x −1
4. x 2 (x + 1)2 (x − 1)
                                             x +1
         7                            12.
5. 24x                                      2x + 1

6. 50x 2 (2x + 1)(2x − 1)             13. {12}

       1                              14. {−1/2, 1}
7.
     x +7
                                      15. {4}
      7
8.
   2(x − 4)                           16. {−3}




                                408
                                            CHAPTER 8


                                            Radicals


                                    1. Introduction to Radicals


                                y
                            9
                                                y = x2
                            8
                            7
                            6
                            5
                            4                                       p
                                                               y=       x
                            3
                            2
                            1
                                                                                x
                                    1   2   3    4     5   6   7    8       9

   1.1. Square Root and Radicals.

  DEFINITION 1.1.1 (Principal Square Root). The principal square root of a non-negative
  real number b is the non-negative number y such that y 2 = b. It is written like so:
                                                p
                                                    b= y
  The fancy sign is known as radical or radix and the expression b is called the radicand of
  the root.


The radical is actually just a traditional notation for exponent:
                                                           1
                                            p
                                                b = (b) 2
This is perhaps the best way to understand the principal square root, and the reason why it
obeys the properties of exponential expressions.

The principal square root of a negative number is not real, but complex.
                                                     409
1. INTRODUCTION TO RADICALS                                                CHAPTER 8. RADICALS

 BASIC EXAMPLE 1.1.1. Here are some easy-to-find square roots.
                                                                          p
 The principal square root of 16 is 4 because 42 = 16, so we can write 16 = 4.
 p
   100 = 10 because 102 = 100.
 p
   0 = 0 because 02 = 0.
 p
   0.25 = 0.5 because 0.52 = 0.25.
 v                       ‹2
 t4 2                    2     4
      =       because        =
    9 3                  3     9
 p
   2 = 1.41421 . . . is irrational, so it cannot be written as a fraction of integers, and we
 can only list finitely many digits of its decimal expansion here.


 THEOREM 1.1.1. Applying the principal square root to an exponential expression with
 base b ≥ 0 amounts to dividing the exponent by 2. For all non-negative real numbers a
 and all positive even integers k,
                                          p               k
                                              a k = (a) 2
 The same is true for exponentiating the principal square root:
                                          p k     k
                                           a = (a) 2
 This is true for odd integers as well, as will be shown in the rational exponent section.


 BASIC EXAMPLE 1.1.2. When taking roots of exponential expressions, we can check the
 results by definition.
 p
    x 14 = x 14/2 = x 7 , which can be checked by definition:
                                        (x 7 )2 = x 7·2 = x 14
 Æ
     y −6 = y −6/2 = y −3 = 1/ y 3 , which can be checked by definition:
                                      ‹2
                                       1         1      1
                                            = 3 2 = 6 = y −6
                                       y 3     (y )    y

 Exponentiating roots works similarly:

  p 4                                                   p −20                     1
   5 = 54/2 = 52 = 25                                     z     = z −20/2 = z −10 = 10
                                                                                   z

                                                410
CHAPTER 8. RADICALS                                             1. INTRODUCTION TO RADICALS
   1.2. Principal nth Root. The nth root is a generalization of the square root. Just like the
square root cancels the square for x ≥ 0
                                         p
                                           x2 = x
the nth root will cancel the nth power
                                               p
                                                   xn = x
                                               n




  DEFINITION 1.2.1 (Principal nth Root). Given a positive even integer n, the principal nth
  root of a non-negative real number b is the non-negative number y such that y n = b.

  Given a positive odd integer n, the principal nth root of a real number b is the number y
  such that y n = b.

  The principal nth root of b is written like so:
                                               p
                                               n
                                                   b= y
  The fancy sign is known as radical or radix, n is called the index and b is called the radicand
  of the root.


Just as was the case with the square root, the radical is actually just a traditional notation for
exponent:
                                                            1
                                        p  n
                                               b = (b) n
                                    p
Note
  p  that the principal square root   b is the principal nth root for n = 2, and could be written
   2
as b, but 2 is traditionally not shown.

  BASIC EXAMPLE 1.2.1. Here are some easy-to-find nth roots.
                                                                            p
                                                                            3
  The principal 3rd root of 125 is 5 because 53 = 125, so we can write        125 = 5
  p
  5
     0 = 0 because 05 = 0
  p
  3
     8 = 2 because 23 = 8
  p
  4
     0.0081 = 0.3 because 0.34 = 0.0081
  p
  17
      1 = 1 because 117 = 1
  v
  3                       ‹3
  t    27    3             3        27
           =     because       =
      125 5                5       125




                                                   411
1. INTRODUCTION TO RADICALS                                                CHAPTER 8. RADICALS

                                                                        p p p
 BASIC EXAMPLE 1.2.2. Roots with even indices, such as the square root , 4 , 6 , and so
 on, were only defined for non-negative radicands, but roots with odd index can be applied
 to negative radicands as well.
 p3
    −1000 = −10 because (−10)3 = −1000.
 p5
    −32 = −2 because (−2)5 = −32
 p7
    −1 = −1 because (−1)7 = −1.


 THEOREM 1.2.1. Applying the principal nth root to an exponential expression with base
 b ≥ 0 amounts to dividing the exponent by n.
                                           p
                                           n               k
                                               b k = (b) n
 Similarly for exponentiating the principal nth root:
                                          €p
                                           n
                                              Šk     k
                                             b = (b) n


 BASIC EXAMPLE 1.2.3. When taking roots of exponential expressions, we can check the
 results by definition.
 p6
    x 12 = x 12/6 = x 2 , which can be checked by definition:
                                         (x 2 )6 = x 2·6 = x 12

 p                              1
     y −15 = y −15/3 = y −5 =
 3
                                   , which can be checked by definition:
                                y5
                                          ‹3
                                            1             1
                                                   =
                                           y5          ( y 5 )3
                                                          1
                                                   =
                                                         y 15
                                                   =     y −15

 Exponentiating roots works similarly:
  p
  4  20
    3 = 320/4 = 35 = 243

  p   −35                    1
           = z −35/5 = z −7 = 7
  5
    z
                             z


                                                 412
CHAPTER 8. RADICALS                                       1. INTRODUCTION TO RADICALS

 EXAMPLE 1.2.1. Simplify the radical expression, assuming that all variable radicands and
 bases are non-negative:               Æ3
                                          (a − 5)18


 SOLUTION:                 Æ
                           3
                               (a − 5)18 = (a − 5)18/3 = (a − 5)6


                                    ANSWER: (a − 5)6


 EXAMPLE 1.2.2. Simplify the radical expression, assuming that all variable radicands and
 bases are non-negative:
                                       p4
                                               −24
                                          x +7


 SOLUTION: The radicand is a sum x + 7, so we have to introduce parentheses when we
 rewrite the radical as an exponent with base x + 7:
                      p4
                             −24                                 1
                         x +7     = (x + 7)−24/4 = (x + 7)−6 =
                                                               (x + 7)6


                                                  1
                                    ANSWER:
                                               (x + 7)6




                                            413
1. INTRODUCTION TO RADICALS                                              CHAPTER 8. RADICALS
      Homework 8.1.
                                                                Š6
Simplify the square root.
                                                        €p
                                                  19.        a+6
   p
1. 81                                                   €p      Š10
   p                                              20.        b+1
2. 25
                                                        Æ
   p                                              21.       (x − 7)2
3.   49
                                                        Æ
                                                  22.       ( y − 13)2
   p
4.   16
   p                                                    Æ
5. 0                                              23.       (m + k)14
   p                                                    Æ
6. 1                                              24.       (2a − b)4


Simplify the radical expression.
   p
   3
7.    27                                          Simplify the radical expression, assuming
   p
   3
                                                  that all variable radicands and bases are
8.    64                                          non-negative.
   p
   3
9. −1000
                                                      p3
                                                  25.      x 30
     p
     3
10. −125
                                                      p3
                                                  26.      y 12
     p
     3
11. −1                                                p5
                                                  27.      a20
     p
     5
12. 0                                                 p5
                                                  28.      b10
     p
     5
13. 32                                                  p3
                                                              18
                                                  29.      a
     p
     4
14. 81                                                   p   25
                                                         5
                                                  30.      u

Simplify the square root, assuming that
                                                        p
                                                        4
                                                  31.       x 2000
all variable radicands and bases are non-
negative.
                                                        p
                                                        4
                                                  32.  u4
     p                                                p
15.     x 12                                      33.   x −12
     p
16.     y 18
                                                      p
                                                  34.   b−16
       p 8                                            p −20
17.     a                                         35. ( 4 y)
      €p Š4                                             €p
                                                         4
                                                             Š−4
18.     b                                         36.      k
                                            414
 CHAPTER 8. RADICALS                            1. INTRODUCTION TO RADICALS
       Homework 8.1 Answers.

1. 9                                 21. x − 7

3. 7                                 23. (m + k)7
5. 0
                                     25. x 10
7. 3
                                     27. a4
9. −10
                                     29. a6
11. −1

13. 2                                31. x 500

15. x 6                                    1
                                     33.
                                           x6
17. a4
                                           1
                                     35.
19. (a + 6) 3
                                           y5




                               415
2. PRODUCTS WITH SQUARE ROOTS                                        CHAPTER 8. RADICALS
                              2. Products with Square Roots
   2.1. Distributivity Over Multiplication.

  THEOREM 2.1.1. Principal square root distributes over multiplication. For all non-negative
  real numbers x and y
                                   p            p p
                                     xy =         x· y
  This property extends naturally to a product with any number of factors.



   PROOF. Using the exponential representation of the root and the properties of the exponent:
                          p             1       1      1  p p
                            x y = (x y) 2 = (x) 2 ( y) 2 = x · y
                                                                                               


We are assuming all variable bases and radicands to be non-negative throughout this chapter
becausepseveral very nice properties fail otherwise. In particular, the distributive property
p          p
   x y = x· y works for positive radicands, but allowing negative radicands leads to nonsense
like
                        p
               1 =        1
                        Æ
                   =      (−1)(−1)
                        p p
                   =      −1 −1                     illegitimate distributivity
                    =   −1
The
p mistake above is in the third equals sign: there is nothing wrong with the complex number
 −1, it is just that the distributive property does not apply to negative radicands.
                                           p
  EXAMPLE 2.1.1. Simplify the expression    8


  SOLUTION: When simplifying expressions involving square roots, it is customary to sim-
  plify
      p the radicand by taking square rootspof factors which happen to be perfect squares.
  So 8 will be written in the answer as 2 2 because
                                     p          p
                                       8 =         4·2
                                                p p
                                           =      4· 2
                                                   p
                                           = 2· 2


                                                p
                                       ANSWER: 2 2

                                              416
CHAPTER 8. RADICALS                                  2. PRODUCTS WITH SQUARE ROOTS
                                          p
 EXAMPLE 2.1.2. Simplify the expression    75


 SOLUTION: We simplify the radicand by finding the largest integer factor which happens
 to be a perfect square, and taking its square root separately by distributivity:
                            p       p          p     p        p
                              75 = 25 · 3 = 25 · 3 = 5 · 3

                                             p
                                    ANSWER: 5 3


 EXAMPLE 2.1.3. Simplify the expression, assuming that all variable bases and radicands
 are non-negative:                       p
                                            x3


 SOLUTION: We simplify the radicand by finding the largest even integer exponent of x,
 and taking its square root separately by distributivity:
                                   p            p
                                      x3 =         x2 · x
                                                p       p
                                           =       x2 · x
                                                   p
                                           = x· x


                                                 p
                                    ANSWER: x        x


 EXAMPLE 2.1.4. Simplify the expression, assuming that all variable bases and radicands
 are non-negative:                       p
                                            x 17


 SOLUTION: The largest even exponent below 17 is 16:
             p            p
               x 17 =        x 16 · x
                          p         p
                     =       x 16 · x
                                   p
                     = x 16/2 · x                 simplified square root
                               p
                     = x8 · x


                                              p
                                   ANSWER: x 8 x

                                           417
2. PRODUCTS WITH SQUARE ROOTS                                        CHAPTER 8. RADICALS

 EXAMPLE 2.1.5. Simplify the expression, assuming that all variable bases and radicands
 are non-negative:                    p       p
                                        2x 3 · 8x


 SOLUTION: Here it helps to simplify the product under the square root before simplifying
 the radicand:
           p       p          p
             2x 3 · 8x =        2x 3 · 8x                          distributivity
                              p
                         =      16x 4
                              p      p
                         =      16 · x 4           distributivity the other way
                          =    4 · x 4/2                    simplify square roots
                          =    4 · x2



                                           ANSWER: 4x 2


 EXAMPLE 2.1.6. Simplify the expression, assuming that all variable bases and radicands
 are non-negative:                  p p        p 
                                   2 6 3+5 2



 SOLUTION: It is important not to confuse the distributivity of multiplication over addition
 with the distributivity of the principal root over multiplication.
        p p          p            p p          p    p
       2 6 3+5 2             = 2 6· 3+2 6·5 2                   distributivity of · over +
                                   p           p                                  p
                             = 2 6 · 3 + 10 6 · 2              distributivity of over ·
                                   p         p
                             = 2 18 + 10 12
 Now we simplify the radicands:
     p         p            p         p
    2 18 + 10 12 = 2 9 · 2 + 10 4 · 3                     finding perfect square factors
                            p p          p p                                    p
                       = 2 9 · 2 + 10 4 · 3                    distributivity of over ·
                                p            p
                       = 2 · 3 · 2 + 10 · 2 · 3                 simplified square roots
                            p      p
                       = 6 2 + 20 3


                                           p      p
                                  ANSWER: 6 2 + 20 3


                                               418
CHAPTER 8. RADICALS                                    2. PRODUCTS WITH SQUARE ROOTS

 EXAMPLE 2.1.7. Simplify the expression, assuming that all variable bases and radicands
 are non-negative:                  p            p
                               −5x x 3 y 4 · (−2) x y 5


 SOLUTION: We will combine the two radicands into one using distributivity, and then
 simplify the result. First, we change the order of multiplication:
      p              p                           p        p
  −5x x 3 y 4 · (−2) x y 5 = (−5)(−2) · x · x 3 y 4 · x y 5
                                       p                                             p
                               = 10x x 3 y 4 · x y 5                distributivity of over ·
                                       p
                               = 10x x 3+1 y 4+5                      properties of exponent
                                       p
                               = 10x x 4 y 9
 Now we can simplify the radicand:
          p                   p
      10x x 4 y 9 = 10x x 4 · y 8 · y                   find largest even power of y
                              p     p       p
                     = 10x x 4 · y 8 · y                                distributivity
                                            p
                     = 10x · x 4/2 · y 8/2 · y                  simplify square roots
                                        p
                     = 10x · x 2 · y 4 · y
                                     p
                     = 10x 1+2 · y 4 y                         properties of exponent
                                    p
                     = 10x 3 · y 4 · y



                                                       p
                                   ANSWER: 10x 3 y 4       y




                                            419
2. PRODUCTS WITH SQUARE ROOTS                                       CHAPTER 8. RADICALS
   Homework 8.2.
                                                        p        p
Simplify the expression, assuming that            12. −5 10 · 15
all variable bases and radicands are non-             p         p
negative.                                         13. 12m · 15m
    p                                                 p             p
1. 12                                             14.   5 y 3 · (−5) 10 y 2
    p                                                 p p
2. 18                                             15.   6( 2 + 2)
    p                                                 p     p      p
3.    27                                          16. 10( 5 + 2)
    p                                                 p         p
4.    60                                          17.   10b · 50b
    p                                                 p       p
5.    72                                          18.   6a · 18a7
    p
6. 80
                                                      p          p
                                                  19.   x2 y3 · x y4
    p
7.    20x 5
                                                      p          p
                                                  20.   a3 b2 · ab
8.
    p
      32 y 7                                           p           p
                                                  21. 5 10(5n + 2)
    p                                                 p     p       p
9.    a10 b13                                     22. 15( 5 − 3 3x)
     p
10.    49x 49
                                                      p          p
                                                  23.   6x y · 12x 2 y 5
      p         p
11. 3 5 · (−4) 16
                                                      p          p
                                                  24.   5ab5 · 20a2 b3




                                            420
CHAPTER 8. RADICALS                   2. PRODUCTS WITH SQUARE ROOTS
    Homework 8.2 Answers.
    p                                   p
1. 2 3                            13. 6m 5
    p                                  p     p
3. 3 3                            15. 2 3 + 2 6
    p                                    p
5. 6 2                            17. 10b 5
       p                                   p
7. 2x 2 5x                        19. x y 3 x y
        p                                p       p
9. a5 b6 b                        21. 25n 10 + 10 5
       p                                    p
11. −48 5                         23. 6x y 3 2x




                            421
3. QUOTIENTS WITH SQUARE ROOTS                                         CHAPTER 8. RADICALS
                              3. Quotients with Square Roots
   3.1. Distributivity Over Division.

  THEOREM 3.1.1. Principal square root distributes over division. For all non-negative real
  numbers a and all positive reals c
                                        s            p
                                           a           a
                                                = p
                                           c           c
  This results generalizes naturally to fractions with any number of factors.


   PROOF. Using the exponential representation of the root and the properties of the exponent:
                                       ‹ 12         1   p
                                                (x) 2
                               v
                               tx       x                  x
                                    =        =       1
                                                       =p
                                  y     y       ( y) 2     y
                                                                                                  

As we simplify quotients with radicals, we will state all answers in a form where radicals are
confined to numerators, and radicands are fully simplified.

  DEFINITION 3.1.1. We say that the denominator is rationalized in a fraction if radicals
  appear in the numerator only, and radicands are simplified.


  EXAMPLE 3.1.1. Simplify and rewrite with rationalized denominator:
                                            1
                                           p
                                             3


  SOLUTION: The radicand is simplified, but it is in the wrong place. To get rid of the radical
  in the monomial
              p p denominator, we can multiply the expression by the radical over itself, in
  this case by 3/ 3:
                             p
               1        1      3
              p    = p ·p
                3        3     3
                           p
                        1· 3                        p p       p         p
                   = p p                      but 3 · 3 = 3 · 3 = 32 = 3
                         3· 3
                       p
                         3
                   =
                        3

                                                p
                                                 3
                                        ANSWER:
                                                3

                                              422
CHAPTER 8. RADICALS                              3. QUOTIENTS WITH SQUARE ROOTS

 EXAMPLE 3.1.2. Simplify and rewrite with rationalized denominator:
                                          2x
                                         p
                                           6x


 SOLUTION: As before, we can rationalize the denominator by multiplying both numerator
 and denominator by the radical we are trying to eliminate:
                              p
           2x           2x      6x
          p       = p ·p
            6x           6x     6x
                             p
                        2x · 6x
                  = p        p
                         6x · 6x
                           p
                       2x 6x                                           p
                  = p                                 distributivity of over ·
                          (6x)2
                          p
                       2x 6x
                  =                            common factors 2 and x cancel
                          6x
                       p
                         6x
                  =
                         3

                                           p
                                            6x
                                   ANSWER:
                                            3


 EXAMPLE 3.1.3. Simplify and rewrite with rationalized denominator:
                                        p
                                          20x 5
                                         p
                                           5x 3


 SOLUTION: Here it makes more sense to apply distributivity and simplify the fraction
 under the radical sign before rationalizing anything:
              p               v
                20x 5         t 20x 5
                                                                     p
               p         =           3
                                                    distributivity of over ÷
                 5x  3           5x
                              p
                         =      4x 2
                       =    2x


                                    ANSWER: 2x

                                         423
3. QUOTIENTS WITH SQUARE ROOTS                                      CHAPTER 8. RADICALS
   Homework 8.3.
                                                         v
Simplify and rewrite with rationalized de-               t 10 y 9
nominator.                                         14.
                                                           18 y 5
    p
     500                                                 v
1. p                                                     t 4x 3
       5                                           15.
                                                           50x
    p
     72
2. p
                                                         v
                                                         t 100
      2                                            16.
    p                                                       49
     50                                                p
3. p                                                    36
      2                                            17. p
    p                                                    6
     40                                                p
4. p                                                    18
     10                                            18. p
    p                                                   32
     55
5. p                                                   p
                                                         63 y 3
      5                                            19. p
    p                                                     7y
     18
6. p                                                   p
      3                                                 8x
                                                   20. p
    p                                                   2x
      5
7. p                                                   p
     20                                                 9b
                                                   21. p
    p                                                   3b
      2
8. p                                                   p
     18                                                 48x 5
                                                   22. p
   v                                                     3x 3
   t4
9.                                                     5
      25                                           23. p
     v                                                  7
     t9
10.                                                    2
       49                                          24. p
                                                        3
                                                       p
     v
     t 2a5
11.                                                     25
       50a                                         25. p
                                                         8
     v
     t 25                                              p
                                                        16
12.                                                26. p
       64                                               27
     v                                                 p
     t 6x 7                                              3
13.                                                27. p
       32x                                              50


                                             424
CHAPTER 8. RADICALS           3. QUOTIENTS WITH SQUARE ROOTS
    p                           p
     5                            27x 5
28. p                       33. p
     7                             3x
    p                           p
     3a                           21a9
29. p                       34. p
      32                           7a3
    p                           p
     2a                           35x 13
30. p                       35. p
      45                           5x 5
    p                           p
     6                            7b5
31. p                       36. p
     5                            28b
    p
      5
32. p
     18




                      425
 3. QUOTIENTS WITH SQUARE ROOTS                      CHAPTER 8. RADICALS
         Homework 8.3 Answers.

1. 10                                   19. 3 y
                                              p
3. 5                                    21.    3
     p                                         p
5.    11                                      5 7
                                        23.
                                               7
     1
7.                                             p
     2                                        5 2
                                        25.
                                               4
     2
9.                                            p
     5                                          6
                                        27.
          2                                   10
         a
11.                                           p
         5                                      6a
                                        29.
     p    3                                    8
    x 3
13.                                           p
     4                                          30
                                        31.
     p                                         5
    x 2
15.
     5                                  33. 3x 2
    p                                          p
17. 6                                   35. x 4 7




                                  426
CHAPTER 8. RADICALS                                         4. SUMS WITH SQUARE ROOTS
                                4. Sums with Square Roots

    4.1. Radical Like Terms. Adding terms with square roots is possible using the distributiv-
ity of multiplication over addition.

  DEFINITION 4.1.1. Terms involving radicals are like (or similar) if they have the same
  variable factors with the same respective exponents, and the same radical factors with
  the same respective radicands.


  BASIC EXAMPLE 4.1.1. Some pairs of like terms:

   p         p                                          p      p
  x 7,     4x 7                                     x    x, −6x x
  p         p                                            p            p
      x,   5 x                                      x 2 y 2ab, 10x 2 y 2ab


  BASIC EXAMPLE 4.1.2. Some pairs of terms which are not like:
    p     p
  y y,       y are not like because they have different exponents for y variable.
  p       p
    2x,     5x are not like because they have different radicands.
  p       p
    8, 3 2 are not like because they have different radicands, but they will be like if we
  simplify the radicands:
                               p     p        p p         p
                                 8= 4·2= 4· 2=2 2
    p         p
  2 2 and 3 2 are indeed like.


  EXAMPLE 4.1.1. Simplify the expression
                                     p   p  p
                                    4 6−5 2− 6


                       p       p
  SOLUTION: The terms 4 6 and − 6 are like, so we can combine them:
      p     p     p         p    p       p
     4 6−5 2− 6 = 4 6− 6−5 2                    terms can be added in any order
                                p       p
                       = (4 − 1) 6 − 5 2                          distributivity
                            p     p
                       = 3 6−5 2
  The remaining terms are not like, so we cannot simplify this expression any further.

                                               p      p
                                   ANSWER: 3       6−5 2

                                             427
4. SUMS WITH SQUARE ROOTS                                        CHAPTER 8. RADICALS

 EXAMPLE 4.1.2. Simplify the expression
                                       p      p
                                      8 18 − 7 2


 SOLUTION: These terms are not like, but the radicand 18 can be simplified:
                          p        p            p        p
                         8 18 − 7 2 = 8 9 · 2 − 7 2
                                                p p       p
                                          = 8 9 2−7 2
                                                  p      p
                                          = 8·3 2−7 2
                                                 p     p
                                          = 24 2 − 7 2
 These terms are like, so we can combine them:
                                p     p            p
                              24 2 − 7 2 = (24 − 7) 2
                                                p
                                            = 17 2


                                              p
                                    ANSWER: 17 2


 EXAMPLE 4.1.3. Simplify the expression
                                       p       p
                                  (3 + 7)(2 − 5 7)


 SOLUTION: We begin by multiplying the sums using the standard binomial pattern, where
 we multiply each term of the first sum by each term of the second sum, and then add all
 of these products together:
         p         p                     p    p        p     p
     (3 + 7)(2 − 5 7) = 3 · 2 − 3 · 5 7 + 7 · 2 − 7 · 5 7
                                     p      p      p p
                          = 6 − 15 7 + 2 7 − 5 7 7                  simplified products
                                     p      p                                 p 2
                          = 6 − 15 7 + 2 7 − 5 · 7                  because     7 =7
                                     p      p
                          = 6 − 15 7 + 2 7 − 35
                                                                       p         p
 We have two pairs of like terms here: the two integers as well as −15 7 and 2 7.
                          p       p                          p      p
                    6 − 15 7 + 2 7 − 35 = 6 − 35 − 15 7 + 2 7
                                                                  p
                                             = −29 + (−15 + 2) 7
                                                           p
                                             = −29 − 13 7


                                                p
                                ANSWER: −29 − 13 7

                                          428
CHAPTER 8. RADICALS                                          4. SUMS WITH SQUARE ROOTS
     4.2. Rationalizing Sums. A sum with two terms can be rationalized if it is multiplied by
its conjugate, which is the sum of the first term and the opposite of the second term.

  DEFINITION 4.2.1. For all real numbers A, B, C, D, the sums
                                        p       p
                                       A B+C D
  and                                     p       p
                                         A B−C D
  are the radical conjugates (or simply conjugates) of each other.


  THEOREM 4.2.1 (Rationalizing the Denominator). In a fraction, a sum with two terms in
  the numerator or the denominator can be rationalized by multiplying both the numerator
  and the denominator by the conjugate of that sum.



   PROOF. We will
                p provepthis statement for a sum of
                                                 p square
                                                       p roots in the denominator. If the
denominator is a b + c d, then its conjugate is a b − c d.
                                                      p     p
                           1                   1     a b−c d
                       p       p    =      p     p · p      p
                      a b+c d            a b+c d a b−c d
                                                  p    p
                                                 a b−c d
                                    =       p     p    p     p
                                         (a b + c d)(a b − c d)

Notice that denominators are sums, and we had to show the invisible fraction parentheses
when we multiplied the denominators. Notice also that the denominator now is a difference
of squares, and can be simplified:
                            p      p                    p        p
                          a b−c d                     a b−c d
                     p      p      p   p      =       p 2        p 2
                   (a b + c d)(a b − c d)           a b − c d
                                                         p        p
                                                        a b−c d
                                              =        p 2         p 2
                                                   a2    b − c2 d
                                                    p        p
                                                   a b−c d
                                              =
                                                    a2 b − c 2 d

The new denominator is free of square roots now, even though they moved into the numerator.

The case with the difference of square roots, as well as the cases with rationalizing the numer-
ator have similar proofs.                                                                     




                                              429
4. SUMS WITH SQUARE ROOTS                                            CHAPTER 8. RADICALS

 EXAMPLE 4.2.1. Rationalize the denominator:
                                          6
                                           p
                                       3+ 5

                               p        p
 SOLUTION: The conjugate of 3 + 5 is 3 − 5:
                               p
        6            6     3− 5
         p   =        p ·      p
     3+ 5         3+ 5 3− 5
                            p
                      6(3 − 5)
             =         p       p        difference of squares in the denominator
                  (3 + 5)(3 − 5)
                         p
                  6(3 − 5)
             =           p 2
                  32 − 5
                         p
                  6(3 − 5)
             =                                          simplify the denominator
                     9−5
                         p
                  6(3 − 5)
             =                                            cancel common factor 2
                       4
                         p
                  3(3 − 5)
             =
                       2

                                                p
                                          3(3 − 5)
                                  ANSWER:
                                              2


 EXAMPLE 4.2.2. Rationalize the denominator:
                                         5x
                                      p     p
                                        x− y


                              p        p          p        p
 SOLUTION: The conjugate of       x−       y is       x+       y:
                                                           p   p
                           5x                        5x      x+ y
                        p     p        =          p     p ·p   p
                          x− y                      x− y     x+ y
                                                         p   p
                                                      5x( x + y)
                                       =           p    p p     p
                                                  ( x − y)( x + y)




                                                  430
CHAPTER 8. RADICALS                                       4. SUMS WITH SQUARE ROOTS
 The denominator is now a difference of squares, so we can simplify it:
                             p     p                    p     p
                         5x( x + y)                  5x( x + y)
                      p    p p         p      =      p 2    p 2
                     ( x − y)( x + y)                  x −     y
                                                        p    p
                                                    5x( x + y)
                                              =
                                                        x−y

                                              p   p
                                           5x( x + y)
                                ANSWER:
                                                 x−y




                                           431
4. SUMS WITH SQUARE ROOTS                                  CHAPTER 8. RADICALS
  Homework 8.4.
                                              p        p
Simplify the expression.             23. 4x       x− x3
    p      p      p                           p
1. 2 5 + 2 5 + 2 5
                                                    p
                                     24.   2 y y + 3 y3
      p       p      p                      p        p
2. −3 6 − 3 3 − 2 3                  25.   ( 5 − 5)(2 5 − 1)
      p      p      p                           p          p
3. −3 2 + 3 5 + 3 5                  26.   (−2 + 3)(−5 + 2 3)
      p     p      p
                                            p                p
4. −2 6 − 3 − 3 6
                                                 p     p
                                     27.   ( 2 + 6)(2 6 − 3 2)
                                              p   p     p    p
      p       p    p
5. −2 6 − 2 6 − 6                    28.   (4 5 − 20)( 20 − 5)
      p      p      p
6. −3 3 + 2 3 − 2 3
    p      p      p
7. 3 5 + 3 5 + 2 5                   Rationalize the denominator and simplify.
    p       p      p
8. − 5 + 2 3 − 2 3                           4
                                     29.      p
    p      p      p                        3+ 5
9. 2 2 − 3 18 − 2
                                            −4
      p       p       p              30.      p
10. − 54 − 3 6 + 3 27                      4−4 2
       p      p       p                    1
11. −3 6 − 12 + 4 3                  31. p
      p     p      p                      2+1
12. − 5 − 5 − 2 45
                                           3
     p       p      p                32. p
13. 3 2 + 2 8 − 3 18                      6−2
     p        p      p
14. 2 20 + 2 20 − 3                          2
                                     33.      p
     p       p      p                      5+ 2
15. 3 18 − 2 − 3 2
                p     p                       3
       p                             34.      p
16. −3 27 + 2 3 − 12                       4−3 3
       p       p     p   p
17. −3 6 − 3 6 − 3 + 3 6                   5
                                     35. p   p
       p     p      p    p                3+4 5
18. −2 2 − 2 + 3 8 + 3 6
                                              5
       p        p     p    p         36.    p   p
19. −2 18 − 3 8 − 20 + 2 20                2 3− 2
       p      p       p   p
20. −3 18 − 8 + 2 8 + 2 8                    x−y
                                     37. p     p
    p         p                              x− y
21. 300x − 3x
    p       p                                 2
                                     38.    p   p
22. 2x + 50x                               2 5+2 3

                               432
CHAPTER 8. RADICALS                           4. SUMS WITH SQUARE ROOTS
   Homework 8.4 Answers.
    p                                 p
1. 6 5                           21. 9 3x
     p     p                              p
3. −3 2 + 6 5                    23. 3x       x
     p                                      p
5. −5 6                          25. 15 − 11 5
    p                                     p
7. 8 5                           27. 6 − 2 3
     p                                     p
9. −8 2                          29. 3 −    5
      p     p                        p
11. −3 6 + 2 3                   31.  2−1
      p                                    p
13. −2 2                             2(5 − 2)
                                 33.
     p                                   23
15. 5 2                                  p    p
          p                          −5( 3 − 4 5)
      p                          35.
17. −3 6 − 3                               77
       p     p                       p     p
19. −12 2 + 2 5                  37. x + y




                           433
5. EQUATIONS WITH SQUARE ROOTS                                         CHAPTER 8. RADICALS
                                5. Equations with Square Roots

    5.1. Squaring Both Sides. Some equations involving principal square roots can be solved
by squaring both sides. This operation does not always produce equivalent equations, and will
create extraneous solutions every now and then.

  THEOREM 5.1.1. If A = B, then the solution set of the equation A2 = B 2 includes all of the
  solutions of A = B, but not necessarily the other way around. This means we can solve
  the original equation by solving A2 = B 2 , and then discarding extraneous solutions, if any.


  BASIC EXAMPLE 5.1.1. It is easy to see that squaring both sides may create additional
  solutions. The solution set of the equation
                                               x =1
  is {1}, while the solution set of
                                           (x)2 = (1)2
  which is equivalent to
                                               x2 = 1
  is {1, −1}.


  EXAMPLE 5.1.1. Determine which of the given numbers are solutions for the equation:
                                            p
                                    x −5= x +7
                                               2,   9


  SOLUTION: If x = 2 then
                                                      p
                                       2−5      =         2+7
                                         −3     =     3
  This is false, so 2 is not a solution, but an extraneous solution.

  If x = 9 then
                                                      p
                                       9−5      =         9+7
                                           4    =     4
  This is true, so 9 is a solution.


                                         ANSWER: {9}




                                                434
CHAPTER 8. RADICALS                                       5. EQUATIONS WITH SQUARE ROOTS
                                             p
 EXAMPLE 5.1.2. Solve the equation               x −6−3=0


 SOLUTION: We want to solve the equation for the term with the radical, or else squaring
 both sides will be pointless.
                 p
                   x −6−3 = 0
                    p
                       x −6 = 3                    added 3 to both sides
                 €p       Š2
                     x −6      = (3)2
                       x −6     =        9
 This looks like a linear equation, which we solve by isolating the variable:
                  x −6      =   9
                       x    =   9+6                       added 6 to both sides
                       x    =   15
 The solution checks out:
                                p                  p
                                    15 − 6 − 3 =       9−3=3−3=0


                                             ANSWER: {15}

                                        p
 EXAMPLE 5.1.3. Solve the equation 6 + 2 x − 4 = 0


 SOLUTION: We start by solving for the term with the radical:
             p
         6+2 x −4 = 0                                 subtract 6 on both sides
             p
            2 x − 4 = −6                     divide both sides by 2 to simplify
             p
               x − 4 = −3                                     square both sides
           p       2
             x −4       = (−3)2
                  x −4      =       9                             add 4 to both sides
                       x    =       13
 Check the solution:
                                              p
                                         6 + 2 13 − 4    =   0
                                                 p
                                              6+2 9      =   0
                                                 6+2·3   =   0
                                                    12   =   0

                                                   435
5. EQUATIONS WITH SQUARE ROOTS                                             CHAPTER 8. RADICALS
  This is false, so 13 is an extraneous solution, and since there are no others, the solution
  set is empty.


                                            ANSWER: ∅


In the previous example, we could have saved ourselves some time if we used the fact that the
principal square root is never negative. We could have concluded there are no solutions from
                                          p
                                         2 x − 4 = −6
which is impossible to satisfy, since the left side is non-negative, while the right side is negative.

  THEOREM 5.1.2. The principal square root is never negative. For any real number b,
                                  p
          • if b is positive,pthen b > 0
                                  p
          • if b = 0, then b = 0 = 0
          • if b is negative, then the principal square root of b is not real, but complex.


  EXAMPLE 5.1.4. Solve the equation
                                          p
                                              x2 − 4 + 4 = 1


  SOLUTION: We start by solving for the term with the radical:
                                 p
                                    x2 − 4 + 4 = 1
                                      p
                                        x 2 − 4 = −3
  We notice that the left side is non-negative, while the right side is negative, so there are
  no solutions possible.


                                            ANSWER: ∅


  EXAMPLE 5.1.5. Solve the equation
                               p          p
                                  3 − 4x − −7x − 6 = 0


  SOLUTION: The left side is a sum of two terms, and squaring a sum should be avoided
  when possible. But if we isolate the radicals, one on each side of the equation, then we




                                                 436
CHAPTER 8. RADICALS                                       5. EQUATIONS WITH SQUARE ROOTS
 can get rid of both of them at the same time:
     p           p
       3 − 4x − −7x − 6 = 0
                   p               p                                   p
                     3 − 4x =         −7x − 6                   add        −7x − 6 to both sides
                €p        Š2       €p          Š2
                   3 − 4x     =        −7x − 6                                 square both sides
                    3 − 4x    =    −7x − 6                       squares cancel square roots
                        3x    =    −9                      isolate the term with x on the left
                          x   =    −3
 Check whether −3 is a solution:
               p           p                      Æ            Æ
                 3 − 4x − −7x − 6           =       3 − 4(−3) − −7(−3) − 6
                                                  p         p
                                            =      3 + 12 − 21 − 6
                                                  p      p
                                            =      15 − 15
                                            =     0
 The equation holds, so −3 is the only solution.


                                      ANSWER: {−3}


 EXAMPLE 5.1.6. Solve the equation
                                                 p
                                   −8 = x −          −2x − 16


 SOLUTION: Isolate the radical on the left before squaring both sides:
                                                   p
                                    −8 = x − −2x − 16
                          p
                             −2x − 16 = x + 8
 Square both sides now, but note that the right side is a sum of two terms, and will have to
 be squared as a binomial:
                           €p           Š2
                              −2x − 16        = (x + 8)2
                                  −2x − 16       =      x 2 + 16x + 64
 This is a quadratic equation, so we will move all non-zero terms to one side, factor, and
 solve using the zero factor property:
                    −2x − 16 + 2x + 16       =        x 2 + 16x + 64 + 2x + 16
                                        0    =        x 2 + 18x + 80
                                        0    =       (x + 8)(x + 10)



                                                437
5. EQUATIONS WITH SQUARE ROOTS                                     CHAPTER 8. RADICALS
 By the zero product property, this quadratic equation has solutions x = −8 and x = −10.
 We have to check them before concluding they are also solutions for the equation with
 radicals.

 If x = −8 then the right side of the original equation is
                           p                           Æ
                       x − −2x − 16 = (−8) − −2(−8) − 16
                                                     p
                                           = −8 − 16 − 16
                                                     p
                                           = −8 − 0
                                            =     −8
 This is equal to the left side, so −8 is a solution.

 If x = −10 then the right side of the original equation is
                         p                             Æ
                     x − −2x − 16 = (−10) − −2(−10) − 16
                                                     p
                                         = −10 − 20 − 16
                                                     p
                                         = −10 − 4
                                           =     −10 − 2
                                           =     −12
 This is not equal to the left side, so −10 is not a solution, although it is an extraneous
 solution.


                                        ANSWER: {−8}




                                                438
CHAPTER 8. RADICALS             5. EQUATIONS WITH SQUARE ROOTS
   Homework 8.5.
                                    p
Solve the equation.           14.      y + 18 = y − 2
   p                                          p
1. x = 7                      15.   x +4=4 x +1
   p                                     p
2. x − 7 = 2                  16.   1+2 a−1= a
   p
3. x − 3 = 9
                                    p
                              17.     x2 + 6 = x − 3
   p                                p
4. 4 − x = 0                  18.     5x + 21 = x + 3
   p                                        p
5. 3x + 1 = 8                 19.   x =1+ 1− x
   p                                     p
6. 5x − 1 = 7                 20.   x = x +5+7
    p                                p      p
7. 4 x = −3                   21.   3 x = x + 32
       p                             p         p
8. 1 − x = 2                  22.   2 x − 7 = 5x
   p          p                     p
9. 8x + 3 = 6x + 7            23.     x +5= x −1
    p           p                   p          p
10.    7b − 9 = b + 3         24.     5x + 3 = 2x − 1
    p                                       p
11. 3a + 1 = a − 3            25.   1 + x = 1 + 5x
            p                       p
12. b − 7 = b − 5             26.     x +2−2= x
            p
13. x − 9 = x − 3




                        439
5. EQUATIONS WITH SQUARE ROOTS                      CHAPTER 8. RADICALS
    Homework 8.5 Answers.

1. {49}                                15. {0, 8}

3. {144}                               17. ∅
5. {21}
                                       19. {1}
7. ∅
                                       21. {4}
9. {2}

11. {8}                                23. {4}

13. {12}                               25. {0, 3}




                                 440
CHAPTER 8. RADICALS                                         6. APPLICATIONS TO RIGHT TRIANGLES
                             6. Applications to Right Triangles
   6.1. Distance Formula.

  DEFINITION 6.1.1 (Euclidean Distance). On a coordinate plane, the distance d between
  points with coordinates (x 1 , y1 ) and (x 2 , y2 ) is

                                      Æ
                                 d=       (x 2 − x 1 )2 + ( y2 − y1 )2




                        y2
                                                    d
                                                                            | y2 − y1 |
                   y




                        y1
                                                  |x 2 − x 1 |


                                       x1                              x2
                                                        x


  This definition is in full agreement with the Pythagorean theorem. Applied to the high-
  lighted right triangle pictured above, the theorem states
                                  d 2 = (x 2 − x 1 )2 + ( y2 − y1 )2
  If we take the principal square root of both sides of the equation, we will get our definition
  of the distance.



Given a right triangle with two known sides, we can use the Pythagorean theorem to solve for
the length of the remaining side.


  EXAMPLE 6.1.1. Given the lengths of two
  sides of a pictured right triangle with
  legs a and b and hypotenuse c, find the
                                                                                      c
  length of the remaining side.                                  a

               a = 8,    b=4
                                                                                  b


                                                 441
6. APPLICATIONS TO RIGHT TRIANGLES                                           CHAPTER 8. RADICALS

  SOLUTION: We state the Pythagorean theorem for this triangle and solve for the unknown
  variable:
              a2 + b2   =    c2
              82 + 42   =    c2                             substitute known lengths
                 80     =    c2               take principal square root of both sides
                p
                 80     =    c
                                                                                 p
  It is true that there is another solution to this quadratic equation, namely − 80, but
  since there are no negative lengths, the negative solution is not applicable and can be
  disregarded. We will state the answer with simplified radicand:
                                               p
                                      c =        80
                                               p
                                      c =        16 · 5
                                               p      p
                                      c =        16 · 5
                                                  p
                                      c = 4· 5


                                                  p
                                         ANSWER: 4 5


  EXAMPLE 6.1.2. Find the distance between the points with coordinates
                                     (4, 7)     and    (−5, 4)


  SOLUTION: Let (x 1 , y1 ) = (4, 7), (x 2 , y2 ) = (−5, 4), and use the distance formula:
                  Æ
          d =          (−5 − 4)2 + (4 − 7)2               simplify sums before squaring
                  Æ
             =         (−9)2 + (−3)2
                  p
             =         81 + 9
                  p
             =         90
  Simplify the radicand for the answer:
                 p           p
                   90 =        9 · 10
                             p p                                             p
                        =      9 · 10                    distributivity of       over ·
                                p
                        = 3 · 10


                                                 p
                                        ANSWER: 3 10


                                                 442
CHAPTER 8. RADICALS                                  6. APPLICATIONS TO RIGHT TRIANGLES
   Homework 8.6.

                                                     7. a = 18, c = 30
                                                                   p
                                                     8. b = 1, c = 2
                        c                                   p        p
        a                                            9. a = 2, b = 6

                                                     10. a = 5, b = 5
                    b

                                                     Find the distance between the given points.
Given the lengths of two sides of a pictured
straight triangle with legs a and b and hy-          11. (2, 3) and (6, 10)
potenuse c, find the length of the remain-
ing side.                                            12. (0, 3) and (4, 0)

1. a = 13, b = 5                                     13. (−3, 2) and (−1, 5)

2. a = 9, c = 15                                     14. (−2, 4) and (−8, −4)
              p
3. b = 1, c = 10                                     15. (0, −5) and (−12, 0)
              p
4. a = 1, c = 3                                      16. (2, −7) and (1, −4)
                 p
5. c = 10, b = 5 3                                   17. (−2, −8) and (6, 7)

6. a = 24, b = 10                                    18. (1, 0) and (7, 3)




                                               443
6. APPLICATIONS TO RIGHT TRIANGLES                        CHAPTER 8. RADICALS
    Homework 8.6 Answers.

1. c = 13
                                                 p
                                           11.       65
3. a = 3                                         p
                                           13.    13
5. a = 5
                                           15. 13
7. b = 26
        p
9. c = 2 2                                 17. 17




                                     444
CHAPTER 8. RADICALS                                                    7. RATIONAL EXPONENT
                                     7. Rational Exponent



                                            3   y

                                                                        2
                                                                y = (x) 3
                                            2



                                            1



                                                                                    x
              −3       −2        −1                         1               2           3



                                           −1


  7.1. Rational Exponent Definition.

 DEFINITION 7.1.1 (Integer Reciprocal Exponent). If b is a real number, and n is a positive
 integer, then the exponential expression (b)1/n is defined as follows:
                                 1
                             (b) n = y,      where         yn = b
 If n is even, then the positive y is chosen. Notationally, these are equivalent:
                                            1      p
                                                   n
                                         (b) =
                                            n
                                                       b


 BASIC EXAMPLE 7.1.1. Some easy-to-find exponents:
     1
 (25) 2 = 5 because 52 = 25.
     1
 (16) 4 = 2 because 24 = 16.
      1
 (−8) 3 = −2 because (−2)3 = −8.
          1
 (0.25) 2 = 0.5 because (0.5)2 = 0.25.
                   1
 (−0.00001) 5 = −0.1 because (−0.1)5 = −0.00001

                                             445
7. RATIONAL EXPONENT                                                          CHAPTER 8. RADICALS

                                 1
  EXAMPLE 7.1.1. Find (225)− 2


  SOLUTION: All properties of exponent still apply, so
                                                        ‹1
                                                     1 2
                                                 
                                      − 12
                                 (225)       =
                                                    225
                                                                     1
                                                       =                  1
                                                             (225) 2
                                                              1
                                                       =
                                                             15


                                                              1
                                              ANSWER:
                                                             15


  DEFINITION 7.1.2 (Rational Exponent). If b is a non-negative real number, and m/n is a
  quotient of an integer and a positive integer in lowest terms, then the exponential expres-
           m
  sion (b) n is defined as follows:
                                               m
                                                        €        1
                                                                     Šm
                                          (b) = (b)
                                               n                 n


  Note that all of these are equivalent:
                        m
                             €       1
                                         Šm              €p
                                                          n
                                                             Šm p
                                                             1  n
                    (b) = (b)
                        n            n        = ((b) ) =m
                                                            b = bm
                                                             n


  Several of the integer exponent properties stop working when we allow the base b to
  be negative, so we are assuming throughout this chapter that all variable bases are non-
  negative.


  THEOREM 7.1.1. Assuming that all bases are non-negative, the rational exponent obeys
  all the properties of the integer exponent.


Recall the properties of the integer exponent we have seen so far:

      • Distributivity over multiplication:
                                              (ab)n = a n b n
      • Distributivity over division:
                                                a n        an
                                                        =
                                                   b         bn
                                                       446
CHAPTER 8. RADICALS                                                                                        7. RATIONAL EXPONENT
     • The product rule:
                                                      b m b n = b m+n
     • The quotient rule:
                                                         bm
                                                            = b m−n
                                                         bn
     • The power rule:
                                                      (b m )n = b mn
     • Negative exponent properties:
                                                              ‹n
                                                              1     1
                                             b   −n
                                                           =      = n
                                                              b    b

 EXAMPLE 7.1.2. Simplify and state the answer with positive exponents only:
                                                                        6
                                                                             13
                                                                    a


 SOLUTION:
                                                      13                                        1
                                             a   6
                                                                     =                 (a)6· 3
                                                                     =                 a2


                                                     ANSWER: a2


 EXAMPLE 7.1.3. Simplify and state the answer with positive exponents only:
                                             €       1
                                                                     Š 32               5
                                                                −1
                                                 x y 3                           · x 2 y2


 SOLUTION:
        €    1
                       Š 23     5
                                                 €          1
                                                                Š 32                    32          5
                  −1
            x y
             3                ·x y
                                2    2
                                         =            x     3                y −1               · x 2 y2     distributivity
                                                       1 3                    3             5
                                         =       x 3 · 2 y −1· 2 · x 2 y 2                                     power rule
                                                       1             3             5
                                         =       x 2 y−2 · x 2 y2
                                                       1        5            3
                                         =       x 2 + 2 y − 2 +2
                                                                 1
                                         =       x3 y 2

                                                                    447
7. RATIONAL EXPONENT                                                                         CHAPTER 8. RADICALS

                                                                                1
                                                   ANSWER:             x3 y 2

 EXAMPLE 7.1.4. Simplify and state the answer with positive exponents only:
                                                         13
                                                        a 2 b0 a−3
                                                               5       1
                                                         b− 7 a 4

 SOLUTION: We can use the product and the quotient rules add and subtract corresponding
 exponents:
         13
        a 2 b0 a−3
                                 a 2 +(−3)− 4 b0−(− 7 )
                                  13                1              5
                                                                                    13     13 6   13 − 6   7
                             =                                                         −3=    − =        =
              − 57       1
                                                                                     2      2  2    2      2
         b           a   4

                                  7    1       5
                                                                                    7 1  14 1   14 − 1   13
                             =   a2−4 b7                                             − =    − =        =
                                                                                    2 4   4  4    4       4
                                  13       5
                             =   a 4 b7

                                                                           13   5
                                                   ANSWER:          a 4 b7




                                                              448
CHAPTER 8. RADICALS                                                                     7. RATIONAL EXPONENT
      Homework 8.7.

Write the expression in radical form.             Simplify and state the answer with positive
                                                  exponents only.
                   4
1.   (2m) 5                                                     1                   3

                       3
                                                  17.   yx3 · x y2
2.   (10r) 4                                                    5

                   3
                                                  18.4v 3 · v −1
3.   (7x) 2                                          € 1 1 Š−1
                   4
                                                  19. a 2 b 2
4.   (6b) 3
                                                        €       5
                                                                                Š0
                                                                        −2
                                                  20.       x y 3




Write the expression in exponential form.              ab3
                                                  21.
     p
     6
                                                      3ab0
5.       v                                                          1       1
   p                                                     2x 2 y 3
6.   5a                                           22.           4            7

   Æ                                                    2x 3 y − 4
7.   (4x)5                                              €       1       3
                                                                            Š 23             2         5
   p
   3
                                                  23.       x y 2       4           · x 3 y−2
8.   c4
                                                        €                                      Š− 12
                                                                3 − 13                  − 73
                                                  24.       g h                  ·g
                                                               € 2 Š3
Evaluate the expression.                          25.   u2 · uv v 3
       2
9.   83
                                                  26.   (x · x y 2 )0
               1
10.   16 4                                              €               1
                                                                            Š 32
                                                                0
           5                                      27.       x y         3           x0
11.   4    2

                                                                                €           Š− 32
           3                                                − 54 2                      2
12.   9    2                                      28.   u           v · u               3


                   1
13.   25− 2                                                 3
                                                      a 4 b−1 · b 4
                                                                                    7


                   3
                                                  29.
14.   16− 2                                               3b−1
                                                                                        5

15.   (−8)             − 43                                     2x −2 y 3
                                                  30.           5            5                   1
                                                        x−4 y−3 · x y 2
                           − 13
16.   (−27)
                                            449
7. RATIONAL EXPONENT                                            CHAPTER 8. RADICALS
         1                    5                                47
      a b 3 · 2b− 4                                 3
                                                        −2
31.                                               m n
                                                    2

            − 12       − 23             33.
       4a          b                              m−1 n− 3
                                                          4


                   5
         3 y−4                                     1
                                                         −2
                                                               23
32.                           1                   y y
                                                    3

      y −1 · 2 y − 3                    34.          5    9
                                                  x−2 y−2




                                  450
 CHAPTER 8. RADICALS                                         7. RATIONAL EXPONENT
         Homework 8.7 Answers.

                                             b
     p
     5
1.  16m4                               21.
   p                                         3
3.   343x 3
                                             x
         1                             23.
5.   v   6
                                             y2
                 5
7.   (4x) 2
                                       25.   u3 v 3
9.   4
                                                 1
                                       27.   y2
11.   32
         1                                   1 7 3
13.                                    29.     b4 a4
         5                                   3
          1                                          3
15.                                           a2
         16                            31.               1
             4       5                       2b 4
17.      x y 3       2

                                                 35
             1                               m8
19.          1   1
                                       33.       7
         a2 b2                                n6




                                 451
7. PRACTICE TEST 8                                                                   CHAPTER 8. RADICALS
                                     Practice Test 8




Simplify each expression, assuming that           Solve each equation:
all variable bases and radicands are non-             p          p
negative:                                         15. 2x + 1 = x + 2
    p                                                 p
1. 121                                            16. 5x + 1 − 6 = 0
                                                              p
                                                  17. x − 2 = 2x − 4
    v
    t4
2.
      9                                               p
                                                  18. 2x − 5 + 4 = x
     p
     3
3.     −8
     p
4.   x 2018
                                                  19. Find the length of the leg b of a
5. ( x − y)2
    p
                                                  straight triangle, if the other leg a is 6 cm,
   p                                              and the hypotenuse c is 14 cm.
   4
6.   81
                                                  20. Find the distance between points
                                                  (−5, −9) and 5, −3.
   p
   5
7.   x 15
     p
8.       72x 5
     p     p
9.  3 y 7 · 27 y 3
                                                  Evaluate the expression:
    p
      7x 7                                                          3
10. p                                             21.   100 2
     28x
                                                                               2
                                                        (−64)− 3
    v
    t 54x                                         22.
11.
       3y
      p    p   p
12.    12(3 2 − 8)
                                                  Simplify the expression and state the an-
                                                  swer with positive exponents only:
Rationalize the denominator and simplify:
                                                        €       1
                                                                               Š2
                                                                        − 12             5

13.
      6
       p                                          23.       x y 3                   · x − 3 y −2
    3+ 7
                                                            1           1
    p   p
     2+ 3                                               a 2 b− 5
14. p   p                                         24.       1           11
     6− 5                                               a 3 b− 5


                                            452
 CHAPTER 8. RADICALS                                         7. PRACTICE TEST 8
         Practice Test 8 Answers.
                                                   p
1. 11                                     13. 9 − 3 7
     2                                          p      p   p      p
2.                                        14.    10 + 2 3 + 15 + 3 2
     3
3. −2                                     15. {1}

4. x 1009                                 16. {7}

5. x − y                                  17. {2, 4}

6. 3                                      18. {7}
                                               p
7. x 3                                    19. 4 10
       p                                          p
8. 6x 2 2x                                20. 2       34

9. 9 y 5                                  21. 1000

10. x 3 /2                                22. 1/16
             p
         3       2x y                            1
11.                                       23.
             y                                  x y3
                                                  1
                                                a 6 b2
         p
12. 2        6                            24.




                                    453
                                         CHAPTER 9


                                  Quadratic Equations


                                    1. Complex Numbers

   1.1. Imaginary Unit. Complex numbers were initially designed to assist in factoring poly-
nomials around 1545 by Gerolamo Cardano. It was observed that an equation such as
                                            x 2 = −1
which is provably without a real solution, can be postulated to have a solution which is not a
real number, and nothing goes wrong as a result. In fact, one could say the situation improves
somewhat. It becomes possible to construct a set of complex numbers, which includes all real
numbers as well as all numbers which happen to be solutions of polynomial equations with
real or complex coefficients. All of the real number axioms hold true for the complex numbers,
so most theorems presented in this text hold for the complex numbers almost word for word.

  DEFINITION 1.1.1 (Imaginary Unit). The solutions of the equation
                                            x 2 = −1
  are complex numbers i and its opposite −i:
                                        i 2 = (−i)2 = −1
  i is usually called the imaginary unit, and it happens to be the principal square root of the
  negative unit:                            p
                                               −1 = i


  THEOREM 1.1.1. If x is a positive real number, then the principal square root of −x is
                                        p     p
                                          −x = x · i

                                            p
  EXAMPLE 1.1.1. Simplify the expression     −100


  SOLUTION:                        p      p
                                    −100 = 100 · i = 10i


                                         ANSWER: 10i

                                              455
1. COMPLEX NUMBERS                                       CHAPTER 9. QUADRATIC EQUATIONS
                                           p
  EXAMPLE 1.1.2. Simplify the expression    −20


  SOLUTION:                        p     p          p
                                    −20 = 20 · i = 2 5 · i

                                                p
                                       ANSWER: 2 5 · i



   1.2. The Set of Complex Numbers. Just like the set of real numbers can be visualized as
a number line, the set of complex numbers can be visualized as a number plane.

                                             5i
                                                  Imaginary
                                             4i
                                             3i
                                                                  3 + 2i
                                             2i
                                              i
                                                                    Real
                        −5 −4 −3 −2 −1               1    2   3     4      5
                                     −i
                                           −2i
                                           −3i
                                           −4i
                                           −5i

This plane works a lot like the Cartesian plane, but the coordinates are interpreted in a par-
ticular way. Every point on this plane represents a distinct complex number. The points on
the real axis are the familiar real numbers. The imaginary unit i is located one unit above the
origin, and the points on the imaginary axis are the real multiples of i. Every other point with
coordinates (a, b), where a and b are real numbers, is a complex number which can be written
as a + bi.

  DEFINITION 1.2.1. A complex number is any number z which can be written in the form
                                            a + bi
  where a is a real number called the real part of z, b is a real number called the imaginary
  part of z, and i is the imaginary unit.


                                              456
CHAPTER 9. QUADRATIC EQUATIONS                                          1. COMPLEX NUMBERS
  1.3. Adding and Multiplying Complex Numbers.

 EXAMPLE 1.3.1. Simplify the expression (3 + 5i) − (2 + 7i)


 SOLUTION: We treat i as if it was a variable, and combine the like terms:
          (3 + 5i) − (2 + 7i)   =     3 + 5i − 2 − 7i           removed parentheses
                                =     1 − 2i                     combined like terms


                                       ANSWER: 1 − 2i


 EXAMPLE 1.3.2. Simplify the expression (1 + 6i)(2 − i)


 SOLUTION: We treat i as if it was a variable, multiply the binomials, and combine the like
 terms:
         (1 + 6i)(2 − i)   =    2 − 1i + 12i − 6i 2
                           =    2 + 11i − 6i 2                i 2 can be replaced by −1
                           =    2 + 11i − 6(−1)
                           =    2 + 11i + 6
                           =    8 + 11i                           combined like terms


                                      ANSWER: 8 + 11i

                                                 p
                                            8+     −16
 EXAMPLE 1.3.3. Simplify the expression
                                                 4


 SOLUTION: We simplify the radicand, and then cancel common factors:
              p
           8 + −16           8 + 4i
                        =                              rewrite in terms of i
               4               4
                                4(2 + i)
                           =                                     factored out GCF
                                   4
                           =    2+i                      canceled common factor 4


                                       ANSWER: 2 + i

                                               457
1. COMPLEX NUMBERS                                    CHAPTER 9. QUADRATIC EQUATIONS
                                               p
                                          20 − −50
 EXAMPLE   1.3.4. Simplify the expression
                                              10


 SOLUTION:
              p                     p
         20 − −50             20 −     50 · i
                           =                                    rewrite in terms of i
             10                     10
                                    p
                               20 − 25 · 2 · i
                           =                                   simplify the radicand
                                     10
                                    p p
                               20 − 25 2 · i                                 p
                           =                               distributivity of over ·
                                      10
                                      p
                               20 − 5 2 · i                                 p
                           =                                                  25 = 5
                                    10
                                     p
                               5(4 − 2 · i)
                           =                                       factored out GCF
                                    10
                                   p
                               4− 2·i
                           =                             canceled common factor 5
                                   2
 If desired, this expression could be stated in the form a + bi by rewriting the fraction as a
 sum:
                                      p                  p
                                  4− 2·i             4     2
                                                =      −     i
                                       2             2    2
                                                         p
                                                          2
                                                = 2−        i
                                                         2
 But we will be content with simplified fractions for answers:

                                                    p
                                               4−     2·i
                                     ANSWER:
                                                    2




                                             458
CHAPTER 9. QUADRATIC EQUATIONS                                           1. COMPLEX NUMBERS
   Homework 9.1.

Simplify the given expression and state the         16. (3 − i)(4 + i)
answer in the form bi, where b is real.
   p                                                17. (10 − 3i)(2i)
1. −5
   p                                                18. (−5i)(2 + 4i)
2.   −6
   p                                                19. 30(−2 + 0.5i)
3.   −49
                                                          2
   p                                                20.     (24 − 30i)
4. −81                                                    3
   p                                                21. (7 + 5i)(7 − 5i)
5. −45
   p                                                22. (3 − 6i)(3 + 6i)
6. −18
   p                                                23. (4 + i)2
7. −120
   p                                                24. (1 − 3i)2
8.   −160
                                                              p
                                                        −6 + −4
                                                    25.
                                                             3
Simplify the given expression.                                p
                                                        15 − −9
                                                    26.
9. (7 + 3i) + (14 + 2i)                                      5
                                                              p
10. (12 − 4i) + (−1 + 6i)                               18 + −8
                                                    27.
                                                            −2
11. (21 − 8i) − (5 + 5i)                                        p
                                                        −10 + −25
                                                    28.
12. (−10 − i) − (2 − 4i)                                     −10
                                                                p
13. (2 − 3i) − (9 − 3i)                                 −24 − −27
                                                    29.
                                                              −6
14. (6 − 11i) + (17i − 6)                                     p
                                                        −4 + −28
15. (1 + i)(1 − 2i)                                 30.
                                                              6




                                              459
 1. COMPLEX NUMBERS                    CHAPTER 9. QUADRATIC EQUATIONS
      Homework 9.1 Answers.
     p
1.    5·i                           17. 6 + 20i

3. 7i                               19. −60 + 15i
    p
5. 3 5 · i                          21. 24
    p
7. 2 30 · i                         23. 15 + 8i

9. 21 + 5i                                  2
                                    25. −2 + i
                                            3
11. 16 − 13i                                   p
                                    27. −9 −    2·i
13. −7                                      p
                                             3
15. 3 − 2i                          29. 4 +    i
                                            2




                              460
CHAPTER 9. QUADRATIC EQUATIONS                                  2. SQUARE ROOT PROPERTY
                                 2. Square Root Property
  2.1. Square Root Property.

 THEOREM 2.1.1. For every non-zero real or complex number z there exist precisely two
 distinct complex
            p     numbers w such that w2 = z. One ofpthe numbers is the principal square
 root of z, z, and the other number is its opposite − z.


 BASIC EXAMPLE 2.1.1. If w2 = 25, then w = 5 or w = −5. In the case when the square is
 positive, both solutions are real.

 If w2 = −4, then w = 2i or w = −2i:
                                      (2i)2    =       22 i 2
                                               =       4(−1)
                                               =       −4
 −2i can be verified as a solution as well. In this case there are no real solutions, but there
 are two distinct complex solutions.


 THEOREM 2.1.2 (Square Root Property). Some equations can be solved by taking square
 roots of both sides. If X is an algebraic expression and r is a real number, then solving
 the equation
                                           X2 = r
 amounts to combining the solution sets of equations
                                               p
                                           X= r
 and                                          p
                                          X =− r


 EXAMPLE 2.1.1. Solve the equation
                                              x2 = 9


 SOLUTION: Take square roots of both sides. Either
                                         p
                                     x = 9=3
 or                                            p
                                        x =−       9 = −3


                                      ANSWER: {−3, 3}


                                               461
2. SQUARE ROOT PROPERTY                                      CHAPTER 9. QUADRATIC EQUATIONS

 EXAMPLE 2.1.2. Solve the equation
                                              y 2 = 12


 SOLUTION: Take square roots of both sides. Either
                                        p        p
                                   y = 12 = 2 3
 or                                          p       p
                                        y = − 12 = −2 3
                                                          p     p
 We can state this solution set using the roster notation −2 3, 2 3 , but it is more tradi-
 tional to use the plus-minus notation when the quadratic solutions are irrational and/or
 complex.

                                                   p
                                         ANSWER: ±2 3


 DEFINITION 2.1.1. Many quadratic equations have irrational solution sets of the form
                                 ¦    p        p ©
                                   a + b, a − b
 and many others have complex solutions sets of the form
                                         {a + bi, a − bi}
 In cases like that we will use a plus-minus notation in order to be more concise:
                             ¦     p        p ©             p
                               a + b, a − b         = a± b
                                    {a + bi, a − bi}     =    a ± bi


 BASIC EXAMPLE 2.1.2. Some solution sets expressed two ways:
                               roster notation         plus-minus notation
                                  p        p                    p
                              2 − 3 5, 2 + 3 5               2±3 5
                                {−6i, 6i}                       ±6i
                             p            p                     p
                         7 − 2 3 · i, 7 + 2 3 · i             7±2 3·i
                              8 − 9i 8 + 9i                    8 ± 9i
                            §               ª
                                    ,
                                5        5                       5




                                                 462
CHAPTER 9. QUADRATIC EQUATIONS                                 2. SQUARE ROOT PROPERTY
  2.2. Solving Equations (nx + k)2 = r.

 EXAMPLE 2.2.1. Solve the equation       (x + 1)2 = 25.


 SOLUTION: By the square root property, either
                                Æ                       p
                                  (x + 1)2 =             25
                                                        p
                                         x +1      =     25
                                         x +1      =    5
                                             x     =    4
 or
                                  Æ                      p
                                      (x + 1)2   =      − 25
                                                         p
                                        x +1     =      − 25
                                        x +1     =      −5
                                             x   =      −6
 The solutions are rational, so we state the solution set in roster notation:


                                      ANSWER: {−6, 4}


 EXAMPLE 2.2.2. Solve the equation       (t + 2)2 = 5


 SOLUTION: By the square root property, either
                                              p
                                t +2 =         5
                                                          p
                                         t   =     −2 +    5
 or
                                                    p
                                    t +2     =     − 5
                                                          p
                                         t   =     −2 −    5
 When the solution set for a quadratic equation is irrational or complex, it is traditional to
 state the answer                       p           p
                                    −2 − 5, −2 + 5
                              p
 in the plus-minus form −2 ± 5


                                                        p
                                     ANSWER: −2 ±        5

                                             463
2. SQUARE ROOT PROPERTY                                        CHAPTER 9. QUADRATIC EQUATIONS

 EXAMPLE 2.2.3. Solve the equation          (2 y − 3)2 = 9


 SOLUTION: By the square root property, either
             Æ                  p
               (2 y − 3)2 =       9
                                p
                  2y − 3 =        9
                     2y − 3    =    3
                         2y    =    6                               added 3 to both sides
                          y    =    3                           divided both sides by 2
 or
                                   Æ                                p
                                       (2 y − 3)2         =    −        9
                                                                    p
                                            2y − 3        =    −        9
                                            2y − 3        =    −3
                                               2y         =    0
                                                  y       =    0


 The solutions are rational, so we state the solution set in roster notation:


                                        ANSWER: {0, 3}


  2.3. Complex Solutions.

 EXAMPLE 2.3.1. Solve the equation          z 2 = −16


 SOLUTION: By the square root property, either
                                            p
                                   z =         −16
                                            p
                                       =       16 · i
                                              =       4i
 or
                                                          p
                                        z     =       −       −16
                                              =       −4i


                                         ANSWER: ±4i

                                                  464
CHAPTER 9. QUADRATIC EQUATIONS                             2. SQUARE ROOT PROPERTY

 EXAMPLE 2.3.2. Solve the equation     (x − 5)2 = −100


 SOLUTION: By the square root property, either
                                                 p
                                  x −5     =      −100
                                  x −5     =     10i
                                       x   =     5 + 10i
 or
                                                  p
                                 x −5      =     − −100
                                 x −5      =     −10i
                                      x    =     5 − 10i


                                     ANSWER: 5 ± 10i




                                           465
2. SQUARE ROOT PROPERTY                                CHAPTER 9. QUADRATIC EQUATIONS
      Homework 9.2.
                                                                      ‹2
Solve the equation by finding all real solu-                      3             1
                                                           
tions.                                               12.       x−          =
                                                                  4            16
1. x 2 = 36

2. y 2 = 144

3. (x − 6)2 = 4                                      Solve the equation by finding all complex
                                                     solutions.
4. (x + 7)2 = 16
                                                     13. z 2 = −2
5. (4n − 5) = 1    2

                                                     14. z 2 = −10
6. (5n − 1) = 49   2

                                                     15. ( y + 2)2 = −25
7. (x − 3) = 282

                                                     16. ( y − 4)2 = −36
8. (x + 9) = 402

                                                     17. (z − 4)2 = −9
9. (−3z + 4) = 0       2

                                                     18. (2z + 5)2 = −49
10. (5z + 8) = 0       2

                   ‹2                                19. (x + 4)2 = −75
               1               9
      
11.       x+               =
               2               4                     20. ( y − 5)2 = −32




                                               466
CHAPTER 9. QUADRATIC EQUATIONS                       2. SQUARE ROOT PROPERTY
    Homework 9.2 Answers.

1. {−6, 6}                             11. {−2, 1}

3. {4, 8}                                   p
                                       13. ± 2 · i
     3
   §      ª
5.     ,1
     2                                 15. −2 ± 5i
         p
7. 3 ± 2 7                             17. 4 ± 3i
     4
   § ª
9.                                               p
     3                                 19. −3 ± 5 3 · i




                                 467
3. COMPLETING THE SQUARE                                          CHAPTER 9. QUADRATIC EQUATIONS
                                   3. Completing the Square


                                                         5
                                                                  y

                                       (x + 2)2          4
                                  y=
                                          2
                                                         3
                                                                          y = 3 − x2
                                                         2

                                                         1
                                                                                       x
                −6    −5     −4       −3   −2     −1                  1      2     3       4
                                                        −1

                                                        −2


    3.1. Completing the Square for x 2 + bx. Our goal for this chapter is to be able to solve
every quadratic equation, and we will be able to do so because every quadratic equation can
be put in a form (x + v)2 = r. The crucial step in that transformation is rewriting an expression
of the type x 2 + bx as a sum of a binomial square and a constant:
                                       x 2 + bx = (x + v)2 − t
As it happens, there is an easy way to figure out v and t from any given b.

  THEOREM 3.1.1 (Completing the Square). For any real or complex number b, the polyno-
  mial x 2 + bx has an equivalent form
                                       x 2 + bx = (x + v)2 − t
                     ‹2
           b         b
  where v = and t =
           2         2


   PROOF.
                                                            ‹2 ‹2
                                                         b       b
                           (x + v) − t
                                  2
                                           =          x+     −
                                                         2       2
                                                              ‹2  ‹2
                                                               b     b
                                           =      x 2 + bx +       −
                                                               2     2
                                           =      x + bx
                                                    2


                                                                                               

                                                  468
CHAPTER 9. QUADRATIC EQUATIONS                              3. COMPLETING THE SQUARE

 EXAMPLE 3.1.1. Complete the square for the expression x 2 + 6x


 SOLUTION: Using the formula from the theorem 3.1.1, b = 6, so in order to complete the
 square we need to add and subtract t, which we compute as follows:
                                   ‹2  ‹2
                                    b       6
                              t=        =       = 32 = 9
                                    2       2
 and so
                             x 2 + 6x   =    x 2 + 6x + 9 − 9
                                        =    (x + 3)2 − 9


                                 ANSWER: (x + 3)2 − 9


 EXAMPLE 3.1.2. Complete the square for the expression x 2 − 14x


 SOLUTION: Using the formula from the theorem 3.1.1, b = −14, so in order to complete
 the square we need to add and subtract t, which we compute as follows:
                               ‹2 
                                         −14 2
                                             ‹
                                b
                           t=       =           = (−7)2 = 49
                                2          2
 and so
                           x 2 − 14x    =   x 2 − 14x + 49 − 49
                                        =   (x − 7)2 − 49


                                ANSWER: (x − 7)2 − 49


 EXAMPLE 3.1.3. Complete the square for the expression x 2 + 5x


 SOLUTION: Using the formula from the theorem 3.1.1, b = 5, so in order to complete the
 square we need to add and subtract t, which we compute as follows:
                                     ‹2  ‹2
                                      b       5     25
                                t=        =       =
                                      2       2      4




                                            469
3. COMPLETING THE SQUARE                                          CHAPTER 9. QUADRATIC EQUATIONS
 and so
                                                                      25 25
                              x 2 + 5x     =       x 2 + 5x +            −
                                                                       4   4
                                                                ‹2
                                                            5            25
                                                   
                                           =           x+            −
                                                            2             4

                                                              ‹2
                                                          5            25
                                                   
                                   ANSWER:             x+          −
                                                          2             4

  3.2. Solving Quadratic Equations by Completing the Square.

 EXAMPLE 3.2.1. Solve the equation         x 2 − 8x + 2 = 0


 SOLUTION: Complete the square for the binomial x 2 − 8 first. Since we are dealing with
 an equation, we can add the constant required to complete the square to both sides. In
 this case, the coefficient for the linear term is −8, and so the constant to complete the
 square is
                                      ‹2
                                       −8
                                             = (−4)2 = 16
                                        2
           x 2 − 8x + 2   =   0                                      constant 2 belongs on the right
      x 2 − 8x + 2 − 2    =   0−2
                2
               x − 8x     =   −2
          x − 8x + 16
           2
                          =   −2 + 16                  add 16 to both sides to complete the square
               (x − 4)2   =   14               complete the square and simplify the right side
 Now we apply the square root property to get the answers. Either
                              Æ                  p
                                 (x − 4)2 =        14
                                                 p
                                    x −4 =         14
                                                    p
                                        x = 4 + 14
 or
                                   Æ                              p
                                       (x − 4)2        =      −       14
                                                                  p
                                         x −4          =      −  14
                                                                 p
                                               x       =      4 − 14


                                                              p
                                       ANSWER: 4 ±                14

                                                   470
CHAPTER 9. QUADRATIC EQUATIONS                                         3. COMPLETING THE SQUARE

 EXAMPLE 3.2.2. Solve the equation x 2 + 6x + 10 = 0


 SOLUTION: Complete the square for the binomial x 2 + 6x first. Since we are dealing with
 an equation, we will add the constant required to complete the square to both sides. In
 our case, the coefficient for the linear term is 6, so the constant to complete the square is
                                       ‹2
                                        −6
                                              = (−3)2 = 9
                                         2
            x 2 + 6x + 10        =   0                        the constant 10 belongs on the right
       x 2 + 6x + 10 − 10        =   0 − 10
                   x 2 + 6x      =   −10
              x + 6x + 9
               2
                                 =   −10 + 9          add 9 to both sides to complete the square
                   (x + 3)2
                                 =   −1
 Now we apply the square root property to get the answers. Either
                                               p
                                  x +3 =         −1
                                           x +3   =      i
                                              x   =      −3 + i
 or
                                                          p
                                           x +3   =      − −1
                                           x +3   =      −i
                                              x   =      −3 − i
 As always in an equation, we can check each answer by comparing the values of the two
 sides of our equation. If x = −3 + i, then
        x 2 + 6x + 10        =   (−3 + i)2 + 6(−3 + i) + 10
                             =   9 − 6i + i 2 − 18 + 6i + 10                       distributed
                             =   1+i   2
                                                                         combined like terms
                             =   0                                                     i 2 = −1
 If x = −3 − i, then
        x 2 + 6x + 10        =   (−3 − i)2 + 6(−3 − i) + 10
                             =   9 + 6i + i 2 − 18 − 6i + 10                       distributed
                             =   1+i   2
                                                                         combined like terms
                             =   0                                                     i 2 = −1


                                           ANSWER: −3 ± i

                                                  471
3. COMPLETING THE SQUARE                              CHAPTER 9. QUADRATIC EQUATIONS

 EXAMPLE 3.2.3. Solve the equation 3x 2 − x − 6 = 0


 SOLUTION: Before we can complete the square, we need to divide both sides by the
 leading coefficient 3:
                                        1
                                  x2 − x − 2 = 0
                                        3
 Now we complete the square by adding
                                      ‹2  ‹2
                                1           1       1
                             
                               − ÷2 = −          =
                                3           6      36
 to both sides.
                       1
                  x2 − x = 2                  constant belongs on the right
                       3
               1     1              1
          x2 −   x+       =    2+
               3    36              36
                   1 2         72   1
                    ‹
                x−        =       +                       found LCD on the right
                   6           36 36
                   1 2         73
                    ‹
                x−        =
                   6           36


 Now we can apply the square root property. This time we will introduce the plus-minus
 notation right into the equation, saving ourselves some time:
                                v                     v
                                        1 2
                                u        ‹           t 73
                                               = ±
                                t
                                    x−
                                        6               36
                                                      v
                                           1          t 73
                                       x−      = ±
                                           6            36
                                                      p
                                           1           73
                                       x−      = ±
                                           6           6
                                                        p
                                                    1     73
                                           x =        ±
                                                    6     6
                                                        p
                                                    1 ± 73
                                           x =
                                                       6

                                                   p
                                              1±     73
                                    ANSWER:
                                                   6

                                          472
CHAPTER 9. QUADRATIC EQUATIONS                               3. COMPLETING THE SQUARE
   Homework 9.3.

Complete the square for the given expres-          9. x 2 − 24x + 21 = 0
sion.
                                                   10. a2 − 10a + 20 = 0
1. x 2 + 8x
                                                   11. b2 − 8b + 16 = 0
    2
2. x − 10x
                                                   12. c 2 + 10c + 25 = 0
3. x 2 − 20x
                                                   13. a2 − 3a − 10 = 0
4. x + 18x
    2

                                                   14. x 2 − 3x − 4 = 0
5. x 2 + 13x
                                                   15. 5x 2 + 6x + 1 = 0
    2
6. x − 3x
                                                   16. 2b2 − 7b − 10 = 0


Solve the given equation by completing the         Find complex solutions for the equation by
square.                                            completing the square.

7. x 2 + 2x − 4 = 0                                17. x 2 − 6x + 25 = 0

8. x 2 + 4x − 5 = 0                                18. x 2 + 2x + 5 = 0




                                             473
3. COMPLETING THE SQUARE             CHAPTER 9. QUADRATIC EQUATIONS
    Homework 9.3 Answers.

1. (x + 4)2 − 16                  11. {4}

3. (x − 10)2 − 100
                                  13. {−2, 5}
5. (x + 6.5) − 42.25
           2

        p                         15. {−0.2, −1}
7. −1 ± 5
        p
9. 12 ± 123                       17. 3 ± 4i




                            474
CHAPTER 9. QUADRATIC EQUATIONS                                      4. QUADRATIC FORMULA
                                     4. Quadratic Formula

 THEOREM 4.1.1 (Quadratic Formula). Let a, b, and c be real numbers, a 6= 0, and consider
 the quadratic equation in one variable x in standard form:
                                       ax 2 + bx + c = 0
 where a is the coefficient of the quadratic term, b is the coefficient of the linear term, c
 is the constant term. Then the solutions of the equation can be computed with
                             p                                     p
                       −b + b2 − 4ac                          −b − b2 − 4ac
                 x1 =                         and       x2 =
                               2a                                   2a
                             2
 Moreover, the radicand b − 4ac is known as the discriminant. If the discriminant is
 positive, then there are two distinct real solutions; if b2 − 4ac = 0, then there is a unique
 real solution; and if b2 − 4ac < 0, then there are two distinct complex solutions.


  PROOF. Deriving the formula amounts to solving this equation by completing the square:
   a x 2 + bx + c    =   0
        a x 2 + bx   =   −c                                   moved the constant to the right

                bx       −c
         x2 +        =                         divided both sides by the leading coefficient a
                a        a
      bx     b2           b2    c                                                              b2
 x2 +    + 2         =      2
                              −                               completed the square with
       a    4a           4a     a                                                             4a2
           b 2            b2   4ac
             ‹
       x+            =       −                       found common denominator on the right
          2a             4a2 4a2
           b 2           b2 − 4ac
             ‹
       x+            =                                       subtracted fractions on the right
          2a                4a2
                           v
                 b         t b2 − 4ac
          x+         =   ±                                   applied the square root property
                2a               4a2
                           p
                 b            b2 − 4ac                                       p
          x+         =   ± p                                   distributed       over the fraction
                2a              4a2
                           p
                 b            b2 − 4ac
          x+         =   ±                                            simplified denominator
                2a              2a
                                  p
                            b       b2 − 4ac
                 x   =   −     ±                                        isolated x on the left
                           2a         2a
                                p
                         −b ± b2 − 4ac
                 x   =                               quadratic formula in plus-minus notation
                                 2a
                                                                                                     
                                               475
4. QUADRATIC FORMULA                                        CHAPTER 9. QUADRATIC EQUATIONS

 EXAMPLE 4.1.1. Solve the equation x 2 − 3x + 2 = 0.


 SOLUTION: a = 1, b = −3, and c = 2, so the discriminant
                            b2 − 4ac = (−3)2 − 4 · 1 · 2 = 9 − 8 = 1,
 and hence the solutions are
                                                       p
                                           −(−3) +      1
                                      x1 =                  =2
                                              2·1
                                                       p
                                           −(−3) −      1
                                      x2 =                  =1
                                              2·1

 Just like for all equations, it is very easy to check the answer. If x 1 and x 2 are the solutions,
 then substituting them for x in the original equation should produce identities:
                                      22 − 3 · 2 + 2   =     0
                                          4−6+2        =     0
                                                   0   =     0


                                      12 − 3 · 1 + 2   =     0
                                          1−3+2        =     0
                                                   0   =     0


                                         ANSWER: {1, 2}


 EXAMPLE 4.1.2. Solve the equation 4x 2 − 1 = 0.


 SOLUTION: a = 4, b = 0, and c = −1, so the discriminant
                                b2 − 4ac = 02 − 4 · 4 · (−1) = 16
 and hence the solutions are
                                           p
                                       −0 + 16 4 1
                                  x1 =        = =
                                         2·4    8 2
                                           p
                                       −0 − 16 −4     1
                                  x2 =        =    =−
                                         2·4     8    2


                                     ANSWER: {−1/2, 1/2}

                                                476
CHAPTER 9. QUADRATIC EQUATIONS                                          4. QUADRATIC FORMULA

 EXAMPLE 4.1.3. Solve the equation
                                         x(x + 8) = 2x − 9


 SOLUTION: First we have to bring the equation into the standard form:
                                            x(x + 8)    =     2x − 9
                                             x + 8x
                                              2
                                                        =     2x − 9
                                 x + 8x − 2x + 9
                                   2
                                                        =     0
                                         x 2 + 6x + 9   =     0
  a = 1, b = 6, and c = 9, and so the discriminant
                              b2 − 4ac = 36 − 4 · 1 · 9 = 36 − 36 = 0
 and the only solution is                            p
                                                 −6 ± 0
                                       x1 = x2 =        = −3
                                                   2·1


                                          ANSWER: {−3}

  4.2. Complex Solutions.

 EXAMPLE 4.2.1. Solve the equation
                                           x 2 − 2x + 2 = 0


 SOLUTION: a = 1, b = −2, and c = 2, so the discriminant
                            b2 − 4ac = (−2)2 − 4 · 1 · (2) = 4 − 8 = −4
 Since −4 is negative, there are no real solutions, but we can still apply the quadratic
 formula to obtain the complex solutions:
                                          p
                                −(−2) ± −4 2 ± 2i
                           x1 =               =        =1±i
                                     2·1           2


                                           ANSWER: 1 ± i




                                                  477
4. QUADRATIC FORMULA                                 CHAPTER 9. QUADRATIC EQUATIONS
   Homework 9.4.

Use the quadratic formula to find all real         12. y 2 − 10 y = −25
solutions.
                                                   13. −9x 2 + 6x + 8 = 0
1. x 2 − 5x + 6 = 0
                                                   14. 2x(x + 3) = 5(x + 2)
2. x 2 + 3x − 40 = 0
                                                   15. x 2 − 4x + 7 = 0
3. x 2 − 2x − 3 = 0
                                                   16. x 2 + 6x − 2 = 0
4. y − 8 y + 20 = 0
    2



5. x 2 − 7x = 0                                    Use the quadratic formula to find complex
                                                   solutions for each equation.
6. 6x 2 = −3x
                                                   17. x 2 + 1 = 0
7. x 2 − 9 = 0
         1                                         18. −3x 2 = 75
8. 5 y =2
         5
                                                   19. x 2 − 6x + 13 = 0
9. 3x 2 − 7x + 2 = 0
                                                   20. x 2 + 4x + 8 = 0
10. 3x 2 + 7x + 4 = 0
                                                   21. z 2 + 2z + 3 = 0
11. x 2 + 8x + 16 = 0
                                                   22. −x 2 + 7x − 12 = 0




                                             478
 CHAPTER 9. QUADRATIC EQUATIONS                           4. QUADRATIC FORMULA
    Homework 9.4 Answers.

                                             2 4
                                           §     ª
1. {2, 3}
                                        13. − ,
                                             3 3
3. {−1, 3}
                                                  p
5. {0, 7}                               15. 2 ±    3·i

7. {−3, 3}                              17. ±i
     1
   §      ª
9.     ,2                               19. −3 ± 2i
     3
                                                   p
11. {4}                                 21. −1 ±    2·i




                                  479
4. PRACTICE FINAL                                       CHAPTER 9. QUADRATIC EQUATIONS
                                        Practice Final




1. Solve the formula for y:                          9. Solve the system:
                                                                  17x + 20 y = 26
                                                              
             4 y + 3x = 5 − y
                                                                −17x + 10 y = 64
2. Solve the inequality
                                                     10. Solve the system:
            2(x − 3) ≥ 5x + 12                                     2(x − 1) = 6 y
                                                              

                                                                 3y − x + 2 = 0
3. Frodo’s journey to Mount Doom was
three times longer than his journey back
home, and the whole trip, there and back
again, took 240 days. Find the duration of           Simplify the expression and state the an-
the journey to the Mount Doom and the du-            swer in scientific notation:
ration of the homeward journey.
                                                           42 × 10−2
                                                     11.
4. One week Alice worked 10 hours wait-                     5 × 10−4
ing tables and 3 hours editing a book, and
                                                     12. (5.5 × 103 )(3.4 × 10−5 )
she made $195. Next week she waited ta-
bles for 5 hours and spent 5 hours editing a
book, and she made $185. Find the hourly
wage for each job.                                   Simplify the expression and state the an-
                                                     swer with positive exponents only:
5. Find an equation of the line with slope                                −2
  1                                                             x 3 y −2
− and y-intercept (0, −2). State the an-             13.
  3                                                            x −1 y −4
swer in the slope-intercept form.
                                                     14. (−3x −5 y 7 )(−5x 0 y −4 )
6. Find an equation of the line with slope
−2 that passes through the point (−1, 3).
State the answer in the slope-intercept
form.                                                Factor the expression completely:

7. Find the slope and an equation of the             15. z 4 − 81 y 4
line that passes through the points (1, −5)
and (3, −1). State the answer in the point-          16. 8x − 4 + 6x 3 − 3x 2
slope form.

8. Solve the system of linear equations by
graphing:

                 y+x = 3
           

               2 y − x = −6
                                               480
CHAPTER 9. QUADRATIC EQUATIONS                                4. PRACTICE FINAL
Solve the equation by factoring:         26.
                                                      p
                                                          x −3+6=4
17.
                6x 2 = −10x              27.
                                                       p   p
                                                      5 x = x +2
18.
               100 − 4x 2 = 0            28.
                                                     p
19.                                                   2x + 5 − 1 = x
               x 3 − 2x 2 = 24x


Simplify the expression:                 29. Solve the equation by taking square
                                         roots of both sides:
    p
20.    100x 2                                          (x + 3)2 = 20
    p
    3
21.    x 24                              30. Solve the equation by completing the
    p                                    square:
22.    45x 2
                                                     x 2 + 18x + 1 = 0
    Æ
23.    242x 6 y 7
    p
      8x 7
24. p                                    Solve the equation by using the quadratic
      2x y 2                             formula:

                                         31.
Solve the equation:                                   x 2 − 4x + 1 = 0
25.                                      32.
                   7    x
                      =                             4x 2 − 4x − 35 = 0
                 x +3   4




                                   481
 4. PRACTICE FINAL                                              CHAPTER 9. QUADRATIC EQUATIONS
     Practice Final Answers.

         5 − 3x                                           11. 8.4 × 102
1. y =
            5
                                                          12. 1.87 × 10−1
2. (−∞, −6]
                                                                  1
                                                          13.
                           −6                                   y4 x8
3.                                                              15 y 3
 x and y are the durations for going there                14.
                                                                 x5
 and back again respectively, in days
 
   x = 3y                                                 15. (z + 3 y)(z − 3 y)(z 2 + 9 y 2 )
   x + y = 240                                            16. (4 + 3x 2 )(2x − 1)
 solution: x = 180,            y = 60
                                                          17. {0, −5/3}
4.
 x and y are the wages for waiting and edit-              18. {−5, 5}
 ing respectively, in dollars per day
                                                          19. {0, −4, 6}
    10x + 3 y = 195
 

    5x + 5 y = 185                                        20. 10x
 solution: x = 12,         y = 25
                                                          21. x 8
        1                                                       p
5. y = − x − 2
        3                                                 22. 3x 5
6. y = −2x + 1                                                            p
                                                          23. 11x 3 y 3       2y
7. y + 5 = 2(x − 1)
                                                                2x 3
8. (4, −1)                                                24.
                                                                 y
                   5
                       y                                  25. {−7, 4}
                   4
                   3                                      26. ∅
                   2
                   1                                      27. {1/4}
                                            x
 −5 −4 −3 −2 −1            1    2   3   4       5         28. {2}
              −1
                  −2                                                p
                                                          29. −3 ± 2 5
                  −3
                                                                    p
                  −4                                      30. −9 ± 4 5
                  −5                                             p
                                                          31. 2 ± 3
9. (−2, 3)
10. no solution                                           32. {−2.5, 3.5}
                                                    482
                                        APPENDIX A


                                      To Instructors


                                      1. Lecture Notes

   1.1. Concepts.

                                        1. Expressions

Define: expressions, equivalent expressions, equations, relations, substitution, sum of terms,
product of factors, coefficient. Show how to split sums into terms, and products into factors.

                                          2. Axioms

Stop before identity to show that terms of sums can be added in any order, and factors of
a product can be multiplied in any order. Stop before distributivity to discuss opposites and
reciprocals. Show distributivity both ways.

  EXAMPLE 1.1.1. Rewrite the expression without parentheses and then simplify by appeal-
  ing to the axioms: x + 3(2 + x)


  SOLUTION:

                    equivalent expressions   axiom used for substitution
                          x + 3(2 + x)
                       x + (3 · 2 + 3 · x)   distributivity
                       x + (3 · x + 3 · 2)   commutativity of +
                         x + (3 · x + 6)     closure
                        (x + 3 · x) + 6      associativity of +
                       (1 · x + 3 · x) + 6   identity of ·
                          (1 + 3)x + 6       distributivity
                             4x + 6          closure


Define subtraction and division, show sums with negative terms and products with reciprocal
factors.
                                             483
1. LECTURE NOTES                                             CHAPTER A. TO INSTRUCTORS
                                             3. Sets

Define sets, roster, integers: positive and non-negative.

Show the number line and the addition/subtraction of negative numbers.

Define set builder, show odd numbers or something like that:
                                 
                                  2k + 1 k is an integer
Show how to list a few members of a set
                               2
                               m m is a positive integer
Define rationals and reals, mention irrational numbers.

Show absolute value and inequality relations.

                                            4. Reals

Show products of negative numbers, opposite of a sum, positive integer exponent, order of
operations, like terms.

                                           5. Primes

Define primes, show prime factorizations

Show cancellation of common factors in fractions, define fractions in lowest terms.

                                          6. Fractions

Show reciprocal of a product, rewriting a fraction as a product, multiplying fractions with
invisible parentheses, applications to areas, dividing fractions, and changing the denominator.

                                             7. LCD

Show distributivity with common denominator, define LCM and LCD.

Show rational addition and mixed numbers.

                                         8. Translation

Show the four arithmetic patterns, mention combinations like “X is 10 units greater than 4
times y”.
                                              484
CHAPTER A. TO INSTRUCTORS                                                1. LECTURE NOTES
   1.2. Linear Equations.

                                         1. Properties

Define solution, equivalent equations, show addition and multiplication properties.

  EXAMPLE 1.2.1. Ivan’s math class has 1.2 times more students than Ivan’s statistics class.
  Find how many people are enrolled in the statistics class has if the math class has 30
  students.


  SOLUTION: Let m and s be the enrollment figures for math and stat class respectively
  (units are students). The statement
                 math class has 1.2 times more students than statistics class
  can be translated as
                                          m = 1.2s
  Since we are given that the math class has 30 students, we can substitute 30 for m in the
  equation and solve for s:
                                        30       =   1.2s
                                        25       =   s
  So the stats class has 25 students enrolled.


                                       ANSWER:
            m and s count students enrolled in math and stats class respectively,
                                    equation: m = 1.2s
                                     solution: s = 25


                                          2. Solving

Define linear equations. Show how to solve by isolating the variable: a simple case, with
parentheses, and with fractions.

  EXAMPLE 1.2.2. Bart, Millhouse, and Martin Prince saved up 149 dollars for a collectible
  comic book. Find how much money each one saved up if Bart contributed 3 times less
  than Millhouse, and Martin contributed 9 dollars more than Millhouse.


  SOLUTION: Let b, m, and p be the amounts contributed by Bart, Millhouse, and Martin
  Prince, in dollars. We can translate the statements as
               b+m+p         =   149                 together they saved up $149
                         b   =   m÷3
                         p   =   m+9

                                             485
1. LECTURE NOTES                                                CHAPTER A. TO INSTRUCTORS
  We can substitute expressions for b and p into the first equation and then solve for m:
                                              b+m+p         =   149
                              (m ÷ 3) + m + (m + 9)         =   149
                                    m + 3m + 3m + 27        =   447
                                                      7m    =   420
                                                        m   =   60
  We can find b and p now using the other two equations which are solved for these vari-
  ables:
                                b     =      m ÷ 3 = 60 ÷ 3 = 20
                                p     =      m + 9 = 60 + 9 = 69


                                       ANSWER:
  b, m, and p be the amounts contributed by Bart, Millhouse, and Martin Prince, in dollars,

                                    b + m + p = 149
                                   

                                      b = m÷3
                                      p = m+9
                                   

                             solution: b = 20, m = 60, p = 69


                                             3. Formulas

Show how to solve linear formulas for a variable. Examples to be solved for y:
                                3x − 4 y       =    x −4
                               2(x − y)        =   4 y + 10x − 1
                                     7x y           4
                                               =
                                      3             b
                                    8y + 2     =   4(x − x y)

                                             4. Percent

Define percent, show decimal representation, show three easy translation patterns. Define
percent increase and decrease, show basic examples and applications.

  EXAMPLE 1.2.3. A quantity increased by 14% and is now 171. Find the quantity before
  the increase.




                                                486
CHAPTER A. TO INSTRUCTORS                                                  1. LECTURE NOTES

  SOLUTION:
                                    171      =    P(1 + 0.14)
                                    150      =    P


  EXAMPLE 1.2.4. A quantity decreased from 140 to 133. Find the percent decrease.


  SOLUTION:
                                                           k
                                    133      =    140 −       · 140
                                                          100
                                                       k
                                     −7      =    −       · 140
                                                      100
                                 7 · 100
                                             =    k
                                  140
                                      5      =    k


                                           ANSWER: 5%


  EXAMPLE 1.2.5. John paid 24% income tax on the total income of $28, 000. Find how
  much money John has made after tax, and the amount he paid to the government.


                 ANSWER: John has made $21, 280 after paying $6, 720 tax.


  EXAMPLE 1.2.6. A contractor buys a residential property, renovates it, and then sells (flips)
  it with 25% markup. If the the sale price is $160, 000, find how much the contractor has
  paid for the property, as well as his profit from flipping it.


           ANSWER: The original price was $128, 000, and the profit was $32, 000.


                                           5. Applications

Show the work equation and give few basic examples of assigning/comparing units:

      •   distance = speed · duration
      •   paycheck = hourly rate · duration
      •   volume of water = pumping rate · duration
      •   total price = unit price · quantity

                                                 487
1. LECTURE NOTES                                             CHAPTER A. TO INSTRUCTORS

 EXAMPLE 1.2.7. Ivan presented 9 examples over 2 hours and 15 minutes. Describe vari-
 ables and units in
                                       w = rt
 and find the rate.


 SOLUTION: w is the number of examples presented, t is duration in hours, r is the rate in
 examples per hour. Note that 15 minutes is 0.25 of an hour.
                                       9   =   r · 2.25
                                       4   =   r
 So the rate is 4 examples per hour.


 EXAMPLE 1.2.8. Shannon’s business serves 14 clients per day on average. Describe vari-
 ables and units in
                                          w = rt
 and find the time needed to serve 518 clients?


 SOLUTION: The rate is in clients per day, so the measure of “work” is just the number of
 clients, and the units of time are days.
                        518    =   14t
                         37    =   t                            days


 EXAMPLE 1.2.9. Two cars, a Lotus and a Renault, are 635 kilometers apart. They start at
 the same time and drive toward each other. The Lotus travels at a rate of 70 kilometers
 per hour and the Renault travels at 57 kilometers per hour. In how many hours will the
 two cars meet?


 SOLUTION:
                 70t + 57t    =    635                    distance equation
                      127t    =    635
                          t   =    5                                   hours


 EXAMPLE 1.2.10. Jane’s monthly car loan payment is one quarter of her monthly house
 loan payment. Find each monthly payment rate if Jane pays 19200 dollars in total over
 the course of the year.




                                           488
CHAPTER A. TO INSTRUCTORS                                                            1. LECTURE NOTES

  SOLUTION: c and h are payment rates for the car and the house respectively, in dollars
  per month.
                      c        =       h÷4            car payment is a quarter of house payment
         c · 12 + h · 12       =       19200                                     “work” equation
  Solve by substitution:
                12(h ÷ 4) + 12h              =    19200                substitute h/4 for c
                           3h + 12h          =    19200
                                   15h       =    19200
                                       h     =    1280                     house payment
  Back to the first equation to find c:
                           c       =    1280/4
                           c       =    320                            car payment


                                                   6. Inequalities

Define linear inequalities and solutions. Show graphs and interval notation. Show addition
and multiplication properties, examples.

   1.3. Graphing.

                                                     1. Reading

Define ordered pair, coordinate plane, graph of a set of points. Plot/read some points, show
axis scaling, show how to find intercepts and extrema.

                                                 2. Linear Equations

Define solution set. Show some fun solution sets, like y = x 2 or y = x 3 . Show vertical and
horizontal lines. Show how to graph linear equations which can be solved for a variable by
finding 2 points.

                                                    3. Intercepts

Define intercepts, show how to find them from equations, and how to use them for plotting.
Mention vertical and horizontal lines.

                                                      4. Slope

Define grade, slope of a segment, slope of a line. Mention positive and negative slopes. Show
how to plot using a rational slope.
                                                         489
1. LECTURE NOTES                                                 CHAPTER A. TO INSTRUCTORS
                                       5. Slope-intercept

Define slope-intercept form. Show how to read, write, and to solve for y to get it. Define
parallel and perpendicular lines, show how to compare given lines using their slopes, mention
vertical and horizontal lines.

                                           6. Point-slope

Define point-slope form, show how to read. Show how to construct an equation of a line given
slope and point, and also given two points. Show how to use point-slope form for plotting on
the grid.

                                      7. Function Notation

This section is optional in the sense that nothing else in the text depends on it.

Define functions and function notation, show examples of use.

  EXAMPLE 1.3.1. Acme CO’s profit in 2000 is 4.7 billion dollars, and 5.3 billion dollars in
  2015. Let x be the year since 2000, and let y(x) be the profit in the corresponding year,
  in billions of dollars. Find a linear model y(x), use the model to predict the profit in
  2018, and to predict the year when the profit is 6 billion dollars.


  SOLUTION:
                   m    =    0.6/15 = 0.04                                     slope
                y(x)    =    0.04x + 4.7                                      model
               y(18)    =    5.42                                     billion dollars
                    6   =    0.04x + 4.7
                  1.3   =    0.04x
                32.5    =    x                              approximately year 2032



   1.4. Systems.

                                           1. Graphing

Define the solution for a system with 2 equations and 2 variables. Show the 3 cases for the
shape of the solution set. Show how to solve by graphing:
                      2x + 4 y = 4
                   
                                                                   (4, −1)
                       2x − y = 9
                      y+x = 3
                   
                                                              no solution
                         2 y = 7 − 2x
                                               490
CHAPTER A. TO INSTRUCTORS                                              1. LECTURE NOTES
                                       2. Substitution

Solve by substitution:
                                y = 5x + 13
                    
                                                                  (−2, 3)
                         4x − 3 y = −17
                         3 y − 2x = −4
                    
                                                                (−4, −4)
                         4x + 5 y = −36

                                       3. Elimination

Show multiplication and addition properties for systems. Solve by elimination:
                      5x − 6 y = 11
                    
                                                                   (7, 4)
                      5x + 6 y = 59
                      3x − 7 y = 11
                    
                                                               (−1, −2)
                      9x + 5 y = −19

                                       4. Applications

  EXAMPLE 1.4.1. Every Monday Helen shares coffee and donuts with the office where she
  works. One week Helen buys 4 cups of coffee and 6 donuts, and pays 18 dollars. The
  next week Helen buys 6 cups of coffee and 12 donuts, and pays 30 dollars. Find the price
  of one cup of coffee and the price of one donut.


  SOLUTION: Let c and d be the price of one cup of coffee and the price of one donut
  respectively, in dollars.
                                    4c + 6d = 18
                               

                                  6c + 12d = 30


                                      ANSWER: (3, 1)


  EXAMPLE 1.4.2 (Volume Mixing). A chemist needs to mix an 18% acid solution with a
  45% acid solution to obtain 12 liters of a 36% solution. How many liters of each of the
  acid solutions must be used?




                                              491
1. LECTURE NOTES                                             CHAPTER A. TO INSTRUCTORS

  SOLUTION: Let x and y be amounts of solutions in liters.
                                          x + y = 12
                                 0.18x + 0.45 y = 0.36(12)


  EXAMPLE 1.4.3 (Value Mixing). How many pounds of chamomile tea that cost $9 per
  pound must be mixed with 8 pounds of orange tea that cost $6 per pound to make a
  mixture that costs $7.80 per pound?


  SOLUTION: Let c be the weight of chamomile tea in pounds, and then
                                   9c + 6(8) = 7.8(c + 8)
  Alternatively, we can also let t be the total weight of the mix, and write a system of two
  equations:
                                           c+8= t
                                      9c + 6(8) = 7.8t


                                       5. Inequalities

Define the solution for an inequality in 2 variables, describe solution sets, show examples,
mention vertical and horizontal lines.

   1.5. Polynomials.

                                        1. Exponent

Define integer exponent, mention base 1. Show product rule and quotient rule. Mention sums
as bases: (x + y)n

                                        2. Properties

Show power rule and distributivity over products and fractions.

                                       3. Polynomials

Define monomial and its simplified form, monomial degree, monomial degree names, polyno-
mial and its standard form, leading coefficient, polynomial names, polynomial degree. Show
how to combine like terms and how to evaluate polynomials.

                                          4. Sums

Show how to add and subtract polynomials by combining like terms.
                                            492
CHAPTER A. TO INSTRUCTORS                                                        1. LECTURE NOTES
                                             5. Products

Show how to multiply monomials, monomial times polynomial, and polynomial times polyno-
mial (show the box).

                                          6. Special Products

Show the difference of squares both ways, and the square of a binomial.

                                             7. Quotients

Show how to divide by a monomial, and the polynomial long division.

                                         8. Negative Exponent

Mention definition again, show how to get rid of negative exponents in products and fractions.
Define scientific notation.

   1.6. Factoring.

                                               1. GCF

Define irreducible and prime polynomials. Show polynomial factorizations. Mention that lin-
ear polynomials are irreducible.

Define GCF for integers and GCF for monomials.

                                             2. Grouping

Show factoring by grouping.
                x y + 2 y + 3x + 6                               4x 3 + 2x 2 + 2x + 1
               24x 3 − 6x 2 + 8x − 2                            3x 3 − 6x 2 − 15x + 30

                                             3. Diamond

Show the diamond pattern. Show the basic factoring strategy combining monomial GCF, group-
ing, and diamond.

                                            4. Trinomials

Show how to factor a trinomial.
                      6z 2 + 11z + 4                             −4h2 + 11h + 3
                      2n2 − 13n − 7                             3x 2 + x y − 14 y 2
                     4r 2 + 11rs − 3s2                           5x 2 + 14x − 3
                                                 493
1. LECTURE NOTES                                                   CHAPTER A. TO INSTRUCTORS
                                        5. Special Products

Show how to factor special products.

                                            6. Strategy

List the types of irreducible polynomials: linear, prime trinomials, and sums of squares. Present
the factoring strategy, combining GCF, grouping, difference of squares, diamond, and trinomial
pattern.
                     6x 4 − 13x 3 + 5x 2                          5x 5 − 20x
                  4x 2 y − 9 y + 8x 2 − 18                    36x 4 − 15x 3 − 9x 2

                                            7. Equations

Show the zero product property and how to solve non-linear polynomial equations.
                                                 3x 2 − 12x    =   0
                                   x + 5x − 9x − 45
                                    3        2
                                                               =   0
                                        3x 5 − 6x 4 + 3x 3     =   0
                                        6x 3 − 4x 2 − 16x      =   0
                                2x 3 + 10x − 3x 2 − 15         =   0

                                          8. Applications

Show applications to areas of rectangles and triangles.

   1.7. Rational Expressions.

                                           1. Simplifying

Define rational expressions. Show how to find which values of a variable make the expressions
undefined. Show how to simplify by canceling common polynomial factors.
                                         6x 3 − 12x 2 − 18x
                                          3x 4 + 6x 3 + 3x 2

                                            2. Products

Show how to multiply and divide rational expressions.
             30x 2 − 15x       x 2 − 49                            x −4     x 2 − x − 12
                         ·                                                ÷
              6x − 42      20x 2 − 20x + 5                         x 2 − 9 x 2 + 6x + 9
                                                 3. LCD

Define common multiple, LCM, and LCD. Show how to find LCMs and LCDs.
                                                  494
CHAPTER A. TO INSTRUCTORS                                                  1. LECTURE NOTES
                                              4. Sums

Show how to simplify sums with the same denominator, and sums with different denominators.


          x2   25                  2     t −2                    2x 2   x   1
             −                       − 2                              +   −
        x −5 x −5                t +1 t − t −2                  x −4 x −2 x +2
                                                                 2



                                      5. Complex Fractions

Define complex fractions, show how to simplify them.
                              9             y x                 1
                       3+                     −                    +4
                            x −3            x    y               x
                             12             1 1                     2
                       4+                     +                 3+ 2
                            x −3            y x                     x

                                         6. Equations

Define extraneous solutions. Show how to detect extraneous solutions for equations involving
rational expressions. Show how to solve the equations.
                                  6                   −3
                                     +9         =
                                x −3                 x −3
                                       3               1   2
                                                =        − 2
                                  x2   − 2x          x −2 x
                        x       2                      5
                           + 2                  =
                      x + 2 x + 5x + 6               x +3
                                         x                 7x        3
                                                =                 −
                                       x −5          x 2 − 3x − 10 x + 2
                                       5              4   1
                                                =       + 2
                                 n3   + 5n2          n+5 n


   1.8. Radicals.

                                              1. Intro

Define principal square root. Mention that the square root of a negative number is not real,
but complex. Show how the square root cancels the even exponent. Define principal nth root
and show how it cancels the exponent which is multiple of n. Mention sums as radicands.

                                          2. Products
                                               495
1. LECTURE NOTES                                               CHAPTER A. TO INSTRUCTORS
Show distributivity over multiplication. Use it to simplify the radicand involving numbers as
well as variables:         p                               p
                              24                              a7
                        p     p                      p     p      p
                          5x · 5x 3                    6(2 3 + 3 6)

                                         3. Quotients

Show distributivity over fractions. Define rationalized denominator. Show how to simplify
fractions and rewrite them with rationalized denominator:
                                            p                 p
                         1                    8                40x
                        p                  p                  p
                          5                  2y                 2x 2
                                            4. Sums

Define radical like terms. Show how to simplify sums by simplifying radicals and combining
like terms:              p    p                      p         p
                           3 + 27                (3 − 2)(5 − 2 2)
                            p    p
Define radical conjugates A B ± C D. Show how to rationalize a denominator with a sum of
two terms.

                                         5. Equations

Show that squaring both sides produces an almost equivalent equation. Show how to check
for extraneous solutions:
                          p
                           2x + 1 = x − 1, check x = 4 and x = 0
Show how to solve equations:
                                        p
                                   7−   3x + 7     =      3
                                p
                           2 + 6 2x 2 − 3x + 5     =      0
                                     p
                                 4 + x 2 − 4x      =      x
                              Æ     p
                                x + x −1+2         =      3
                                      p                        p
                                        3x − 5     =      2−       x −1

                                        6. Applications

Show the distance formula and applications to right triangles.

                                    7. Rational Exponent

Define rational exponent and show why it has to be in lowest terms before it can be evaluated.
Show how to change expressions from radical to rational exponent form and back. Show
how to evaluate expressions. Show how to simplify expressions involving rational exponents.
Optionally, define the imaginary unit and show how the distributivity and the exponent of
exponent property fail for negative bases.
                                             496
CHAPTER A. TO INSTRUCTORS                                                  1. LECTURE NOTES
   1.9. Quadratics. It is possible to skip introducing complex numbers, if so desired, by skip-
ping the first section. The homework in the sections that follow is divided into two parts: first
part with real solutions, and the second part with complex solutions, so one can easily assign
one or the other or both topics to practice.

                                     1. Complex Numbers

Simplify square roots of negative
                           p      numbers, sums and products of complex numbers, and ex-
                      −b ± D
pressions of the form
                         2a
                                    2. Square Root Property

State that every w2 = z has two solutions for non-zero z. Show the square root property and
the plus-minus notation. Show how to solve equations (nx + k)2 = r.

                                   3. Completing The Square

Show how to complete the square x 2 + bx as well as ax 2 + bx. Show how to solve equations
using this method.

                                     4. Quadratic Formula

Derive the quadratic formula by completing the square in
                                       ax 2 + bx + c = 0
Show a few examples.




                                              497
                                          APPENDIX B


                                        To Developers


                                        1. Contributing

    1.1. Submitting Contributions. If you would like to suggest an edit, please do not hesitate
to contact the maintainer:

"Ivan G. Zaigralin" <melikamp@melikamp.com>

Both this text and its source are licensed under free and libre licenses of the copyleft type,
meaning that anyone can use them in any way whatever, up to and including distributing
modified copies, as long as they use the same licenses for derivative works. All your contri-
butions automatically assume the same project license, but nontrivial LATEX source copy may
qualify you for becoming a co-author and a consequent copyright assignment.

Awesome ideas about content, structure, and appearance are welcome, and so is constructive
criticism. If you want more exercises or examples of a certain type, we urge you strongly to
do as much work as possible, and describe the statements, the solutions, and the answers in
detail, so that we can typeset your idea with minimal research. And if you happen to know
LATEX and opt to submit code contributions which are formatted as described below, you should
be prepared to be listed as a co-author.

    1.2. Document Hierarchy. Standard book template hierarchy is used, with chapters, sec-
tions, and subsections. The chapter source files are included from basic-algebra.tex. Each
chapter file contains the title and the list of included sections. Each section source lives in its
own file. When working on a specific section, it is possible to greatly speed up the build time
by commenting out other chapters in basic-algebra.tex, and other sections in the corre-
sponding chapter source file. The chapter title is printed on the same page as the first section
in that chapter, so do not forget to comment out the title as well when working on sections
other than the first, or else the document will re-flow incorrectly.




                                               499
2. DOCUMENT FORMATTING GUIDELINES                           CHAPTER B. TO DEVELOPERS
                            2. Document Formatting Guidelines

   2.1. Mathematical Notions. We use fairly vanilla LATEX throughout the text, only defining
special commands when we really need to enforce the way a particular concept is typeset.

      • Interval graphs
        \InfOpen{17}

                               17
        \OpenInf{17}

                               17
        \InfClosed{17}

                               17
        \ClosedInf{17}

                               17
      • Interval notation
        \opop{1,2}
            (1, 2)
        \opcl{1,2}
            (1, 2]
        \clop{1,2}
            [1, 2)
        \clcl{1,2}
           [1, 2]
      • Large delimiters
        \pipes{x+\sqrt x}
                  p
             x+       x
        \brackets{x+y}
            [x + y]
        \parens{\frac1x + 3}
                1
                   ‹
                  +3
                x




                                            500
CHAPTER B. TO DEVELOPERS                          2. DOCUMENT FORMATTING GUIDELINES
      • Mixed number
        \mixed{3}{4}{5}
            3 45
      • Set builder
        \setb{2k}{k\in A}
            
             2k k ∈ A
      • Set roster
        \aggr{1,2,3}
          {1, 2, 3}
      • Complex fractions set in normal font size
        \compfrac{\frac 12 + \frac 34}{\frac1x - \frac2x}
            1 3
              +
            2 4
            1 2
              −
            x x
      • System of two equations
        \twoEqSystem{x + y}{2x + 2}{y}{4x - 5}
                 x + y = 2x + 2
            

                     y = 4x − 5


    2.2. Sets of Equations and Expressions. Equivalent expressions and relations are typeset
using align, with commands defined to make the relation appear nice:

\begin{align*}
  x - 1 &\ee 5 &\ecomment{ecomment is for annotations in equations}\\
  x &\ee 6 &\ecomment{ee command makes equal sign with space around it}
\end{align*}

         x −1        =   5                ecomment is for annotations in equations
             x       =   6      ee command makes equal sign with space around it

\begin{align*}
  x - 1 &\qq\geq 5\\
  x &\qq\geq 6 &\ecomment{qq command inserts space around the argument}
\end{align*}

          x −1 ≥         5
                 x   ≥   6        qq command inserts space around the argument
                                            501
2. DOCUMENT FORMATTING GUIDELINES                             CHAPTER B. TO DEVELOPERS
    2.3. Frames. There are several mdframed environments defined, and should be used con-
sistently throughout.

\begin{axiom}[This is an axiom]
  This environment is used for the real and complex number axioms only.
\end{axiom}

  AXIOM 2.3.1 (This is an axiom). This environment is used for the real and complex number
  axioms only.


\begin{definition}[This is a definition]
  This environment is used for definitions.
\end{definition}

  DEFINITION 2.3.1 (This is a definition). This environment is used for definitions.


\begin{theorem}[This is a true fact]
  This environment is used for all general enough true statements
  which can be proven from the axioms and the logical principles.
\end{definition}

  THEOREM 2.3.1 (This is a true fact). This environment is used for all general enough true
  statements which can be proven from the axioms and the logical principles.


\begin{trivia}[This is an easy example]
  This environment is used for examples which are trivial enough.
\end{trivia}

  BASIC EXAMPLE 2.3.1 (This is an easy example). This environment is used for examples
  which are trivial enough.




                                             502
CHAPTER B. TO DEVELOPERS                         2. DOCUMENT FORMATTING GUIDELINES
\begin{example}[This is a nontrivial example]
  This environment is used for stating examples which are similar to
  the homework. It is almost always followed by the other two
  environments: the solution and the answer.
\end{example}
\begin{exsol}
  This environment provides a solution for the example stated above.
\end{exsol}
\begin{exans}
  This environment states the answer to the question above in an
  optimal way which the readers are expected to follow as they are
  working through the homework.
\end{exans}

 EXAMPLE 2.3.1 (This is a nontrivial example). This environment is used for stating exam-
 ples which are similar to the homework. It is almost always followed by the other two
 environments: the solution and the answer.


 SOLUTION: This environment provides a solution for the example stated above.


  ANSWER: This environment states the answer to the question above in an optimal way
   which the readers are expected to follow as they are working through the homework.




                                           503
2. DOCUMENT FORMATTING GUIDELINES              CHAPTER B. TO DEVELOPERS
  2.4. Homework and Answers.

\begin{hw-and-answers}
  \begin{exercise}
    These three environments allow to typeset exercise sections with
    or without answers. They are numbered as subsections and they insert
    page breaks. Does this exercise have an answer?
    \begin{answer}
      Yes.
    \end{answer}
  \end{exercise}

  \begin{exercise}
    This exercise does not have an answer. Why should it?
  \end{exercise}
\end{hw-and-answers}




                                    504
CHAPTER B. TO DEVELOPERS                           2. DOCUMENT FORMATTING GUIDELINES
   Homework B.2.

1. These three environments allow to type-          and they insert page breaks. Does this ex-
set exercise sections with or without an-           ercise have an answer?
swers. They are numbered as subsections
                                                    2. This exercise does not have an answer.
                                                    Why should it?




                                             505
 2. DOCUMENT FORMATTING GUIDELINES         CHAPTER B. TO DEVELOPERS
    Homework B.2 Answers.

1. Yes.




                                     506
CHAPTER B. TO DEVELOPERS                                    4. WORKING WITH THE SOURCE
                                           3. To Do

   3.1. To Do Items.

       •   More notes for applications of w = r t.
       •   More applications of difference in chapter 1.
       •   More applications of slope as rate of change.
       •   More mixture notes.
       •   Add 1-variable mixture problems in chapter 2.
       •   Add applications of equations with rational expressions?
       •   Add practice test 9?

                                 4. Working with the Source

   4.1. Getting the Source. The easiest way to get the source is to git it. You can clone the
repository with

git clone https://git.albertleadata.org/melikamp/basic-algebra

    4.2. Building the Source with LATEX. This text is typeset in LATEX, texlive to be specific,
and a GNU+Linux environment which provides a minimal userland such as ls and a bash-
like shell, a pdflatex command and GNU make. If you use any kind of vanilla GNU+Linux
OS such as Freenix or Debian, then all you need to do is fire up the console, change into the
directory with the source, and run

make
to build print/basic-algebra.pdf, which is the version intended as the final result.

make clean
will remove temporary build files, staged in build/.

If you happen to use a bizarro OS without a GNU-style make and/or texlive, you may still
be able to build the PDF. At the very least you need some kind of pdflatex and makeindex
commands, and all the LATEX packages used. Then you just need to run something like

pdflatex basic-algebra
pdflatex basic-algebra
makeindex basic-algebra
pdflatex basic-algebra
pdflatex basic-algebra
to build the final version of the PDF in the current directory. Good luck.
                                              507
5. ACKNOWLEDGMENTS                                             CHAPTER B. TO DEVELOPERS
    4.3. Branches. The development happens in the branch numbered after the next edition
(first, second, and so on). Once the new edition is ready for use, the changes are merged into
the master branch, and a corresponding tag is created. The top of the master branch is then
merged into the color and mono branches. The master branch, while printable, is intended
to be viewed on a screen; color is a print-ready branch with decolored web-links, and mono
is a print-ready branch done in shades of gray.


                                    5. Acknowledgments

Many homework exercises were shamelessly borrowed from Beginning and Intermediate Al-
gebra, a free and libre (CC-BY) textbook by Tyler Wallace, available for free download at
http://wallace.ccfaculty.org/book/book.html

Many thanks to the Elementary Algebra class of Spring 2018 at Cosumnes River College,
where students graciously reviewed a very early draft of the Concepts chapter.

Even more thanks to the Elementary Algebra class of Summer 2018 at CRC, and especially
Michelle Cao, Margaret Nyambura, and Brittany Pace for proofing the early draft.

Immense amount of thanks to Carlos Campa Navarro for proofing most of the text methodically
and thoroughly, as well as Duy Ly, Gloria Williams, Samreen Javed, and the rest of the Fall 2018
Elementary Algebra class at CRC.

Muito obrigado to Gilberta Iorg and the rest of the Spring 2020 Elementary Algebra class at
CRC.

Yet more thanks to the Fall 2020 classat CRC, in particular Kalsum Mirzai, Katharine Owens-
Byrd, Nguyen Pham, Thuy Vuong, and especially Alla Tkhay.




                                              508
                                                      Index



∞, 139                                                       dividing fractions, 73
∅, 34                                                        division, 29

absolute value, 40                                           element of a set, 34
addition property for equations, 97                          elimination for linear systems, 226
addition property for inequalities, 141                      empty set, 34
addition property for systems, 226                           equation, 15
additive identity, 24                                        equations with rational expressions, 399
additive inverse, 24                                         equivalent equations, 96
algebraic expression, 15                                     equivalent expressions, 15
algebraic substitution, 17                                   equivalent inequalities, 141
associativity of addition, 23                                equivalent systems, 226
associativity of multiplication, 24                          Euclidean distance, 441
axioms, 23                                                   expression, 15
                                                             extraneous solution, 399
binary relation, 16
binomial, 267                                                factor (of a product), 19
bivariate polynomial, 266                                    factoring ax 2 + bx + c by guessing, 332
                                                             factoring x 2 + bx + c by guessing, 325
Cartesian plane, 149                                         factoring by grouping, 321
closure of arithmetic operations, 23                         formula, 117
coefficient, 20                                              fraction in lowest terms, 64
common multiple for polynomials, 377                         fraction notation, 30
commutativity of addition, 23                                function defined by an expression, 202
commutativity of multiplication, 23                          function notation, 202
completing the square, 468                                   fundamental theorem of arithmetic, 62
complex numbers, 456
composite numbers, 62                                        Gaussian elimination, 226
conjugate of a sum with radicals, 429                        GCF, 314
constant monomial, 266                                       GCF for monomials, 316
constant rate applications of systems, 232                   grade of the road, 174
coordinate, 149                                              graph, 150
coordinate plane, 149                                        graph of a relation, 160
cubic monomial, 266                                          greatest common factor for monomials, 316
                                                             greatest common factor, 314
difference of squares, 284                                   grouping, 321
dimensional analysis, 130
discriminant, 475                                            i, 455
distance, 441                                                imaginary unit, 455
distributivity of exponent over division, 260                improper fraction, 82
distributivity of exponent over multiplication, 258          inequality relations, 42
distributivity of multiplication over addition, 25           infinity, 139
                                                       509
                                                                                  CHAPTER B. INDEX
integer divisor, 62                                   number line, 35
integer exponent, 249                                 numerical coefficient, 20
integer reciprocal exponent, 445
integers, 35                                          one, 24
intercepts, 168                                       opposite, 24
interval notation, 139                                opposite of a sum, 49
invisible parentheses, 30                             opposite of opposite, 28
irrational numbers, 39                                order of arithmetic operations, 54
irreducible over integers, 313                        order of operations in fractions, 30
irreducible over reals, 313                           ordered pair, 149
irreducible polynomial, 313                           ordered pair solution, 159
isolating a variable, 105                             orthogonal lines, 188

                                                      parallel lines, 186
leading coefficient, 266
                                                      percent, 123
least common multiple, 80
                                                      percent increase and decrease, 125
least common multiple for polynomials, 377
                                                      perimeter of a rectangle, 117
like terms, 57
                                                      perpendicular lines, 188
like terms with radicals, 427
                                                      plus-minus notation, 462
linear equation, 104
                                                      point on a plane, 149
linear equation graph, 162
                                                      point-slope form, 195
linear function, 204
                                                      polynomial factorization, 314
linear inequality, 138
                                                      polynomial common multiple, 377
linear inequality in two variables, 238
                                                      polynomial degree, 267
linear model, 204
                                                      polynomial division, 290
linear monomial, 266
                                                      polynomial expression, 266
linear system, 213
                                                      polynomial in standard form, 266
linear systems and substitution, 222
                                                      polynomial least common multiple, 377
lowest common denominator, 81
                                                      polynomial long division, 292
lowest common denominator for rational
                                                      positive integer exponent, 50
     expressions, 377
                                                      positive integers, 34
lowest terms fraction, 64
                                                      power rule for exponents, 257
                                                      prime numbers, 62
member of a set, 34
                                                      prime polynomial, 313
mixed numbers, 82
                                                      principal nth root, 411
mixing applications of systems, 233
                                                      principal square root, 409
monomial, 267
                                                      product (expression), 19
monomial coefficient, 265
                                                      product of fractions, 71
monomial degree, 266
                                                      product rule for exponents, 250
monomial expression, 265
                                                      proper fraction, 82
monomial GCF, 316
movement at constant rate, 130                        Q, 37
multiplication property for equations, 98             quadratic formula, 475
multiplication property for inequalities, 142         quadratic monomial, 266
multiplication property for systems, 226              quartic monomial, 266
multiplicative identity, 24                           quintic monomial, 266
multiplicative inverse, 25                            quotient of fractions, 73
multiplying fractions, 71                             quotient rule for exponents, 252

natural numbers, 34                                   R, 38
negative reciprocal, 188                              radical, 409
negative integer exponent, 300                        radical conjugate, 429
non-negative integers, 34                             radix, 409
non-strict inequality, 42                             rational exponent, 446
                                                510
CHAPTER B. INDEX
rational expression, 363                           zero, 24
rational numbers, 37                               zero product in equations, 348
rationalizing the denominator, 429                 zero product property, 348
real number axioms, 23
real numbers, 38
reciprocal, 25
reciprocal of a product, 70
reciprocal of reciprocal, 29
relation, 16
relatively prime, 314
relatively prime monomials, 316
rise, 174
rise over run, 174
roster notation, 34
run, 174

scientific notation, 305
set, 34
set roster notation, 34
set-builder notation, 36
similar terms, 57
similar terms with radicals, 427
simplifying fractions, 64
slope formula, 174
slope of a line, 175
slope of a segment, 174
slope-intercept form, 182
solution, 95
solution set, 95
solution set for bivariate inequality, 238
solving a formula for a variable, 117
square of a binomial, 286
square root, 409
square root property, 461
strict inequality, 42
substitution, 17
substitution for linear systems, 222
subtraction, 29
sum (expression), 18
systems of linear equations, 213

term (of a sum), 18
trinomial, 267

unique factorization domains, 314
unit, 24
univariate polynomial, 266

value of an expression, 15
variable restrictions in expressions, 364
volume of a box, 117

whole numbers, 34
work at constant rate, 130
                                             511